You are on page 1of 139

1

 यूपीएससी अभ्यर्थिय ों के र्िए मार्सक पर्िका के सन्दर्ि में महत्वपूर्ि अपडे ट 

र्प्रय यूपीएससी अभ्यथी,

हमें आर्ा है यक आप आगामी परीक्षा के यिए उत्सायहत और तैिार हैं। करें ट अफेिसन मैगज़ीन के डे ििपसन के रूप में , हम आपकी तैिारी
की िात्रा कद और भी अयिक प्रभािी बनाने के यिए कुछ रदमाोंचक एिों फिदािी ख़बरें तर्ा घटनाक्रम साझा करने के यिए उत्सायहत हैं ।
यहााँ नए सोंस्करर् में कुछ महत्वपूर्ि अपडे ट र्कए गए हैं:
• हाि के रुझान ों के अनुसार र्िकास: हम िूपीएससी परीक्षा की पररितननर्ीि प्रकृयत और उभरते रुझानदों कद समझते हैं , जद प्रश्न-पत्र
कद आकार दे ते हैं। इन पररितननदों कद ध्यान में रखते हुए आपकद सबसे अयिक प्रासोंयगक और परीक्षा-उन्मुख सामग्री प्रदान करने के यिए
हमने अपनी मैगज़ीन कद अनुकूयित यकिा है ।
• थीम-आधाररि दृर्िक र्: िूपीएससी परीक्षा में यियर्ष्ट यिषिदों पर ध्यान केंयित करने की हायििा प्रिृयि के प्रयतउिर में, हम हमने अपनी
अध्यिन सामग्री कद उन आिश्यक यिषिदों के इदन -यगदन सोंरयचत यकिा है जद अक्सर प्रश्न-पत्रदों में पूछे जाते हैं ।
 उदाहरण के यिए, हाि की प्रारों यभक परीक्षा के प्रश्न-पत्र में “यियि की सम्यक प्रयक्रिा के यसद्धाों त” के बारे में पूछा गिा र्ा, इसयिए
हमारे द्वारा “यियि की सम्यक प्रयक्रिा” पर चचान कद र्ायमि यकिा गिा है ; इसी तरह, अन्य टॉयपक; जैसे-खाद्य प्रसोंस्करण क्षेत्र और
भारतीि प्रिायसिदों की भूयमका आयद कद भी जदडा गिा है ।
• महत्वपूर्ि घटनाओों पर दृर्िक र् आधाररि आिेख: इस सोंस्करण में, हमने एक व्यापक फीचर आिेख र्ायमि यकिा है जद महत्वपूणन
घटनाओों तर्ा समाज, अर्नव्यिस्र्ा और पिानिरण पर उनके प्रभाि पर प्रकार् डािता है । िह आिेख आपकद इस बात की समग्र समझ
प्रदान करता है यक ऐसी घटनाएों नीयतगत यनणनिदों कद कैसे आकार दे ती हैं और यिश्व स्तर पर िदगदों के जीिन कद कैसे प्रभायित करती हैं ।
• प्रारम्भिक परीक्षा के र्िए सोंर्क्षप्त ख़बरें : हम आपकी परीक्षा की तैिारी में समि के मूल्य कद समझते हैं । इसयिए, हमारे द्वारा
महत्वपूणन समसामयिक ख़बरदों कद सोंयक्षप्त और अयत सोंयक्षप्त ददनदों प्रारूपदों में तैिार यकिा गिा है । िे सोंयक्षप्त खबरें आपके अध्यिन
कािनक्रम से समझौता यकए यबना आपकद अपडे ट रहने में सहािक हदोंगे।
• र्ििर् और आों कड़ का व्यापक उपय ग: अध्यिन िदग्य और समझ कद बढाने के यिए, हमारे द्वारा सोंपूणन मैगज़ीन में अयिक यचत्र,
इन्फदग्रायफक्स और महत्वपूणन आों कडदों कद र्ायमि यकिा गिा है । यचत्रात्मक सामग्री आपकी समझ कद और बेहतर करने तर्ा जयटि
सूचनाओों कद िाद रखने में सहािता प्रदान करे गी।
• नीर्िशास्त्र सम्बन्धी आिेख: इस सोंस्करण में नीयतर्ास्त्र से सम्बोंयित एक आिेख भी र्ायमि है , जहाँ हम आपकद सैद्धाों यतक ज्ञान का
प्रिदग िास्तयिक जीिन के पररदृश्यदों में करने के यिए मागनदर्नन प्रदान करें गे । हम आपकद स्विों के अनुभि से उदाहरण यनकािने के
यिए प्रदत्सायहत करते हैं क्दोंयक इससे आपकद एयर्क्स की तैिारी के दौरान अपने यिचारदों कद अयिक प्रभािी ढों ग से व्यक्त करने में मदद
यमिेगी ।
 हमने एर्ेज सीरीज की हायििा घटना कद यििा है और इसे एयर्क्स (जीएस IV पेपर) की माों ग के अनुसार िागू यकिा है ।
• प्रीर्िम्स और मेन्स के र्िए प्रश्न: हमने मेन्स के यिए आिेख पढने के बाद आपकी समझ का परीक्षण करने के यिए प्रश्न जदडे हैं । इसके
अिािा, हम ‘टे स्ट िदरसेल्फ’ अनुभाग में प्रारों यभक परीक्षा के यिए प्रश्न र्ायमि यकिे हैं , जद अोंत में एक पररयर्ष्ट के रूप में यदिा यकिा
गिा है ।

हमारी प्रर्िबद्धिा:
समसामयिक मुद्दों के यिए आपके यिश्वसनीि अध्यिन स्रदत के रूप में, PWOnlyIAS परीक्षा-पुनरुत्पादन िदग्य सामग्री प्रदान करने
के यिए प्रयतबद्ध है । जद िूपीएससी के बदिते पैटनन के अनुरूप है । हम आपके ज्ञान भागीदार और सफिता की ओर आपकी िात्रा के
समर्नक के रूप में अपनी भूयमका पर गिन करते हैं। िाद रखें यक िूपीएससी परीक्षा केिि तथ्दों कद ददहराने के बारे में नहीों है , बल्कि
यियभन्न मुद्दों की व्यापक समझ का प्रदर्नन से सम्बोंयित है । हम आपसे आग्रह करते हैं यक समािदचनात्मक समझ यिकयसत करें और
मैगज़ीन में र्ायमि यिषिदों से गहन रूप से सम्बद्ध रहें । हम आपकी तैिारी के यिए आपकद र्ुभकामनाएँ दे ते हैं ! ध्यान केंयित करें ,
सकारात्मक रहें और उत्कृष्टता के यिए प्रिास करते रहें ।

2
 र्िषय सूिी 
1. राजव्यवस्था और शासन _____________________________________ 5 3.10 र्ारि में खाद्य प्रसोंस्करर् उद्य ग ______________________ 53
1.1 र्िर्ध की सम्यक प्रक्रिया _________________________________ 5 3.11 र्ारि में सेमीकोंडरर र्िर्नमािर्_______________________ 55
1.2 वैवाक्रिक बलात्कार का अपराधीकरण ____________________ 7 िघु समािार
1.3 क्रिक्रजटल व्यक्तिगत िे टा संरक्षण (िीपीिीपी) क्रवधेयक 2022 3.12 यूर्निसिि बेर्सक इनकम _____________________________ 57
____________________________________________________________ 9 3.13 िैर्िक प्रर्िस्पधाित्मकिा सूिकाोंक _____________________ 57
1.4 ट् ांसजेंिर समुदाय के क्रलए आरक्षण _____________________ 12 3.14 र्ौग र्िक सोंकेि टै ग _________________________________ 58
िघु समािार 3.15 एआई के र्िए टर ाई की र्सिाररश ______________________ 58
1.5 क्रवधायक ं की अय ग्यता ________________________________ 13 3.16 र्ििररि निीकरर्ीय ऊिाि ____________________________ 58
1.6 क्रनयम 176 बनाम क्रनयम 267 ___________________________ 14 3.17 मूल् म्भस्थरीकरर् क ष ________________________________ 59
1.7 मौन रिने का अक्रधकार _________________________________ 14 डे टा पॉइों ट्स
1.8 जनजातीय सलािकार पररषदें (टीएसी) __________________ 14 3.18 थ क मूल् सूिकाोंक मुद्रास्फीर्ि के रुझान_____________ 59
1.9 पररसीमन अभ्यास _____________________________________ 14 3.19 िषि की पहिी र्िमाही में र्नयािि में र्गरािट _____________ 60
िार्िका में सोंर्क्षप्त समािार 3.20 िस्तु एिों सेिा कर के छह िषि __________________________ 60
1.10 रािरगान: उच्चिम न्यायािय के र्िशार्निे श_____________15 िार्िका में सोंर्क्षप्त समािार
1.11 शहरी 20 मेयरि र्शखर सम्मेिन_____________________ 15 3.21 सेबी र्शकायि र्निारर् प्रर्ािी (स्क र) प्लेटिामि ______60
1.12 रािरीय ई-गिनेंस सेिा र्ििरर् मूल्ाोंकन (NeSDA) 3.22 र्गफ्ट र्नफ्टी__________________________________________61
प टि ि_____________________________________________________15 3.23 अोंिराििरीय र्ििीय सेिा केंद्र प्रार्धकरर्
1.13 ई-सरस म बाइि ऐप_________________________________15 (आईएिएससीए)__________________________________________61
1.14 पोंिायि र्िकास सूिकाोंक_____________________________16 3.24 र्टयर 2 बाोंड__________________________________________61
1.15 र्डर्िटि टाइम िाउिर-20 िुिाई ____________________16 3.25 ग्रीडफ्लेशन___________________________________________61
2. अंतरााष्ट््ीय संबंध ____________________________________________ 17 3.26 एकीकृि िॉर्िम्भस्टक्स इों टरफेस प्लेटफॉमि ______________62
2.1 शंघाई सिय ग संगठन (SCO) के राष्ट््ाध्यक्ष ं की पररषद का 3.27 स्टीि स्लैग र ड टे क्न िॉिी____________________________62
23वां क्रशखर सम्मेलन ______________________________________ 17 3.28 साोंम्भिकी पर स्थायी सर्मर्ि
2.2 भारत-श्रीलंका _________________________________________ 19 (एससीओएस)_____________________________________________62
2.3 भारतीय प्रवासी ________________________________________ 21 3.29 िैंर्पयोंस 2.0 प टि ि____________________________________62
2.4 शरणाथी संकट पर यूएनएचसीआर ररप टा _______________ 23 3.30 प्राथर्मक कृर्ष ऋर् सर्मर्ियााँ (पीएसीएस)_____________62
2.5 भारत-फ्ांस संबंध ______________________________________ 25 3.31 नोंिी प टि ि___________________________________________ 62
2.6 बहुपक्षीय क्रवकास बैंक (एमिीबी) सुधार _________________ 26 3.32 र्िय क र्डों ग_________________________________________ 63
िघु समािार 3.33 उत्प्रेरक ______________________________________________63
2.7 अंतरााष्ट््ीय उत्तर दक्रक्षण व्यापार गक्रलयारा (आईएनएसटीसी) 3.34 GeM प टि ि _________________________________________63
___________________________________________________________ 29 3.35 िस्ट एनिी टर ाोंर्िशन पाटि नरर्शप (JET-P)_____________63
2.8 भारत-यूएई संबंध ______________________________________ 29 समािार में सूिकाोंक/ररप टि
2.9 नाट __________________________________________________ 29 3.36 र्नयािि िैयारी सूिकाोंक (ईपीआई) 2022 ______________64
िार्िका में सोंर्क्षप्त समािार 3.37 ऊिाि सोंक्रमर् सूिकाोंक ______________________________64
2.10 हेनिे पासप टि इों डेक्स 2023__________________________ 30 3.38 र्कसान सोंकट सूिकाोंक ______________________________64
2.11 टर ाोंस-पैर्सर्िक पाटि नरर्शप के र्िए व्यापक और प्रगर्िशीि 3.39 र्िि र्निेश ररप टि ____________________________________64
समझौिा (सीपीटीपीपी)____________________________________30 4. पयाििरर् और र्ूग ि _______________________________________ 65
2.12 पूिी एर्शया र्शखर सम्मेिन___________________________30 4.1 िििायु र्ििप षर् _____________________________________ 65
2.13 अोंिरसरकारी िािाि रूपरे खा __________________________30 4.2 आपिा ि म्भखम में कमी ________________________________ 67
2.14 शहरी 20 मेयरि र्शखर सम्मेिन______________________30 4.3 महासागर सोंसाधन और इसकी क्षमिा __________________ 69
2.15 यूर्निसिि प स्टि यूर्नयन (UPI)______________________ 30 4.4 िििायु स्माटि कृर्ष (सीएसए) __________________________ 71
2.16 र्ारि-इों ड नेर्शया ईएिडी____________________________31 िघु समािार
2.17 ग्ल बि साउथ________________________________________31 4.5 एम्बरग्रीस _____________________________________________ 73
3. अथाव्यवस्था ________________________________________________ 32 4.6 कछु ए: गोंगा में सिाई एिेंट _____________________________ 73
3.1 रुपये का अंतरााष्ट््ीयकरण _______________________________ 32 4.7 कून में िीिा की मृत्यु िर _______________________________ 73
3.2 बैटरी इिेम्भरर क िाहन __________________________________ 34 4.8 ग्ल बि र्गब्बन नेटिकि (िीिीएन) ______________________ 74
3.3 ऑनिाइन गेर्मोंग ______________________________________ 36 4.9 ग्ल बि िॉरे स्ट िॉि ____________________________________ 74
3.4 पररसोंपर्ि मुद्रीकरर् ___________________________________ 39 4.10 काबिन कैप्िररों ग और र्ोंडारर् _________________________ 75
3.5 क यिा गैसीकरर्______________________________________ 42 4.11 एों थ्र प सीन युग _______________________________________ 75
3.6 बहुआयामी गरीबी ______________________________________ 44 4.12 पमािफ्रॉस्ट र्िगिन: बटागाइका क्रेटर __________________ 75
3.7 िाि उत्पािन __________________________________________ 46 5. फ कस आलेख ____________________________________________ 77
3.8 र्डर्िटि अथिव्यिस्था __________________________________ 47 5.1 चरम मौसम घटना _____________________________________ 77
3.9 र्ारि में र्गग अथिव्यिस्था ______________________________ 50

3
6. र्िज्ञान और प्रौद्य र्गकी _____________________________________ 83 8.10 बच्चे और सशस्त्र सोंघषि: सोंयुक्त रािर ररप टि ____________115
6.1 िीन म अनुक्रमर् ______________________________________ 83 9. किा एिों सोंस्कृर्ि _________________________________________116
6.2 िोंद्रयान-3______________________________________________ 84 9.1 पुरािशेष और किा खिाना अर्धर्नयम ________________116
6.3 र्ारि 6G एिायोंस______________________________________ 86 िार्िका में सोंर्क्षप्त समािार
6.4 नेशनि ररसिि िाउों डेशन र्िधेयक, 2023 _______________ 88 9.2 रुद्रर्गरर पहाड़ी_______________________________________118
6.5 र्क्रप्ट करें सी ____________________________________________ 91 9.3 र्सिे हुए िहाि र्नमािर् र्िर्ध (टों काई र्िर्ध)____________118
6.6 िािि र्ाषा मॉडि-िैटिीपीटी ___________________________ 93 9.4 नमिा किा __________________________________________118
िघु समािार 9.5 िम्बानी कढ़ाई किा__________________________________119
6.7 क्वासर _________________________________________________ 94 9.6 िैर्िक शाोंर्ि सूिकाोंक 2023 _________________________119
6.8 डाकि पैटनि _____________________________________________ 94 9.7 हुि र्ििस____________________________________________119
6.9 र्िशाि मीटरिेि रे र्डय टे िीस्क प ______________________ 95 9.8 मीरा पैर्बस __________________________________________119
6.10 नॉन-िोंर्गबि ट कन __________________________________ 95 9.9 रािरीय समुद्री र्िरासि पररसर_________________________119
6.11 र्ारि में मधुमेह मेर्िटस (डीएम) और िपेर्िक (टीबी)। 95 10. नीर्िशास्त्र (एर्थक्स) ______________________________________120
6.12 म जेक िायरस _______________________________________ 96 10.1 खेि के र्नयम बनाम खेि की र्ािना _________________120
6.13 र्िमाग खाने िािा अमीबा _____________________________ 96 11. समािार ों में व्यम्भक्तत्व______________________________________123
6.14 बाह्य अोंिररक्ष मामि ों के र्िए सोंयुक्त रािर कायाििय 11.1 ि कमान्य र्ििक ___________________________________123
(यूएनओओएसए) __________________________________________ 96 11.2 िोंद्रशेखर आिाि ___________________________________123
6.15 सागर सोंपकि __________________________________________ 96 11.3 रॉबटि ओप्पेहाइमर __________________________________123
6.16 एस्पाटे म _____________________________________________ 97 11.4 श्यामा प्रसाि मुखिी________________________________123
6.18 ओजेम्भिक औषर्ध ____________________________________ 97 11.5 अल्लूरी सीिाराम रािू ______________________________123
6.19 पीबीडब्ल्यू आरएस1: गेहों की नई र्कस्म ________________ 97 11.6 र्मर्हर र् ि ________________________________________123
िार्िका में सोंर्क्षप्त समािार 11.7 रानी िु गािििी_______________________________________124
6.20 अोंिररक्ष में काबिन अर्ु ________________________________98 12. समािार ों में स्थान _________________________________________125
6.21 हाइडर िन ईोंधन सेि__________________________________98 12.1 कच्चार्ििू द्वीप ______________________________________125
6.22 स्माटि बैंडेि __________________________________________98 12.2 ओडे सा का यूक्रेन बोंिरगाह __________________________125
6.23 यूम्भिड र्मशन _______________________________________98 12.3 क्रीर्मया र्िि _______________________________________125
6.24 डिेन मस्कुिर र्डस्टर ॉिी (डीएमडी) ___________________98 12.4 इराक ______________________________________________126
6.25 घन सोंग्रहर्___________________________________________98 12.5 कास पठार _________________________________________127
6.26 थ्रेड्स ________________________________________________98 12.6 स ि मन इस्लैंडस __________________________________126
6.27 उपय ग प्र ट कॉि ____________________________________99 12.7 पैंग ग ों त्स झीि_____________________________________127
6.28 िेप्ट स्पाइर र्सस _____________________________________99 13. ख़बर ों में य िनाएों _________________________________________128
6.29 रािरीय अोंग एिों ऊिक प्रत्यार पर् सोंगठन (NOTTO) ___99 13.1 राज् ों में अर्िशमन सेिाओों के र्िस्तार और
7. रक्षा और सुरक्षा __________________________________________ 100 आधुर्नकीकरर् की य िना _______________________________128
7.1 र्ारि में नशीिी ििाओों का खिरा ____________________ 100 13.2 उिर पूिी क्षेि के र्िए र्मशन िैर्िक मूल् श्ृोंखिा
िघु समािार र्िकास__________________________________________________128
7.2 सस्पेंशन ऑि ऑपरे शोंस (एसओओ) _________________ 102 13.3 अटि िय अभ्युिय य िना__________________________128
7.3 र्ारिीय नौसेना के र्िए रािेि _______________________ 102 13.4 समथि ______________________________________________128
7.4 िस्टर युद्ध सामग्री __________________________________ 103 13.5 नाबार्िग बिात्कार पीर्ड़ि ों की सहायिा के र्िए
7.5 सूिना सुरक्षा प्रथाओों पर र्िशार्निे श: सीईआरटी-इन __ 103 य िना___________________________________________________128
समाचार में अभ्यास: 13.6 पशुधन क्षेि के र्िए ऋर् गारों टी य िना ______________128
7.6 र्िमेक्स 23___________________________________________103 13.7 पीएम-प्रर्ाम य िना ________________________________129
7.7 साल्वेक्स _____________________________________________103 13.8 हररि ऋर् य िना___________________________________129
8. समाि और सामार्िक न्याय ______________________________ 104 13.9 पूोंिी र्निेश 2023-24' य िना के र्िए राज् ों क र्िशेष
8.1 इच्छामृत्यु ____________________________________________ 104 सहायिा _________________________________________________129
8.2 र्िि िनसोंिा र्ििस और र्ारि िनसोंिा नीर्ि ______ 106 13.10 सेिा र् ि य िना__________________________________129
8.3 र्ारि में प्रशामक िे खर्ाि क सुदृढ़ बनाना ___________ 109 13.11 अर्ग्रम प्रार्धकरर् य िना __________________________129
8.4 स शि स्टॉक एक्सिेंि _______________________________ 111 13.12 स्वामी र्निेश क ष_________________________________129
8.5 घरे िू पेयिि, सिाई िथा स्वच्छिा (िॉश) 2000-2022 पर 14. र्िर्िध ____________________________________________________130
प्रगर्ि ____________________________________________________ 113 14.1 र्ििे श में पहिा आईआईटी कैंपस ____________________130
िघु समािार समाचार में पुरस्कार
8.6 स्वच्छ सिेक्षर् 2023 _________________________________ 114 14.1 पेन र्पोंटर पुरस्कार_______________________________130
िार्िका में सोंर्क्षप्त समािार 14.2 रािरीय र्ूर्िज्ञान पुरस्कार_________________________130
8.9 प्रिशिन ग्रेर्डों ग सूिकाोंक 2.0___________________________115

FIND TEST YOURSELF (PAGE NO. 131-135) AT THE END…

4
1. राजव्यवस्था और शासन
1.1 र्िर्ध की सम्यक प्रक्रिया
संदभा:
समान नागरिक संहिता पि चचाा के आलोक में, यि तका हिया जाता िै हक यियि की सम्यक प्रहिया, यियि द्वािा स्थाहपत प्रहिया की तुलना में
मौहलक अहिकािों की सुिक्षा का अहिक प्रभावी सािन िै ।
र्िर्ध की सम्यक प्रक्रिया के बारे में:
• यि एक ऐसी कानूनी प्रहिया िै जो व्यक्तिगत अक्रधकार ं क
बनाए िखने औि उसके बचाव के हलए र्िर्ध के एक हनकाय
में स्थाहपत हिशाहनिे शों औि हनयमों का पालन किती िै ।
• सम्यक प्रहिया िीवानी औि आपिाहिक िोनों मामलों पि
लागू िोती िै ।
• यि यियि का एक मूलभूत हसद्ां त िै जो व्यक्तिय ं क
मनमानी सरकारी कारा वाई से बचाता िै।
• संहविान हनमाा ता संहविान के अनुच्छेि 21 में र्िर्ध की
सम्यक प्रक्रिया के बजाय र्िर्ध द्वारा स्थाक्रपत प्रक्रिया क
आगे बढाते रिे:
 "र्िर्ध द्वारा स्थाक्रपत प्रक्रिया के अलावा हकसी भी व्यक्ति
को उसके जीवन या व्यक्तिगत स्वतंत्रता से वंहचत निीं
हकया जाएगा। "
र्िर्ध द्वारा स्थाक्रपत प्रक्रिया और र्िर्ध की उक्रचत प्रक्रिया के
बीच अंतर:

र्िर्ध द्वारा स्थाक्रपत प्रक्रिया र्िर्ध की सम्यक प्रक्रिया

• अिालतें केवल यि मूल्ां कन किती िैं हक र्िर्ध हनमाा ता औि • न्यायालय हकसी हवशेष र्िर्ध को बनाने के पीछे के हविायी ज्ञान को
अहिकािी उहचत प्रहियाओं का पालन कि ििे िैं या निीं। िे खेगा।
• यहि कोई र्िर्ध प्राकृहतक न्याय या समानता का उल्लंघन किता • यहि कोई र्िर्ध अनुहचत औि अन्यायपूर्ा िै तो वि यि घोहषत कि
िै , तो भी वि इसके पीछे के हविायी तका पि चचाा निीं किे गा। सकता िै हक यि प्राकृहतक न्याय के हसद्ांत के क्तखलाफ िै औि इस
• इसे जापानी संहविान से हलया गया। तिि इसे खारिज हकया जा सकता िै ।
• भाितीय संहविान के अनुच्छेि 21 में स्पष्ट रूप से उपलब्ध िै । • यि अमेरिकी संहविान में उक्तल्लक्तखत िै ।
• संहविान में कोई स्पष्ट उल्लेख निीं िै ।

भारतीय संदभा में र्िर्ध द्वारा स्थाक्रपत प्रक्रिया क र्िर्ध की की सम्यक प्रक्रिया से अक्रधक प्राथक्रमकता क् ं दी गई?
• क्रवधानमंिल प्राक्रधकार क कमज र करना: भाितीय संहविान के हनमाा ता हविायी प्राहिकाि को चुनौती िे ने वाले सम्यक प्रक्रिया क्रसद्ांत
क लेकर क्रचंक्रतत थे।
• न्यायालय ं क मनमानी शक्ति: अमेरिका ने अन्यायपूर्ा कानूनों को पलटने के हलए सम्यक प्रहिया हसद्ां त का उपयोग हकया िै । भाितीय
संहविान के हनमााताओं को हचंता थी हक इससे अिालतों को यियि को प्रभाहवत किने की बहुत अहिक शक्ति हमल जाएगी।
• र्िर्ध द्वारा स्थाक्रपत प्रक्रिया का लचीलापन: "यियि द्वािा स्थाहपत प्रहिया" अहिक लचीली िै , क्ोंहक इसके हलए केवल यि आवश्यक िै
हक र्िर्ध उन सिी प्रहियाओं का पालन किें हजससे अिालतों के पास उन कानूनों को िद्द किने का हववेकाहिकाि कम िो जाता िै हजन्हें वे
मनमाना मानते िैं।
• र्िर्ध द्वारा स्थाक्रपत प्रक्रिया भारतीय न्याक्रयक परं परा के अक्रधक अनुरूप िै : भाितीय न्यायपाहलका पािं परिक रूप से हविाहयका के
हनर्ायों में िस्तक्षेप किने में अहनच्छु क ििी िै ।
 इस प्रकाि, यियि द्वािा स्थाहपत प्रहिया सम्यक प्रहिया हसद्ांत की तुलना में न्याहयक संयम की इस पिं पिा के अहिक अनुरूप िै ।

5
र्िर्ध द्वारा स्थाक्रपत प्रक्रिया से जुडी भ्ांक्रतयां :
• न्याय का उल्लंघन: यियि द्वािा स्थाहपत प्रहिया किती िै हक जो
र्िर्ध हवहि पूवाक बनाया गया िै वि वैि िै , भले िी वि न्याय औि
न्यायसंगत आिशों का उल्लंघन किता िो।
• स्वतंत्रता क कमज र करना: कानूनी प्रहिया का कडाई से पालन
किने से र्िर्ध बनाने वाले प्राहिकािी द्वािा बनाए गए अन्यायपूर्ा
कानूनों के कािर् व्यक्तियों के जीवन औि व्यक्तिगत स्वतंत्रता के
खतिे में पडने का खतिा बढ़ सकता िै ।
• अत्यक्रधक शक्ति: यहि संसि कोई र्िर्ध पारित किती िै , तो उस
र्िर्ध के प्राविानों औि प्रहियाओं के अनुसाि हकसी व्यक्ति की
जीवन या व्यक्तिगत स्वतंत्रता छीनी जा सकती िै ।

सम्यक प्रक्रिया क्रसद्ांत और भारतीय न्यायपाक्रलका की भूक्रमका:


• सवोच्च न्यायालय की भूक्रमका: संहविान हनमााताओं द्वािा
जानबूझकि छोडे जाने के बावजूि, भाित का सवोच्च न्यायालय
भाितीय संहविान में सम्यक प्रहिया को पढ़ने के हलए अनुच्छेद 14
और 21 की व्याख्या करता िै।

भारत में र्िर्ध की सम्यक प्रक्रिया से जुडे न्याक्रयक मामले :


• ए. के. ग पालन बनाम मद्रास राज्य (1950):
 पृष्ठभूक्रम: हनवािक हनिोि अहिहनयम 1950 ने कम्युहनस्ट नेता
ए. के. गोपालन को कैि कि हलया। याहचकाकताा ने अनुच्छेि
32 के तित एक रिट याहचका िायि की औि आिोप लगाया हक
यि अहिहनयम अनुच्छेि 19 (1) (डी) के तित आवागमन की
स्वतंत्रता औि अनुच्छेि 21 के तित व्यक्तिगत स्वतंत्रता का
उल्लंघन किता िै ।
 व्याख्या: उच्चतम न्यायालय ने क्रनवारक क्रनर ध अक्रधक्रनयम
1950 क संवैिाहनक घोहषत हकया औि सम्यक प्रक्रिया के
क्रसद्ांत और र्िर्ध द्वारा स्थाक्रपत प्रक्रिया के बीच अंतर क क्रवस्तार से बताया।
 क्रनष्कषा: इस प्रकाि, सम्यक प्रहिया का हसद्ां त भाित में लागू निीं हकया गया क्ोंहक ए. के. गोपालन मामला एक हमसाल बन गया।
• आरसी कूपर मामला:
 पृष्ठभूक्रम: 1970 तक, आरसी कूपर मामले में उच्चतम न्यायालय ने गोपालन मामले के कठोि दृहष्टकोर् को गंभीिता से कमजोि कि
हिया।
 व्याख्या: इसने घोहषत हकया हक संहविान के प्रमुख मौहलक अहिकाि - अनुच्छेद 14 में समानता, अनुच्छेद 19 की मौहलक स्वतंत्रता ,
औि अनुच्छेद 21 के तित जीवन औि स्वतंत्रता की सुिक्षा - एक-िू सिे से जुडे हुए िैं ।
• मेनका गांधी बनाम भारत संघ (1978)
 पृष्ठभूक्रम: मेनका गांधी ने पासप टा अक्रधक्रनयम, 1967 की धारा 10(3)(ग) की संवैिाहनकता पि सवाल उठाया, जो सावाजहनक हित
में पासपोटा को जब्त किने का अहिकाि िे ता िै ।
 व्याख्या: सात न्यायाधीश ं की पीठ ने माना क्रक अनुच्छेद 21 के तित स्थाहपत प्रहिया न्यायसंगत, हनष्पक्ष औि उहचत िोनी चाहिए।
इसके अलावा ऐसी प्रहियाओं का पिीक्षर् संक्रवधान के अनुच्छेद 14 और 19 के तित क्रकया जाना चाक्रिए।
✓ क्रनष्कषा: इस प्रकाि भाित में तका स्थाहपत िोता िै - र्िर्ध द्वारा स्थाक्रपत प्रक्रिया + प्रक्रिया क्रनष्पक्ष और उक्रचत ि नी चाक्रिए न
क्रक मनमानी।
• केएस पुट्टास्वामी बनाम भारत संघ (2017): न्यायालय ने इस बात पि जोि हिया हक वाक्ां श "र्िर्ध की सम्यक प्रक्रिया" कुछ
व्याख्यात्मक कहठनाइयों को जन्म िे ता िै औि अस्पष्ट िै ।

6
• केशर क्रसंि रामकृष्ण पाक्रटल और अन्य बनाम मिाराष्ट्् राज्य और अन्य। (2017): न्यायालय ने फैसला सुनाया क्रक यहि सम्यक
प्रहिया के हनयमों का पालन किने के बाि अवैि हनमाा र् िटा हिए जाते िैं तो संहविान के अनुच्छेद 300-ए के तित अक्रधकार ं का उल्लंघन
निीं िोता िै ।

र्िर्ध की उक्रचत प्रक्रिया का मित्व:


• व्यक्तिगत अक्रधकार ं की सुरक्षा: सम्यक प्रक्रिया यि गारं टी दे ती िै क्रक व्यक्तिय ं क जीवन, स्वतंत्रता या संपक्रत्त से हकसी भी प्रकाि
वंहचत किने से पिले नोहटस औि सुनवाई का अवसि हिया जाता िै ।
• र्िर्ध का शासन: उहचत प्रहिया एक ऐसा ढााँचा स्थाहपत किती िै जिााँ र्िर्ध सभी व्यक्तियों पि लगाताि औि हनष्पक्ष रूप से लागू िोते िैं ,
चािे उनकी सामाहजक क्तस्थहत, िन या प्रभाव कुछ भी िो।
• क्रनष्पक्षता और न्याय: सम्यक प्रहिया के हलए सिकाि को स्थाहपत प्रहियाओं का पालन किना, साक्ष्य प्रस्तुत किना औि व्यक्तियों को
अपना बचाव किने का अवसि िे ना आवश्यक िै ।
• शक्ति के दु रुपय ग के क्रवरुद् सुरक्षा: उहचत प्रहिया सिकािी प्राहिकाि के प्रयोग पि सीमा तय किती िै औि मनमाने कायों को िोकती
िै जो व्यक्तिगत अहिकािों का उल्लंघन कि सकते िैं।
• मानवीय गररमा: सम्यक प्रहिया यि सुहनहित किके प्रत्येक व्यक्ति की अंतहनाहित गरिमा को पिचानती िै हक उनके साथ िूि, अमानवीय
या अपमानजनक व्यविाि निीं हकया जाता िै।
• संगक्रत और समानता: यि कानूनों के कायाा न्वयन में क्तस्थिता को बढ़ावा िे ता िै औि यि सुहनहित किता िै हक सभी व्यक्तियों के साथ
उनकी सामाहजक क्तस्थहत, जातीयता, िमा या हकसी अन्य हवशेषता की पिवाि हकए हबना, र्िर्ध के तित समान व्यविाि हकया जाता िै ।
• कानूनी क्रनक्रितता: सम्यक प्रहिया कानूनों के कायाान्वयन के हलए स्पष्ट हनयम औि प्रहियाएाँ प्रिान किके कानूनी हनहितता में योगिान
किती िै ।
प्र. भाितीय यियि संिभा में, "कानून द्वािा स्थाहपत
क्रनष्कषा: प्रहिया" को "यियि की सम्यक प्रहिया" हसद्ांत से
• उहचत प्रहिया का हसद्ां त पूिी तिि से लागू निीं हकया गया िै जैसा हक संयुि िाज्य अहिक प्राथहमकता क्ों िी जाती िै , औि भाितीय
अमेरिका में लागू हकया गया िै , लेहकन भाित में हसद्ां त के पीछे की प्रेिक शक्तियों का न्यायपाहलका एवं कानूनी प्रर्ाली के हलए इस
पालन हकया जाता िै औि इस प्रकाि भाित में लोगों के अहिकािों की िक्षा की जाती िै । प्राथहमकता के क्ा हनहिताथा िैं ?

1.2 वै व ाक्रिक बलात्कार का अपराधीकरण


संदभा:
िाल िी में, उच्चतम न्यायालय वैवाहिक बलात्काि को अपिाि मानने की मां ग किने वाली कई याहचकाओं पि सुनवाई किने के हलए सिमत
हुआ।
वैवाक्रिक बलात्कार के बारे में:
• वैवाहिक बलात्काि या पयत द्वारा पत्नी का बलात्काि, हकसी पत्नी की सिमहत के हबना उसके पयत द्वारा यौन संबंि बनाना िै ।
• वैवाहिक बलात्काि को अब यौन हिं सा माना जाता िै , लेहकन भारत में वैवाक्रिक बलात्कार क अपराध घ क्रषत करना क् ं मित्वपूणा िै?
ऐहतिाहसक रूप से, पहत-पत्नी को यौन संबंिों का अहिकाि था। • िाष्टरीय परिवाि स्वास्थ्य सवेक्षर् (एनएफएचएस) के तीसिे (2005-06) औि
भारत में बलात्कार से संबंक्रधत र्िर्ध: चौथे (2015-16) िौि में पाया गया हक हवहभन्न िाज्यों में मक्रिलाओं के
क्तखलाफ अंतरं ग साथी क्रिंसा (आईपीवी) 3% से 43% तक िै।
आईपीसी की धारा 375:
• 2019-20 में लगभग 6,37,000 नमूना घिों में हकए गए सवेक्षर् के पांचवें
• यि िािा यौन सिमहत की सात िािर्ाओं को सूचीबद् किता िै,
िौि में पाया गया हक 18-49 आयु वगा की 3 में से 1 भारतीय मक्रिला
जो अगि टू टती िैं , तो क्रकसी पुरुष द्वारा बलात्कार का अपराध
क वैवाक्रिक क्रिंसा का अनुभव ि ता िै, क्रजसमें 5% -6% ने यौन क्रिंसा
माना जाएगा। की सूचना दी िै।
• िालााँक्रक, इस प्रावधान में एक मित्वपूणा छूट शाक्रमल िै :
हकसी पुरुष द्वािा अपनी िी पत्नी, क्रजसकी उम्र अठारि वषा से कम न ि , के साथ यौन संबंध या यौन कृत्य बलात्काि निीं िै ।
• वैवाहिक बलात्काि अपवाि म टे तौर पर द धारणाओं पर आधाररत िै :
 शाश्वत सिमक्रत: यि मानता िै हक शािी के बाि महिला की सिमहत अपरिवतानीय िै । औपहनवेहशक र्िर्ध का यि हवचाि हक एक
महिला पहत की "संपहि" िै , इस अविािर्ा का आिाि िै ।
 सेक्स की अपेक्षा: चूंहक हववाि संतानोत्पहि के हलए िोता िै , इसहलए यि माना जाता िै हक एक महिला यौन हजम्मेिारियों को पूिा किे गी।
प्राविान का तात्पया यि िै हक एक महिला हववाि में सेक्स से इनकाि निीं कि सकती क्ोंहक पहत इसकी अपेक्षा किता िै ।

7
वैवाक्रिक बलात्कार क अपराध घ क्रषत करने के क्रलए तका: क्रवक्रभन्न संस्थान ं द्वारा वै वाक्रिक बलात्कार पर क्रवचार:
• शारीररक अखंिता का अक्रधकार: हकसी महिला का अपने भारत के क्रवक्रध आय ग का रुख:
शरीर और प्रजनन पर स्वायत्तता का अक्रधकार उसकी • भाित के हवहि आयोग ने अपनी 172 वीं रिपोटा में िािा 375 के अपवाि में
समानता, हनजता औि शािीरिक अखंडता के हलए मौहलक िै । संशोिन किके वैवाहिक बलात्काि को अपिाि घोहषत किने की अनुशंसा
• सिमक्रत और स्वायत्तता: वैवाहिक बलात्काि र्िर्ध इस बात पि निीं की।
जोि िे ते िैं हक हववाि का मतलब यौन गहतहवहि के हलए स्थायी जेएस वमाा सक्रमक्रत की क्रसफाररशें :

सिमहत निीं िै । • 2013 में, हिल्ली में हनभाया सामूहिक बलात्काि के बाि गहठत जेएस वमाा
सहमहत ने वैवाहिक बलात्काि को एक दाल्किक अपिाि बनाने की
• लैंक्रगक असमानता क संब क्रधत करना: वैवाहिक बलात्काि
हसफारिश की थी।
र्िर्ध ऐहतिाहसक लैंहगक असंतुलन को स्वीकाि किते िैं औि
भारतीय न्यायपाक्रलका:
उसका समािान किते िैं , हजसने अक्सि हववाि संस्था के भीति
• अरनेश कुमार बनाम क्रबिार राज्य एवं अन्य (2014): न्यायालय ने
महिलाओं को िाहशये पि िकेल हिया िै । फैसला हकया हक वैवाहिक बलात्काि को अपिाि घोहषत किने से
• दु व्याविार से सुरक्षा: वैवाहिक बलात्काि घिे लू हिं सा का एक सामाहजक औि पारिवारिक व्यवस्था टू ट जाएगी।
चिम रूप िो सकता िै , औि अपिािीकिर् पीहडतों को कानूनी • केरल उच्च न्यायालय: 2021 में, केिल उच्च न्यायालय ने फैसला सुनाया
सुिक्षा प्रिान किता िै। हक वैवाहिक बलात्काि तलाक का आिाि िो सकता िै , भले िी इसके हलए
• कानूनी खाक्रमय ं क खत्म करना: वैवाहिक बलात्काि को िं ड न हिया जाए।
अपिाि घोहषत किके, र्िर्ध मानता िै हक बलात्काि एक • क्रनमेशभाई भरतभाई दे साई बनाम गुजरात राज्य (2017) मामला:
अपिाि िै , भले िी इसमें शाहमल पक्षों के बीच संबंि कुछ भी  न्यायालय ने किा हक "वैवाहिक बलात्काि को अपिाि बनाने से
िो। वैवाहिक बलात्काि को बढ़ावा िे ने वाली हवनाशकािी मनोवृहि िू ि िो
• स्वस्थ वैवाक्रिक संबंध ं क बढावा दे ना: वैवाहिक बलात्काि जाएगी"।

के क्तखलाफ स्पष्ट र्िर्ध आपसी सम्मान, हवश्वास औि संचाि पि • इं क्रिपेंिेंट थॉट बनाम यूक्रनयन ऑफ इं क्रिया (2017) मामला:
 इस मामले में उच्चतम न्यायालय ने 15 से 18 साल की नाबाहलग पत्नी
आिारित स्वस्थ संबंिों को बढ़ावा िे ते िैं ।
के साथ यौन संबंि को अपिाि किाि हिया िै ।
वैवाक्रिक बलात्कार क अपराध घ क्रषत करने के क्रवरुद् तका: • लेहकन, िािा 375 के अपवाि की जांच किते समय उच्चतम न्यायालय ने
• क्रववाि संस्था क कमज र करना: वैवाहिक बलात्काि को वयस्क महिलाओं के वै वाहिक बलात्काि के सवाल पि हवचाि किने से
अपिाि घोहषत किने से हववाि औि पािं परिक मूल् कमजोि इनकाि कि हिया।
िोंगे। इससे परिवाि टू ट सकते िैं औि समाज कमजोि िो सकता
िै ।'
• प्रवतान की कक्रठनाई: वैवाहिक रिश्ते के भीति गैि-सिमहत वाले यौन संबंि को साहबत किना जहटल िो सकता िै , औि कानूनी प्रहिया के
परिर्ामस्वरूप साक्ष्य की कमी या पिस्पि हविोिी गवािी जैसी कहठनाइयां िो सकती िैं ।
• पाररवाररक मूल् ं का क्षरण: यि हववाि औि पारिवारिक जीवन की पहवत्रता में िस्तक्षेप किके पािं परिक पारिवारिक मूल्ों के क्षिर् का
कािर् बन सकता िै ।
• झूठे आर प ं का अनुमान: आलोचकों का िावा िै हक वैवाहिक बलात्काि को अपिाि घोहषत किने से झूठे आिोपों का िास्ता खुल सकता
िै , जो बदला लेने, तलाक की कायावािी, या बच्चे की अक्रभरक्षा की लडाई में लाभ िाक्रसल करने जैसे उद्दे श् ं से प्रेररत ि ग ं े।
• प्रचक्रलत कानून ं का दु रुपय ग: वतामान कानूनों जैसे घिे लू हिं सा अहिहनयम 2005 का अक्सि लोगों को फंसाने के हलए िु रुपयोग हकया
जाता िै ।
 इस प्रकाि, यि मित्वपूर्ा िो जाता िै हक वैवाहिक बलात्काि
अंतरााष्ट््ीय प्राथक्रमकता:
को अपिाि घोहषत किते समय, इसके िु रुपयोग को िोकने • एमनेस्टी इं टिनेशनल के डे टा के अनुसाि, 185 में से 77 (42%) दे श
के हलए उहचत सुिक्षा उपाय िोने चाहिए। कानून के माध्यम से वैवाहिक बलात्काि को अपिाि मानते िैं ।
• क्रनजता और घुसपैठ: कुछ हविोहियों का तका िै हक वैवाहिक • घाना, भारत, इं ि नेक्रशया, जॉिा न, लेस थ , नाइजीररया, ओमान,
बलात्काि के क्तखलाफ र्िर्ध िाज्य द्वािा वैवाहिक जीवन के क्रसंगापुर, श्रीलंका और तंजाक्रनया जैसे िस िे श स्पष्ट रूप से हकसी
अंतिं ग औि हनजी पिलुओं में अनुहचत घुसपैठ को जन्म िे महिला या लडकी के पहत द्वािा वैवाहिक बलात्काि की अनुमहत िे ते िैं।
सकता िै । • वैवाहिक बलात्काि को अपिाि घोहषत किने वाला पिला िे श स क्रवयत
आगे की राि: संघ (1922) था औि यूके (1991) औि अमेररका (1993) ऐसा किने
वाले अंहतम पहिमी िे शों में से थे।
• राज्य का िस्तक्षेप: जब िाज्य ििे ज, िूिता, तलाक आहि जैसे
वैवाहिक मामलों में िस्तक्षेप किता िै , तो उसे बलात्काि जैसे जघन्य अपिाि में भी िस्तक्षेप किना चाहिए।

8
• जागरूकता अक्रभयान ं पर ध्यान: वैिाहनक सुिाि के साथ-साथ सिमहत, हचहकत्सा िे खभाल औि पुनवाा स पि जनता को जागरूक किना।
 उदािरण के क्रलए: एनजीओ शक्ति शाक्रलनी, वैवाहिक बलात्काि से पीयडत लोगों का समथान किती िै ।
• स्वाक्रमत्व की धारणा क संब क्रधत करना: पत्नी को पहत की हवशेष संपहि के रूप में िे खने को चुनौती िे ने के हलए आपिाहिक र्िर्ध से
वैवाहिक बलात्काि की छूट को समाप्त किना।
• बदलाव के क्रलए र्िर्ध: वैवाहिक बलात्काि को अपिाि घोहषत किने औि पीहडतों की सुिक्षा के हलए हविायी उपायों के माध्यम से बिलाव
की वकालत किना।
प्र. िम िे श में महिलाओं के क्तखलाफ यौन हिं सा के बढ़ते मामले
• अनुच्छेद 142 के तित असाधारण शक्तियां: यि सवोच्च न्यायालय
िे ख ििे िैं । इसके क्तखलाफ मौजूिा कानूनी प्राविानों के बावजूि
को असािािर् शक्तियां िे ता िै। इस प्रकाि, संसिीय र्िर्ध के हबना,
ऐसी घटनाओं की संख्या बढ़ ििी िै । इस खतिे से हनपटने के
अिालतें िािा 375 के अपवाि को पलट सकती िैं।
हलए कुछ नवीन उपाय सुझाएाँ । (2014)

1.3 क्रिक्रजटल व्यक्तिगत िे टा सं र क्षण (िीपीिीपी) क्रवधे य क 2022


संदभा:
िाल िी में केंद्रीय मंहत्रमंडल ने क्रिक्रजटल पसानल िे टा प्र टे क्शन (िीपीिीपी) क्रवधेयक क मंजूरी दे दी।
िे टा और िे टा सुरक्षा के बारे में:
• िे टा: डे टा कंप्यूटि-पठनीय प्रारूप में संग्रिीत जानकािी के संग्रि को
संिहभात किता िै , हजसमें सोशल मीहडया संिेश, ऑनलाइन आितें,
लेनिे न, मेहडकल रिकॉडा औि व्यक्तिगत हवविर् शाहमल िैं ।
• िे टा सुरक्षा: िे टा सुरक्षा का उद्दे श् व्यक्तिगत ग पनीयता अक्रधकार ं
और िे टा उपय ग के बीच संतुलन बनाते हुए व्यक्तिगत डे टा की सुिक्षा
किना िै ।
भारत में िे टा सुरक्षा:
• समक्रपात ढांचे का अभाव: भाित के पास डे टा सुिक्षा के हलए कोई हवहशष्ट
कानूनी ढांचा निीं िै ।
• मौजूदा अक्रधक्रनयम: कुछ मौजूिा अहिहनयम सीहमत डे टा सुिक्षा प्रिान
किते िैं
 सूचना प्रौद्य क्रगकी अक्रधक्रनयम 2000: िािा 43 ए उपयोगकताा डे टा
को कॉपोिे ट संस्थाओं द्वािा िु रुपयोग से बचाती िै लेहकन सिकािी
एजेंहसयों पि लागू निीं िोती िै । इसमें संवेिनशील व्यक्तिगत डे टा, जैसे
मेहडकल इहतिास औि बायोमेहटर क जानकािी शाहमल िै ।
 अन्य अक्रधक्रनयम: उपभ िा संरक्षण अक्रधक्रनयम 2015 और
कॉपीराइट अक्रधक्रनयम 1957 जैसे अहिहनयम भी व्यक्तिगत जानकािी के हलए कुछ सुिक्षा प्रिान किते िैं।
क्रिक्रजटल व्यक्तिगत िे टा संरक्षण (िीपीिीपी) क्रवधेयक 2022 की मुख्य क्रवशेषताएं :
• िे टा क्रप्रंक्रसपल और िे टा प्रत्ययी: यि हविेयक उस व्यक्ति को संिहभात किने के हलए "िे टा क्रप्रंक्रसपल" शब्द का उपय ग करता िै
क्रजसका िे टा एकत्र क्रकया जा रिा िै।
 िे टा प्रत्ययी: यि उस इकाई (जैसे क्रक एक व्यक्ति, कंपनी, फमा या राज्य) क संदक्रभात करता िै जो व्यक्ति के व्यक्तिगत डे टा को
प्रोसेस किने के उद्दे श्य औि लक्ष्य को हनिाा रित किता िै ।
 बच्च ं के क्रलए प्रावधान: उन्हें 18 वषा से कम आयु के उपयोगकताा ओं के रूप में परिभाहषत हकया गया िै , उनके माता-हपता या वैि
अहभभावकों को उनका िे टा क्रप्रंक्रसपल माना जाता िै।
• व्यक्तिगत िे टा और उसकी प्र सेक्रसंग क पररभाक्रषत करना:
 व्यक्तिगत िे टा क "कोई भी डे टा हजसके द्वािा या हजसके संबंि में हकसी व्यक्ति की पिचान की जा सकती िै" के रूप में परिभाहषत
हकया गया िै ।
 प्र सेक्रसंग से तात्पया संग्रि औि भंडािर् सहित व्यक्तिगत डे टा के संबंि में हकए गए संचालन के पूिे चि से िै ।

9
• सीमा पार िे टा प्रवाि: डे टा सुिक्षा औि सिकािी पहुं च के आिाि
पि, केंद्र उन क्षेत्रों को सूहचत किे गा जिां भाितीय डे टा स्थानां तरित
हकया जा सकता िै ।
• िे टा संरक्षण ब िा : यि हविेयक अनुपालन को लागू किने के हलए
डे टा संिक्षर् बोडा की स्थापना का प्रस्ताव किता िै । डे टा संिक्षर् बोडा
"हडजाइन से हडहजटल" िोगा।
• उपय गकताा अक्रधकार और ग पनीयता: उपयोगकतााओं को
व्यवसायों द्वािा िखे गए अपने व्यक्तिगत डे टा को सुिािने औि हमटाने
का अहिकाि िै ।
 बडे आकाि की कंपहनयों को अनुपालन का मूल्ांकन किने के
हलए एक िे टा सुरक्षा अक्रधकारी औि एक स्वतंत्र डे टा ऑहडटि
हनयुि किना िोगा।
 यि हविेयक बच्चों को नुकसान पहुं चाने वाले व्यक्तिगत डे टा के
प्रसंस्किर् औि 18 वषा से कम उम्र के बच्च ं पर लक्रक्षत
क्रवज्ञापन पर प्रक्रतबंध लगाता िै।
• छूट और राष्ट््ीय सुरक्षा: इस हविेयक में िाष्टरीय सुिक्षा कािर्ों से छूट
शाहमल िै , जो सिकाि को िाज्य एजेंहसयों को िाज्य की संप्रभुता,
अखंडता औि सुिक्षा के हित में प्राविानों का पालन किने से छूट िे ने
की अनुमहत िे ती िै ।
 उपयोगकतााओं की संख्या औि संसाहित डे टा की मात्रा के आिाि
पि कुछ व्यवसायों को भी छूट िी जा सकती िै । यि स्टाटा -अप
इकोहसस्टम की हचंताओं का समािान किता िै ।
• दं ि:
 उपय गकतााओं के क्रलए: हविेयक उन उपयोगकतााओं के हलए िं ड हनिाा रित किता िै जो साइन-अप के िौिान झूठे िस्तावेज जमा
किते िैं या तुच्छ हशकायत िजा किते िैं, हजसमें 10,000 रुपये तक का जुमाा ना िै ।
 इकाई के क्रलए: डे टा उल्लंघनों औि गैि-अनुपालन के हलए जुमाा ना 50 किोड रुपये से 500 किोड रुपये तक कि हिया गया िै ।

क्रवधेयक से जुडी क्रचंताएं :


आरटीआई की धारा 8(1)(जे ): यि व्यक्तिगत जानकािी को
• सूचना का अक्रधकार (आरटीआई) अक्रधक्रनयम का कमज र ि ना:
अस्वीकाि किने की अनुमहत िे कि हनजता की िक्षा किती िै यहि
 यि हविेयक िािा 8(1)(जे) में संशोिन किके इसके िायिे का हवस्ताि इसका सावाजहनक गहतहवहि या हित से कोई संबंि निीं िै या यहि
किने औि सभी व्यक्तिगत जानकारी क आरटीआई अक्रधक्रनयम इसके प्रकटीकिर् से गोपनीयता का अनुहचत उल्लंघन िोता िै ।
के दायरे से मुि करने का प्रयास करता िै।
 यि संशोिन गोपनीयता की िक्षा किने औि आिटीआई अहिहनयम के तित व्यक्तिगत जानकािी तक पहुं च से इनकाि किने के हलए
िािा 8(1)(जे) को लागू किने की क्षमता को समाप्त कि िे गा।
• क्रनजता के अक्रधकार का अपयााप्त संरक्षण: यि हविेयक हनयमों का मसौिा तैयाि किने में केंद्र सिकाि को व्यापक हववेकािीन शक्तियां
प्रिान किता िै , हजससे व्यक्तियों के हनजता के अहिकाि पि संभाहवत उल्लंघन के बािे में हचंताएं बढ़ जाती िैं।
 उिाििर् के हलए, धारा 18 के तित, यि केंद्र सिकाि को केवल एक अहिसूचना जािी किके हकसी भी सिकािी, या यिां तक हक हनजी
क्षेत्र की संस्थाओं को हविेयक के प्राविानों से छूट िे ने का अहिकाि िे ता िै ।
• िे टा संरक्षण ब िा के क्रलए स्वायत्तता का अभाव: बोडा की हनयुक्ति केंद्र सिकाि के अिीन िै , जो इसकी स्वतंत्रता औि संस्था पि सिकाि
के सीिे हनयंत्रर् पि सवाल उठाता िै ।
 सिकाि अन्य कानूनों के तित भी बोडा को काया सौंप सकती िै , हजससे इसकी स्वायिता पि औि असि पडे गा।
• इं टरनेट तक सीक्रमत पहुंच: इस हविेयक में हशकायत प्राक्तप्त औि समािान के हलए डे टा प्रोटे क्शन बोडा को "हडजाइन से हडहजटल" िोने
की आवश्यकता िै , लेहकन व्यापक आबािी तक सीहमत इं टिनेट पहुंच एक समस्या िै ।
 उिाििर् के हलए, आईटीयू के हवश्व िू िसंचाि/आईसीटी संकेतक डे टाबेस के अनुसाि, भाित में केवल 43 प्रहतशत आबािी इं टिनेट का
उपयोग किती िै ।

10
• जागरूकता की कमी: भाित में डे टा सुिक्षा सुहनहित किने में एक बडी चुनौती डे टा सुिक्षा के मित्व औि इससे जुडे जोक्तखम के बािे में
व्यक्तियों औि संगठनों के बीच जागरूकता की कमी िै ।
• कमज र अनुपालन प्रवतान: भाित में डे टा सुिक्षा के हलए मौजूिा कानूनी ढांचे में अनुपालन सुहनहित किने के हलए मजबूत तंत्र का अभाव
िै ।
िे टा सुरक्षा की चुनौक्रतयां :
िे टा सुरक्षा की आवश्कता:
• बुक्रनयादी ढांचे और क्रवक्रनयमन की कमी: महत्पूणन बुहनयािी ढांचे औि
• अथाव्यवस्था का बढता क्रिक्रजटलीकरण: स्माटा फ न अपनाने, हवहनयमन की कमी भाित में एमआईटी टे क्नोलॉजी रिव्यू साइबि हडफेंस
इं टरनेट कनेक्तिक्रवटी और सरकार की क्रिक्रजटल अथाव्यवस्था क इं डेक्स द्वािा पिचानी गई मित्वपूर्ा चुनौहतयां िैं ।
बढ़ावा इत्याहि ने भाित के हडहजटल परिदृश्य को बिल हिया िै । • समक्रपात मंत्रालय का अभाव: भाित में वतामान में साइबि सुिक्षा मुद्दों के
 आईटी उद्य ग की वृक्तद्: क्रवत्त वषा 2022 में भाित का आईटी समािान के हलए एक समहपात मंत्रालय का अभाव िै ।
उद्योग 15% से अहिक की सीएजीआि से बढ़कि $196 हबहलयन • अपयााप्त िे टा सुरक्षा ढांचा: भाित का डे टा सुिक्षा ढांचा, जो वतामान में
से बढ़कि $225 हबहलयन से अहिक िो गया। यहि यि हवकास िि 2000 के आईटी अहिहनयम द्वािा शाहसत िै , आिुहनक हडहजटल खतिों
जािी ििी तो 2027 तक आईटी उद्योग का मूल् 394 हबहलयन को संबोहित किने औि व्यक्तिगत डे टा गोपनीयता की पयााप्त सुिक्षा किने
के हलए अपयााप्त िै ।
डॉलि िोगा।
 उपय गकताा: इं टरनेट एं ि म बाइल एस क्रसएशन ऑफ • क्रवदे शी स्वाक्रमत्व वाले बु क्रनयादी ढांचे पर क्रनभारता: सोशल मीहडया औि
हविे शी िाडा वेयि पि हनभािता िे श को साइबि खतिों औि डे टा उल्लंघनों
इं क्रिया (IAMAI) के अनुसाि, 346 हमहलयन भाितीय ई-कॉमसा
के प्रहत उजागि किता िै ।
औि हडहजटल भुगतान जैसी ऑनलाइन सेवाओं का उपयोग किते
• कैंक्रिज एनाक्रलक्रटका: एक िाजनीहतक कंसक्तटंग फमा ने लाखों भाितीय
िैं । फेसबुक उपयोगकतााओं की सिमहत के हबना उनका व्यक्तिगत डे टा प्राप्त
• क्रनजता से समझौता: . प्रभावी डे टा सुिक्षा के हबना, व्यक्तियों की किके औि उसका िु रुपयोग किके डे टा घोटाला हकया।
सिमहत के हबना उनकी हनगिानी औि प्रोफाइहलंग बढ़ने का जोक्तखम  पेगासस स्पाइवेयर: 2019 में, इजिायली फमा एनएसओ ग्रुप ने भाित
िै । में पत्रकािों, कायाकतााओं औि सिकािी अहिकारियों की जासूसी किने
• आक्रथाक िाक्रन: आईबीएम का अनुमान िै हक भाित में 2021 में डे टा के हलए पेगासस स्पाइवेयि का इस्ते माल हकया।
उल्लंघन की औसत लागत USD 2.2 क्रमक्रलयन थी। • क्रिक्रजटल क्रनरक्षरता की उच्च दर: भाितीय आबािी के एक बडे हिस्से में
• साइबर अपराध: भाित में साइबि अपिािों की प्रकृहत में बिलाव हडहजटल साक्षिता का अभाव िै , हजससे हडहजटल िु हनया की जहटलताओं
िे खा जा ििा िै , जो अहिक संगहठत औि सियोगात्मक िो गए िैं । की समझ की कमी के कािर् उनका डे टा जोक्तखम में िै ।

 जामताडा और नूंि जैसे शििों में साइबि अपिाि केंद्रों में वृक्तद् • 2018 एएसईआि अध्ययन के अनुसाि, स्कूलों में कंप्यूटि तक पहुं च
सीहमत थी, केवल 21.3% स्कूली बच्च ं के पास ऐसी पहुंच थी।
हुई िै । ये शिि अक्सि हवहभन्न िाज्यों के हत्र-जंक्शन पि क्तस्थत िोते
िैं जो उन्हें एक म बाइल िाका स्पॉट बनाते िैं।
• डे टा स्थानीयकिर्: यि सुहनहित किता िै हक नागरिकों द्वािा उत्पन्न डे टा को िाष्टरीय संपहि माना जाए औि दे श के क्रित में सुरक्षा, ग पनीयता
और वाक्रणज्य की सुरक्षा के क्रलए राष्ट््ीय सीमाओं के भीतर संग्रिीत क्रकया जाए।
 हडहजटल िे टा संरक्षण क्रवधेयक, 2022, हपछले संस्किर् के सख्त डे टा स्थानीयकिर् आवश्यकताओं में छूट िे ता िै । यि पूवाहनिाा रित
आकलन के आिाि पि "हवश्वसनीय भूगोल" किे जाने वाले हवहशष्ट अंतिाा ष्टरीय स्थानों पि डे टा स्थानां तिर् की अनुमहत िे ता िै ।
आगे की राि:
• अक्रधकार ं और सुरक्षा क संतुक्रलत करना: डे टा संिक्षर् कानूनों को गोपनीयता औि सूचना के संवैिाहनक अहिकािों को संतुहलत किना
चाहिए।
• जैसे-जैसे भाितीय डे टा सुिक्षा हनयामक ढांचा हवकहसत िोगा, हजम्मेिाि डे टा प्रशासन के हलए कायाान्वयन, प्रवतान और अनुपालन
मित्वपूणा ि गा।
• क्षेत्राक्रधकार और वाह्य क्षेत्रीय पहुंच: भाितीय नागरिकों को सामान या सेवाएं प्रिान किने वाले या उनकी प्रोफाइहलंग किने वाले हविे शी
डे टा प्रोसेहसंग केंद्रों पि सख्त कायाा न्वयन औि क्षेत्राहिकाि।
प्र. बढ़ते साइबि अपिािों के कािर् हडहजटल िु हनया में
• िे टा सुरक्षा ब िा क अक्रधकार दे ना: उल्लंघनकताा संस्थाओं पि हविीय िं ड
डे टा सुिक्षा को काफी मित्व हमल गया िै । न्यायमूहता बी.
सहित जुमाा ना लगाया जा सकता िै । बाि-बाि अपिाि किने पि अहिक जुमाा ना
एन. श्रीकृष्ण सहमहत की रिपोटा डे टा सुिक्षा से संबंहित मुद्दों
लग सकता िै।
को संबोहित किती िै । आपके हवचाि में, साइबि स्पेस में
• जागरूकता और क्रशक्षा क बढावा दे ना: सावाजहनक जागरूकता, गोपनीयता व्यक्तिगत डे टा की सुिक्षा से संबंहित रिपोटा की ताकत औि
साक्षिता औि अहिकािों को बढ़ाने के हलए शैक्षहर्क संस्थानों के साथ पाटा निहशप कमजोरियां क्ा िैं ? (2018)
किना।

11
• अंतरााष्ट््ीय सिय ग क बढावा दे ना: वैहश्वक गोपनीयता सुिक्षा को मजबूत किने, वैहश्वक ढां चे के साथ गोपनीयता मानकों को संिेक्तखत किने,
डे टा टर ां सफि पि सियोग किने औि अंतििाष्टरीय गोपनीयता मंचों में सहिय रूप से भाग लेने के हलए अंतिाा ष्टरीय सियोग को बढ़ावा िे ना
बहुत जरूिी िै ।

1.4 ट् ां स जें ि र समु द ाय के क्रलए आरक्षण

संदभा:
मिािाष्टर सिकाि को हशक्षा औि सावाजहनक िोजगाि में ट् ांसजेंिर व्यक्तिय ं क अक्रतररि आरक्षण प्रदान करने में कक्रठनाइय ं का सामना
करना पड रिा िै।
ऊर्ध्ााधर और क्षैक्रतज आरक्षण:
ट् ांसजेंिर कौन िै? • हशक्षा औि िोजगाि में आिक्षर् को िो व्यापक श्रेहर्यों में हवभाहजत हकया
• टर ां सजेंडि उन लोगों के हलए एक व्यापक शब्द िै क्रजनकी जा सकता िै , अथाात ऊर्ध्ाािि औि क्षैहतज।
लैंक्रगक पिचान और अक्रभव्यक्ति पारं पररक रूप से जन्म के • ऊर्ध्ााधर आरक्षण: यि जाहत पिानुिम, एससी, एसटी औि ओबीसी पि
समय उन्हें हिए गए क्रलंग से जुडे मानदं ि ं और अपेक्षाओं के आिारित सामाहजक हवषमता औि हपछडे पन को संबोहित किता िै ।

अनुरूप निीं िै । • क्षैक्रतज आरक्षण: यि वंहचत समूिों के हलए सकािात्मक कािा वाई प्रिान
किने के हलए ऊर्ध्ाािि श्रेहर्यों में कटौती किता िै । उदािरण के क्रलए,
ट् ांसजेंिर समुदाय के सामने चुनौक्रतयां : हवकलांग व्यक्तियों को सभी ऊर्ध्ाािि श्रेहर्यों में क्षैहतज आिक्षर् की गािं टी
• भेदभाव और िाक्रशए पर जाना: टर ां सजेंडि व्यक्तियों को िी जाती िै ।
िोजगाि, हशक्षा औि आवास में भेिभाव का सामना किना पडता
िै , हजससे उनके अवसि औि संसािनों तक पहुंच सीहमत िो जाती नालसा बनाम भारत संघ (2014): उच्चतम न्यायालय ने टर ांसजेंडि
िै । व्यक्तियों को सामाहजक औि शैहक्षक रूप से हपछडे वगा के रूप में मान्यता
• पिचान के मुद्दे और कानूनी सुरक्षा का अभाव: हलंग-हवहशष्ट िी, अहिकािों औि सम्मान के साथ तीसरे क्रलंग के रूप में उनकी पुक्रष्ट् की।
पिचान िस्तावेज प्राप्त किना कहठन िै , हजससे मतिान के अहिकाि • यि सिकाि को ऊर्ध्ाािि या क्षैहतज हनहिा ष्ट हकए हबना हशक्षा औि
औि सामाहजक भागीिािी प्रभाहवत िोती िै। कानूनी क्तस्थहत के कािर् िोजगाि में "सभी प्रकार के आरक्षण" दे ने का क्रनदे श दे ता िै।
वे हिं सा औि उपेक्षा के प्रहत संवेिनशील िो जाते िैं ।
• सामाक्रजक बक्रिष्कार: टर ां सजेंडि, लैंहगक गैि अनुरूपता के ट् ांसजेंिर व्यक्ति (अक्रधकार ं का संरक्षण) अक्रधक्रनयम 2019:
कािर् हकसी चीज को हविासत या हकसी को गोि निीं ले सकते। • ट् ांसजेंिर व्यक्ति की पररभाषा: यि अहिहनयम एक टर ांसजेंडि व्यक्ति को
• बेर जगारी और आक्रथाक संघषा : उच्च बेिोजगािी, कायास्थल ऐसे व्यक्ति के रूप में परिभाहषत किता िै हजसका हलंग जन्म के समय हनहिा ष्ट
भेिभाव औि सावाजहनक सुहविाओं की कमी आहथाक कहठनाइयों हलंग से मेल निीं खाता िै।

का कािर् बनती िै ।  इसमें टर ांस-पुरुष औि टर ांस-महिलाएं , इं टिसेक्स हभन्नता वाले व्यक्ति, हलंग-
हवषमता वाले व्यक्ति औि हकन्नि एवं हिजडा जैसे सामाहजक-सांस्कृहतक
• उत्पीडन, क्रिंसा और कलंक: टर ां सजेंडि व्यक्तियों को सामाहजक
पिचान वाले व्यक्ति शाहमल िैं।
उपिास, कलंक औि भेिभाव का सामना किना पडता िै , हजससे
• भेदभाव के क्तखलाफ क्रनषेध: यि हशक्षा, िोजगाि, स्वास्थ्य िे खभाल,
मौक्तखक औि शािीरिक िु व्याविाि िोता िै औि िाहनकािक सावाजहनक सुहविाओं,संचाि, आवास, सावाजहनक या हनजी कायाालय औि
रूहढ़वाहिता को बढ़ावा हमलता िै । उनकी िे खभाल के हलए हजम्मेिाि प्रहतष्ठानों तक पहुंच सहित हवहभन्न क्षेत्रों में
• स्वास्थ्य दे खभाल और कल्ाण में बाधाएं : टर ां सजेंडि लोगों को टर ांसजेंडि व्यक्तियों के क्तखलाफ भेिभाव पि िोक लगाता िै।
हचहकत्सा पेशेविों से भेिभाव का सामना किना पडता िै , हजससे • क्रनवास का अक्रधकार: टर ांसजेंडि व्यक्तियों को अपने परिवाि के साथ ििने का
उनके शािीरिक औि मानहसक स्वास्थ्य से समझौता िोता िै । अहिकाि िै। यहि परिवाि िे खभाल प्रिान निीं कि सकता िै तो एक सक्षम
अिालत पुनवाास का आिे श िे सकती िै।
ट् ांसजेंिर समुदाय द्वारा क्षैक्रतज आरक्षण की मांग: • र जगार: हकसी भेिभाव की अनुमहत निीं; प्रत्येक प्रहतष्ठान को अहिहनयम से
• टर ां सजेंडि समुिाय िाहशए पि िोने वाले भेिभाव को िू ि किने औि संबंहित हशकायतों को संभालने के हलए एक हशकायत अहिकािी हनयुि किना
अपनी सामाहजक पिचान को बनाने के हलए आिक्षर् चािता िै। िोगा।
• नालसा के फैसले क अक्सि अन्य क्रपछडा वगा (ओबीसी) श्रेणी • ट् ांसजेंिर व्यक्ति के क्रलए पिचान प्रमाण पत्र: टर ांसजेंडि व्यक्ति 'टर ांसजें डि'
में ट् ांसजेंिर व्यक्तिय ं के क्रलए आरक्षण के क्रनदे श के रूप में के रूप में पिचान प्रमार् पत्र के हलए हजला महजस्टर े ट को आवेिन कि सकता
िै। हलंग परिवतान सजािी के बाि संशोहित प्रमार्पत्र संभव िो जाता िै।
दे खा जाता िै।
• ट् ांसजेंिर व्यक्तिय ं के क्रलए राष्ट््ीय पररषद (एनसीटी): इस अहिहनयम में
• इसको लेकि अब तक कोई मित्वपूर्ा कायाान्वयन प्रगहत निीं हुई एनसीटी का प्राविान िै जो केंद्र सिकाि को सलाि िे ने के साथ-साथ टर ांसजेंडि
िै । व्यक्तियों के संबंि में नीहतयों, कानून औि परियोजनाओं के प्रभाव की हनगिानी
• िहलत, बहुजन औि आहिवासी टर ां सजेंडि व्यक्तियों के क्रलए भी किता िै।
संभाक्रवत बराबरी के बारे में क्रचंताएं बढी ं िैं।
12
ट् ांसजेंिर ल ग ं के क्रलए आरक्षण में चुनौक्रतयां:
• नालसा के फैसले में अस्पष्ट्ता: चूंक्रक नालसा के फैसले में ट् ांस ल ग ं क 'सामाक्रजक और शैक्षक्रणक रूप से क्रपछडे वगा' के रूप में
मानने का आह्वान हकया गया था , हजससे उन्हें ओबीसी के साथ जोडने का जोक्तखम था।
• स्पष्ट्ता और कायाान्वयन की कमी: टर ां सजेंडि आिक्षर् पि केंद्र सिकाि की ओि से स्पष्ट हिशाहनिे शों औि नीहतयों की कमी प्रभावी
कायाा न्वयन में बािा डालती िै।
• जाक्रत और जनजातीय पिचान के साथ समझौता: हवहभन्न समुिायों जैसे हक दक्रलत, बहुजन और आक्रदवासी लोगों के टर ां सजेंडि को
जाहत औि हलंग-आिारित आिक्षर् के बीच चयन किना िोगा। यि समझौता उनके लाभों को कम कि सकता िै औि उन्हें िाहशए पि जाने
को मजबूि कि सकता िै ।
• सीक्रमत राजनीक्रतक इच्छाशक्ति: ट् ांसजेंिर व्यक्ति (अक्रधकार ं का संरक्षण) अक्रधक्रनयम, 2019, टर ां सजेंडि लोगों के हलए क्षैहतज आिक्षर्
का उल्लेख निीं किता िै , जो िाजनीहतक इच्छाशक्ति की कमी को िशााता िै ।

आगे की राि:
• नीक्रतगत सुधार: ट् ांसजेंिर-क्रवक्रशष्ट् आरक्षण नीक्रतयां बनाना एक िीघाकाहलक समािान िै । साथ िी, नीहत हनमाा र् में टर ांसजेंडि आवाजों
का प्रहतहनहित्व शाहमल किना जरूिी िै ।
• जागरूकता और संवेदनशीलता: टर ां सजेंडि लोगों औि उनके ट् ांसजेंिर व्यक्तिय ं के क्रलए पिल:
अहिकािों के हलए जागरूकता औि सिानुभूहत बढ़ाना, सिकािी • ट् ांसजेंिर व्यक्तिय ं के क्रलए राष्ट््ीय प टा ल: यि सामाहजक न्याय औि
अहिकारियों, नीहत हनमााताओं औि समाज को हशहक्षत किना। अहिकारिता मंत्रालय का एक पोटा ल िै ।
• िे टा संग्रि और अनुसंधान: साक्ष्य-आिारित नीहतयों औि  यि पोटा ल िे श में किीं से भी हडहजटल रूप से प्रमार् पत्र औि पिचान
पत्र के हलए आवेिन किने में टर ांसजेंडि समुिाय के व्यक्तियों की
िस्तक्षेपों को सूहचत किने के हलए, दक्रलत और आक्रदवासी
सिायता किता िै ।
समुदाय ं से संबंक्रधत ट् ांसजेंिर पर व्यापक अनुसंधान और
• गररमा गृि य जना:
िे टा संग्रि करना।
 उद्दे श्: आश्रय, भोजन, हचहकत्सा िे खभाल औि मनोिं जन सुहविाओं
• जबरदस्ती के उपाय ं क संब क्रधत करना: संगहठत हभक्षावृहि
जैसी बुहनयािी सुहविाओं के साथ आश्रय प्रिान किना।
को िं हडत किने के प्राविान पि हफि से हवचाि किना ताहक यि ✓ क्षमता-हनमाार्/कौशल हवकास के हलए सिायता प्रिान किना।
सुहनहित हकया जा सके हक टर ांसजेंडि व्यक्तियों, हजनके पास • ट् ांसजेंिर व्यक्तिय ं के क्रलए राष्ट्ीय पररषद: ट् ांसजेंिर व्यक्तिय ं

अक्सि व्यविाया आजीहवका हवकल्ों की कमी िोती िै , के साथ (अक्रधकार ं का संरक्षण) अक्रधक्रनयम, 2019 के तित प्रिि शक्तियों का
जबििस्ती न िो , औि वैकक्तल्क तिीकों की खोज िो। प्रयोग किते हुए केंद्र सिकाि ने टर ांसजेंडि व्यक्तियों के हलए एक िाष्टरीय
• स्वीकृक्रत: टर ां सजेंडि समुिाय को समाज के एक अहनवाया अंग परिषि का गठन हकया िै ।
के रूप में मान्यता िी जानी चाहिए, औि सामाहजक कलंक को • अध्यक्ष (पदे न): केंद्रीय सामाहजक न्याय औि अहिकारिता मंत्री।
समाप्त किने की आवश्यकता िै ।
लघु समाचार

1.5 क्रवधायक ं की अय ग्यता • अनुच्छेद 102(2) संहविान की दसवी ं अनुसूची को हकसी भी


संदभा: िाल िी में, िाष्टरवािी कां ग्रेस पाटी (एनसीपी) ने मिािाष्टर सिस्य को अयोग्य घोहषत किने का अहिकाि िे ता िै । िसवीं
सिकाि में शाहमल हुए नौ एनसीपी हविायकों के क्तखलाफ अयोग्यता अनुसूची को दलबदल क्रवर धी र्िर्ध के रूप में जाना जाता
याहचका िायि की। िै।
क्रवधायक ं क अय ग्य ठिराने के संवैधाक्रनक प्रावधान:  दसवी ं अनुसूची अंततः 1985 में संहविान के 52 वें संशोिन
• संहविान का अनुच्छेद 102 और अनुच्छेद 191 िमशः अहिहनयम के माध्यम से अक्तस्तत्व में आई।
सांसद ं और क्रवधायक ं की अय ग्यता से संबंक्रधत िै। दल-बदल क्रवर धी र्िर्ध की क्रवशेषताएं :
• अनुच्छेद 102(1) ऐसे उिाििर्ों का हवस्तृत वर्ान किता िै • दल-बदल के आधार पर अय ग्यता: हकसी िाजनीहतक िल से
जब ऐसी अयोग्यता की जा सकती िै संबंहित हविायक को अयोग्य ठििाया जाएगा यहि वि:
 यहि व्यक्ति सिकाि के अिीन कोई अघोहषत लाभ का पद  स्वेच्छा से अपनी पाटी की सिस्यता छोड िे ता िै ।
धारण करता िै।  अपने िाजनीहतक िल द्वािा जािी हनिे श के हवपिीत सिन में
 यहि सक्षम न्यायालय द्वारा उसे क्रवक्रक्षप्त घ क्रषत कि हिया मतिान किता िै /मतिान से अनुपक्तस्थत ििता िै ।
जाए।
 यहि वि अनुन्म क्रचत क्रदवाक्रलया िै आहि।
13
 पाटी द्वारा 15 क्रदन ं के भीतर मतिान या अनुपक्तस्थत ििने नागररक ि या क्रवदे शी या कंपनी या क्रनगम जैसा कानूनी
को माफ कि हिया जाता िै , तो उसे अयोग्य निीं ठििाया व्यक्ति ि । इसमें उस हिशा में तीन प्राविान शाहमल िैं:
जाता िै ।  क ई कायोत्तर र्िर्ध निी ं, क ई द िरा ख़तरा निी ं, क ई
 स्वतंत्र सदस्य सिन के हलए हनवाा हचत िोने के बाि हकसी आत्म-द षार पण (Self-Incrimination) निी ं।
िाजनीहतक िल में शाहमल िोते िैं तो उन्हें अयोग्य घोहषत • अनुच्छेद 20(3): इसमें किा गया िै हक "हकसी भी अपिाि के
कि हिया जाएगा। आिोपी व्यक्ति को अपने क्तखलाफ गवाि बनने के हलए मजबूि
 नाम-क्रनदे क्रशत सदस्य- यहि नाम-हनिे हशत िोने के छि निीं हकया जाएगा।"
मिीने बाि हकसी भी िाजनीहतक िल में शाहमल िोते िैं तो  इस अनुच्छेि का संिक्षर् केवल आपराक्रधक कायावािी
उन्हें अयोग्य घोहषत कि हिया जाएगा। तक िी सीक्रमत िै। क्रसक्रवल कायावािी के तित , कोई
• क्रवलय के मामल ं में छूट: व्यक्ति अनुच्छेि 20(3) के बचाव का उपयोग किके हकसी
 सिस्यों को छूट तब हमलती िै जब मूल िाजनीहतक िल का प्रश्न का उिि िे ने से इनकाि निीं कि सकता िै ।
कम से कम द क्रतिाई हकसी अन्य िाजनीहतक िल में
1.8 जनजातीय सलािकार पररषदें (टीएसी)
हवलय िो जाता िै।
• क्रनणाय लेने का प्राक्रधकार: हकसी सिस्य को सिन से अयोग्य संदभा - िाल िी में ओहडशा के मुख्यमंत्री नवीन पटनायक ने
घोहषत किने का क्रनणाय सदन के अध्यक्ष का ि ता िै। जनजाक्रत सलािकार पररषद की बैठक में जरूरतमंद एसटी-
एससी छात्रों को हडग्री कोसा किने के हलए 50 िजाि रुपये िे ने की
1.6 क्रनयम 176 बनाम क्रनयम 267 घोषर्ा की।
संदभा: संसद के मानसून सत्र का पिला क्रदन महर्पुि की क्तस्थहत जनजातीय सलािकार पररषद(टीएसी) के बारे में
पि चचाा के प्रारूप पर सिकाि औि हवपक्ष के बीच मतभेि के • उत्पक्रत्त: ये िाज्यों में अनुसूहचत जनजाहतयों के कल्ार् औि
कािर् बाहित हुआ। उन्नहत की िे खिे ख के हलए पांचवी ं अनुसूची के तित गक्रठत
क्रनयम 267 और क्रनयम 176 क्ा िै? संवैधाक्रनक क्रनकाय िैं।
• हनयम 267 क्रवक्रशष्ट् कामकाज क समाय क्रजत करने के क्रलए • भाित के संहविान की पांचवी ं अनुसूची के अनुच्छेद 244(1)
क्रनयम ं के क्रनलंबन से संबंक्रधत िै, जबहक हनयम 176 के तित , टीएसी की स्थापना अनुसूक्रचत क्षेत्र ं वाले प्रत्येक
सावाजक्रनक मित्व के तत्काल मामल ं पर अल्पकाक्रलक चचाा राज्य में की जाएगी औि, यहि िाष्टरपहत ऐसा हनिे श िे ते िैं , तो
की अनुमहत िे ता िै । ऐसे हकसी भी राज्य में भी स्थाक्रपत क्रकया जाएगा, जिां
• हनयम 267 िाज्यसभा सां सि को सभापहत की मंजूिी से सिन अनुसूक्रचत जनजाक्रतयां िैं, लेक्रकन अनुसूक्रचत क्षेत्र निी ं िैं।
के पूवा-हनिाा रित एजेंडे को हनलंहबत किने की हवशेष शक्ति िे ता • सदस्यता: इसमें 20 से अक्रधक सदस्य निी ं ि ग ं े, हजनमें से
िै । लगभग तीन-चौथाई िाज्य हविानसभा में अनुसूहचत
• अल्पावक्रध चचाा: यि संसिीय कायावािी के हलए उपयोग हकया जनजाक्रतय ं के प्रक्रतक्रनक्रध ि ग ं े , बशते क्रक यक्रद राज्य
जाने वाला एक उपकिर् िै , जो वषा 1953 से मौजूि िै । क्रवधानसभा में एसटी के प्रक्रतक्रनक्रधय ं की संख्या ऐसे
 अत्यावश्क सावाजक्रनक मित्व के मामल ं पि सिकाि प्रक्रतक्रनक्रधय ं द्वारा भरी जाने वाली टीएसी के सीट ं की संख्या
का ध्यान आकहषात किने औि उस पर औपचाररक प्रस्ताव से कम ि , शेष सीटें उन जनजाक्रतय ं के अन्य सदस्य ं द्वारा
या व ट के क्रबना छोटी अवहि की चचाा शुरू किने के हलए भरी जाएं गी।
उपलब्ध िै । • वतामान क्तस्थक्रत: आं ध्र प्रदे श, तेलंगाना, छत्तीसगढ, गुजरात,
क्रिमाचल प्रदे श, झारखंि, मध्य प्रदे श, मिाराष्ट््, ओक्रिशा
1.7 मौन रिने का अक्रधकार और राजस्थान जैसे अनुसूहचत क्षेत्रों वाले 10 (िस) िाज्यों में
संदभा: िाल िी में, उच्चतम न्यायालय(SC) ने किा हक सभी जनजातीय सलािकाि परिषि (टीएसी) का गठन हकया गया िै ।
आिोहपयों को 'मौन रिने का अक्रधकार' िै और जांचकताा उन्हें
आत्म-द षार पण (Self-Incrimination) के क्तखलाफ 1.9 पररसीमन अभ्यास
अक्रधकार के तित बोलने या अपिाि स्वीकाि किने के हलए मजबूि संदभा: उच्चतम न्यायालय (एससी) प्रथम दृष्टया भाित के चुनाव
निीं कि सकते। आयोग (ईसी) के कानूनी रुख से असिमत िै हक वि केंद्र से
संवैधाक्रनक प्रावधान: प्राहिकाि प्राप्त किने के बाि िी महर्पुि, असम, नागालैंड औि
• अनुच्छेि 20: यि हकसी आिोपी व्यक्ति को मनमानी और अरुर्ाचल प्रिे श में परिसीमन प्रहिया शुरू कि सकता िै ।
अत्यक्रधक सजा के क्तखलाफ सुिक्षा प्रिान किता िै , चािे वि

14
पृष्ठभूक्रम: पररसीमन क्ा िै?
• 2001 की जनगर्ना के आं कडों के बािे में हचंताओं के कािर् • परिसीमन जनसंख्या में परिवतान का प्रहतहनहित्व किने के हलए
हपछले परिसीमन (2002-08) में अरुणाचल प्रदे श, असम, लोकसभा औि िाज्य हविानसभा सीटों की सीमाओं को हफि से
मक्रणपुर और नागालैंि क बािर रखा गया था। हनिाा रित किने का काया िै ।
• चाि िाज्यों में आहिवासी समुिायों को हचंता थी हक परिसीमन पररसीमन आय ग
प्रहिया से उनकी सीट ं का अनुपात बदल जाएगा, हजससे • परिसीमन किने के हलए स्वतंत्र हनकाय, भाित के चुनाव आयोग
उनके चुनावी हित प्रभाहवत िोंगे। के सियोग से काम किता िै ।
प्रमुख क्रबंदु: • परिसीमन आयोग अहिहनयम के प्राविानों के तित इस आयोग
• भाित के मुख्य न्यायािीश ने जन प्रक्रतक्रनक्रधत्व अक्रधक्रनयम, को भाित के िाष्टरपहत द्वारा क्रनयुि क्रकया जाता िै।
1950 की धारा 8 ए का िवाला िे ते हुए किा हक चुनाव आयोग • संरचना: एक सेवाहनवृि उच्चतम न्यायालय न्यायािीश, भाित
को परिसीमन किने के हलए भाित सिकाि के प्राहिकिर् की के मुख्य चुनाव आयुि औि संबंहित िाज्य चुनाव आयुि।
आवश्यकता निीं िै । • संवैधाक्रनक प्रावधान:
जन प्रक्रतक्रनक्रधत्व अक्रधक्रनयम, 1950 की धारा 8 ए  अनुच्छेद 82 के तित, संसि को प्रत्येक जनगर्ना के बाि
• 4 राज्य ं में पररसीमन: यि अरुणाचल प्रदे श, असम, एक परिसीमन अहिहनयम बनाना िै । एक बाि अहिहनयम
मक्रणपुर या नागालैंि राज्य ं में लोकसभा औि हविानसभा लागू िो जाने के बाि, केंद्र सिकाि परिसीमन आयोग का
हनवााचन क्षेत्रों के परिसीमन से संबंहित िै । गठन किती िै ।
• चुनाव आय ग की भूक्रमका: धारा 8ए(2) में प्राविान िै हक  इस आयोग के आिे श अंहतम िोते िैं औि हकसी भी अिालत
जैसे िी िाष्टरपहत आिे श िद्द किते िैं , चुनाव आयोग को चाि के समक्ष इस पि सवाल निीं उठाया जा सकता क्ोंहक
िाज्यों में संसिीय औि हविानसभा हनवााचन क्षेत्रों को हनिाा रित इससे चुनाव अहनहित काल के हलए रुक जाएगा।
किने के हलए परिसीमन प्रहिया शुरू किनी िोगी।
िार्िका में सों र्क्षप्त समािार

राष्ट््गान: उच्चतम न्यायालय • श्रीनगि में एक कायाकािी महजस्टर े ट ने कहथत तौि पि िाष्टरगान के हलए निीं खडे िोने पि 11 लोगों को
के क्रदशाक्रनदे श हििासत में लेकि जेल भेज हिया िै ।
 राष्ट््ीय गौरव अपमान क्रनवारण अक्रधक्रनयम, 1971:
• राष्ट््ीय र्ध्ज, राष्ट््गान, भारतीय मानक्रचत्र जैसे िाष्टरीय प्रतीकों के अपमान या अनािि के साथ-साथ
भारत के संक्रवधान की अवमानना करने क दं क्रित करता िै ।

शिरी 20 (Urban 20 ) • िाल िी में, गांधीनगर में यि हशखि सम्मेलन आयोहजत हकया गया था।
मेयरल क्रशखर सम्मेलन • िाष्टरीय शििी मामलों के संस्थान (एनआईयूए), यू20 तकनीकी सहचवालय के सियोग से आवास और
शिरी मामल ं का मंत्रालय ने इसे आय क्रजत क्रकया।
• क्तस्थिता की हिशा में कायों पि हवचाि-हवमशा किने के हलए G20 दे श ं के शिर ं के नेताओं क एक
साथ एक मंच पि लाया गया।

राष्ट््ीय ई-गवनेंस सेवा क्रवतरण • प्रशासक्रनक सुधार और ल क क्रशकायत क्रवभाग (DAPRG) ने NeSDA ढांचे का तीसिा संस्किर्
मूल्ांकन (NeSDA) प टा ल लॉन्च हकया िै।
• उद्दे श्: नागरिक के दृहष्टकोर् से मौजूिा ई-गवनेंस सेवा क्रवतरण तंत्र की गििाई औि प्रभावशीलता
को मापना।
• एनईएसडीए ढां चा संयुि राष्ट्् ई-सरकारी सवेक्षण के ऑनलाइन सेवा सूचकांक (ओएसआई)
पर आधाररत िै।

ई-सारस म बाइल ऐप • िीनियाल अंत्योिय योजना-िाष्टरीय ग्रामीर् आजीहवका हमशन (DAY-NRLM) ने eSARAS मोबाइल
ऐप लॉन्च हकया औि e-SARAS पूहता केंद्र का उि् घाटन हकया।

15
• उद्दे श्: स्वयं सिायता समूिों की महिलाओं द्वािा बनाए गए उत्पािों के हवपर्न के हलए अहिक प्रभावी
मंच प्रिान किना।

पंचायत क्रवकास सूचकांक • केंद्रीय पंचायती राज राज्य मंत्री ने पंचायत हवकास सूचकां क पि िाष्टरीय कायाशाला में पंचायत हवकास
(पीिीआई) सूचकां क पि एक रिपोटा जािी की।
• आाँ कडे : पीडीआई के पास स्थानीयकृत एसडीजी के 9 हवषयों पि 144 स्थानीय लक्ष्य, 577 स्थानीय
संकेतक िैं ।
• नौ हवषयों में शाहमल िैं : गिीबी मुि औि बढ़ी हुई आजीहवका वाला गां व , स्वस्थ गांव, बच्चों के अनुकूल
गां व, पयााप्त पानी वाला गां व
• स्वच्छ औि ििा-भिा गांव, आत्महनभाि बुहनयािी ढांचे वाला गां व, सामाहजक रूप से न्यायसंगत औि
सामाहजक रूप से सुिहक्षत गां व, सुशासन वाला गांव, महिला हमत्रवत गां व।

क्रिक्रजटल टाइम वाउचर • भाित का चुनाव आयोग िाजनीहतक िलों के हलए िू ििशान औि आकाशवार्ी पि प्रसािर् समय का
लाभ उठाने के हलए हडहजटल टाइम वाउचि (डीटीवी) पेश किता िै ।
• डीटीवी एक हडहजटल टोकन िै जो िू ििशान या ऑल इं हडया िे हडयो पि एक हनहित मात्रा में प्रसािर्
समय का प्रहतहनहित्व किता िै ।

16
2. अं त राा ष्ट्् ीय सं ब ं ध
2.1 शं घ ाई सिय ग सं ग ठन (SCO) के राष्ट्् ाध्यक्ष ं की पररषद का 23वां क्रशखर सम्मे ल न
संदभा:
िाल िी में प्रिानमंत्री ने शंघाई सिय ग संगठन (SCO) के राष्ट््ाध्यक्ष ं की पररषद के 23वें क्रशखर सम्मेलन क वीहडयो कॉन्फ्रेंहसंग के जरिए
संबोहित हकया।
• बैठक के िौिान ईरान आक्रधकाररक तौर पर शंघाई सिय ग
संगठन (एससीओ) का पूणा सदस्य बन गया िै ।

प्रधानमंत्री के संब धन की मुख्य बातें:


• चाबिार बंदरगाि: प्रिान मंत्री ने एससीओ समूि में ईिान के
शाहमल िोने के बाि बढ़ी हुई व्यापाि गहतहवहियों के हलए चाबिाि
बंििगाि के उपयोग को बढ़ाने का सुझाव हिया।

चाबिाि बंििगाि ओमान की खाडी के तट पि िहक्षर् पूवी ईिान


में क्तस्थत एक िर्नीहतक रूप से मित्वपूर्ा बंििगाि िै ।

• आतंकवाद पर नई क्रदल्ली घ षणा: बैठक के अंत में सिस्य िे शों


द्वािा आतंकवाि पि नई हिल्ली घोषर्ा पि िस्ताक्षि हकए गए,
हजसमें किा गया हक अंतिााष्टरीय समुिाय को "आतंकवािी,
अलगाववािी औि चिमपंथी समूिों की गहतहवहियों का मुकाबला
किने के हलए” एक साथ आना चाहिए।
• चीन-पाक्रकस्तान आक्रथाक गक्रलयारा (CPSE): प्रिान मंत्री ने यि
भी िे खां हकत हकया हक कनेक्तिहवटी परियोजनाओं को हियाक्तन्वत
किते समय, एससीओ के सिस्य िे शों की "संप्रभुता औि क्षेत्रीय
अखंडता का सम्मान किना" आवश्यक िै ।
• आतंक क्रवत्तप षण: आतंक हविपोषर् से हनपटने के हलए
एससीओ के आिएटीएस (क्षेत्रीय आतंकवाि हविोिी संिचना) तंत्र
के माध्यम से आपसी सियोग बढ़ाने की आवश्यकता िै ।
• अफगाक्रनस्तान: भाित अफगाहनस्तान के संबंि में अपनी
हचंताओं औि अपेक्षाओं को अहिकां श एससीओ िे शों के साथ
साझा किता िै ।

भारत की अध्यक्षता में एससीओ:


• शंघाई सियोग संगठन की भाित की अध्यक्षता का क्रवषय
SECURE िै क्रजसका अथा िै -
S: सुिक्षा(Security)
E: आहथाक हवकास(Economic Development)
C: कनेक्तिहवटी(Connectivity)
U: एकता(Unity)
R: संप्रभुता औि क्षेत्रीय अखंडता के हलए सम्मान (Respect for Sovereignty and Territorial Integrity)
E: पयाा विर् संिक्षर् (Environmental Protection)
भारत क एससीओ सदस्यता का लाभ:
• बहु-आयामी जुडाव: एससीओ भारत क रूस के साथ संबंिों को गििा किने के हलए एक अनूठा मंच िे ता िै ; चीन और पाक्रकस्तान के
प्रभाव की हनगिानी किना औि उसका मुकाबला किना औि मध्य एक्रशयाई गणराज्य ं (सीएआर) के साथ आहथाक औि सांस्कृहतक
सियोग का हवस्ताि किना।

17
• कनेक्तिक्रवटी: बडे पैमाने पि बुहनयािी ढांचे के हवकास के माध्यम से मध्य एक्रशया क दक्रक्षण और दक्रक्षण पूवा एक्रशया के साथ एकीकृत
किने से इस क्षेत्र के हलए कई वषों तक व्यापक आहथाक हवकास सुहनहित िो सकता िै।
• ऊजाा संसाधन: दु क्रनया की लगभग 10% ऊजाा औि तेल सीएआि से आता िै । सबसे अहिक ऊजाा -हनभाि िे शों में से एक िोने के नाते,
भाित के पास एससीओ में शाक्रमल ि कर क्षेत्रीय कूटनीक्रत के माध्यम से अपनी ऊजाा जरूरत ं क पूरा करने का अवसर िै।
• आतंकवाद प्रत्युत्तर: भाित एससीओ के क्षेत्रीय आतंकवाद-र धी ढांचे (RATS) के माध्यम से खुक्रफया जानकारी साझा करने, सवोत्तम
प्रथाओं और प्रौद्य क्रगक्रकय ं क क्रवकक्रसत करने, प्रत्यपाण व्यवस्था आक्रद की क्रदशा में काम करके अपने आतंकवाद-क्रवर धी अनुभव
क बेितर बना सकता िै।
• क्रववादास्पद मुद्दे पर बातचीत: एससीओ चाटा ि हकसी भी हद्वपक्षीय हववाि को उठाने की अनुमहत निीं िे ता िै , हजससे भाित-पाहकस्तान के
बीच वषों से रुकी हुई बातचीत की सुहविा हमल सकती िै ।
• चीन क मात दे ना: एससीओ का पूर्ा सिस्य िोने के नाते, भाित चीन की मित्वाकांक्षी पिल "वन बेल्ट, वन र ि" औि "चाइना
पाक्रकस्तान इक नॉक्रमक कॉररि र (सीपीईसी)" से प्रभावी ढं ग से क्रनपटने के क्रलए अपनी नीक्रत की य जना बनाने की क्तस्थक्रत में
ि गा।
• क्रवक्रभन्न क्षेत्रीय मुद्द ं पर बातचीत के क्रलए स्थान: यि यूिेहशया क्षेत्र में िो ििी घटनाओं के संिभा में मित्वपूर्ा स्थान िखता िै जैसे यूिेन में
रूस का सैन्य अहभयान, अफगाहनस्तान में ताहलबान का अहिग्रिर्।
• अवसर की क्तखडकी: एससीओ को "अवसि की क्तखडकी" औि एक ऐसे संगठन के रूप में िे खा जाता िै जो िो बडी शक्तियों - रूस औि
चीन - को एक साथ लाता िै औि क्षेत्र में सुिक्षा औि संस्कृहत के क्षेत्रों में घहनष्ठ सियोग के हलए क्तस्थहत हनिाा रित किता िै ।

भारत के सामने एससीओ से संबंक्रधत चुनौक्रतयां : एससीओ बनाम नाट :


• पाक्रकस्तान का उलझाव: एससीओ में पाहकस्तान की मौजूिगी • आपसी सिय ग के क्रवचार पर आधाररत: एससीओ की स्थापना
औि चीन का प्रभुत्व, भाित को संगठन में एक गौर् भूहमका तक एक नए प्रकाि के अंतिाा ष्टरीय संगठन को बनाने के हलए की गई थी,
सीहमत कि िे ता िै। जो शून्य-संचय सोच औि आहिपत्य पि आिारित निीं था, बक्ति
• पक्रिम क्रवर धी चररत्र: एससीओ को मध्य एहशया औि यूिेहशया में नाटो के हवपिीत हवहविता के हलए सियोग औि सम्मान पि
अमेरिका औि पहिमी प्रभाव के प्रहतकाि के रूप में िे खा जाता िै । आिारित था।
पहिम के साथ अपनी बढ़ती हनकटता के कािर् भाित को इस • सवासम्मक्रत आधाररत क्रनणाय: सिस्यों को सुिक्षा के हलए अपनी
समूि के साथ काम किने में संघषा किना पड सकता िै । संप्रभुता छोडने की आवश्यकता निीं िै। सिस्य एक-िू सिे के
• सीएआर से कनेक्तिक्रवटी: भाित का ध्यान अंतिाा ष्टरीय उिि- आं तरिक मामलों में िस्तक्षेप निीं किते। हनर्ाय बहुमत के बजाय
िहक्षर् परिविन गहलयािे (आईएनएसटीसी), अश्गाबात समझौते, आम सिमहत से िी हलए जाते िैं, जैसा हक नाटो के मामले में िै ।
चाबिाि बंििगाि पि िै जबहक बाकी सिस्यों ने चीन की बेट औि • क ई सामूक्रिक रक्षा तंत्र निी ं: एससीओ के भीति सामूहिक िक्षा
िोड पिल को अपनाया िै । जैसी कोई चीज निीं िै , जो नाटो से एक बु हनयािी अंति िै , जैसा हक
• आतंकवाद का मुकाबला: भाित के साथ पाहकस्तान के छद्म युद् इसके प्रहसद् अनुच्छेि 5 के तित शाहमल िै ।
ने संगठन को आतंकवाि से लडने से िोक हिया िै । एससीओ द्वािा • रक्षात्मक और आिामक कारा वाई: नाटो शीत युद् की िे न िै ।
भाित को अफगाहनस्तान शांहत प्रहिया से भी बािि िखा गया िै। इसका एक काल्हनक शत्रु िै : सोहवयत संघ के नेतृत्व वाला
• न्यूनतम भागीदारी: चीन की तुलना में इस क्षेत्र में भाित का क्षेत्रीय समाजवािी खेमा। अक्तस्तत्व में आने के बाि से, नाटो ने बाि-बाि
प्रभाव सीहमत िै । 2017 में, भारत ने मध्य एक्रशया के साथ $2 आिामक कािा वाई की िै , हजसका अथा यि िै हक वि िक्षात्मक
क्रबक्रलयन और रूस के साथ $10 क्रबक्रलयन का व्यापार क्रकया, संगठन निीं िै हजसका वि िावा किता िै ।
जबक्रक चीन ने $50 क्रबक्रलयन और $100 क्रबक्रलयन का व्यापार • एससीओ ने हकसी भी िे श के क्तखलाफ कोई आिामक कािा वाई निीं
क्रकया। की िै ।
• एससीओ के सदस्य दे श ं के बीच संघषा: भाित-चीन औि
हकहगास्तान औि ताहजहकस्तान सीमा हववाि साझा किते िैं जो एससीओ सिस्यों के बीच सियोग में बािा डालते िैं।
• क्रववाद समाधान तंत्र का अभाव: एससीओ में सिस्य िे शों के बीच हववािों को सुलझाने के हलए एक औपचारिक तंत्र का अभाव िै ।
• धन की कमी: मध्य एहशयाई गर्िाज्यों (सीएआि) की हवहभन्न आहथाक परियोजनाओं, सांस्कृहतक मतभेिों औि घिे लू चु नौहतयों के हलए
पयाा प्त िन की कमी।
• भू-राजनीक्रतक चुनौती: भाित के एससीओ में शाहमल िोने पि उसे चीन औि रूस के बाि कम मित्व की भूहमका हनभाने की चुनौती का
सामना किना पडे गा।

18
आगे की राि:
• क्रसद्ांत ं पर आधाररत सिय ग क बढावा दे ना: एससीओ के भीति सियोग को औि बढ़ाने के हलए, भाित को सावाभौहमक रूप से
मान्यता प्राप्त अंतिाा ष्टरीय मानिं डों, सुशासन, र्िर्ध के शासन, खुलेपन, पाििहशाता औि समानता के मित्व पि जोि िे ना चाहिए।
• चाटा र क्रसद्ांत ं का पालन: भाित को एससीओ के चाटा ि में उक्तल्लक्तखत हसद्ां तों का सख्ती से पालन किने की वकालत किनी चाहिए।
• एससीओ क्रशखर सम्मेलन की भूक्रमका क बढाना: भाित को संबंहित संस्थाओं पि परिषि द्वािा लगाए गए प्रहतबंिों को लागू किने में
केंद्रीय औि समन्वयकािी भूहमका हनभाने के हलए एससीओ हशखि सम्मेलन पि जोि िे ना चाहिए।
• काया समूि ं की स्थापना: इसमें शाहमल सभी संस्थाओं की हचंताओं को िू ि किने के हलए, भाित को एससीओ ढांचे के भीति समहपात काया
समूिों की स्थापना का प्रस्ताव िे ना चाहिए।
• अक्रधक क्रवत्तीय संसाधन ं का आवंटन: एससीओ के बजट को बढ़ाने, एक स्थायी एससीओ हवश्लेषर्ात्मक केंद्र स्थाहपत किने की तत्काल
आवश्यकता िै जो सामान्य समस्याओं का हवश्लेषर् किे गा, अगले 10 वषों के हलए िर्नीहत हवकहसत किे गा औि हवश्ले षर्ात्मक कायों पि
नजि िखने के हलए एक समन्वय हनकाय बनाएगा।
प्र. एससीओ के सामने आने वाली प्रमुख चुनौहतयों पि चचाा कीहजए
• व्यापार क्षेत्र की स्थापना: हनर्ायों के प्रभावी कायाा न्वयन के हलए एक
औि क्षेत्रीय क्तस्थिता औि आहथाक सियोग को बढ़ावा िे ने में इसकी
तंत्र का हनमाार् औि एक तिजीिी व्यापाि क्षेत्र की स्थापना।
प्रभावशीलता बढ़ाने के उपाय सुझाएं ।
• संघषा और प्रक्रतस्पधाा का प्रबंधन: एससीओ की सफलता सिस्य िे शों,
हवशेष रूप से रूस औि चीन औि अन्य क्षेत्रीय औि अहतरिि-क्षेत्रीय हिग्गजों के प्रहतस्पिी हितों के प्रबंिन पि हनभाि किती िै ।

2.2 भारत-श्रीलं क ा
संदभा:
िाल िी में श्रीलंका के िाष्टरपहत राक्रनल क्रविमक्रसंघे आहिकारिक िौिे पि हिल्ली आये।
भारत-श्रीलंका संबंध ं के प्रमुख स्तंभ:
• ऐक्रतिाक्रसक संबंध: भाित औि श्रीलंका के बीच प्राचीन काल से िी सांस्कृक्रतक, धाक्रमाक और व्यापाररक संबंध ं का एक लंबा इक्रतिास
रिा िै।
 िोनों िे शों के बीच मजबूत सांस्कृहतक संबंि िैं , कई श्रीलंकाई लोग अपनी हविासत भाित से जोडते िैं । बौद् िमा, हजसकी उत्पहि भाित
में हुई, श्रीलंका में भी एक मित्वपूर्ा िमा िै।
• आक्रथाक सिय ग: अमेरिका औि हिटे न के बाि भाित श्रीलंका का तीसरा सबसे बडा क्रनयाात गंतव्य िै। श्रीलंका के 60% से अहिक हनयाा त
भाित-श्रीलंका मुि व्यापाि समझौते का लाभ उठाते िैं ।
 िु हनया के साथ श्रीलंका के कुल व्यापाि में 16 प्रहतशत हिस्सेिािी के साथ भाित श्रीलंका का सबसे बडा व्यापारिक भागीिाि िै ।
 िाहलया आहथाक संकट के िौिान, भाित ने आवश्यक वस्तुओं की गंभीि कमी के बीच िाित प्रिान किते हुए, लगभग 4 क्रबक्रलयन िॉलर
की आपातकालीन क्रवत्तीय सिायता प्रदान की।
• क्रवकास सिय ग: श्रीलंका में भाित की प्रमुख परियोजना, भारतीय आवास पररय जना, युद्ग्रस्त औि वृक्षािोपर् क्षेत्रों में 50,000 घि
बनाएगी।
• संचार के समुद्री मागों (sloc) क सुरक्रक्षत करना: हिं ि मिासागि से गुजिने वाले एसएलओसी को सुिहक्षत किना, उच्च समुद्र ं में समुद्री
सुरक्षा और क्रनयम आधाररत व्यवस्था क बढाने में केंद्रीय मित्व का िै।
• क्षेत्रीय सिय ग: िोनों िे श दक्रक्षण एक्रशयाई क्षेत्रीय सिय ग संगठन (साका) के सदस्य िैं औि िहक्षर् एहशया में आहथाक औि सामाहजक
क्तस्थहतयों में सुिाि लाने के उद्दे श्य से क्षेत्रीय पिल में सहिय रूप से शाहमल िैं ।
• पयाटन: 2015 में, भाित सिकाि ने श्रीलंकाई ई-टू ररस्ट वीजा (eTV) य जना शुरू की।
 2016 में, भाित औि श्रीलंका ने एक ओपन स्काई समझौते पि िस्ताक्षि हकए, हजससे श्रीलंकाई एयिलाइं स को भाित के हलए असीहमत
उडान भिने की अनुमहत हमल गई।
• मानव संसाधन क्रवकास: श्रीलंकाई छात्रों को प्रहतवषा छात्रवृहि हमलती िै । "स्टिी इन इं क्रिया" कायािम के तित आयुवेि, योग औि बौद्
अध्ययन पाठ्यिम पेश हकए जाते िैं ।

भारत-श्रीलंका संबंध ं में चुनौक्रतयां :


• आतंकवाद: हलबिे शन टाइगसा ऑफ तहमल ईलम (LTTE) उििी श्रीलंका में एक उग्रवािी समूि था जो उिि औि पूवा में जातीय तहमल
अल्संख्यकों के हलए एक स्वतंत्र िाज्य बनाने की मांग कि ििा था।

19
• िाजीव गां िी की ित्या के बाि भाित-श्रीलंका संबंि खिाब िो गए।
• तक्रमल जातीय मुद्दे: भाित को श्रीलंका के तहमल अल्संख्यकों की
हचंता िै , हजनका तहमलनाडु से सां स्कृहतक औि भाषाई संबंि िै। संघषा
के िौिान तहमलों के साथ हकया गया व्यविाि हववाि का हवषय िै ।
• समुद्री सीमा क्रववाद: पाक जलडमरूमध्य में एक छोटा-सा हनजान
द्वीप कच्चाथीवू पि िोनों पक्ष िावा किते िैं , वतामान में श्रीलंका
हववाहित क्षेत्र की िे खिे ख किता िै ।
• मत्स्य पालन क्रववाद: भाित औि श्रीलंका के बीच पाक खाडी में मछली
पकडने के अहिकाि को लेकि लंबे समय से हववाि चल ििा िै।
• चीन का बढता प्रभाव: क्रनवेश और बुक्रनयादी ढांचा पररय जनाओं
के माध्यम से श्रीलंका में चीन का बढ़ता प्रभाव भाित के हलए
हचंताजनक िै ।
 उदािरण के क्रलए: चीन ने श्रीलंका की 2021 जीडीपी का 18%
हनवेश हकया औि उसके हविे शी ऋर् का 10.8% चीन पि बकाया
िै ।
• राजनीक्रतक अक्तस्थरता: श्रीलंका में िाजनीहतक अक्तस्थिता का भी
संबंिों पि प्रभाव पडा िै , सिकाि में बिलाव के कािर् हविे श नीहत में
बिलाव आया िै ।
• श्रीलंका के संक्रवधान के 13वें संश धन के कायाान्वयन क भारत
का समथान: गृियुद् के बाि, श्रीलंकाई प्रांतों, हवशेष रूप से उििी औि पूवी क्षेत्रों ने सिा िस्तांतिर् पि चचाा की िै। भाित एक ऐसा
िाजनीहतक समािान चािता िै जो तहमल अल्संख्यकों की जरूितों को पूिा किे ।

आगे की राि:
• तक्रमलनािु की राजनीक्रत से अलगाव: भाित की नीहत तहमलनाडु की गठबंिन िाजनीहत से तय निीं िोनी चाहिए, बक्ति उसे श्रीलंका के
तहमल समुिाय के कल्ार् के हलए अपने लक्ष्य स्पष्ट रूप से बताने चाहिए।
• चीन के प्रभाव का प्रबंधन: गलत संचाि औि अहवश्वास को िोकने के हलए चीन सहित अन्य िे शों के साथ अपने संबंिों के बािे में श्रीलंका
के साथ िचनात्मक बातचीत।
 एकल भागीिाि पि िे श की हनभािता को कम किने के हलए भारत-श्रीलंका समझौता और श्रीलंका के संक्रवधान में 13वां संश धन:
श्रीलंका को बुहनयािी ढां चे के हवकास औि हनवेश में लागत • इस पि कोलंबो में प्रधानमंत्री राजीव गांधी और राष्ट््पक्रत जेआर
जयवधाने के बीच िस्ताक्षर क्रकया गया।
प्रभावी हवकल् प्रिान किना।
• इसमें तहमल बहुल उििी औि पूवी श्रीलंका को कुछ शक्तियां िे ने के हलए
• श्रीलंका के साथ गैर-पारस्पररक जुडाव: भाित को श्रीलंका
संहविान में संशोिन किने की मांग की गई, हजससे गृियुद् संवैिाहनक
के साथ गैि-पािस्परिक जुडाव, उसकी समस्याओं को समझना रूप से समाप्त िो सके।
औि उसकी मिि किनी चाहिए लेहकन उसके द्वािा इसे कभी • उििी (जाफना प्रायद्वीप) औि पूवी (हत्रंकोमाली) प्रांतों को अस्थायी रूप से
भी ििे में निीं हलया जाना चाहिए। भाित को इस बात पि जोि हवलय हकया जाना था, हजसके बाि यि तय किने के हलए जनमत संग्रि
िे ना चाहिए हक कोलंबो की नीहत घोषर्ा भाित की हचंताओं औि किाया गया हक क्ा उन्हें एक साथ ििना चाहिए या हवभाहजत िोना चाहिए।
हितों को संबोहित किे ।
• क्रववादास्पद मत्स्य पालन मुद्दे का समाधान: िोनों िे शों को हववािों को सुलझाने औि हटकाऊ मछली पकडने की प्रथाओं को बढ़ावा िे ने
के हलए तंत्र स्थाहपत किना चाहिए।
• बहुआयामी सिय ग: िोनों िे शों को समुद्री डकैती, समुद्री आतंकवाि से हनपटने औि हिंि मिासागि क्षेत्र में समुद्री मागों की सुिक्षा सुहनहित
किने में सियोग को प्राथहमकता िे नी चाहिए।
प्र. ‘भाित श्रीलंका का सहियों पुिाना हमत्र िै ।' कथन के आलोक में
• सतत जुडाव और सिय ग: संबंिों में आगे बढ़ने के िास्ते में अपने लोगों
श्रीलंका के िाहलया संकट में भाित की भूहमका पि चचाा कीहजए।
की भलाई औि क्षेत्र की क्तस्थिता को बढ़ाने के हलए आपसी सम्मान औि
(2022)
समझ पि आिारित हनिं ति जुडाव औि सियोग शाहमल िै ।

20
2.3 भारतीय प्रवासी
संदभा:
िाल िी में प्रिानमंत्री ने हमस्र में भाितीय समुिाय के साथ बातचीत की।
प्रवासी भारतीय ं के बारे में :
• ‘डायस्पोिा’ शब्द की उत्पहि ग्रीक डायस्पेइिो से हुई िै , हजसका • एनआरआई: -ऐसे भाितीय नागरिक जो हकसी भी उद्दे श्य से अहनहित
अथा िै फैलाव। काल के हलए हविे श में िि ििे िैं । हवशेष रूप से खाडी िे शों में ऐसे नागरिक
• भाितीय प्रवासी से तात्पया उन लोगों से िै हजनकी उत्पहि भाित काफी संख्या में पाए जाते िैं ।
से मानी जाती िै या जो हविे श में ििने वाले भाितीय नागरिक • पीआईओ:- प्रवासी भाितीय जो अपने ििने वाले िे श के नागरिक बन गए
िैं । िैं ।
• इसमें अक्रनवासी भारतीय (एनआरआई),भारतीय मूल के • एसपीआईओ:- भाितीय मूल का िाज्यहविीन व्यक्ति। उनके पास अपने
व्यक्ति (पीआईओ), औि भारतीय मूल के राज्यक्रविीन भाितीय मूल को प्रमाहर्त किने के हलए कोई िस्तावेज निीं िै । ऐसे
व्यक्तियों की सबसे ज्यािा संख्या म्यांमाि औि श्रीलंका में िै ।
व्यक्ति (एसपीआईओ) शाक्रमल िैं।

भारतीय प्रवासी समुदाय के गठन का कारण:


 दासता उन्मूलन का प्रभाव: िमशः 1834, 1846 औि 1873 में हिहटश, रां सीसी औि डच उपहनवेशों में िासता के उन्मूलन ने उनके
उपहनवेशों की बागान अथाव्यवस्थाओं में श्रम की अत्यहिक कमी पैिा कि िी।
 क्रगरक्रमक्रटया व्यवस्था: 'हगिहमहटया' व्यवस्था ने बडी संख्या में भाितीयों को हगिहमहटया मजिू िों के रूप में भती हकया औि गन्ने के बागानों
औि कृहष संपिा पि काम किने के हलए कैिे हबयन, अरीका औि प्रशां त क्षेत्र के हिहटश उपहनवेशों में ले जाया गया।
 व्यापार और अन्वेषण: सोने, चांिी औि मसालों जैसी कीमती िातुओं के व्यापाि के कािर् भाितीय शुरू में िहक्षर् पूवा एहशया, मध्य
एहशया औि अरीका के िे शों औि यूिोप के कुछ हिस्सों में चले गए।
 उच्च क्रशक्षा गंतव्य: पहिमी िे श भी उच्च हशक्षा के हलए एक मित्वपूर्ा गंतव्य के रूप में उभिे िैं ।
✓ उदािरण के क्रलए: अमेरिका में 1965 के आप्रवासन अहिहनयम औि 1967 में कनाडा में लागू हनयमों ने इन िोनों िे शों में बडी
संख्या में पेशेविों के बसने का मागा प्रशस्त हकया।
 आक्रथाक क्रवकास और तकनीकी क्रशक्षा: तीव्र आहथाक हवकास औि हवशेष रूप से आईटी में तकनीकी हशक्षा के हवस्ताि ने हवकहसत
िे शों में प्रवासन को बढ़ावा हिया िै ।

भारतीय प्रवासी समुदाय का मित्व: मुख्य तथ्य: भारतीय प्रवासी

• राजनीक्रतक प्रभाव: भाितीय प्रवासी भी अपनी • संयुि िाष्टर के आहथाक औि सामाहजक मामलों के हवभाग (यूएन डीईएसए) द्वािा जािी
'अंतरााष्ट््ीय प्रवासन 2020 ररप टा ' की मुख्य हवशेषताएं :
गोि ली हुई भूहम पि िाजनीहतक प्रभाव प्राप्त कि
 भाित में िु हनया की सबसे बडी प्रवासी आबादी िै, 2020 में लगभग 18 क्रमक्रलयन
ििे िैं । उदािरण के क्रलए:
लोग अपनी मातृभूहम से बािि िि ििे िैं ।
 संयुि िाज्य अमेरिका की उपिाष्टरपहत कमला
• भाित के प्रवासी संयुि अिब अमीिात (3.5 हमहलयन), अमेरिका (2.7 हमहलयन) औि
दे वी िैररस का चुनाव। सऊिी अिब (2.5 हमहलयन) सहित कई प्रमुख िे शों में हवतरित िैं ।
 मॉरीशस के राजनेता प्रक्रवंद कुमार िगन्नाथ
2017 से प्रिानमंत्री पि पि िैं ।
 प्रदीप क्रसंि रूपन 2019 से मॉरीशस के सातवें िाष्टरपहत िैं ।
• नीक्रतगत वकालत: अमेरिका-भारत परमाणु समझौते क अमेरिका में भाितीय प्रवाहसयों का समथान प्राप्त था, हजन्होंने सीनेटिों औि
कां ग्रेहसयों को समझौते पि िस्ताक्षि किने के हलए िाजी हकया, हजससे साहबत हुआ हक भाितीय प्रवासी भाित को लाभ पहुं चाने वाली नीहतयों
को प्रभाहवत किने में मित्वपूर्ा भूहमका हनभा सकते िैं ।
• धारणा बदलना: भाितीय प्रवाहसयों ने भाित के प्रहत िु हनया की िािर्ा को सकािात्मक रूप से बिल हिया िै।
 उदािरण के क्रलए: सुंिि हपचाई, सत्या नडे ला जैसे हिग्गजों के साथ हसहलकॉन वैली में भाितीय प्रभुत्व, एक प्रौद्योहगकी पावििाउस औि
गुर्विापूर्ा मानव संसािनों के स्रोत के रूप में भाित की छहव को मजबूत किता िै ।
• धन प्रेषण: हवश्व बैंक की एक रिपोटा के अनुसाि, भाित को 2020 में प्रेषर् में USD83 हबहलयन से अहिक िन प्राप्त हुआ।
• क्रशक्षा और संस्कृक्रत में य गदान: हशक्षा औि संस्कृहत अन्य िो क्षेत्र िैं जिां प्रवासी, भाित औि उनके हनवास वाले िे शों के बीच संबंिों को
मजबूत किने में मित्वपूर्ा भूहमका हनभा सकते िैं ।
• उदािरण के क्रलए:

21
 इं हडया हबजनेस स्कूल, िैििाबाि प्रबंिन अध्ययन के हवकास में डायस्पोिा के भूहमका का एक अच्छा उिाििर् िै ।
 उन्होंने योग, आयुवेि, भाितीय व्यंजन आहि जैसी भारतीय संस्कृक्रत और परं पराओं क क्रवदे श ं में फैलाया।
 व्हाइट िाउस, यूएसए में हिवाली जैसे हिंिू त्योिािों का जश्न।
 उन िे शों से भाित में पयाटन को बढ़ाने में योगिान िे ना।
✓ उदािरण के क्रलए: संयुि िाज्य अमेरिका 2021 में 429.8 िजार से अक्रधक आगंतुक ं के साथ भारत आने वाले क्रवदे शी पयाटक ं
का सबसे बडा स्र त था।
चुनौक्रतयां:
• घरे लू राजनीक्रत पर प्रभाव: बडे हविे शी समुिायों की उपक्तस्थहत के कािर्, प्रत्येक घिे लू िाजनीहतक हवकास एक बाििी आयाम प्राप्त कि
लेता िै ।
• प्रवासी और आतंकवाद: कभी-कभी, िवाला मागा के माध्यम से आतंकवाि के हविपोषर् के हलए प्रवासी एक प्रमुख स्रोत बन जाते िैं । वे
नाको व्यापाि को सुहविाजनक बनाने में भी भूहमका हनभाते िैं ।
 उदािरण के क्रलए: खाहलस्तान आं िोलन को हविे शों में हसख समुिायों के बडे वगा से प्रमुख िाजनीहतक, हविीय औि भौहतक समथान
प्राप्त िोता िै ।
• क्रवदे शी खुक्रफया एजेंक्रसय ं के क्रलए संपक्रत्त के रूप में प्रवासी: पाहकस्तान के अलावा, अमेरिका, हिटे न औि कनाडा आहि की खुहफया
एजेंहसयों ने भी अपनी गहतहवहियों के हलए भाितीय प्रवाहसयों का उपयोग हकया िै ।
• भारतीय प्रवाक्रसय ं की दू सरी पीढी के बदलते क्रवचार: संयुि िाज्य अमेरिका में भाितीय िाजिू त के साथ भाित का कॉकस भाितीय-
अमेरिकी समुिाय के बिलते िवैये के बािे में एक अंतदृाहष्ट प्रिान किता िै ।
 उदािरण के क्रलए, भाितीय-अमेरिकी समुिाय के प्रहतहनहियों वाले कॉकस ने भाित सिकाि से यि सुहनहित किने का आग्रि हकया हक
ल कतंत्र के मानदं ि ं क बनाए रखा जाए।

प्रवासी भारतीय ं तक पहुंच बढाने के क्रलए सरकारी पिल:


• प्रवासी भारतीय क्रदवस (PBD): भाित के हवकास में हविे शों में बसे भाितीय समुिाय के योगिान को हचहित किने के हलए िि साि 9
जनविी को आयोहजत हकया जाता िै ।
• भारत क जानें कायािम: इसका उद्दे श्य आहथाक, औद्योहगक, हशक्षा, हवज्ञान औि प्रौद्योहगकी, संचाि औि सूचना प्रौद्योहगकी, संस्कृहत जैसे
हवहभन्न क्षेत्रों में जागरूकता को बढ़ावा िे ना िै।
• प्रवासी तीथा दशान य जना: 45 से 65 वषा की आयु के बीच पीआईओ (भाितीय मूल के व्यक्तियों) को अपनी जडों से हफि से जुडने का
मौका प्रिान किती िै ।
• पीआईओ और ओसीआई कािों की क्लक्रबंग: केंद्र ने भाितीय मूल के लोगों के हलए भाित में वीजा-मुि प्रवेश को आसान बनाने के हलए
ओविसीज हसटीजन ऑफ़ इं हडया (OCI) औि पसान्स ऑफ इं हडयन ओरिहजन (PIO) काडों को एक िी योजना में हमला हिया िै ।
• प्रवासी भारतीय ं के साथ सांस्कृक्रतक संबंध ं क बढावा दे ना (पीसीटीिी): पीसीटीडी भाितीय संस्कृहत, नई पिल औि आिुहनक भाित
के हवकास को प्रिहशात किने वाले सांस्कृहतक कायािमों के आयोजन में सिायता किता िै ।
• प्रवासी कौशल क्रवकास य जना (पीकेवीवाई): इसका लक्ष्य िै प्रवासी भाितीय श्रहमकों के कौशल हवकास की प्रहिया को संस्थागत बनाना।
• ई-माइग्रेट प्रणाली: सभी हविे शी हनयोिाओं को डे टाबेस में पंजीकिर् किना आवश्यक िै , जो प्रवाहसयों के कल्ार् औि शोषर् पि िोक
सुहनहित किता िै ।
• मदद प टा ल: यि हविे श में ििने वाले लोगों की हशकायतों पि समय पि औि त्वरित कािा वाई किता िै ।
आगे की राि:
• प्रवासी नेटवका और प्लेटफॉमा स्थाक्रपत करना: हवहभन्न िे शों में भाितीय प्रवाहसयों से जुडने के हलए औपचारिक नेटवका औि प्लेटफ़ॉमा
बनाए जाने चाहिए।
• क्रनवेश के अवसर प्रदान करना: प्रहियाओं को सिल बनाकि, प्रोत्सािन िे कि औि अनुकूल कािोबािी मािौल प्रिान किके भाित में हनवेश
के अवसिों को बढ़ावा िें ।
• द िरी नागररकता और आसान वीजा क्रनयम प्रदान करना: प्रवासी भाितीयों के हलए िोििी नागरिकता हवकल् प्रिान किने पि हवचाि
किें , हजससे वे भाित के साथ मजबूत संबंि बनाए िख सकें।
प्र. प्रवासी भाितीयों के सामने क्ा चुनौहतयां िैं औि यि भाित के
• नीक्रतगत संवाद में शाक्रमल ि ना: भाित सिकाि औि प्रवासी
भाितीयों के बीच नीहतगत संवाि के हलए मंच स्थाहपत किना, ताहक
हवकास के हलए क्ा अवसि प्रस्तुत किता िै ?
उनकी हचंताओं को िू ि हकया जा सके, उनके इनपुट मां गे जा सकें औि उन्हें हनर्ाय लेने की प्रहियाओं में शाहमल हकया जा सके।
22
2.4 शरणाथी सं क ट पर यू ए नएचसीआर ररप टा

संदभा:
िाल िी में, संयुि राष्ट्् शरणाथी उच्चायुि (यूएनएचसीआर) ने "वैक्रश्वक रुझान: 2022 में जबरन क्रवस्थापन" ररप टा जारी की।
ररप टा की मुख्य बातें
• वैक्रश्वक मजबूरन क्रवस्थापन: िु हनया भि में हवस्थाहपत लोगों की संख्या
ररकॉिा 110 क्रमक्रलयन तक पहुंच गई िै । प्रत्येक 74 लोगों में से एक से
अहिक अब हवस्थाहपत िैं ।
 हपछले एक िशक में जबिन क्रवस्थाक्रपत ल ग ं की संख्या द गुनी ि
गई िै।
 वतामान में यूिेक्रनयन का क्रवस्थापन हद्वतीय हवश्व युद् के बाि से
वैहश्वक स्ति पि सबसे तेज िै , जो 2021 के अंत में 27,300 से बढ़कि
2022 के अंत में 5.7 हमहलयन िो गया।
• 2022 में सभी नए हवस्थापनों में से आिे से अहिक (54 प्रहतशत) के हलए
आपदा-संबंधी आं तररक क्रवस्थापन क्रजम्मेदार िै।
 2021 में, भाित में लगभग पााँच क्रमक्रलयन ल ग आं तररक रूप से
क्रवस्थाक्रपत हुए। इसमें बताया गया कािर् आपदा और जलवायु
पररवतान िै।
• क्रवस्थापन का कारण: यूिेन में युद् 2022 में हवस्थापन का मुख्य कािर्
था।
 अन्य कारण: िु हनया के अन्य हिस्सों में संघषा, भोजन की कमी,
मुद्रास्फीहत औि जलवायु आपातकाल ने हवश्व स्ति पि बढ़ते हवस्थापन
में योगिान हिया िै ।
• शरणाथी: हवस्थाहपत लोगों की कुल संख्या में से, 35.3 क्रमक्रलयन शरणाथी थे, या वे लोग जो संयुि िाष्टर एजेंहसयों के आिे श के तित सुिक्षा
पाने के हलए अंतििाष्टरीय सीमा पाि कि गए थे।
 कुल शिर्ाहथायों में से, लगभग आिे केवल तीन दे श ं से आए थे: सीररया, यूिेन और अफगाक्रनस्तान।
• राज्यक्रविीनता: 2022 के अंत में, िु हनया भि में अनुमाहनत 4.4 क्रमक्रलयन ल ग िाज्यहविीन या अहनिाा रित िाष्टरीयता वाले थे - 2021 के अंत
की तुलना में 2 प्रक्रतशत अक्रधक।
 िाज्यहविीन लोग वे िोते िैं हजन्हें क्रकसी भी सरकार द्वारा नागररक के रूप में मान्यता निी ं दी जाती िै।
• शरण चािने वाले: वैहश्वक स्ति पि 5.4 हमहलयन शिर् चािने वाले थे।
 आवेदक ं का दे श: सवाा हिक संख्या पाहकस्तान (28,500), हमस्र (27,300) औि लीहबया (20,200) से िजा की गई।
• जनसांक्तख्यकीय क्रवस्थापन
 बच्चे: वे सभी जबरन क्रवस्थाक्रपत ल ग ं का 41 प्रक्रतशत िैं , लेहकन िु हनया की आबािी का केवल 30 प्रहतशत िैं ।
 2018 से 2022 के बीच, 1.9 हमहलयन से अहिक बच्चे शरणाथी के रूप में पैदा हुए - प्रहत वषा अनुमाहनत 3,85,000 बच्चे।
 मक्रिलाएाँ और लडक्रकयााँ: वे 2022 के अंत तक जबिन हवस्थाहपत हकए गए सभी शिर्ाहथायों में से आधे से अक्रधक या 51 प्रक्रतशत का
प्रक्रतक्रनक्रधत्व करती िैं।
• पड सी या क्रनम्न और मध्यम आय वाले दे श ं पर असंगत ब झ: 2022 के अंत में, 70 प्रहतशत शिर्ाहथायों, हजनमें शिर्ाथी जैसी क्तस्थहत
वाले लोग औि अंतिााष्टरीय सुिक्षा की आवश्यकता वाले अन्य लोग शाहमल थे, की मेजबानी पड सी दे श ं द्वारा की गई थी।
 2022 में, क्रनम्न और मध्यम आय वाले दे श ं ने िु हनया के 76 प्रहतशत शिर्ाहथायों औि अंतिाा ष्टरीय सुिक्षा की आवश्यकता वाले अन्य
लोगों की मेजबानी की।
 कम आय वाले दे श, हजनके पास क्तस्थहत को अनुकूहलत किने के हलए सीहमत संसािन िैं औि वैहश्वक सकल घिे लू उत्पाि का केवल
0.5 प्रक्रतशत क्रिस्सा िै, ने 16 प्रक्रतशत शरणाक्रथाय ं की मेजबानी की।
• मेजबान दे श: 3.6 हमहलयन शिर्ाहथायों के साथ तुकी िु हनया में सबसे अक्रधक शरणाक्रथाय ं की मेजबानी करने वाला िे श बना हुआ िै,
जो कुल शिर्ाहथायों के 10 प्रहतशत से अहिक िै ।

23
• क्रसल्वरलाइक्रनंग: 2022 में, सुरक्रक्षत तीसरे दे श में पुनवााक्रसत शरणाक्रथाय ं की संख्या हपछले वषा से िोगुनी िोकि 1,14,300 िो गई।
भारत और शरणाथी:
• भाित अन्य िे शों के लगभग द
लाख शरणाक्रथाय ं और शरण
चािने वाल ं औि लगभग 20,000
िोहिं ग्याओं की मेजबानी किता िै ।
 जबहक कुछ शिर्ाहथायों, अथाात्
श्रीलंका और क्रतब्बत से आए
ल ग ं को सिकािी सेवाओं औि
िोजगाि तक पहुंच प्राप्त हुई िै ,
र क्रिंग्याओं क सबसे बुक्रनयादी
सेवाओं और सुरक्षा से भी
वंक्रचत कर क्रदया गया िै।
• भारत की नीक्रत: भाित में
शिर्ाहथायों की समस्या के समािान
के हलए क्रवक्रशष्ट् र्िर्ध का अभाव िै।
 भाित शरणाथी कन्वेंशन,1951
औि इसके 1967 के प्रोटोकॉल,
शिर्ाथी संिक्षर् से संबंहित
प्रमुख कानूनी िस्तावेजों का एक पक्षकार निीं िै ।
 गैर-वापसी का क्रसद्ांत: जब भी असिाय शिर् चािने वालों
ने इसके ििवाजे खटखटाए िैं , भाित ने गैि-वापसी के • प्रवासी: वि व्यक्ति ज काम, क्रशक्षा, पररवार या जीवनशैली जैसे क्रकसी
भी कारण से एक स्थान से दू सरे स्थान पर जाता िै , या तो िे श के भीति
हसद्ां त का पालन हकया िै , इस तथ्य के बावजूि हक भारत
या अंतििाष्टरीय सीमा के पाि।
1951 के संयुि राष्ट्् शरणाथी सम्मेलन का
 प्रवाहसयों को अपने गंतव्य दे श में कानूनी दजाा प्राप्त ि भी सकता
िस्ताक्षरकताा निी ं िै।
िै औि निीं भी औि वे विां स्थायी रूप से बसने का इिािा िख भी
 अंतरााष्ट््ीय र्िर्ध में गैर-वापसी की अवधारणा का तका िै सकते िैं या निीं भी।
हक अपने िी िे श में उत्पीडन से भाग ििे हकसी व्यक्ति को • शरणाथी (Refugee): वि व्यक्ति जो उत्पीडन, युद् या हिं सा के उहचत
उसकी इच्छा के हवरुद् विां वापस निीं भेजा जाना चाहिए। भय के कािर् अपने मूल दे श से भाग गया िै और ज विां सुरक्रक्षत
 तदथा शरणाथी नीक्रत: यि नई हिल्ली क शरणाक्रथाय ं के रूप से वापस निी ं लौट सकता िै।
साथ उनके संबंि के आिाि पि अलग-अलग व्यविाि किने  शरणाक्रथाय ं क अंतरराष्ट््ीय कानून द्वारा संरक्रक्षत क्रकया जाता िै
औि मानवीय हितों की तुलना में अन्य हितों को प्राथहमकता औि उन्हें िू सिे िे श में शिर् लेने का अहिकाि िै ।
िे ने में सक्षम बनाती िै। • शरण चािने वाला (Asylum Seeker): ऐसा व्यक्ति हजसने अपना मूल
 भाित ने एक मानक संचालन प्रक्रिया (एसओपी) दे श छ ड क्रदया िै और िू सिे िे श में शिर्ाथी के रूप में सुरक्षा की मांग
प्रकाक्रशत की िै क्रजसका शिर्ाथी िावा किने वाले हविे शी कर रिा िै, लेक्रकन क्रजसके दावे पर अभी तक फैसला निी ं हुआ िै।

नागरिकों के साथ बातचीत किते समय सभी संबंहित  शिर् चािने वालों को शरणाक्रथाय ं के समान अक्रधकार और लाभ
निी ं क्रमलते िैं जब तक हक उनकी क्तस्थहत को मान्यता निीं िी जाती
अहिकारियों को पालन किना िोगा।
िै ।
आगे की राि: • क्रवस्थाक्रपत व्यक्ति: ऐसा व्यक्ति हजसे सशस्त्र संघषा, क्रिंसा,
• क्रटकाऊ समाधान: इसका मुख्य लक्ष्य क्रवस्थाक्रपत समुदाय ं मानवाक्रधकार ं के उल्लंघन या प्राकृक्रतक या मानव क्रनक्रमात आपदाओं
की जरूरत ं और कमज ररय ं क धीरे -धीरे कम करना िै, के कारण अपना घर या हनवास स्थान छोडने के हलए मजबूि हकया गया
साथ िी उनकी क्षमताओं, कौशल औि लचीलेपन को बढ़ाना िै , िो।
हजससे उन्हें हवस्थापन के आिाि पि भेिभाव के हबना अपने • हवस्थाहपत व्यक्ति या तो आं तररक रूप से क्रवस्थाक्रपत (अपने दे श के
मानवाहिकािों का पूिी तिि से आनंि लेने में सक्षम बनाया जा भीतर) या बािरी रूप से क्रवस्थाक्रपत (अंतरााष्ट््ीय सीमा के पार) ि
सकते िैं।
सके।

24
• संघषों क र कना और उसका हि र्नकािना: इसमें शाहमल शरणाक्रथाय ं के क्रलए संयुि राष्ट्् उच्चायुि (यूएनएचसीआर)
सभी पक्षों से िाजनीहतक इच्छाशक्ति, संवाि, कूटनीहत औि • यि एक वैहश्वक सं गठन िै जो जीवन बचाने, अहिकािों की िक्षा किने औि
मानवीय कािा वाई की अपेक्षा िै । शिर्ाहथायों, जबिन हवस्थाहपत समुिायों औि िाज्यहविीन लोगों के बेिति
 अंतिाा ष्टरीय समुिाय को भी शां हत हनमाा र् प्रयासों का समथान भहवष्य के हनमाार् के प्रहत समहपात िै ।
किना चाहिए, हिं सा औि अक्तस्थिता के मूल कािर्ों का • मुख्यालय: हजने वा, क्तस्वट् जिलैंड.
समािान किना चाहिए। • 1951 शिर्ाथी कन्वेंशन औि इसका 1967 का प्रोटोकॉल प्रमुख कानूनी
• सुरक्षा और सिायता: स्वास्थ्य िे खभाल, हशक्षा, पानी, स्वच्छता िस्तावेज िैं जो यूएनएचसीआि का आिाि बनते िैं ।
औि आजीहवका के अवसिों जैसी बुहनयािी सेवाओं तक पहुंच संयुि राष्ट्् शरणाथी कन्वेंशन 1951

प्रिान किना। • शिर्ाथी की परिभाषा, शरण पाने वाल ं के अक्रधकार और शरण प्रदान
करने वाले दे श ं के दाक्रयत्व सभी एक बहुपक्षीय संयुि राष्ट्् संक्रध में
• अंतरााष्ट््ीय सिय ग क मजबूत करना: शिर्ाथी क्तस्थहतयों पि
क्रनधााररत िैं।
अहिक पूवाा नुमाहनत औि न्यायसंगत प्रहतहियाओं के हलए एक
• इसके तित, जो लोग अपनी जातीयता, िमा, िाष्टरीयता, हकसी हवशेष
रूपिे खा के रूप में, शरणाक्रथाय ं पर ग्ल बल कॉम्पैि क
सामाहजक समूि की सिस्यता या िाजनीहतक हवश्वास के कािर् उत्पीडन
लागू किने के माध्यम से िाज्यों औि अन्य अहभकताा ओं के बीच से भाग ििे िैं , उन्हें कुछ अक्रधकार क्रदए जाते िैं ।
हजम्मेिािी साझा किना। • 1967 प्र ट कॉल के रूप में सम्मेलन में एक संशोिन हकया गया िै ।
• जलवायु पररवतान के प्रभाव ं क संब क्रधत करना: इसके शरणाक्रथाय ं पर वैक्रश्वक समझौता (जीसीआर), 2018
क्रलए ग्रीनिाउस गैस उत्सजान क कम किना, ग्लोबल वाहमिंग • इसे शिर्ाहथायों को बेिति समथान िे ने के हलए मेजबान िे शों औि समुिायों
के प्रभावों से अनुकूलन किना , आपदा ज क्तखम में कमी और के बीच क्रजम्मेदारी-साझाकरण क बढावा दे ने के क्रलए क्रिजाइन क्रकया
तैयाररय ं का समथान करना औि पयााविर्ीय कािकों से गया िै।
हवस्थाहपत लोगों की सुिक्षा बढ़ाना आवश्यक िै । यूके-रवांिा शरण य जना
• क्रनजी क्षेत्र क शाक्रमल करना: हनजी क्षेत्र हवस्थाहपत लोगों को • यि पांच साल की परीक्षण य जना िै , हजसे अप्रैल 2022 में पेश हकया
गया था।
नौकरियों औि बुहनयािी ढांचे के समथान के माध्यम से संकट से
• इसका लक्ष्य चुक्रनंदा शरण चािने वाल ं क एकतिफा हटकट के साथ
उबिने में मिि किने में अमूल् योगिान िे सकता िै ।
िवांडा ले जाना िै , जिां वे अपने शिर् के िावे िजा किा सकें।
 2018 में, क्रफलीपी ंस क्रिजास्टर रे क्रजक्रलएं स फाउं िेशन,
जो उस िे श में हनजी क्षेत्र के अहभकताा ओं का एक नेटवका िै , ने मिावी क्षेत्र में संघषा से प्रभाहवत लोगों को 3,000 िोजगाि के प्रस्ताव
प्रिान हकए।

2.5 भारत-फ्ां स सं ब ं ध

संदभा:
िाल िी में भाितीय प्रिानमंत्री द क्रदवसीय आक्रधकाररक यात्रा पर पेररस पहुंचे।
यात्रा का मुख्य आकषाण:
• बैक्तस्टल िे परे ि: भाितीय प्रिानमंत्री निें द्र मोिी पेररस में फ्ांस के बैक्तस्टल िे परे ि में सम्माक्रनत अक्रतक्रथ िैं।
 यि फ्ांस का राष्ट््ीय क्रदवस िै, ज 14 जुलाई क मनाया जाता िै, क्रजसे बैक्तस्टल डे या फेटे नेशनेल फ़्रैन्काइज के नाम से भी जाना
जाता िै ।
 क्रदन का मित्व: यि फेटे डे ला फेडिे शन की वषागांठ िै , जो 1790 में रां सीसी लोगों की एकता का जश्न मनाने के हलए आयोहजत एक
कायािम था। इसे राजशािी के अंत के प्रतीक के तौर पर दे खा जाता िै.
 सैन्य पिे ड में क्रत्र-सेवा भारतीय दल भाग लेगा। परे ि के दौरान भारतीय वायुसेना के तीन हवमान फ्लाईपास्ट भी किें गे।
• फ्ांस में UPI: भाित औि रां स यूहनफाइड पेमेंट इं टिफेस (UPI) भुगतान तंत्र का उपयोग किने पि सिमत हुए िैं ।
 यूक्रनफाइि पेमेंट्स इं टरफेस (UPI) भारत की म बाइल-आधाररत भुगतान प्रणाली िै औि लोगों को वचुाअल भुगतान पते के माध्यम
से चौबीसों घंटे भुगतान किने की अनुमहत िे ती िै ।
• ग्रैंि िॉस ऑफ द लीजन ऑफ ऑनर: प्रिानमंत्री निें द्र मोिी को रां स के िाष्टरपहत इमैनुएल मैिॉन ने फ्ांस के सवोच्च सम्मान ग्रैंि
िॉस ऑफ द लीजन ऑफ ऑनर से सम्माक्रनत क्रकया िै।
 इसकी स्थापना 1802 में,पूवा रां सीसी सम्राट नेपोहलयन बोनापाटा ने की थी।
 प्रिानमंत्री निें द्र मोिी यि सम्मान पाने वाले पिले भारतीय प्रधानमंत्री िैं।
• नौसेना के क्रलए राफेल: िाल के वषों में रां सीसी एयिोस्पेस प्रमुख डसॉट एहवएशन से यि िू सिी ऐसी लडाकू जेट खिीि िोगी।

25
 2016 में ,भारतीय वायु सेना के क्रलए 36 राफेल लडाकू जेट खिीिने का सौिा हकया गया था।

भारत के क्रलए फ्ांस का मित्व:


• समृद् संबंध: वषा 2023 भाित-रां स िर्नीहतक साझेिािी की 25वी ं वषागांठ
िै । िोनों िे श िक्षा, अंतररक्ष, नागररक परमाणु, संस्कृक्रत और ल ग ं से ल ग ं
के संबंध ं सक्रित क्रवक्रभन्न क्षेत्र ं में क्रनकटता से सिय ग करते िैं।
• मजबूत वैक्रश्वक समथाक: 1998 में परमाणु परीक्षण के बाद भारत के
सामररक मित्व क पिचानने वाला रां स पिला िे श था। रां स के साथ
साझेिािी यूर प में भारत की सबसे मित्वपूणा रणनीक्रतक साझेदारी िै।
• व्यापार और वाक्रणज्य: 2021-22 में वाहषाक व्यापाि की मात्रा 12.42 क्रबक्रलयन
िॉलर तक पहुंच गई, औि हपछले िो िशकों में 10.31 हबहलयन डॉलि के संचयी
हनवेश के साथ रां स भाित में 11वां सबसे बडा क्रवदे शी क्रनवेशक िै।
• रक्षा और सुरक्षा में भूक्रमका: फ्ांस ने िहथयािों के सि-हवकास औि सि-
उत्पािन के माध्यम से भारत के रक्षा औद्य क्रगक आधार क आधुक्रनक बनाने
में मित्वपूर्ा भूहमका हनभाई िै ।
 यि 2017 औि 2021 के बीच दू सरा सबसे बडा रक्षा आपूक्रताकताा बन
गया।
 मित्वपूणा रक्षा सौदे , जैसे हक रां सीसी स्कॉपीन पनिु क्तब्बय ं क शाक्रमल
करना औि 36 िाफेल लडाकू जेट की खिीि,रक्षा सिय ग की गिराई क उजागर करते िैं।
 संयुि सैन्य अभ्यास और संवाद इन संबंिों को औि मजबूत किते िैं ।
• जलवायु: जलवायु परिवतान से हनपटने के हलए भाित की प्रहतबद्ता फ्ांस के लक्ष्य ं के अनुरूप िै, जैसा क्रक पेररस समझौते और 2015
में अंतरााष्ट््ीय सौर गठबंधन की स्थापना के तित उनके संयुि प्रयासों से प्रिहशात िोता िै ।
• ऊजाा क्षेत्र: भाितीय पिमार्ु ऊजाा हनगम (NPCIL) औि अिे वा , 9.6 गीगावॉट की कुल स्थाहपत क्षमता वाले छि ईपीआर (यूर पीय
दबावयुि ररएिर) पिमार्ु ऊजाा रिएिि हवकहसत किने के हलए बातचीत कि ििे िैं ।
• वैक्रश्वक क्रनकाय ं में समथान: रां स संयुि िाष्टर सुिक्षा परिषि में भाित की स्थायी सिस्यता और परमाणु आपूक्रताकताा समूि में प्रवेश का
समथान करता िै।
• इं ि -पैक्रसक्रफक रणनीक्रत: रांस यूर पीय संघ का एकमात्र दे श िै हजसके पास इं डो-पैहसहफक में क्षेत्र िैं ।
 रां स तीन क्षेत्रीय नेटवका का हिस्सा िै - क्रिंद मिासागर आय ग, 1986 से, क्रिंद मिासागर नौसेना संग ष्ठी 2008 में इसके हनमाार्
के समय से औि क्रिंद मिासागर ररम एस क्रसएशन (1995 में बनाया गया) 2020 से।
• चीन पर नजर रखना: द न ं दे श चीन के क्षेत्रीय और वैक्रश्वक व्यविार पि हचंता साझा किते िैं औि इं डो-पैहसहफक में संतुलन बनाए
िखने के हलए हमलकि काम किने के हलए प्रहतबद् िैं।

भक्रवष्य की कायावािी: फ्ांस, भारत और क्रवश्व:


• भारत और फ्ांस द न ं अपनी रणनीक्रतक स्वायत्तता क मित्व दे ते िैं, अपनी हविे श नीहतयों में स्वतंत्रता का प्रयास किते िैं औि एक
बहुध्रुवीय िु हनया चािते िैं।
• यूिेन युद्: रूस के आिमर् औि उसके कािर् हुए भू-िाजनीहतक परिवतानों ने भाित के िर्नीहतक मित्व के बािे में यूिोपीय (हवशेष रूप
से रां स) में एक नई जागरूकता ला िी िै औि साथ िी फ़्रां स के बािे में भाित में भी।
• व्यापार और क्रनवेश क बढाना: व्यापाि औि हनवेश के क्षेत्र में भािी संभावनाएं िैं औि िोनों िे शों को लंबे समय से लंक्रबत मुि व्यापार
समझौते (एफटीए) के त्वररत समाधान पर ध्यान केंक्रद्रत करना चाक्रिए।

2.6 बहुपक्षीय क्रवकास बैं क (एमिीबी) सु ध ार


संदभा:
िाल िी में, G20 के एक स्वतंत्र क्रवशेषज्ञ समूि ने बहुपक्षीय क्रवकास बैंक ं (MDBs) क मजबूत किने पि अपनी रिपोटा जािी की।
बहुपक्षीय क्रवकास बैंक (एमिीबी) के बारे में :

26
• यि गिीब िे शों में आहथाक हवकास को प्रोत्साहित किने के उद्दे श्य से द या द से एमिीबी के उदािरण
अक्रधक दे श ं द्वारा चाटा िा एक अंतििाष्टरीय हविीय संस्थान िै । • हवश्व बैंक
• इनकी उत्पक्रत्त क्रद्वतीय क्रवश्व युद् के बाद, युद् से तबाि िे शों के पुनहनामाा र् औि • यूिोपीय हनवेश बैंक
वैहश्वक हविीय प्रर्ाली को क्तस्थि किने के हलए हुई थी। • एहशयाई हवकास बैंक
• एहशयन इन्फ्रास्टर क्चि इन्वेस्टमेंट बैंक
बहुपक्षीय क्रवकास बैंक ं (एमिीबी) क मजबूत करने पर जी 20 क्रवशेषज्ञ समूि की

इस्लाहमक डे वलपमेंट बैंक
ररप टा की मुख्य क्रवशेषताएं

आहथाक एकीकिर् के हलए सेंटरल अमेरिकन बैंक
• क्रवशेषज्ञ समूि के बारे में: अथाशास्त्री लॉरें स समसा और एनके क्रसंि की अध्यक्षता

नव हवकास बैंक
में हवशेषज्ञ पैनल को एमिीबी के क्रलए सुधार ं का प्रस्ताव दे ने के क्रलए G20 दे श ं
के समूि द्वािा हनयुि हकया गया था।
• क्रवशेषज्ञ समूि ने क्रट् पल मैंिेट अपनाने का सुझाव क्रदया:
 अत्यक्रधक गरीबी क खत्म करना, साझा समृक्तद् क बढावा दे ना औि वैक्रश्वक सावाजक्रनक वस्तुओं में य गदान दे ना।
 2030 तक स्थायी ऋण स्तर क तीन गुना किना।
 एमडीबी एजेंडे के तत्वों का समथान किने के हलए लचीली औि नवीन व्यवस्था की अनुमहत िे ने के हलए तीसरा क्रवत्त प षण तंत्र बनाना।
• जी20 की भूक्रमका: 2030 में एमिीबी प्रणाली के स्थायी ऋण स्तर ं क एसडीजी लक्ष्यों को प्राप्त किने में हनवेश किने के हलए
हवकासशील िे शों द्वािा आवश्यक हविीय सिायता से जोडना औि िर तीन साल में ऐसे ऋण ं की समीक्षा करना।
• क्रनजी कंपक्रनय ं के साथ साझेदारी: एमडीबी को अपनी सतत हवकास िर्नीहतयों के केंद्र में हनजी पूंजी जुटाने औि उत्प्रेरित किने को
िखना चाहिए।
 वे सरकार ं क नीक्रतगत और क्रवक्रनयामक ज क्तखम क कम करने औि हविीय उत्पाि पेशकशों को हनजी पूंजी बाजाि अंतिाल के
अनुरूप बनाने में मिि कि सकते िैं ।
• सभी क्रितधारक दृक्रष्ट्क ण: अंतिाा ष्टरीय मुद्रा कोष औि हवश्व बैंक की
तिि एमडीबी को जोक्तखमों को कम किने, साझा किने औि प्रबंहित
किने के हलए सिकािों औि हनजी क्षेत्र के साथ काम किना चाहिए
और इस प्रकार पूंजी की लागत में कमी लानी चाक्रिए।

एमिीबी का मित्व
• क्रवकासशील दे श ं क समथान: एमडीबी हवकासशील िे शों को
आक्रथाक प्रबंधन क मजबूत किने औि गिीबी कम किने में मिि
किने के हलए क्रवत्तीय और तकनीकी सिायता प्रदान करता िै।
 सिायक भूक्रमका: 2008 के वैहश्वक हविीय संकट के िौिान,
एमडीबी ने $222 क्रबक्रलयन का हविपोषर् प्रिान हकया, जो
वैक्रश्वक क्तस्थरीकरण प्रयास ं के क्रलए मित्वपूणा था।
• संस्थागत क्षमता का क्रनमााण: एमडीबी शासन, नीहतगत ढां चे औि
संस्थागत क्षमता को बढ़ाने में सिस्य िे शों का समथान किते िैं।
• राजनीक्रतक सीमाओं से परे : एमडीबी उन समस्याओं को िल किने
पि अहिक ध्यान केंहद्रत कि ििे िैं जो िाष्टरीय सीमाओं से पिे िैं ।
• जलवायु कारा वाई का समथान: वे ग्ल बल वाक्रमिंग क कम करने
औि इसके अनुरूप ढलने, नवीकिर्ीय ऊजाा का उपयोग किने,
पयााविर् की िक्षा किने, जैव क्रवक्रवधता क संरक्रक्षत करने और
क्रटकाऊ बुक्रनयादी ढांचे के क्रनमााण के उद्दे श् से कायािम ं का
समथान करते िैं।
 उदािरण के क्रलए- हवश्व बैंक ने राष्ट््ीय स्वच्छ गंगा क्रमशन (एनएमसीजी) या नमाक्रम गंगे पररय जना के क्रलए पांच साल के ऋण
(दू सरे चरण के क्रलए) क मंजूरी दे दी िै।
• क्रनजी क्षेत्र की भागीदारी का समथान: जोक्तखमों को कम किके, शासन को बढ़ाकि औि सावाजहनक-हनजी भागीिािी को बढ़ावा िे कि, वे
एक ऐसे वाताविर् के हवकास में योगिान किते िैं जो हनजी हनवेश के हलए अनुकूल िै ।

27
एमिीबी में सुधार की आवश्कता:
• एक आकार सभी के क्रलए क्रफट दृक्रष्ट्क ण(One Size Fit Approach): न्यायपूर्ा औि सतत
हवकास को आगे बढ़ाने के हलए, हवकासशील िे शों में क्रवक्रभन्न प्रकार की अलग-अलग और
बदलती जरूरतें हैं, औि एमडीबी के वतामान परिचालन तिीके औि व्यवसाय मॉडल उन
जरूितों को पूिा किने के हलए सबसे उपयुि निीं िैं ।
• पारं पररक प्राथक्रमकताओं क संतुक्रलत करना: जैसे-जैसे एमडीबी अपने अहििे शों का हवस्ताि
कि ििे िैं , हचंता यि िै हक गरीबी उन्मूलन और असमानता क संब क्रधत करने जैसी
पारं पररक प्राथक्रमकताओं से समझौता यकिे जाने का डर है ।
• 21वी ं सदी के क्रलए उपयुि निी ं: एमिीबी का वतामान संस्थागत और क्रनयामक ढांचा पुिाना
िै औि क्रिक्रजटल वातावरण की जक्रटलता और त्वररत-बदलती प्रकृक्रत क संभालने में
असमथा िै।
 उदािरण: नई सीमा पाि चुनौहतयों - जलवायु पररवतान, मिामारी, मंदी, प्रवासन इत्याहि
के हलए नई हवि क्षमता औि नए दृहष्टकोर् की आवश्यकता िै ।
• सीक्रमत संसाधन: संसािन सीहमत िैं , हजससे एमडीबी के हलए हवकास हनहि की बढ़ती मां ग को
पूिा किना मुक्तिल िो गया िै ।
 हवश्व बैंक औि अन्य एमडीबी से हवकासशील िे शों को समथान का कुल प्रवाि 2022 में 192
क्रबक्रलयन िॉलर था, िालां हक यि 2009 में प्राप्त हवकासशील िे शों के सकल घिे लू उत्पाि के
एक हतिाई हिस्से से भी कम था।
• क्रनजी क्षेत्र की भागीदारी का अभाव: एमडीबी को हवकास पिलों के हलए हनजी क्षेत्र की फंहडं ग
िाहसल किने में कहठनाइयों का सामना किना पडता िै ।
• प्रक्रियात्मक प्रक्रतबंध: नौकिशािी प्रहियाओं में फंसने के कािर् एमडीबी अक्सि आलोचना का
हशकाि िोते िैं , जो पररय जनाओं के कायाान्वयन और क्रनणाय लेने में बाधा उत्पन्न कर सकता
िै।
• प्रक्रतबंक्रधत समथान: वतामान में, एमिीबी अपने खाते पि उिाि हिए गए प्रत्येक डॉलि के हलए
केवल 0.6 िॉलर क्रनजी पूंजी जुटाते िैं।

एमिीबी में सुधार के क्रलए अन्य क्रसफाररशें


• क्रनवेश क प्राथक्रमकता दे ना: एमडीबी को अपने अहििे श को फैलाने के बजाय उन क्षेत्र ं में क्रनवेश क प्राथक्रमकता दे नी चाक्रिए ज
क्रवकास और जलवायु पररणाम ं के क्रलए मित्वपूणा िैं।
• जलवायु-संबंधी उद्दे श् ं का समथान करना: एमडीबी को बडे जलवायु हनवेशों का समथान किने के हलए ररयायती धन मुिैया कराने में
मदद करनी चाक्रिए।
• दे श क्रवक्रशष्ट् रणनीक्रतयााँ: दे श ं क क्षेत्रीय और वैक्रश्वक चुनौक्रतय ं पर सामूक्रिक कारा वाई की आवश्कता क अपनी िर्नीहतयों में
एकीकृत किने में मिि किने में इसे मित्वपूर्ा भूहमका हनभानी चाहिए।
 एमडीबी िे शों को यि पिचानने में मिि कि सकता िै क्रक उनके मौजूदा अंतराल और क्रनवेश के अवसर किां िैं ।
 सिमत दे श रणनीक्रत के तित प्रदशान के आिाि पि सिकािों को िन हवतरित हकया जाना चाहिए।
• अंतरााष्ट््ीय क्रवत्तीय संरचना में सुधार: इसे अहिक समावेशी, कुशल और न्यायसंगत बनाना औि हवकासशील िे शों को इन संस्थानों के
शासन हनकायों में अहिक प्रहतहनहित्व औि एक मजबूत आवाज प्रिान किना।
• हवकासशील िे शों की ऋर् पुनगाठन आवश्यकताओं को बेिति ढं ग से पूिा किने के हलए िेटन वुि्स संस्थान ं के बीच घक्रनष्ठ समन्वय।

क्रनष्कषा
प्र. वे कौन-सी प्रमुख चुनौहतयां िैं हजन्होंने बहुपक्षीय हवकास बैंकों
• 21वी ं सदी की चुनौक्रतय ं से क्रनपटने के क्रलए एमिीबी क और
(एमडीबी) में सुिाि की आवश्यकता को प्रेरित हकया िै औि इन
अक्रधक प्रासंक्रगक बनाने से मानव कल्ार् को बढ़ाने में योगिान चुनौहतयों का समािान किने औि उनके संचालन में सुिाि किने के
हमलेगा। हलए हकन हवहशष्ट उपायों पि हवचाि हकया जा ििा िै ?
• इसमें क्रवक्रभन्न क्रितधारक ं के साथ गिरा एकीकरण मित्वपूर्ा िै ।

28
लघु समाचार

2.7 अं त राा ष्ट्् ीय उत्तर दक्रक्षण व्यापार गक्रलयारा साथ-साथ रुपे क्तस्वच औि यूएईक्तस्वच के साथ ज डने में
(आईएनएसटीसी) सिय ग करें गे।
संदभा: भारत और ईरान ने रूस के साथ व्यापाि को बढ़ावा िे ने • सीमा पार लेनदे न: पिले समझौते द्वािा स्थानीय मुद्राओं
के हलए अंतरााष्ट््ीय उत्तर दक्रक्षण व्यापार गक्रलयारे (INSTC) के (रुपया और क्रदरिम) में सीमा पार लेनदे न क बढ़ावा िे ने के
संचालन को आगे बढ़ाने की योजना बनाई िै । हलए एक रूपिे खा प्रिान की जाएगी।
अंतरााष्ट््ीय उत्तर दक्रक्षण व्यापार गक्रलयारे के बारे में: • क्तस्वफ्ट का क्रवकल्प: िोनों िे श एक मैसेहजंग हसस्टम बनाने के
• अंतरााष्ट््ीय उत्तर दक्रक्षण व्यापार गक्रलयारे (INSTC) की हलए काम किें गे जो क्तस्वफ्ट (स साइटी फॉर वर्ल्ा वाइि
शुरुआत 2000 में रूस, भारत और ईरान द्वारा की गई थी, इं टरबैंक फाइनेंक्रशयल टे लीकम्युक्रनकेशन) के साथ
यि एक बहु-मॉिल पररविन मागा िै ज क्रिंद मिासागर प्रक्रतस्पधाा कर सके।
और फारस की खाडी क ईरान के माध्यम से कैक्तस्पयन भारत-यूएई व्यापक आक्रथाक साझेदारी समझौता (सीईपीए)
सागर से जोडता िै औि आगे रूस में सेंट पीटसाबगा के माध्यम • सीईपीए िो या िो से अहिक िे शों के बीच एक व्यापाि समझौता
से उििी यूिोप तक जोडता िै । िै जो आहथाक सियोग औि एकीकिर् को आगे बढ़ाने के प्रयास
• सदस्य दे श: इसे 13 दे श ं अजरबैजान, बेलारूस, में व्यापार और क्रनवेश बाधाओं क कम करता िै। यि एक
बुल्गाररया, आमेक्रनया, भारत, ईरान, कजाक्रकस्तान, संपूर्ा व्यापाि समझौता िोता िै जो उत्पाद ,ं सेवाओं और
क्रकक्रगास्तान, ओमान, रूस, ताक्रजक्रकस्तान, तुकी और यूिेन क्रनवेश क संब क्रधत करता िै।
द्वारा अनुम क्रदत क्रकया गया िै।  मई 2022 में यूएई और भारत के बीच सीईपीए के बाि से
िोनों िे शों के बीच व्यापाि लगभग 15% बढ़ गया िै ।
 क्रद्वपक्षीय व्यापार में तेल हनयाा त सहित लगभग 85 हबहलयन
डॉलि का हनयाात संयुि अिब अमीिात से भाित को हकया
जाता िै ।

2.9 नाट
संदभा: क्रलथुआक्रनया के क्रवक्रनयस में िाल िी में संपन्न नाटो हशखि
सम्मेलन में सिस्य, यूिेन क नाट के करीब लाने पर सिमत
हुए।
नाट के बारे में:
• नाटो इकतीस सदस्य िे शों से बना एक ट् ान्सअटलांक्रटक
सुरक्षा गठबंधन िै।
• स्थापना: इसकी स्थापना 1949 में उत्तरी अटलांक्रटक संक्रध
पि िस्ताक्षि के साथ हुई थी, हजसे वाक्रशंगटन संक्रध भी किा
छक्रव िेक्रिट: द वायर जाता िै।
2.8 भारत-यू ए ई सं ब ं ध • मुख्यालय: िुसेल्स, बेक्तियम.
संदभा: िाल िी में, प्रिानमंत्री निें द्र मोिी की यात्रा के िौिान भारत- • सदस्य: बेक्तियम, कनाडा, डे नमाका, रां स, आइसलैंड, इटली,
यूएई ने रुपया-क्रदरिम समझौते पर िस्ताक्षर क्रकए। लक्जजमबगा, नीििलैंड, नॉवे, पुतागाल, यूनाइटे ड हकंगडम, संयुि
भारत और संयुि अरब अमीरात के बीच समझौता ज्ञापन िाज्य अमेरिका, ग्रीस, तुकी, जमानी, स्पेन, चेहकया, िंगिी, पोलैंड,
(एमओयू): बुल्गारिया, एस्टोहनया, लातहवया , हलथुआहनया, िोमाहनया,
● सिय ग के क्षेत्र: िू सिे एमओयू के अनुसाि, िोनों केंद्रीय स्लोवाहकया, स्लोवेहनया, अल्बाहनया, िोएहशया, मोंटेनेग्रो, सबसे
बैंक भाित के यूक्रनफाइि पेमेंट इं टरफेस (यूपीआई) िाहलया जुडाव 2020 में उत्तरी मैसेि क्रनया और 2023 में
क यूएई के इं स्टेंट पेमेंट प्लेटफॉमा (आईपीपी) के क्रफनलैंि थे।

29
िार्िका में सों र्क्षप्त समािार

िेनले पासप टा इं िेक्स • यि िु हनया के सभी पासप टों क उन गंतव्यों की संख्या के आिाि पि िैं क किता िै , जिां उनके िािक हबना
2023 पूवा वीजा के पहुं च सकते िैं ।
• जारीकताा: िेनले और पाटा नसा।
• आधार: यि सूचकां क इं टिनेशनल एयि टर ां सपोटा एसोहसएशन (आईएटीए) से डे टा इकट्ठा किता िै जो वैहश्वक
स्ति पि अंति-एयिलाइन सियोग का प्रबंिन किता िै।
• िे नले पासपोटा इं डेक्स 2023 में भाित ने 80वी ं रैं क िाक्रसल की।
• दु क्रनया के शीषा 5 सबसे शक्तिशाली पासप टा :
 हसंगापुि>जमानी>ऑक्तस्टरया>डे नमाका>बेक्तियम

ट् ांस-पैक्रसक्रफक • यूनाइटे ड हकंगडम ने औपचारिक रूप से एक प्रमुख इं डो-पैहसहफक ब्लॉक सीपीटीपीपी में शाक्रमल ि ने के
पाटा नरक्रशप के क्रलए क्रलए एक संक्रध पर िस्ताक्षर क्रकया।
व्यापक और • इसके हलए िे शों को टै ररफ क खत्म करने या उल्लेखनीय रूप से कम करने औि सेवाओं औि हनवेश
प्रगक्रतशील समझौता बाजािों को खोलने के हलए मजबूत प्रहतबद्ता बनाने की आवश्यकता िै ।
(सीपीटीपीपी) • सदस्य: ऑस्टर े हलया, िुनेई, कनाडा, हचली, जापान, पेरू, मलेहशया, मैक्तक्सको, न्यूजीलैंड, हसंगापुि औि हवयतनाम।
• क्रवदे शी कंपक्रनय ं के क्रलए प्रक्रतस्पधाा, बौक्तद्क संपदा अक्रधकार और सुरक्षा क संब क्रधत करने वाले हनयम
बनाए।
• भू-राजनीक्रतक मित्व: सीपीटीपीपी को क्षेत्र में चीन के प्रभुत्व के क्तखलाफ एक िीवाि के रूप में िे खा जाता िै।
• लाभ: सीपीटीटीपी एहशया प्रशांत िे शों में यूके के हनयाात के हलए टै रिफ में कटौती किे गा।

पूवी एक्रशया क्रशखर • िाल िी में, भाितीय हविे श मंत्री ने जकाताा, इं ि नेक्रशया में 13वी ं ईएएस हविे श मंहत्रयों की बैठक में भाग हलया।
सम्मेलन (ईएएस) • यि रणनीक्रतक बातचीत के क्रलए इं ि -पैक्रसक्रफक का प्रमुख मंच िै।
• सदस्य दे श: ऑस्ट्े क्रलया, चीन, भारत, जापान, न्यूजीलैंि, क ररया गणराज्य, रूस और संयुि राज्य
अमेररका के साथ िस आक्रसयान दे श (िुनेई, कंब क्रिया, इं ि नेक्रशया, लाओस, मलेक्रशया, म्यांमार,
क्रफलीपी ंस, क्रसंगापुर, थाईलैंि, क्रवयतनाम)
• बैठक के प्रमुख पररणाम: भाग लेने वाले काउं हटयों ने दक्रक्षण चीन सागर की क्तस्थक्रत पर क्रचंता व्यि की,
नेक्रवगेशन और ओवरफ्लाइट की स्वतंत्रता के मित्व औि अंतििाष्टरीय र्िर्ध के अनुसाि क्रववाद ं के शांक्रतपूणा
समाधान का उल्लेख क्रकया।

अंतरसरकारी वाताा • यूएनजीए अध्यक्ष ने किा हक संयुि राष्ट्् अंतरसरकारी वाताा की रूपिे खा बैठकों का पिला खंड अब वेबकास्ट
रूपरे खा हकया गया िै ।
• यि संयुि िाष्टर के भीति संयुि राष्ट्् सुरक्षा पररषद (यूएनएससी) में सुधार के क्रलए काम करने वाले राष्ट््-
राज्य ं का एक समूि िै।
• संरचना: आईजीएन कई अलग-अलग अंतििाष्टरीय संगठनों से बना िै :
 अफ़्रीकी संघ, G4 िाष्टर (भाित, िाजील, जापान औि जमानी), यूनाइहटं ग फ़ॉि कंसेंसस ग्रुप (UfC), L69 ग्रुप
ऑफ़ डे वलहपंग कंटर ीज, अिब लीग, कैिे हबयन कम्युहनटी (CARICOM)।

शिरी 20 (U20) • िाल िी में, शिरी 20 मेयरल क्रशखर सम्मेलन गांधीनगर में आयोहजत हकया गया था।
मेयरल क्रशखर • मेजबान: आवास और शिरी मामल ं का मंत्रालय, िाष्टरीय शििी मामलों के संस्थान (एनआईयूए), यू20
सम्मेलन तकनीकी सहचवालय के सियोग से।
• क्तस्थिता की हिशा में कायों पि हवचाि-हवमशा किने के हलए G20 दे श ं के शिर ं के नेताओं क एक साथ लाया।

यूक्रनवसाल प स्टल यूक्रनवसाल प स्टल यूक्रनयन (यूपीयू), ग्ल बल प स्टल नेटवका का इस्तेमाल करते हुए सीमा पार प्रेषण के हलए
यूक्रनयन (यूपीयू) एकीकृत भुगतान इं टिफेस (यूपीआई) का आकलन किे गा।
यूपीयू के बारे में:

30
• यि संयुि राष्ट्् की एक क्रवशेष एजेंसी िै, क्रजसकी स्थापना 1874 में हुई थी औि मुख्यालय बना
(क्तस्वट् जरलैंि) में िै, यि िु हनया भि में िू सिा सबसे पुिाना अंतिाा ष्टरीय संगठन िै ।
• सदस्य: 192 सिस्य िे श
 संयुि िाष्टर का कोई भी सिस्य िे श यूपीयू का सिस्य बन सकता िै ।
 द -क्रतिाई सदस्य ं द्वारा अनुम क्रदत ि ने पि गैि-संयुि िाष्टर िे श यूपीयू में शाहमल िो सकते िैं ।
 भारत 1876 में यूपीयू में शाक्रमल हुआ था।
• यूक्रनफाइि पेमेंट्स इं टरफेस (UPI) के बारे में: यि भाित की मोबाइल-आिारित भुगतान प्रर्ाली िै औि लोगों
को वचुाअल भुगतान पते के माध्यम से चौबीसों घंटे भुगतान किने की अनुमहत िे ती िै।

भारत-इं ि नेक्रशया भाित औि इं डोनेहशया ने "भारत-इं ि नेक्रशया आक्रथाक और क्रवत्तीय वाताा" शुरू करने की घ षणा की िै।
ईएफिी आक्रथाक और क्रवत्तीय संवाद (ईएफिी) के बारे में :
• मित्व: ईएफिी संवाद भारत और इं ि नेक्रशया के बीच हद्वपक्षीय संबंिों को गििा किे गा, साथ िी दक्रक्षण पूवा
एक्रशया और दु क्रनया की व्यापक आहथाक औि हविीय क्तस्थिता में भी योगिान िे गा।

ग्ल बल साउथ • यूिेन में युद् पि, नाटो के साथ खडे िोने के हलए अरीका, एहशया औि लैहटन अमेरिका के कई प्रमुख िे शों की
अहनच्छा ने "ग्लोबल साउथ" शब्द को सामने लाया िै ।
ग्ल बल साउथ के बारे में:
• ग्लोबल साउथ िु हनया भि के हवहभन्न िे शों क संदक्रभात करता िै क्रजन्हें कभी-कभी 'क्रवकासशील', 'कम
क्रवकक्रसत' या 'अक्रवकक्रसत' के रूप में वक्रणात क्रकया जाता िै।
• क्षेत्रीय प्रसार: वैहश्वक िहक्षर् के कई िे श िहक्षर्ी गोलािा में िैं , मुख्यतः अरीका, एहशया औि लैहटन अमेरिका
में।
वैक्रश्वक उत्तर:
• उत्तरी अमेररका औि यूर प में क्तस्थत िैं , ओहशहनया औि अन्य जगिों पि कुछ अन्य िैं।

31
3. अथा व्य वस्था
3.1 रुपये का अं त राा ष्ट्् ीयकरण

सोंिर्ि:
भारतीि ररजिन बैंक (RBI) के अयिकाररिदों के एक अोंिर र्िर्ागीय समूह (आईडीिी) ने एक ररप टि में आगाह र्कया है र्क रुपये के
अोंिराििरीयकरर् के पररणामस्वरूप र्ुरुआती दौर में रुपिे की यियनमि दर में अल्कस्र्रता बढ सकती है।
अोंिर-र्िर्ागीय समूह (आईडीिी) के बारे में:
• अोंिर-र्िर्ागीय समूह (आईडीिी) का गठन आरबीआई के यडप्टी गिननर द्वारा यकिा गिा र्ा।
• उद्दे श्य:
अिीि में रुपये का अोंिराििरीयकरर्:
 अोंतरान ष्टरीि मुिा के रूप में रुपिे की ल्कस्र्यत की समीक्षा करना
• 1970 के िशक की र्ुरुआत तक भारतीि रुपिा खाड़ी िे श ों
 घरे िू मुिा के अोंतरान ष्टरीिकरण के यिए एक रदड मैप तैिार करना। (कुिैि, बहरीन, किर, सोंयुक्त अरब अमीराि) में िैध मुद्रा था।
आईडीिी की र्सफाररशें: • 1959 में , आरबीआई ने खाडी क्षेत्र के यिए यिर्ेष नदट जारी यकए
• र्िशेष आहरर् अर्धकार (एसडीआर) ट करी में रुपये क शार्मि र्िन्हें खाड़ी रुपया या बाहरी रुपया कहा िािा था, यजसका
मूल्य भारतीि रुपिे के समान हदता र्ा।
करना, जद सदस्य दे र्दों के आयिकाररक भोंडार के पूरक के यिए
आईएमएफ द्वारा बनाई गई एक अोंतरराष्टरीि आरयक्षत सोंपयि है ।
• र्ारिीय रुपये के नदट हज िात्रा पर िे जा सकते र्े और उन्हें
सऊदी ररिाि से बदि सकते र्े; बाद में, यिर्ेष "हज नदट् स" पेर्
 एसडीआर का मूल्य पाोंच मुिाओों की एक टदकरी पर आिाररत है -
यकए गए।
अमेररकी डॉिर, िूरद, चीनी रॅ ल्कन्मन्बी, जापानी िेन और यियटर् पाउों ड
• 1966 में र्ारिीय मुद्रा का अिमूल्न र्कया गया, यजसके कारण
स्टयििंग। खाडी दे र्दों ने खाडी रुपिे का उपिदग बोंद कर यदिा।
• रुपिे के अोंतरान ष्टरीिकरण की गयत में तेजी िाने के यिए र्ििे शी • 1970 के िशक की शुरुआि तक, सभी खाडी दे र्दों ने खाडी
प टि ि र्िय र्निेशक (एिपीआई) व्यिस्था क सुदृढ़ करना। रुपिे कद अपनी मुिा के रूप में उपिदग करना बोंद कर यदिा।
• समकक्ष दे र्दों की रुपये और स्थानीय मुद्राओों में र्नपटान और
र्ुगिान के यिए यद्वपक्षीि और बहुपक्षीि व्यापार समझौतदों/व्यिस्र्ाओों से जुडे प्रस्तािदों की जाोंच के यिए एक मानकीकृि दृर्िक र्
अपनाना।
• रुपिे का अोंतरान ष्टरीिकरण करने के यिए एर्शयाई िीयररों ग यूर्नयन (एसीयू) िैसे मौिूिा र्द्वपक्षीय और बहुपक्षीय र्ुगिान और
र्नपटान िोंि का उपय ग।
 एसीिू एक भुगतान व्यिस्र्ा है यजसके तहत प्रयतभागी र्ुद्ध बहुपक्षीि आिार पर भाग िेने िािे केंिीि बैंकदों के बीच अोंतर-क्षेत्रीि िेनदे न
के यिए भुगतान का यनपटान करते हैं ।
• मुिा के अोंतरान ष्टरीिकरण कद सुयििाजनक बनाने के यिए गैर-र्निार्सय ों क रुपया खािा ख िने की अनुमर्ि िे ना।
 एनआरआई (अयनिासी भारतीि) के यिए एक रुपिा खाता एक भारतीि नागररक द्वारा रखे गए बैंक खाते कद सोंदयभनत करता है जद
रदजगार, व्यिसाि िा अन्य उद्े श्यदों के यिए भारत से बाहर खदिा जाता है ।
• अोंतरराष्टरीि िेनदे न के यनपटान के यिए ररयि-टाइम ग्रॉस सेटिमेंट (आरटीिीएस) प्रर्ािी का र्िस्तार।
• सीएिएस प्रणािी में रुपये क शार्मि करना।
 सीएिएस प्रणािी एक यिश्वव्यापी यििीि नेटिकन है जद यिदे र्ी मुिा (एफएक्स) िेनिे न क सुरर्क्षि और िेि बनािा है।
 इसे एफएक्स बाजार में यििोंयबत भुगतान के जदल्कखम कद कम करने और िैयश्वक यििीि प्रणािी की ल्कस्र्रता में सुिार करने के यिए बनािा
गिा र्ा।
मुद्रा अोंिराििरीयकरर्:
• एक अोंतरान ष्टरीि मुिा का उपिदग जारीकतान दे र् की सीमाओों से परे यकिा जाता है और िह यियनमि के माध्यम, खाते की इकाई और मूल्य
के भोंडार के रूप में बुयनिादी कािन कर सकता है ।
 प्रमुख मुद्रा: USD> िूरद> जापानी िेन> पाउों ड स्टयििंग।
रुपये के अोंिराििरीयकरर् के बारे में:
• िह भारतीि मुिा के बढ़िे सीमा पार िेनिे न की प्रर्क्रया क सोंिर्र्िि करिा है , यिर्ेष रूप से आयाि-र्नयािि व्यापार में इसके बाद
अन्य चािू खाता िेनदे न और यफर पूोंजी खाता िेनदे न।
• इससे यिदे र्ी व्यापार में अमेररकी डॉिर सयहत अन्य मुिाओों के यिपरीत भारतीि रुपिे में व्यापार का अोंतरराष्टरीि यनपटान सोंभि हद सकेगा।
32
अोंिराििरीयकरर् की प्रर्क्रया:
• मुद्रा व्यापार की स्विोंििा: मुिा खरीद और यबक्री पर प्रयतबोंि हटाना।
• घरे िू मुद्रा में िािान: घरे िू और यिदे र्ी कोंपयनिदों कद उनकी मुिा में यनिानत का
चािान करने की अनुमयत दे ना।
• र्ििे शी स्वार्मत्व और ह म्भडोंग्स: यिदे र्ी सोंस्र्ाओों कद दे र् की मुिा और यििीि
उपकरण रखने की अनुमयत दे ना।
• र्िपर्न य ग्य र्िखि ों का र्ििे शी र्नगिम: यिदे र्ी फमों और सोंस्र्ानदों कद दे र्
की मुिा में इल्किटी और ऋण यिखत जारी करने की अनुमयत दे ना।
• र्ििे शी बाजार ों पर घरे िू र्नगिम: घरे िू सोंस्र्ानदों कद यिदे र्ी बाज़ारदों में अपनी
मुिा में यिखत जारी करने में सक्षम बनाना।
• अोंिराििरीय र्ििीय सोंस्थान ों का सोंिािन: अोंतरान ष्टरीि यििीि सोंस्र्ानदों कद ऋण
यिखत जारी करने और अपने सोंचािन में दे र् की मुिा का उपिदग करने की
अनुमयत दे ना। रुपये के अोंिराििरीयकरर् की र्िशा में प्रयास:
• मुद्रा बास्केट समािेशन: दे र् की मुिा कद अन्य दे र्दों की मुिा • आरबीआई ने र्ारिीय मुद्रा में र्ििे शी व्यापार के र्नपटान की
बास्केट में र्ायमि करना। अनुमर्ि िी: आरबीआई ने भागीदार दे र्दों के बीच यिदे र्ी व्यापार
 आरर्क्षि मुद्रा: ितनमान में, अमेररकी डॉिर, िूरद, जापानी िेन कद भारतीि मुिा में यनपटाने की अनुमयत दे दी है ।
और यियटर् पाउों ड दु यनिा में प्रमुख आरयक्षत मुिाएों हैं ।  आरबीआई ने यिर्ेष रुपिा िदस्टर द खातदों (एसआरिीए) के
रुपये के अोंिराििरीयकरर् के िार्: माध्यम से बाहरी व्यापार के रुपिे में यनपटान की अनुमयत दी
है । एक िदस्टर द खाता यकसी दू सरे दे र् के बैंक की ओर से चिािा
• मुद्रा ि म्भखम क कम करिा है: सीमा पार िेनदे न में रुपिे का
जाता है
उपिदग भारतीि व्यिसािदों कद मुिा जदल्कखम का प्रबोंिन करने में मदद
 कुि 18 दे र्दों कद एसआरिीए खदिने की अनुमयत दी गई है ।
करता है । मुिा की अल्कस्र्रता से सुरक्षा द्वारा व्यापार करने की िागत
• एफटीपी 2023: हार्िया र्ििे श व्यापार नीर्ि (एफटीपी)
कम हद जाती है और िैयश्वक स्तर पर बेहतर व्यापार िृल्कद्ध की सुयििा
2023, भारतीि रुपिे में व्यापार के चािान, भुगतान और यनपटान
यमिती है । का प्रस्ताि करती है ।
• र्ििे शी मुद्रा र्ोंडार की आिश्यकिा कम ह िािी है: रुपिे के • रुपये की व्यिस्था का र्िस्तार: भारत की भूटान और नेपाि के
अोंतरान ष्टरीिकरण से यिदे र्ी मुिा भोंडार पर यनभनरता कम हद जाती है । सार् रुपिे की व्यिस्र्ा र्ी और हाि ही में श्रीिोंका ने औपचाररक
िह भारत कद बाहरी झटकदों के प्रयत कम सोंिेदनर्ीि बनाता है और रूप से रुपिे कद एक यनयदन ष्ट यिदे र्ी मुिा के रूप में र्ायमि करने
ल्कस्र्रता कद बढाता है । का यनणनि यििा।
• सौिे बािी की शम्भक्त में सुधार: जैसे-जैसे रुपिे का उपिदग • BHIM UPI QR की सीमा पार स्वीकृर्ि: एनआईपीएि
महत्वपूणन हदता जाता है , भारतीि व्यिसािदों कद अोंतरान ष्टरीि िेनदे न में (एनपीसीआई इों टरनेर्नि पेमेंट्स यियमटे ड), नेर्नि पेमेंट्स
बेहतर सौदे बाजी की र्ल्कक्त यमिती है । कॉरपदरे र्न ऑफ इों यडिा (एनपीसीआई) की पूणन स्वायमत्व िािी
सहािक कोंपनी ने व्यापाररक प्रयतष्ठानदों पर भीम िूपीआई क्ूआर
• व्यापार और र्निेश क सुगम बनािा है: रुपिे की अोंतरान ष्टरीि
की सीमा पार स्वीकृयत कद सक्षम करने के यिए राष्टरदों में यियभन्न पहि
स्वीकृयत सीमा पार व्यापार और यनिेर् िेनदे न कद सरि बनाती है ,
की है ।
यजससे भारतीि व्यिसािदों के यिए अोंतरान ष्टरीि बाजारदों तक पहुोंच और
• ितनमान में, BHIM UPI QR कद र्सोंगापुर, यूएई, मॉरीशस, नेपाि
यिदे र्ी यनिेर् आकयषनत करना आसान हद जाता है । और र्ूटान में स्वीकृर्ि र्मि गई है।
• र्ििीय एकीकरर् क बढ़ािा: यिश्व स्तर पर स्वीकृत रुपिा यििीि
एकीकरण कद बढािा दे ता है और यिदे र्ी यनिेर्कदों कद भारतीि यििीि बाजारदों की ओर आकयषनत करता है , यजससे पूोंजी प्रिाह और यनिेर्
के अिसर बढते हैं ।
• अमेररकी डॉिर की घटिी र्हस्सेिारी: यद्वपक्षीि और क्षेत्रीि आयर्नक सहिदग समझौते िैयश्वक यििीि प्रणािी का बहु-ध्रुिीकरण कर रहे
हैं । व्यापार चािान और यनपटान में गैर-अमेररकी डॉिर का उपिदग बढ रहा है ।
• र्ू-रािनीर्िक कारक: रूस पर िगाए गए प्रयतबोंिदों ने दे र्दों कद सोंभायित भयिष्य के प्रयतबोंिदों के बारे में सतकन कर यदिा है। इसने दे र्दों कद
अमेररकी डॉिर-प्रभुत्व िािी िैयश्वक मुिा प्रणािी पर सिाि उठाने के यिए प्रेररत यकिा है ।
मुद्रा के आों िररककरर् से िुड़ी िुनौर्ियााँ:
• मुद्रा अोंिराििरीयकरर् के कारर् बाहरी झटके बढ़ सकिे हैं: दे र् के अोंदर और बाहर और एक मुिा से दू सरे मुिा में िन के प्रिाह के खुिे
चैनि कद दे खते हुए, यकसी मुिा का अोंतरान ष्टरीिकरण बाहरी झटके क बढ़ा सकिा है।

33
• र्हि ों का टकराि: अल्पकार्िक घरे िू और िीघिकार्िक
अोंिराििरीय आर्थिक उद्े श्यदों के बीच यहतदों का टकराि हद सकता है ।
• बाजार की गहराई और िरििा: र्ारिीय रुपया बाजार की
गहराई और तरिता अभी तक अमेररकी डॉिर िा िूरद जैसी प्रमुख
िैयश्वक मुिाओों से तुिनीि नहीों है ।
• र्नयामक ढाोंिा: भारत में यिदे र्ी मुिा िेनदे न, पूोंजी प्रिाह और यििीि
बाजारदों कद यनिोंयत्रत करने िािे यनिामक ढाोंचे कद रुपिे के
अोंतरान ष्टरीिकरण का समर्नन करने के यिए और अयिक उदार और
सुव्यिल्कस्र्त करने की आिश्यकता है ।
• आर्थिक म्भस्थरिा: र्ारिीय अथिव्यिस्था की म्भस्थरिा, र्िसमें
मुद्रास्फीर्ि िर, रािक षीय अनुशासन और समग्र आर्थिक
र्िकास जैसे कारक र्ायमि हैं , अोंतरराष्टरीि बाजारदों में रुपिे की
यिश्वसनीिता और आकषनण कद बढाने में महत्वपूणन भूयमका यनभाती
है ।
• र्निेशक ों का र्ििास: राजनीयतक ल्कस्र्रता, नीयत पूिान नुमान और
यनिेर्क-अनुकूि सुिार जैसे कारक िैयश्वक यनिेर्कदों के यिए
अनुकूि माहौि बनाने में िदगदान करते हैं ।

आगे की राह:
• रुपये की पररिििनीयिा: पूणन पररितननीिता के िक्ष्य के सार् रुपिे
कद अयिक स्वतोंत्र रूप से पररितननीि बनािा जाना चायहए, यजससे
यििीि यनिेर् भारत और यिदे र्दों के बीच स्वतोंत्र रूप से स्र्ानाों तररत
हद सके।
• िरि रुपया बाोंड बािार पर ध्यान िे ना: आरबीआई कद एक गहरे
और अयिक िरि रुपया बाोंड बािार क आगे बढ़ाना चायहए, रुपये की पररिििनीयिा का अर्न है िस्तुओों और सेिाओों में व्यापार के
यिए भारतीि मुिा (रुपिे) का यिदे र्ी मुिा में और यिदे र्ी मुिा का
यजससे यिदे र्ी यनिेर्कदों और भारतीि व्यापार भागीदारदों कद रुपिे में
भारतीि मुिा में स्वतोंत्र रूप से यियनमि करने की क्षमता। इसके दद
अयिक यनिेर् यिकल्प यमि सकें, यजससे इसका अोंतरराष्टरीि उपिदग
प्रकार हैं
सोंभि हद सके।
• िािू खािा पररिििनीयिा: िह िस्तुओों और सेिाओों के व्यापार के
• रुपये में िेनिे न क प्र त्सार्हि करना: भारतीि यनिान तकदों और यिए रुपिे कद स्विोंि रूप से यिदे र्ी मुिा में पररियतनत करने की
आिातकदों कद रुपिे में अपने िेनदे न का यबि बनाने के यिए अनुमयत दे ता है ।
प्रदत्सायहत यकिा जाना चायहए - रुपिे के आिात/यनिान त िेनदे न के • पूोंिी खािा पररिििनीयिा: िह रुपिे कद यिदे र्ी यनिेर् और
यिए व्यापार यनपटान औपचाररकताओों कद अनुकूयित करने से काफी यििीि पररसोंपयििदों जैसे पूोंजी प्रिाह और बयहिानह के यिए आों र्शक
मदद यमिेगी। रूप से पररिििनीय बनाने की अनुमर्ि िे िा है।
• मुद्रा र्िर्नमय समझौिे: अयतररक्त मुिा यियनमि समझौते (जैसा यक श्रीिोंका के सार्) भारत कद डॉिर जैसी आरयक्षत मुिा का सहारा यिए
यबना, रुपिे में व्यापार और यनिेर् िेनदे न का यनपटान करने की अनुमयत दें गे।
• कर प्र त्साहन: इसके अयतररक्त, भारत में पररचािन में रुपिे का उपिदग करने के यिए यिदे र्ी व्यिसािदों कद कर प्रदत्साहन से भी मदद
यमिेगी।

3.2 बै ट री इिे म्भरर क िाहन


सोंिर्ि:
दु यनिा के र्ुद्ध-र्ून्य उत्सजनन में पररितनन के मद्े नजर हाइयिड इिेल्कररक िाहन यिकासर्ीि दे र्दों के यिए एक बडा अिसर पेर् करते हैं।
ई-िाहन ों के बारे में: • शुद्ध-शून्य िक्ष्य से िात्पयि उस ल्कस्र्यत से है यजसमें यकसी िे श के
• ई-िाहन ऐसे िाहन हैं जद जीिाश्म ईोंिन पर चिने िािे उत्सििन की र्रपाई िािुमोंडि से ग्रीनहाउस गैस ों के अिश षर् और
आों तररक दहन इों जन (आईसीई), के बजाि यबजिी से चिते यनष्कासन द्वारा की जाती है ।
हैं ।

34
• िे िाहन कद चिाने और यियभन्न घटकदों कद र्ल्कक्त प्रदान करने के यिए इिेल्कररक मदटर और ररचाजेबि बैटरी का उपिदग करते हैं ।
ईिी के प्रकार:
• बैटरी इिेम्भरर क िाहन (बीईिी): बीईिी तकनीक का उपिदग करने िािे इिेल्कररक िाहन पूरी तरह से बैटरी चायित इिेल्कररक डर ाइिटर े न
पर चिते हैं । उिाहरर् के र्िए: टाटा नेक्सन
• हाइर्िड इिेम्भररक िाहन (एिईिी): एचईिी में इों जन कद ईोंिन से ऊजान यमिती है , और मदटर कद बैटरी से यबजिी यमिती है । उिाहरर्
के र्िए: टदिदटा के िेररएों ट
• प्लग-इन हाइर्िड इिेम्भरर क िाहन (पीएिईिी): पीएचईिी में, कदई भी ईोंिन, पारों पररक ईोंिन (जैसे पेटरदि) िा िैकल्कल्पक ईोंिन (जैसे
बािदडीजि) में से चुन सकता है। उिाहरर् के र्िए: टदिदटा यप्रिस
• ईोंधन सेि इिेम्भररक िाहन (एिसीईिी): एफसीईिी कद र्ून्य-उत्सजनन िाहन के रूप में भी जाना जाता है । िे िाहन चिाने के यिए
आिश्यक यबजिी उत्पन्न करने के यिए 'ईोंिन सेि प्रौद्यदयगकी' का उपिदग करते हैं । उिाहरर् के र्िए: ई-ररक्शा
इिेम्भरर क िाहन ों के िार्:
• पयाििरर्ीय प्रर्ाि: इिेल्कररक िाहन र्ून्य टे िपाइप उत्सजनन उत्पन्न
करते हैं , यजसका अर्न है यक िे ऑपरे र्न के दौरान हिा में काबनन
डाइऑक्साइड (सीओ2), नाइटर दजन ऑक्साइड (एनओएक्स), िा
पीएम जैसे प्रदू षक नहीों छदडते हैं ।
• ऊिाि िक्षिा: इिेल्कररक मदटर आों तररक दहन इों जन की तुिना में
अयिक कुर्ि हदते हैं, जद बैटरी से ऊजान के उच्च प्रयतर्त कद
प्रणददन में पररियतनत करते हैं।
• निीकरर्ीय ऊिाि एकीकरर्: सौर िा पिन ऊजान जैसे
निीकरणीि ऊजान स्रदतदों से यबजिी का उपिदग करके इिेल्कररक
िाहनदों कद चाजन यकिा जा सकता है ।
• उपर् क्ताओों क सरकारी प्र त्साहन: कई सरकारें इिेल्कररक
िाहनदों कद अपनाने कद प्रदत्सायहत करने के यिए टै क्स क्रेयडट, छूट
िा सल्किडी जैसे प्रदत्साहन प्रदान करती हैं । िे प्रदत्साहन ईिी खरीदने
की अयग्रम िागत कद कम करने और उन्हें अयिक यकफािती बनाने
में मदद कर सकते हैं ।
• ऊिाि स्विोंििा: इिेल्कररक िाहनदों कद व्यापक रूप से अपनाने से
आिायतत जीिाश्म ईोंिन पर यनभनरता कम हद जाती है ।
इिेम्भरर क िाहन ों से िुड़ी िुनौर्ियााँ:
• उच्च क्रय िागि: 2 और 4 पयहिा िाहनदों के इिेल्कररक िेररएों ट की
कीमत अक्सर यनियमत ईोंिन यिकल्पदों की तुिना में बहुत अयिक
हदती है ।
• उच्च रखरखाि िागि: मुि रूप से आिश्यक सुर्िधाओों की
कमी के कारण रखरखाि िागत अयिक है।
 उिाहरर् के र्िए: भारत में 65,000 से अयिक पेटरदि पोंप हैं िेयकन केिि 1640 ईिी चायजिंग स्टे र्न हैं ।
• अपयािप्त बुर्नयािी ढााँिा: हमारे दे र् में पिान प्त चायजिंग बुयनिादी ढाँचे की कमी ईिी की पैठ बढाने में एक बडी बािा है ।
• कम पुनर्ििक्रय मूल्: ितनमान में, ईिी बाजार स्वतोंत्र डीिरयर्प के सार् यिभायजत है , यजससे सेकेंड-हैं ड यबक्री के यिए उयचत बुयनिादी
ढाों चा तैिार करना मुल्किि हद जाता है ।
• उपिब्धिा की कमी: भारत में अब तक ईिी केिि पहिे से उपिब्ध जीिाश्म-ईोंिन चायित 2 और 4 पयहिा िाहनदों के िेररएों ट हैं ।
 टे स्ला जैसे उच्च प्रदर्नन िािे िक्जरी िेररएों ट िा सुपरकार अभी भी भारतीि बाजारदों में नहीों आए हैं ।
• सीर्मि सोंसाधन: ऐसा अनुमान है यक यियर्िम उत्पादन कद बढाना एक चुनौती हदगी, यजससे आपूयतन में कमी हदगी यजसके कारण यनमानता
कम गुणििा िािे खयनज इनपुट का उपिदग कर सकते हैं , जद ईिी के बैटरी प्रदर्नन पर प्रयतकूि प्रभाि डािेगा।
• र्बििी आपूर्िि: गाों िदों और छदटे र्हरदों तर्ा यदल्ली और एनसीआर जैसे महानगरदों में यनरों तर यबजिी आपूयतन की कमी के कारण िगातार
घोंटदों यबजिी गुि रहती है ।
35
आगे की राह:
• सरकारी नीर्ियाों और प्र त्साहन: भारत सरकार कद सहािक ई-िाहन बाजार की सोंर्ािनाएाँ :
नीयतिदों कद िागू करना जारी रखना चायहए यजसमें सल्किडी, • नीर्ि आय ग के अनुमान के मुतायबक, 2030 तक भारत में 80%
कर क्रेयडट और छूट जैसे यििीि प्रदत्साहन और ईिी यियनमान ण ि पर्हया और र्िपर्हया िाहन, 40% बसें और 30 से 70% कारें
कद बढािा दे ने िािी नीयतिाों र्ायमि हद सकती हैं । इिेम्भररक िाहन ह ग ों ी।
• िार्ििंग इों फ्रास्टर क्चर र्िकास: ईिी के यिकास कद समर्नन दे ने • एक हायििा अध्यिन के अनुसार, 2025 तक इिेम्भररक िाहन ों (ईिी) का
के यिए एक मजबूत और व्यापक चायजिंग इों फ्रास्टर क्चर नेटिकन बािार कम से कम ₹475 र्बर्ियन का हदने की उम्मीद है ।
स्र्ायपत करना महत्वपूणन है । • इिेल्कररक ददपयहिा िाहनदों की पहुोंच ितन मान में 1% से 2025 िक 15%
• सािििर्नक पररिहन र्िि् युिीकरर्: इिेल्कररक बसदों, िक पहुोंिने का अनुमान है।

टै ल्कक्सिदों और सािनजयनक पररिहन के अन्य रूपदों कद अपनाने • ऐसी उम्मीद है यक ईिी उद्यदग में 2030 तक 5 कर ड़ प्रत्यक्ष और
अप्रत्यक्ष र िगार सृ र्िि ह ग ों े।
कद प्रदत्सायहत करने से उत्सजनन कद कम करने और ईिी कद
• घरे िू इिेम्भररक िाहन (ईिी) बािार के 2022 और 2030 के बीि 49%
बढािा दे ने पर महत्वपूणन प्रभाि पड सकता है ।
की िक्रिृम्भद्ध िार्षिक िृम्भद्ध िर (सीएिीआर) से बढने की उम्मीद है और
• घरे िू र्िर्नमािर् और आपूर्िि श्ृोंखिा: ईिी और उनके घटकदों
2030 तक िायषनक यबक्री एक करदड िूयनट तक पहुों चने की उम्मीद है।
के घरे िू यियनमान ण कद बढािा दे ने से स्र्ानीि अर्नव्यिस्र्ा कद • इस क्षेत्र के महत्व का अोंदाजा इस तथ् से िगािा जाता है यक िह 2021
बढािा यमि सकता है , रदजगार के अिसर पैदा हद सकते हैं के अोंत में 3.7 करदड का प्रत्यक्ष और अप्रत्यक्ष रदजगार पैदा करते हुए
और आिात पर यनभनरता कम हद सकती है । समग्र सकि घरे िू उत्पाि में 7.1% और र्िर्नमािर् सकि घरे िू उत्पाि
• अनुसोंधान और र्िकास: ईिी प्रौद्यदयगकी पर केंयित में 49% का य गिान िे िा है।"
अनुसोंिान और यिकास पहि में यनिेर् करना और उद्यदग,
यर्क्षा और अनुसोंिान सोंस्र्ानदों के बीच सहिदग स्वदे र्ी
प्र. भारत में बैटरी इिेल्कररक िाहनदों के सामने आने िािी चुनौयतिदों पर चचान
प्रौद्यदयगयकिदों कद यिकयसत करने और भारत की अनूठी कीयजए और उन्हें अपनाने कद बढािा दे ने के यिए कौन-सी सरकारी पहि
पररल्कस्र्यतिदों से सोंबोंयित यियर्ष्ट चुनौयतिदों का समािान करने के िागू की गई हैं ?
यिए महत्वपूणन है।
• र्नयामक सुधार: ईिी से सोंबोंयित यनिमदों और नीयतिदों की िगातार समीक्षा और अद्यतन करने से एक अनुकू ि पाररल्कस्र्यतकी तोंत्र बनाने में
मदद यमि सकती है ।

3.3 ऑनिाइन गे र्मों ग


सोंिर्ि:
हाि ही में, िस्तु एिों सेिा कर (िीएसटी) पररषि ने ऑनिाइन गेर्मोंग, कैसीन और घुड़िौड़ के यिए पूणन अोंयकत मूल्य पर एक समान 28
प्रर्िशि कर िगाने का र्नर्िय र्िया।
ऑनिाइन गेर्मोंग के बारे में:
• ऑनिाइन गेयमोंग से तात्पिन उन खेिदों से है जद केिि इों टरनेट कनेक्शन के साथ ही खेिे िा सकिे हैं।
• ऑनिाइन गेर्मोंग के प्रकार:
 ई-स्प ट्ि स: सुव्यिल्कस्र्त इिेररॉयनक खेि यजसमें पेर्ेिर ल्कखिाडी र्ायमि हदते हैं ।
 काल्पर्नक खेि: िास्तयिक जीिन के खेि ल्कखिायडिदों कद चुनना और ल्कखिायडिदों के प्रदर्नन के आिार पर अोंक जीतना।
 कौशि आधाररि: मानयसक कौर्ि।
 सोंर्ािना आधाररि: पासा पिटने जैसी िादृल्किक गयतयियि पर आिाररत ऑनिाइन गेम।

कौशि का खेि मौका या र्कस्मि का खेि

• र्सद्धाोंि दृर्िक र्: जीतने िा जीतने की सोंभािना बढाने के यिए • िे गेम भाग्य पर आिाररत हैं िहाों िीिने की सोंर्ािना र्कसी
व्यल्कक्त के पास खेि का आिश्यक अनुर्ि और व्यापक ज्ञान म्भखिाड़ी के कौशि, अनुभि और/िा ज्ञान से नहीों बढाई जा
ह ना आिश्यक है। सकती।
• उिाहरर्: र्तरों ज, कैरम, रम्मी, तीन पिी, घुडदौड और • उिाहरर् ों में बैकारे ट, स्लॉट गेम, िॉटरी और टे बि गेम जैसे
काल्पयनक खेि कौर्ि के खेि कहिाते हैं। ब्लैकजैक, तीन पिी और रूिेट र्ायमि हैं।

36
र्ारि की ऑनिाइन गेर्मोंग क्षमिा
• सनराइि सेरर: भारतीि गेयमोंग बाजार 2022 में 2.8 र्बर्ियन डॉिर से बढ़कर
2025 में 5 र्बर्ियन डॉिर िक पहुोंिने की उम्मीि है, ि 28-30% की
सीएिीआर से बढ़ रहा है।
• िे श में गेमसि की सोंिा 2022 में 420 यमयििन से बढकर 2023 िक 450
र्मर्ियन और 2025 िक 500 र्मर्ियन िक पहुोंिने की उम्मीि है।
• रािस्व सृिन: करािान और यियनिमन के माध्यम से, ऑनिाइन गेयमोंग में भारत
सरकार के यिए बड़ी मािा में धन िाने की क्षमिा है ।
 गेयमोंग सेरर ने 2020 से 2022 के बीि िीडीपी में अपना य गिान 0.1% से
बढ़ाकर 0.5% कर र्िया है।
• र िगार: भारत में गेयमोंग उद्यदग ने 40,000 से अर्धक कररयर अिसर पैिा र्कए
हैं और 2025 तक िह सोंख्या बढकर 2,50,000 हदने की उम्मीद है ।
• भारत में ऑनिाइन गेयमोंग का पहिा उत्कृििा केंद्र र्शिाोंग में स्र्ायपत यकिा गिा
है ।
र्ारि में ऑनिाइन गेर्मोंग के र्िकास के कारर्:
• स्माटि ि न की पैठ: र्िि आर्थिक मोंि (डब्ल्यूईएि) के अनुसार, मदबाइि
यडिाइस भारत के गेयमोंग उद्यदग का प्रार्यमक प्रेरक हैं ।
 भारत में ितनमान में 600 र्मर्ियन से अर्धक स्माटि ि न उपय गकिाि हैं और
2026 िक 1 र्बर्ियन स्माटि ि न उपय गकिािओों िक पहुोंिने की उम्मीि
है।
• इों टरनेट की सामर्थ्ि और उपिब्धिा: भारत एक डे टा-समृद्ध बाजार है , जहाों
िैयश्वक औसत की तुिना में मदबाइि डे टा सस्ता है ।
• स्थानीयकृि सामग्री: िीन पिी, िूड र्कोंग और अन्य िदकयप्रि गेम र्होंिी, गुिरािी, मराठी आयद जैसी स्र्ानीि भाषाओों में गेम खेिने
के यिकल्प प्रदान करते हैं , इस प्रकार एक बड़े स्थानीय िशिक ों क िर्क्षि करिे हैं।
• युिा आबािी: भारत में युिा आबािी में गेयमोंग कद अयिक अपनािा जाता है।
 भारत की 50% से अर्धक िनसोंिा 25 िषि से कम आयु की है और 65% से अयिक जनसोंख्या 35 िषन से कम आिु की है ।
• र्डर्िटि र्ुगिान क अपनाना: इसने गेम में यडयजटि खरीदारी के यिए परे र्ानी कम कर दी है और गेयमोंग पदटन ि पर ऑनिाइन भुगतान
में तेजी से िृल्कद्ध में िदगदान यदिा है ।
 भारतीि ररज़िन बैंक के अनुसार, यडयजटि भुगतान िेनदे न यिि िषन 2017-18 में 2,071 करदड से बढ़कर र्िि िषि 2021-22 में 8,840
कर ड़ ह गया है।
• ई-स्प ट्ि स के र्िए मान्यिा: इसे र्ारिीय ओिोंर्पक सोंघ द्वारा दे र् में ई-स्पदट्न स की र्ासी यनकाि के रूप में ई-स्प ट्ि स िेडरे शन ऑि
इों र्डया की स्र्ापना करके आयिकाररक तौर पर मान्यता दी गई है ।
• र्निेश में िृम्भद्ध: भारत में गेयमोंग कोंपयनिदों ने यपछिे पाोंच िषों में घरे िू और यिदे र्ी यनिेर्कदों से 2.8 र्बर्ियन डॉिर की िोंर्डों ग िुटाई है,
र्िससे उन्हें पररिािन स्तर हार्सि करने में मिि र्मिी है।

ऑनिाइन गेर्मोंग से िुड़ी र्िोंिाएाँ :


• सट्टे बािी और िुआ: पारों पररक िूड पर आिाररत ऑनिाइन गेम यििाददों में रहे हैं और सट्टे बािी और िुए के आर प िगे हैं।
• र्होंसा, यौन सामग्री, र्िोंग या नस्लीय रूर्ढ़िार्ििा िैसी अनुर्िि सामग्री िक पहुाँि, और अन्य ल्कखिायडिदों से अनुयचत व्यिहार हदने का
खतरा।
• साइबरबुर्िोंग और उत्पीड़न: एक अध्यिन में पािा गिा यक अयिकाों र् मयहिा गेमसन कद खेिते समि उत्पीडन का अनुभि हदता है , यजनमें
से अयिकाों र् पीयडतदों की उम्र 18 से 24 िषन के बीच है।
• व्यिहाररक पररिििन: ऑनिाइन गेम आिि बनाने िािे बन सकिे हैं क्दोंयक िे यिर्ेष रूप से उपिदगकतानओों कद बार-बार िौटने के
यिए िुभाने के मकसद से बनाए जाते हैं ।

37
• डीएमके साोंसि टीआर बािू के अनुसार, यपछिे तीन िषों में, ऑनिाइन जुए में बडी रकम हारने के बाद िर्मिनाडु में 40 से अर्धक
ि ग ों ने आत्महत्या कर िी है।
• सामार्िक असोंि ष: जुए की ित के कारण घरे िू र्होंसा, मानर्सक स्वास्थ्य समस्याएों और घरे िू ऋर् की समस्याएों पैिा हुई हैं, उन्हदोंने
कहा यक बच्चे अक्सर ऑनिाइन गेर्मोंग के र्िए नकिी पहिान और अपने मािा-र्पिा के क्रेर्डट काडि का उपय ग करिे हैं।
• िायरस और मैििेयर/रैं समिेयर: है कसन सोंक्रयमत फाइिें डाउनिदड करने िा गेयमोंग फदरम पर बच्चदों/िुिाओों कद सोंक्रयमत िेबसाइटदों पर
जाने के यिए प्रदत्सायहत करने का प्रिास करते हैं।
• ग पनीयिा: व्यल्कक्तगत जानकारी तीसरे पक्ष के सार् साझा की जा सकती है । कुछ गेयमोंग प्लेटफॉमन उपिदगकतान ओों कद अपने गेर्मोंग खाि ों
क अन्य स शि नेटिकि प्लेटफॉमि से ि ड़ने के र्िए प्र त्सार्हि करिे हैं।
,

र्ारि में ऑनिाइन गेर्मोंग का र्िर्नयमन:


• साििी ों अनुसूिी: सोंर्िधान राज् सरकार ों क
साििी ों अनुसूिी की राज् सूिी की प्रर्िर्ि 34 के
िहि सट्टे बाजी और जुए और प्रयियष्ट 33 के तहत खेि
से सोंबोंयित मामिदों पर र्िर्ध बनाने का अयिकार दे ता
है ।
 हािाँयक, राज् कौशि के खेि पर र्िर्ध नही ों
बना सकिा क् र्ों क यह केिि सट्टे बािी, िुआ
और खेि से सोंबोंयित है , जद सोंय ग के खेि के
दािरे में आते हैं ।
• ऑनिाइन गेर्मोंग पर अिाििें: यियभन्न अदाितदों के
फैसिे के अनुसार, कौर्ि के खेि पर सट्टे बाजी
कानूनी है , जबयक मौके के खेि पर सट्टे बाजी कानूनी
नहीों है ।
 सोंर्िधान के अनुच्छेि 19 (1) (िी) और
अनुच्छेि 21 का यहस्सा हैं ।
 राज्य के पास र्िर्ध बनाने और जुए की गयतयियि पर सट्टे बाजी कद यियनियमत करने की शम्भक्त है।
• कानूनी इकाई: कदई भी ऑनिाइन गेयमोंग प्लेटफॉमन - घरे िू िा यिदे र्ी - भारतीि उपिदगकतान ओों कद िास्तयिक पैसे िािे ऑनिाइन गेम
की पेर्कर् करने के यिए र्ारिीय र्िर्ध के िहि र्नगर्मि एक कानूनी इकाई ह ना आिश्यक ह गा।
 ररप र्टिं ग इकाई: इन प्लेटिामों क 2002 के धन श धन र्निारर् अर्धर्नयम के तहत 'ररपदयटिं ग इकाई' के रूप में भी माना जाएगा।
● सूिना प्रौद्य र्गकी (मध्यििी र्िशार्निे श और र्डर्िटि मीर्डया आिार सोंर्हिा) सोंश धन र्नयम, 2023: िह ऑनिाइन गेयमोंग
इकद-यसस्टम के यिए एक व्यापक ढाों चा है ।
 पररर्ाषा: िह एक 'ऑनिाइन गेम' कद "एक ऐसे गेम के रूप में पररभायषत करता है जद इों टरनेट पर पेर् यकिा जाता है और एक
उपिदगकतान द्वारा कोंप्यूटर सोंसािन िा मध्यस्र् के माध्यम से एक्सेस हदने िदग्य है"।
 स्व-र्नयामक र्नकाय (एसआरबी): इसके र्िए िास्तर्िक धन िािे गेर्मोंग प्लेटफॉमि क स्व-र्नयामक र्नकाय (एसआरबी) के
सार् पोंजीकरण करने की आिश्यकता हदती है जद िह यनिान ररत करे गा यक गेम "अनुमेि" है िा नहीों।
 प्रर्िबोंर्धि खेि: िह यकसी भी ऐसे खेि पर प्रयतबोंि िगाता है यजसमें सट्टे बािी और िुआ शार्मि है।
 र्बिौर्िय ों की र्ूर्मका: यबचौयििदों के यिए यकसी भी ऑनिाइन गेम कद हदस्ट, प्रकायर्त िा साझा नहीों करना अयनिािन है जद नुकसान
पहुों चा सकता है िा अनुमयत के अनुसार सत्यायपत नहीों यकिा गिा है ।

र्ारि में सुरर्क्षि ऑनिाइन गेर्मोंग क बढ़ािा िे ने के र्िए सरकारी पहि:


• र्डर्िटि गेर्मोंग अनुसोंधान पहि: र्िज्ञान और प्रौद्य र्गकी र्िर्ाग के र्िज्ञान और इों िीर्नयररों ग अनुसोंधान ब डि (एसईआरबी) ने
र्ारिीय र्डर्िटि गेर्मोंग अनुसोंधान क्षेि और उद्य ग का समर्नन करने के यिए तीन मुख्य यदर्ाओों की पहचान की है ।
1. गेयमोंग प्लेटफामों में अनुसोंिान एिों यिकास,
2. भारतीि सोंस्कृयत और मूल्यदों पर जदर दे ने िािे इमयसनि गेम प्रदटदटाइप।
3. सहिदगात्मक तकनीकी यडज़ाइन प्रयक्रिा, यजसे SERB गेम िैि द्वारा बनािा गिा र्ा।

38
• एर्नमेशन, र्ििुअि इिेर् स, गेर्मोंग और कॉर्मक (AVGC) प्रम शन टास्क ि सि: सूचना और प्रसारण मोंत्रािि ने एर्नमेशन,
र्ििुअि इिेर् स, गेर्मोंग और कॉर्मक (AVGC) प्रम शन टास्क ि सि की स्थापना की है।
 भारतीि एिीिीसी उद्य ग में "मेक इन इों र्डया" और "िाोंड इों र्डया" बैनर िे िाने की क्षमिा है।
आगे की राह
● कानून ों में एकरूपिा: इसमें र्ायमि सभी पक्षदों के यिए उयचत गेयमोंग यियनिमन की तत्काि आिश्यकता है , अन्यर्ा केंि कद अनुच्छेि
252 के तहत राज्यदों के यिए कानून बनाने की र्ल्कक्त तर्ा अनुच्छेि 248 के तहत अिर्शि शम्भक्तय ों का उपय ग करके र्नयम और
र्िर्नयम बनाने का अर्धकार र्िया िाना िार्हए।
○ समन्वय और सहय ग: ऑनिाइन गेयमोंग उद्यदग की क्षमता का पूरी तरह से एहसास करने के यिए, राज्य और केंि सरकारदों
कद उद्य ग र्हिधारक ों के साथ र्मिकर उर्िि र्िशार्निे श र्नधािररि करने के र्िए र्मिकर काम करना िार्हए।
● र्नगरानी: पूरे भारत में यनिमदों कद मानकीकृत करने और कानूनदों में एकरूपता बनाए रखने के यिए, पूरे ऑनिाइन जुआ क्षेत्र की
िे खरे ख के र्िए एक र्नयामक प्रार्धकरर् ह ना िार्हए।
● अर्र्र्ािक की र्ूर्मका: माता-यपता कद अपने बच्चदों की गेयमोंग आदतदों और ऑनिाइन यबताए गए समि के यिए जिाबदे ह हदने की
आिश्यकता है ।
● बच्च ों की सुरक्षा: एक आयु रे र्टों ग प्रर्ािी िागू करें जद नाबायिगदों कद उनके माता-यपता की अनुमयत के यबना आगे बढने से रदकती
है । आधार पर ओटीपी सत्यापन का उपिदग करके इस समस्या कद हि यकिा जा सकता है।
● िागरूकिा: गेयमोंग व्यिसािदों कद म्भखिार्ड़य ों क सोंर्ार्िि ि म्भखम ों के बारे में आक्रामक रूप से सूर्िि करना िार्हए और
उन पररल्कस्र्यतिदों कद कैसे पहचानना चायहए जहाों िदखािडी और दु व्यनिहार हदने की सोंभािना है ।
● पारिर्शििा: प्रयतभायगिदों की गुमनामी कद दू र यकिा िस्तु एिों सेिा कर (िीएसटी) पररषि के बारे में:
जाना चायहए, और एक मिबूि र्शकायि प्रबोंधन • र्ारिीय सोंर्िधान के अनुच्छेि 279ए के अनुसार, जीएसटी का प्रर्ासन और
िोंि यिकयसत यकिा जाना चायहए। सोंचािन करने के यिए राष्टरपयत द्वारा जीएसटी पररषद का गठन यकिा जाएगा।

3.4 पररसों प र्ि मु द्रीकरर्


सोंिर्ि:
केंिीि ऊजान मोंत्रािि का िक्ष्य र्ििीय िषि 2023-24 में पररसोंपर्ि मुद्रीकरर् के माध्यम से िगर्ग ₹15,000 कर ड़ िुटाने का है।
• भारत सरकार ने र्िि िषि 2022-23 में ₹1.32 र्टर र्ियन की सोंपर्ि का मुद्रीकरर् र्कया है, जबयक िक्ष्य ₹1.62 र्टर र्ियन का है, जबयक
इसने र्िि िषि 2023-24 के र्िए ₹1.79 र्टर र्ियन का िक्ष्य
रखा है।

पररसोंपर्ि मुद्रीकरर् के बारे में:


• िह कम उपिदग की गई सािििर्नक सोंपर्िय ों के आयर्नक
मूल्य कद अनिॉक करके रािस्व के नए/िैकम्भल्पक स्र ि
बनाने की प्रर्क्रया है।
 यकसी सािििर्नक र्नकाय के स्वार्मत्व िािी क ई र्ी
सोंपर्ि, सडक, हिाई अड्डे , रे ििे, स्टे र्न, पाइपिाइन,
टर ाों सयमर्न िाइनें आयद हद सकती है , िा िहाों तक यक
अप्रिुक्त भूयम भी हद सकती है ।
• उद्दे श्य: ऐसी पररसोंपर्ि का निीनीकरर् और सोंिधिन करना
जद आयर्नक मूल्य दे ने के मामिे में अनुिरदािी बनी हुई है।
• उद्दे श्य:
 आयर्नक मूल्य बढाने के यिए र्निी क्षेि की िक्षिा बढ़ाना।
 मूल्य इों जीयनिररों ग के माध्यम से अर्धक र्ििीय िीिरे ि बनाना।
 ि म्भखम आिोंटन का अनुकूिन करना।
 र्निेश अोंिराि क पाटने के र्िए धन का उपिदग करना।
• पररसोंपर्ि मुद्रीकरर् का मििब िरूरी नही ों र्क सोंपयि का यियनिेर् हद।

39
पररसोंपर्ि मुद्रीकरर् मॉडि
• प्रत्यक्ष सोंर्ििात्मक मॉडि: िाउनिीड सािििर्नक-र्निी र्ागीिारी (पीपीपी) ररयायिें
 सोंिािन, रखरखाि और स्थानाोंिरर् (ओएमटी): यनजी यनिेर्क सड़क ों के सोंिािन और रखरखाि में र्निेश करिे हैं और टदि
टै क्स, भूयम मूल्य पर कब्जा आि आयद के माध्यम से अपनी यनिेर् िागत िसूि करते हैं।
 ट ि, ऑपरे ट और टर ाोंसिर (टीओटी): सािनजयनक रूप से यिि पदयषत पररचािन पररिदजनाएों , जैसे राजमागन, का मुिीकरण यकिा
जाता है और बदिी िगाई जाती है । यनिेर्क यियनिदग र्ुि अर्ििि करने के 15 से 30 साि के अर्धकार के बििे में एकमुश्त
र्निेश करिे हैं।
 सोंिािन, रखरखाि और र्िकास (ओएमडी): पररचािन पररसोंपयि कद ररिाित अियि के दौरान उन्नयन और ओ एों ड एम के र्िए
एक र्निी पाटी क पट्टे पर र्िया िािा है।
 पुनिािस सोंिािन रखरखाि स्थानाोंिरर् (आरओएमटी): सोंचािन और राजस्व सोंग्रह यफर से र्ुरू करने से पहिे एक मौजूदा सोंपयि
कद आिशि रूप से उन्नि या सोंिर्धिि र्कया िाना िार्हए।
• सोंरिना र्ििप षर् मॉडि
 इों फ्रास्टर क्चर इन्वेस्टमेंट टर स्ट (InvITs): इों फ्रास्टर क्चर इन्वेस्टमेंट टर स्ट (InvITs) बुर्नयािी ढाोंिे में प्रत्यक्ष र्निेश करने के र्िए र्िर्र्न्न
र्निेशक ों से एकर्िि धन का उपय ग करिे हैं।
 ररयि एस्टे ट र्निेश टर स्ट (REIT): फमन ररिि एस्टे ट का मायिकाना हक रखती है , प्रबोंिन करती है िा यिि पदषण करती है और
यनिेर्कदों से पूोंजी जुटाई जाती है , यजसे बाद में यनिेर् यकिा जाता है ।

सोंपर्ि मुद्रीकरर् के िार्:


• रािस्व स्र ि: कम उपिदग यकए गए सािनजयनक यनिेर् के मुिीकरण से पीएसिू कद यििीि सोंसािन प्राप्त करने में मदद यमिेगी और िोंबी
अिर्ध में, इन सोंसाधन ों का अर्धक प्रर्ािी उपय ग ह सकेगा।
• गुर्क प्रर्ाि: यनजी मायिक उत्पादकता बढाते हैं और खचन कम करते हैं । यनजीकरण के इस पैमाने के प्रभाि से उत्पािक र िगार की
मात्रा में िृल्कद्ध हदने की सोंभािना है ।
• िक्षिा में सुधार: िह यनजी क्षेत्र की र्िशेषज्ञिा और निािार का िाभ उठाकर पररसोंपर्िय ों की उत्पािकिा और िक्षिा क बढ़ा
सकिा है।
• आर्थिक समृम्भद्ध: जैसे-जैसे यनजी क्षेत्र अयिक सोंसािनदों का उत्पादन करने में सक्षम हदगा, अथिव्यिस्था पुनिीर्िि ह िाएगी, और
मुद्रीकृि पररसोंपर्िय ों से प्राप्त रािस्व नए बुयनिादी ढाोंचे के यनमान ण और भयिष्य की पररसोंपयििदों के पूणन पुनचनक्रण में िदगदान दे गा।

र्ारि में सोंपर्ि मुद्रीकरर्


1. र्ििय केिकर पैनि: भारत में, पररसोंपयि मुिीकरण का यिचार
पहिी बार 2012 में रािक षीय समेकन के र डमैप पर र्ििय
केिकर के नेिृत्व िािी एक सर्मर्ि द्वारा सुझाया गया था।
 आगे के र्िकास और बुर्नयािी ढाोंिे की िरूरि ों के
र्ििप षर् के यिए सोंसािन जुटाने के यिए मुिीकरण की
यसफाररर् की।
2. केंद्रीय बिट 2021-22: दे र् में बेहतर और यटकाऊ बुयनिादी ढाों चे
के यिि पदषण के यिए पररसोंपयििदों के मुिीकरण कद िीन स्तोंर् ों में
से एक के रूप में रे खाोंर्कि र्कया गया था।
3. रािरीय मुद्रीकरर् पाइपिाइन (एनएमपी): िह एक पररसोंपयि मुिीकरण कािनक्रम है , यजसकी घदषणा बिट 2021 में की गई थी।
• उद्दे श्य: सडक, रे ििे, यिमानन, यबजिी, तेि और गैस और भोंडारण सयहत कई क्षेत्रदों में कई िाउनिीड बुर्नयािी ढाोंिा पररसोंपर्िय ों के
पररसोंपर्ि मुद्रीकरर् के यिए एक रदडमैप के रूप में कािन करना।
 िह रणनीयतक यनिेर्कदों िा यनजी क्षेत्र की कोंपयनिदों द्वारा उपिदग के यिए तैिार की गई सभी सोंपयििदों के बारे में िानकारी प्रिान करने
के र्िए एक र्ूर्म बैंक के रूप में कायि कर सकिा है।
 िह अप्रिुक्त और कम उपिदग की गई सरकारी सोंपयििदों के सोंर्ार्िि मूल् का आकिन करे गा।
• एनएमपी का िक्ष्य: र्िि िषि 22 से र्िि िषि 25 िक, चार साि की अियि में, केंि सरकार की सोंपर्ि मुद्रीकरर् के माध्यम से 6
र्टर र्ियन रुपये जुटाना।

40
• िह उन िाउनफीड सोंपर्िय ों पर ध्यान केंर्द्रि करिा है यजनमें यनिेर् पहिे ही यकिा जा चुका है िेयकन उनका कम उपिदग यकिा गिा
है ।
• इसके िहि सोंपर्िय ों का स्वार्मत्व केंि के पास रहे गा।
रािरीय मुद्रीकरर् पाइपिाइन (एनएमपी) के िार्
• सकि घरे िू उत्पाि क बढ़ािा िे ना: इससे सरकार कद नए सािनजयनक बुयनिादी ढाोंचे के यिकास, राजस्व के सािन यिकयसत करने,
िृल्कद्धर्ीि पूोंजी उत्पादन अनुपात (आईसीएआर) कद कम करने आयद में यनिेर् करने में मदद यमिेगी।
• एनआईपी का समथिन: एनएमपी से हदने िािी आि रािरीय अिसोंरिना पाइपिाइन (एनआईपी) के िहि बुर्नयािी ढाोंिे के र्िए कुि
पररव्यि का िगभग 14% हदने की उम्मीद है ।
• र िगार सृिन: 360 यडग्री की िृल्कद्ध से र्हरदों, कस्दों और गाोंिदों में र िगार के अिसर बढ़ें गे, र्िससे िीिन की गुर्ििा बेहिर ह गी।
• एिडीआई क आकर्षिि करना: िह बुयनिादी ढाोंचे यनिेर् टर स्ट (इनयिट् स) जैसे िाहनदों के माध्यम से दे र् में उच्च र्ििे शी सोंस्थागि
र्निेश क आकर्षिि करने में मिि कर सकिा है।
• सहय गात्मक र्िकास: यह सािििर्नक और र्निी क्षेि ों के बीि सोंरर्िि साझेिारी बना सकता है , जद आपसी यिश्वास और सहिदग कद
बढािा दे सकता है ।
• बेहिर ग्राहक अनुर्ि: िह सोंपयििदों के बेहतर सोंचािन और रखरखाि कद सुयनयित करके नागररकदों के यिए सेिाओों की गुर्ििा और
उपय गकिाि अनुर्ि में सुधार कर सकिा है।
पररसोंपर्ि मुद्रीकरर् से सोंबोंर्धि िुनौर्ियााँ
• र्िर्नयामक बाधाएाँ : यियनिामक िातािरण की स्पष्टता और ल्कस्र्रता का अभाि मुद्रीकरर् से प्राप्त मूल् क प्रर्ार्िि करिा है या िरम
मामि ों में, यकसी सोंपयि का मुिीकरण करना असोंभि बना सकता है जैसा यक िात्री टर े न यनजीकरण के मामिे में हदता है ।
• एकार्धकार: महत्वपूणन बुयनिादी ढाोंचे के यियनिेर् के कारण एकार्धकार और मुद्रास्फीर्ि ह सकिी है।
 उिाहरर्: एक ही बदिी िगाने िािे कद 50 िषों के र्िए छह हिाई अड् ों क पट्टे पर दे ने से करदाताओों द्वारा यिि पदयषत सोंपयििदों
कद मुट्ठी भर व्यािसायिक समूहदों कद हस्ताोंतररत करने की आर्ोंका पैदा हद गई है ।
• िैर्मिी ग ड कॉन्सेप्ट: यपछिे 70 िषों में करदाताओों के पैसे से सािनजयनक सोंपयििाों बनाई गई हैं , यजन्हें 25-50 िषों के िीघिकार्िक
पट्टे के माध्यम से र्निी कोंपर्नय ों क पेश र्कया िा रहा है।
• िार् अर्धकिमकरर्: र्निी र्निेशक पट्टे की अिर्ध के िौरान अपने िार् क अर्धकिम करने के र्िए पट्टे पर िी गई सोंपर्िय ों
पर आिश्यकता से अयिक यनिेर् करने में आर्ोंयकत हदोंगे।
• उिाहरर्: यसोंगापुर कद अपनी उपनगरीय टर े न ों और र्सिर्िोंग प्रर्ार्िय ों का रािरीयकरर् करना पड़ा, क्दोंयक यनजी ऑपरे टर ने
रखरखाि में कम यनिेर् यकिा र्ा।
• उपर् क्ता के र्हि ों पर िाोंि: सफि बदिीदाता उपय र्गिा मूल् में िेिी से िृम्भद्ध कर सकिा है और उपभदक्ता कद िागत िहन करनी
पडती है ।
 उिाहरर्: न्यू साउथ िेल्स में, खोंभदों और तारदों के यनजीकरण के बाद यबजिी की कीमतें पाोंच साि में ददगुनी हद गईों।
• र्पछिा ख़राब अनुर्ि: भारतीि रे ििे ने 150 रे िगार्ड़यााँ ििाने के र्िए र्निी पार्टि य ों क आमोंर्िि र्कया, िेयकन नौ क्लस्टरदों के यिए
बदिीदाताओों कद आकयषनत करने में यिफि रही और केिि तीन क्लस्टरदों के यिए दद बदयििाँ प्राप्त हुईों।

आगे की राह
• ििाबिे ही सुर्नर्िि करना: सरकार एक ऐसा केंिीि यनकाि बना सकती है ि उद्य ग ों में सोंपर्ि के मुद्रीकरर् की प्रगर्ि की र्नगरानी
कर सकिी है।
 र्िर्ागीय ढ़रों क ि ड़ना: इससे मुिीकरण की प्रयक्रिा र्ुरू करने में आने िािी बािाओों की पहचान करने और उन्हें सोंबदयित करने
में यियभन्न सोंबोंर्धि मोंिािय ों और र्िर्ाग ों क सहायिा र्मिेगी।
• सोंपर्िय ों की पहिान: 'एसेट मैनेिमेंट प टि ि' जैसी पहि के सार् आईटी सक्षम समर्नन, जद सभी सोंबोंयित सोंपयििदों कद एक मोंच पर
िाता है और मौजूदा बुयनिादी ढाों चे के यनमान ण की पहि कद आगे बढाने के यिए दृश्यता और यनणनि िेने के मामिे में इसे पारदर्ी बनाता
है ।
• नि न्वेषी र्ििीय मॉडि: बुयनिादी ढाोंचे के ऋण फोंडदों की तरह बाों डदों पर िक्षिा हार्सि करने के र्िए अर्धक र्ििार करने की
आिश्यकिा है।
• र्निेशक क आिासन: यनिेर्कदों कद उनकी सोंपयि की ि म्भखम र्िशेषिाओों के बारे में आिासन प्रदान करें ।

41
 सोंरर्िि गारों टी या पररसोंपर्ि बोंडि ों के र्नमािर् के रूप में केंि सरकार से सहािता- जद केंिीि और राज्य पररसोंपयििदों का यमश्रण है
- फािदे मोंद हद सकती है ।
• इन्फ्फ्रास्टर क्चर इन्वेस्टमेंट टर स्ट (InvIT) मुद्रीकरर् का मागि उपभदक्ता के यिए गित मूल्याोंकन िा कीमत में िृल्कद्ध के जदल्कखम कद कम कर
सकता है ।
 InvIT एक म्यूिुअि िोंड के समान है, ि ररटनन के रूप में आि का एक छदटा यहस्सा अयजनत करने के यिए बुयनिादी ढाोंचे में सोंभायित
व्यल्कक्तगत/सोंस्र्ागत यनिेर्कदों से छदटी मात्रा में िन के सीिे यनिेर् कद सक्षम बनाता है ।
• िैकम्भल्पक र्ििाि-समाधान िोंि: कुर्ि और प्रभािी यििाद समािान तोंत्र स्वार्ार्िक रूप से और स्विार्िि रूप से पररय िना के
र्डिाइन और र्नष्पािन के र्िए उपय गी ह ग ों े।

3.5 क यिा गै सीकरर्


सोंिर्ि:
हाि ही में, क यिा मोंिािय ने अपनी ऊजान सोंक्रमण िदजनाओों के अनुरूप यिि िषन 2030 तक 100 यमयििन टन कदििे कद गैसीकृत करने
का िक्ष्य रखा है ।
रािरीय क यिा गैसीकरर् र्मशन
मुि बािें • सरकार का िक्ष्य 2030 िक 100 र्मर्ियन टन (एमटी) क यिा गैसीकरर्
• उद्दे श्य: अन्य क्षेत्रदों कद प्रदत्सायहत करते हुए दे र् की करना है, र्िसमें 4 र्टर र्ियन रुपये से अर्धक का र्निेश ह गा।
भयिष्य की ऊजान जरूरतदों कद पूरा करना। • इसके तहत, ईोंधन के स्वच्छ स्र ि ों के उपय ग क प्रदत्सायहत करने के यिए
• यह मोंिािय सािनजयनक क्षेत्र के उपक्रमदों (पीएसिू) और िायणल्कज्यक कदििा ब्लॉक नीिामी के तहत रािस्व र्हस्सेिारी पर 20% की
यनजी क्षेत्र ददनदों के यिए क यिा और र्ििाइट ररयायि िी गई।
गैसीकरर् पररय िनाओों क बढ़ािा िे ने के यिए • सीआईएि ने िैयश्वक यनयिदा के माध्यम से बीओओ (र्बड-ओन-ऑपरे ट)
आधार पर कम से कम 3 गैसीकरर् सोंयोंि स्थार्पि करने की र्ी य िना
₹6,000 करदड के पररव्यि के सार् एक िदजना पर
बनाई है और र्सोंथेर्टक प्राकृर्िक गैस के र्िपर्न के र्िए गेि के साथ एक
यिचार कर रहा है ।
समझौिा ज्ञापन पर हस्ताक्षर र्कया है।
• गैसीकरर् के र्िए पीपीपी मॉडि: गैसीकरर्
पररय िनाओों की बजटीि सहािता के यिए पीएसयू और र्निी क्षेि की
कोंपर्नय ों का चिन टै ररि-आधाररि ब िी प्रर्क्रया के माध्यम से यकिा जाएगा।

क यिा गैसीकरर् की आिश्यकिा: प्रिुर मािा में स्विे शी क यिा स्र ि


• स्विे शी स्र ि: कदििा भारत का प्रार्यमक ऊजान स्रदत है , ि िे श की िार्र्म्भज्क
ऊिाि खपि का िगर्ग 67% है।
 िह अिसादी चट्टान में पािा जाने िािा जीिाश्म ईोंिन है और इसे आमिौर
पर "कािा स ना" कहा िािा है।
 भारत कदििा उद्यदग में एक प्रमुख दे र् है , जद उत्पािन, खपि और आयाि
में िीन के बाि िू सरे स्थान पर है।
 2020 तक की ल्कस्र्यत के अनुसार, भारत के पास पाोंििाों सबसे बड़ा क यिा
र्ोंडार है।
• ऊिाि सुरक्षा: घरे िू कदििा भोंडार से कच्चे तेि और प्राकृयतक गैस जैसे आिायतत
ईोंिन पर भारत की यनभनरता कम हद जाएगी।
• क यिे का सिि उपय ग: र्ारि 2050 िक अपनी ऊिाि िरूरि ों के यिए
कदििे पर यनभनर रहे गा और इसका उपिदग िर्ासोंभि पयाििरर् अनुकूि िरीके
से करना महत्वपूर्ि है।
क यिा गैसीकरर् के बारे में:
• प्रर्क्रया: िह एक थमो-रासायर्नक प्रर्क्रया है र्िसमें गैसीिायर का िबाि और ऊष्मा, क यिे क उसके रासाियनक घटकदों में यिघयटत
कर दे ती है।
• घटक: पररणामी "यसनगैस" ज्यादातर काबिन म न ऑक्साइड (सीओ) और हाइडर िन है, सार् ही कुछ अन्य गैसीि पदार्न जैसे हाइडर दजन
सल्फाइड और काबनन डाइऑक्साइड भी हैं ।

42
• क यिे का स्वच्छ र्िकल्प: ददहरे उपिदग
के कारण िह पारों पररक क यिा ििाने
की िुिना में अर्धक िक्ष है:
 ऊजान उत्पन्न करने के यिए टबान इन के
अोंदर जिाए जाने से पहिे क यिा
गैस ों क पहिे िू र्षि पिाथों से शुद्ध
र्कया िािा है।
 र्द्विीयक उपय ग: गैस टबािइन
र्नकास ऊष्मा क एकि र्कया िा
सकिा है और र्ाप टबािइन-िनरे टर
के र्िए र्ाप का उत्पािन करने के
र्िए उपय ग र्कया िा सकिा है।

क यिा गैसीकरर् के िार्:


• क यिा गैस का औद्य र्गक अनुप्रय ग
 ऊिाि का कच्चा स्र ि: यसनगैस का
उपिदग र्बििी उत्पन्न करने के र्िए,
ऊिाि-िक्ष ईोंधन सेि प्रौद्य र्गकी में, िा औद्यदयगक अनुप्रिदगदों के यिए रासाियनक "यनमान ण घटकदों" के रूप में यकिा जा सकता है ।
 हाइडर िन अथिव्यिस्था: हाइडर दजन का उपिदग हाइडर िन अथिव्यिस्था क शम्भक्त प्रिान करने के र्िए र्कया िा सकिा है।
 ऑट म बाइि ईोंधन: क यिा गैस क गैस िीन के प्रर्िस्थापन के रूप में ऑट म बाइि में उपय ग र्कए िाने िािे पररिहन
ईोंधन में पररियतनत यकिा जा सकता है ।
 इस्पाि उद्य ग: यसनगैस (CO और H2) इस्पात उत्पादन के यिए आिश्यक अपिायक एिेंट है और इसे पयाििरर् के अनुकूि
िकनीक माना िािा है (र्ट्ठी के िेि के उपय ग क कम करने में)।
 र्बििी पैिा करना: सोंश्लेषण गैस का उपिदग यबजिी पैदा करने के यिए एकीकृि गैसीकरर् सोंयुक्त िक्र (IGCC) प्रर्ािी में र्कया
िा सकिा है।
 र्िर्िध उत्पाि: क यिा गैसीकरर् के माध्यम से मेथनॉि, डाइर्मथाइि ईथर (डीएमई), ओिेर्िन और उििरक ों के यनमान ण में
इस्तेमाि हदता है । कदििा यबजिी सोंिोंत्र की राख/अिर्ेषदों से बने काबिन िाइबर और प्लाम्भस्टक कोंप र्िट र्ी खूब माोंग में हैं।
• िामािस्युर्टकि उद्य ग: र्सनगैस का उपय ग सयक्रि फामानस्युयटकि सामग्री (एपीआई) बनाने में यकिा जाता है ।
• स्वच्छ और पयाििरर् के अनुकूि: इसमें क ई स्क्रबर कीिड़ उत्पन्न नहीों हदता है और अयिकाों र् ध ने के पानी क पुन: सोंसार्धि
र्कया िािा है, और शेष अपर्शि िि का प्रभािी ढों ग से उपचार यकिा जा सकता है ।
• पेररस प्रर्िबद्धिा क पूर्ि करना: कदििा गैसीकरण उत्सििन स्तर क कम करने और गैर-जीिाश्म ईोंिन-आिाररत ऊजान सोंसािनदों की
उन्नयत में सहािता करता है ।
क यिा गैसीकरर् में िुनौर्ियााँ:
• क यिे की गुर्ििा: र्ारिीय क यिे की उच्च राख सामग्री उपयुक्त प्रौद्य र्गकी के र्िकास में एक महत्वपूणन बािा है ।
• प्रेररि धोंसाि: प्रर्क्रया के िौरान गहरे खनन से उत्पन्न र्ून्य र्ेष कदििे और आसपास की चट्टानदों ददनदों में महत्वपूर्ि र्िरूपर् का कारण
बन सकता है ।
 िापन, शमन, िि प्रिाह और सोंर्ार्िि छि और दीिार ढहने से कैयिटी की अखोंडता गोंभीर रूप से प्रभायित हद सकती है , यजससे
िोंसाि हद सकता है।
• पयाििरर्ीय कारक: कुछ अध्यिनदों के अनुसार, कदििा गैसीकरण िास्ति में पारों पररक क यिा र्बििी स्टे शन की िुिना में अर्धक
CO2 उत्पन्न करिा है।
• िि िनाि: यह अर्धक िि-सघन ऊिाि उत्पादन यियििदों में से एक है ।
• पररय िना का अथिशास्त्र: िह स्वार्ार्िक रूप से 'अम्भस्थर' म्भस्थर्ि िािी प्रर्क्रया है और उत्पाि गैस की प्रिाह दर और ताप मूल्य
ददनदों समि के सार् अिग-अिग हदोंगे, यजसका पररिदजना के अथिशास्त्र पर महत्वपूणन प्रभाि पडे गा।
 इससे व्यािसार्यक खिरे की सोंभािना भी बढ जाती है ।

43
आगे की राह:
• बेहतर कदििा गुणििा ल्कस्र्रता, यनरों तर आपूयतन और यनकट खनन और पररिहन िागत यनिोंत्रण के यिए कदििा गैसीकरण पररिदजनाओों
(नीिामी यिोंकेज के माध्यम से प्रदान की जाने िािी) के यिए 'समर्पिि, बोंि क यिा खिानें' र्नधािररि की िाएों गी।
• व्यिहायििा अोंिर र्ििप षर्, बुर्नयािी ढाोंिे की म्भस्थर्ि: 'ऊिाि सुरक्षा' (स्वच्छ) पररिदजनाओों की व्यिहािनता में सुिार के यिए आिश्यक
गैसीकरण पररिदजनाओों के बहुत उच्च पूोंजीगत व्यि का समर्नन करने के यिए सरकार की ओर से यििीि प्रदत्साहन।
• समान अिसर: पिानिरण-अनुकूि कदििा गैसीकरण पररिदजनाओों के यिए कदििा फीडस्टॉक कीमतदों पर ितनमान में िागू उपकर/शुल्क
से छूट प्रिान की िाएगी।
• कदििा गैसीकरण आिाररत पररिदजनाओों के तेज और सुचारू कािानन्विन के यिए ' क यिा गैसीकरर् और द्रिीकरर् पर रािरीय नीर्ि'
ित्काि िैयार और िागू की िानी िार्हए।
• 'गैस िीन और एिपीिी के साथ मेथनॉि, डीएमई के र्मश्र्' की य िनाओों क जल्द से जल्द िागू यकिा जाना चायहए, सार् ही
कदििा गैसीकरण के सभी उत्पाि ों के र्िए नीर्िगि ढाोंिे की र्ी आिश्यकिा है।

3.6 बहुआयामी गरीबी


सोंिर्ि:
'रािरीय बहुआयामी गरीबी सूिकाोंक: एक प्रगर्ि समीक्षा 2023' ररपदटन जारी की है ।

बहुआयामी गरीबी क्ा है?


• बहुआिामी गरीबी में गरीब ि ग ों द्वारा अपने
िै र्नक िीिन में अनुर्ि र्कए िाने िािे
र्िर्र्न्न अर्ाि शार्मि हैं - िैसे र्क खराब
स्वास्थ्य, र्शक्षा की कमी, अपयािप्त िीिन
स्तर, अशक्तिा, काम की खराब गुर्ििा,
र्होंसा का खिरा और ऐसे क्षेि ों में रहना ि
पयाििरर् के र्िए खिरनाक हैं।

िैर्िक बहुआयामी गरीबी सूिकाोंक (MPI):


• िारीकिाि: सोंयुक्त रािर र्िकास कायिक्रम
(यूएनडीपी) और ऑक्सि डि गरीबी और
मानि र्िकास पहि (ओपीएिआई)।
• सोंकेिक - इसमें तीन व्यापक सोंकेतक मापे जाते
हैं जद स्वास्थ्य, र्शक्षा और िीिन स्तर हैं, यजन्हें
आगे 10 उप-सोंकेतकदों में यिभायजत यकिा गिा है ।
• मािृ स्वास्थ्य और बैंक खािा रािरीय एमपीआई
का र्हस्सा हैं।

नीर्ि आय ग द्वारा रािरीय बहुआयामी गरीबी


सूिकाोंक:
• निीनतम रािरीय पररिार स्वास्थ्य सिेक्षर्
[NFHS-5 (2019-21)] के आिार पर, राष्टरीि
MPI का िह दू सरा सोंस्करण दद सिेक्षणदों, NFHS-
4 (2015-16) और NFHS-5 के बीच बहुआिामी
गरीबी कद कम करने में भारत की प्रगयत का प्रयतयनयित्व करता है । (2019-21)।
● सोंकेिक: राष्टरीि एमपीआई मॉडि िैयश्वक एमपीआई मॉडि के दस सोंकेतकदों कद बरकरार रखता है , िह राष्टरीि प्रार्यमकताओों के
अनुरूप दद सोंकेतक, अर्ान त् मािृ स्वास्थ्य और बैंक खािे र्ी ि ड़िा है ।

44
रािरीय बहुआयामी गरीबी सूिकाोंक का महत्व:
• गरीबी क समझना: गरीब ों और सबसे गरीब ि ग ों (अोंत्य िय) पर
गरीबी के प्रभािदों कद पूरी तरह से समझने के यिए गरीबी के मौयिक और
गैर-मौयिक उपािदों की आिश्यकता है ।
• प्रर्ािी प्रर्िर्क्रया: जब मौयिक गरीबी उपािदों के सार् पूरक यकिा जाता
है , तद राष्टरीि एमपीआई नीयत यनमान ताओों कद दे र् में गरीबी की व्यापकता
और जयटिता पर यिचार करने और प्रभािी ढों ग से प्रयतयक्रिा दे ने में
सक्षम बनाता है ।
• िर्क्षि गरीबी न्यूनीकरर् रर्नीर्ि: गरीबी पर नीयत-प्रासोंयगक
जानकारी की उपिब्धता यजतनी अयिक हदगी, नीयत यनमान ता इसे कम
करने के यिए उतने ही बेहतर ढों ग से सुसल्कित हदोंगे।
 उदाहरण के यिए, गरीबी कम करने की रर्नीर्ियााँ उन क्षेि ों के
र्िए र्र्न्न ह िी हैं िहााँ अर्धकाोंश ि ग ों के पास र्शक्षा और
आिास की कमी है।
• गरीबी की िर्टििाओों क समझना: एमपीआई गरीबी की
िर्टििाओों क समझने और समािेशी समाधान बनाने में एक
महत्वपूणन मीि का पत्थर है ।
• नीर्ि र्नमािर्: िह सूचकाों क नीयत यनमान ताओों और जनता कद सोंयुक्त
रािर के सिि र्िकास िक्ष्य ों (एिेंडा 2030) क प्राप्त करने और
बहुआिामी गरीबी कद कम करने में मदद करता है ।
• गरीबी कम करने के र्िए बहु-क्षेिीय दृर्िक र् का प्रिशिन: िे उपिल्कब्धिाँ गरीबी कम करने की यदर्ा में भारत के प्रिासदों की
पररितननकारी र्ल्कक्त कद दर्ान ती हैं , जद िदगदों की स्विता, खाना पकाने के ईोंिन और यबजिी तक पहुोंच में सुिार के यिए बडे यनिेर् में स्पष्ट
है ।

आगे की राह:
• अिसर की समानिा: आिश्यक सुयििाओों और सामायजक सेिाओों की अिी गुणििा और सािनभौयमक सािनजयनक प्राििान से अिसर
समानता बढनी चायहए।
• र्निेश में िृम्भद्ध: निीनतम एनएसओ ररिीज में, 2022-23 के यिए मौजूदा कीमतदों में सकि ल्कस्र्र पूोंजी यनमान ण दर जीडीपी का 29.2% है ।
जबयक सािनजयनक यनिेर् में सुिार हुआ है , यनजी क्षेत्रदों से यनिेर् में उल्लेखनीि िृल्कद्ध की आिश्यकता है ।
• र्निेश आिोंटन: यनिेर् कद उन क्षेत्रदों पर ध्यान केंयित करने की आिश्यकता है जद यिकास कद बढािा दे ते हैं और रदजगार पैदा करते हैं ।
• मानि सोंसाधन र्िकास में िेिी िाना: मानि सोंसािन यिकास के यिए र्ैयक्षक सुयििाओों जैसे साक्षरता कद बढािा दे ने के यिए स्कूिदों,
तकनीकी प्रयर्क्षण सोंस्र्ानदों और िदगदों में कौशि र्िकर्सि करने के र्िए व्यािसार्यक कॉिेि ों में अर्धक र्निेश की आिश्यकिा
ह िी है।
• उत्पीर्ड़ि समूह ों क आिाि िे ना: पारों पररक रूप से उत्पीर्ड़ि और िर्मि समूह ों क एक बडी आिाज प्रदान करके, जायत और यिोंग
के ल्कखिाफ पूिानग्रह से यनपटने की जरूरत है ।
• प्रौद्य र्गकी की शम्भक्त का उपय ग: प्रौद्यदयगकी ने भारत में कुछ पहुों च बािाओों कद कम करने में मदद की है , र्िशेष रूप से सूिना िक
पहुोंि के सोंबोंध में। उदाहरण-स्विों पदटन ि, Mygov.in
• ऋर् िक पहुोंि: गरीबदों कद आसान र्तों पर ऋण की उपिब्धता छदटे यकसानदों के यिए उत्पादक सोंसािनदों तक पहुोंच प्राप्त करने की ल्कस्र्यत
पैदा करे गी।
• र्ारिीय आर्ििार्सय ों और अल्पसोंिक ों का उत्थान: िह सुयनयित करने की आिश्यकता है यक दयित, मुल्कस्लम और आयदिासी पीछे
न रहें । सोंिुक्त राष्टर की निीनतम ररपदटन के अनुसार, दे र् में िगभग 50% आयदिासी गरीब हैं , सार् ही 33% दयित और 33% मुल्कस्लम भी
गरीब हैं ।

45
3.7 िाि उत्पािन
सोंिर्ि:
अरहर की माोंग घरे िू उत्पादन से अयिक हद रही है , ऑस्टरे र्िया और िािीि के र्कसान र्ारिीय उपर् क्ताओों के र्िए ि कर्प्रय पीिी
िाि उगाने की य िना बना रहे हैं।

समािार के बारे में अर्धक िानकारी:


• भारतीि ििहन और अनाि सोंघ (आईपीिीए) ऑस्टर े यििाई व्यापार यनकाि पल्स ऑस्टर े यििा की मदद से क्वी ोंसिैंड में अरहर की खेिी
क बढािा दे गा।
• ऑस्टर े यििा ने पहिे 2019 में अरहर दाि का प्रािदयगक उत्पादन यकिा र्ा और िहाों गुणििा अिी दे खी गई र्ी। हािाँ यक, िे इसे जारी रखने
में असमर्न र्े क्दोंयक भारत में कीमतें उनके यिए िाभदािक नहीों र्ीों। उन्हदोंने अब एक बार यफर भारतीि बाजार के यिए अरहर उगाने पर
काम र्ुरू कर यदिा है ।

िाि ों के बारे में:


• दािें िायषनक फयििदों िािी फसिें िा यियभन्न आकार, आकृयत और रों ग के बीज हैं यजनका उपिदग भदजन और चारे ददनदों के यिए यकिा जाता
है ।
 "दािें" र्ब्द केिि उन फसिदों पर िागू हदता है जद सूखे अनाि के र्िए उगाई िािी हैं। इसका मतिब िह है यक र् िन के र्िए
उगाई िाने िािी हरी िसिें, र्िन्हें सब्जी िसिें कहा िािा है, साथ ही ज्ािािर िेि र्नकािने के र्िए उगाई िाने िािी
िसिें और केिि बुआई के र्िए उगाई िाने िािी िर्ियाों र्ायमि नहीों हैं।
• र्ारि में उगाई िाने िािी िाि ों के प्रकार: बोंगाि चना (दे सी चना), अरहर / तू अर / िाि चना, हरी फयििाँ (मूोंग), चना मटर (काबुिी
चना), कािी मटपे (उडद / मह / कािी चना), रे ड यकडनी बीन्स (राजमा), ब्लैक आइड पीज़ (िदयबिा), मसू र, सफेद मटर भारत में उगाई
और खाई जाने िािी प्रमुख दािें हैं
र्ारि में िाि ों की म्भस्थर्ि:
िाि ों की अनूठी र्ूर्मका: • र्ारि की रैं र्कोंग: भारत िु र्नया में िाि ों का सबसे बड़ा उत्पािक
• पयाििरर् की दृर्ि से र्टकाऊ: दािें यियभन्न प्रकार की यमट्टी (िैर्िक उत्पािन का 25%), उपर् क्ता (र्िि खपि का 27%) और
और जििािु पररल्कस्र्यतिदों में उगती हैं । िे यियभन्न कृयष आयािक (14%) है।
प्रणायििदों, जैसे िसि िक्र, र्मर्श्ि और अोंिरिसि प्रर्ािी • क्षेि: खाद्यान्न के अोंिगिि आने िािे क्षेि में दािदों की यहस्सेदारी िगभग
के र्िए उपयुक्त हैं। 20 प्रयतर्त है और दे र् में कुि खाद्यान्न उत्पािन में इसका य गिान
 कम काबिन िुटर्प्रोंट: दािदों में काबिन उत्सििन और पानी िगर्ग 7-10 प्रर्िशि है। िाि ों की उत्पािकिा 764 र्कग्रा/हेरेयर
की आिश्यकता कम हदती है । हाि के अनुमानदों के है।

अनुसार, एक र्कि माोंस के उत्पािन के र्िए पानी की • खेिी का मौसम: हािााँर्क िािें ख़रीफ और रबी ि न ों मौसम ों में उगाई
जाती हैं , रबी की दािें कुि उत्पािन में 60 प्रर्िशि से अर्धक का
िरूरि िाि ों की िुिना में पाोंि गुना अर्धक है।
य गिान िे िी हैं।
 नाइटर िन म्भस्थरीकरर्: िर्ियाों िायुमोंडिीय
• प्रमुख ििहन: चना सबसे प्रमुख ििहन है (कुि उत्पािन में 40
नाइटर िन क र्मट्टी में ल्कस्र्र करने और यमट्टी से जुडे प्रर्िशि), इसके बाि िुअर/अरहर 15 से 20 प्रर्िशि और
फास्फदरस कद छदडने में मदद करती हैं । िे पिी के साोंचे के उड़ि/कािी मटपे और मूोंग िगर्ग 8-10 प्रर्िशि है।
रूप में यमट्टी में काबिर्नक पिाथि र्ी र्मिािे हैं। • राज् ों की रैं र्कोंग: मध्य प्रदे र्, महाराष्टर, राजस्र्ान, उिर प्रदे र् और
 मृिा क्षरर् क र कना: िे मृिा-क्षरर् कद रदकने में मदद कनानटक र्ीषन पाोंच दाि उत्पादक राज्य हैं ।
करती हैं , यमट्टी की उिनरता स्तर कद बनाए रखने में िदगदान • प्रमुख र्नयािि गोंिव्य (2022-23): बाोंग्लादे र्, चीन, सोंिुक्त अरब
करती हैं । अमीरात, अमेररका और नेपाि।
• गरीब आिमी का प्र टीन: िह उन िदगदों के यिए प्रदटीन का
स्रदत है यजनकी पहुोंच माों स और डे िरी उत्पाददों तक नहीों है ।
• उद्य ग ों के र्िए कच्चा माि: औद्यदयगक फसिदों के रूप में दािें उद्यदगदों कद कच्चा माि प्रदान करती हैं , िैसे िाि उद्य ग, र्ुना अनाि
उद्य ग, पापड़ उद्य ग, आर्ि।
• िैकम्भल्पक उपय ग: िे मिेयर्िदों के यिए पौयष्टक चारे के समृद्ध स्रदत के रूप में काम करते हैं । कुछ दािें हरी खाद िािी फसिदों के रूप में
उपिुक्त हैं जैसे ि र्बया, कुिथी और मूोंग।

46
र्ारि में ििहन उत्पािन की िुनौर्ियााँ:
• प्रर्ि व्यम्भक्त उपिब्धिा: भारत में दािदों की प्रयत
व्यल्कक्त र्ुद्ध उपिब्धता 51.1 ग्राम/र्िन (1971) से
घटकर 41.9 ग्राम/र्िन (2013) ह गई है, िबर्क
डब्ल्यूएिओ की र्सिाररश 80 ग्राम/र्िन है।
• एमएसपी की अप्रर्ािकाररिा: नेफेड द्वारा दािदों
की िास्तयिक खरीद यपछिे कुछ िषों में न्यूनतम रही
है क्दोंयक मोंडी की कीमतें एमएसपी से काफी ऊपर
चि रही हैं ।
• अकुशि नीर्ि: खुिी आयाि नीर्ि के साथ-साथ
कुशि र्िपर्न व्यिस्था और उत्पादन बािाओों के
अभाि के कारण माोंग और आपूयतन के बीच भारी
अोंतर पैदा हद गिा है । भारतीि माों ग और आपूयतन में
करीब 17 िाख टन की कमी है ।
• सुर्नर्िि र्सोंिाई का अर्ाि: 1966-67 और
2012-13 के बीच, चािि के यिए यसोंचाई का क्षेत्र
38% से बढ़कर 59% और गेहों के र्िए 48% से
93% ह गया, िबर्क िाि ों के र्िए यह 9% से
16% था। ििहन ों का िगभग 84% क्षेत्र िषान
आिाररत है और िहाँ की यमट्टी अपेक्षाकृत कम उिनरता िािी है ।
• िििायु पररिििन: जििािु सूखा और गमी का तनाि यिर्ेष रूप से िे श के शुष्क और अधि -शुष्क क्षेि ों में बीि की पैिािार में 50%
की कमी िािा है।
• खराब िि र्नकासी प्रर्ािी: खराब िि र्नकासी के कारर् यूपी, र्बहार, पर्िम बोंगाि, छिीसगढ़, एमपी और झारखोंड राज्यदों में
बाररर् के मौसम में जिभराि हद जाता है , यजससे यिर्ेष रूप से अरहर की उपज का काफी नुकसान हदता है ।
• कीट और बीमाररयााँ: नाइटर िन और िास्फ रस से भरपूर हदने के कारण दािें कीडदों, कीटदों और बीमाररिदों का यर्कार हदती हैं जद चना,
अरहर और मसूर की उत्पादकता कद गोंभीर रूप से कम कर दे ती हैं ।
• सोंस्थागि ऋर् की कमी: छ टे , सीमाोंि और र्करायेिार र्कसान ों की सोंस्र्ागत ऋण तक पहुों च में असमर्नता के कारण उच्च उपि िािे
बीि ,ों िॉस्फेर्टक उििरक ों और बेहतर प्रौद्यदयगकी का यनम्न उपिदग हदता है ।
आगे की राह:
• प्रर्ािी एमएसपी: सरकार ों क िह सुर्नर्िि करने की आिश्यकिा है र्क आयार्िि िाि ों की िागि एमएसपी से कम न ह । िाि ों
के र्िए एमएसपी में सोंश धन और घ र्षि मूल् पर प्रर्ािी खरीि ह ।
• इनपुट की समय पर उपिब्धिा: अपेयक्षत उपज की गारों टी दे ने िािी गुर्ििापूर्ि इनपुट और र्िपर्न सेिाओों की समय पर उपिब्धिा
के सार् सोंस्र्ागत ऋण प्रदान करने के सार्-सार् तकनीकी सहािता दे ना।
• ईसीए का प्रिििन: आिश्यक िस्तु अर्धर्नयम (ईसीए) के प्रिििन और पीडीएस या प्रत्यक्ष िार् हस्ताोंिरर् य िना के माध्यम से
आिश्यकिा-आधाररि र्ििरर् के माध्यम से खुिे बाजार में साि भर दाि प्र. भारत में दिहन उत्पादन में बािा डािने िािी प्रमुख
की उपिब्धता सुयनयित करना।
चुनौयतिाँ क्ा है , और उन रणनीयतिदों का सुझाि दीयजए यजन्हें
• आयाि र्नर्िरिा क कम करना: िाि ों के घरे िू उत्पािन क बढ़ाकर
इन चुनौयतिदों से पार पाने और दे र् में दिहन उत्पादन बढाने के
समि के सार् िाि ों के आयाि पर यनभनरता कद िीरे -िीरे कम करने की
यिए िागू यकिा जा सकता है ।
आिश्यकता है ।
• बफर स्टॉक: यबना यकसी बबानदी के उयचत प्रबोंिन की जिाबदे ही के सार् आिश्यकता-आिाररत बफर स्टॉक का यनमानण।

3.8 र्डर्िटि अथि व्य िस्था


सोंिर्ि:
सरकार यडयजटि अर्नव्यिस्र्ा के िास्तयिक आकार कद मापने के यिए एक एिेंसी क र्नयुक्त करने की य िना बना रही है । यडयजटि
अर्नव्यिस्र्ा 2025 िक िगर्ग 1 र्टर र्ियन अमेररकी डॉिर ह ने का अनुमान है।
47
• िह ररपदटन राज् ों की रैं र्कोंग और यडयजटि अर्नव्यिस्र्ा में उनकी यहस्सेदारी के सार्-सार् ऐसी अर्नव्यिस्र्ा पर ई-कॉमसन और उभरती
प्रौद्यदयगकी के प्रभाि पर भी गौर करे गी।

र्ारि की र्डर्िटि अथिव्यिस्था पर र्िर्र्न्न ररप टि :


ररप टि िााँि - पररर्ाम

• आईटी बुयनिादी ढाोंचे, सॉफ्टिेिर क्षमताओों और स्वास्थ्य दे खभाि तर्ा यर्क्षा क्षेत्रदों सयहत 30
र्डर्िटि र्िषय ों की पहचान करता है ।

MeitY का 'र्ारि के र्टर र्ियन


डॉिर र्डर्िटि अिसर'

• 2025 िक 1 र्टर र्ियन अमेररकी डॉिर का आर्थिक मूल् सृयजत करने के यिए यडयजटि पररितनन
का अनुमान है , यजससे 60 से 65 र्मर्ियन नौकररयाों पैिा ह ग
ों ी।

• कृर्ष, स्वास्थ्य और र्शक्षा जैसे क्षेत्रदों कद 390-500 यबयििन अमेररकी डॉिर का िाभ हदने की
उम्मीद है ।

• भारत की इों टरनेट अर्नव्यिस्र्ा छह गुना िृल्कद्ध दजन करते हुए 2030 िक 1 र्टर र्ियन अमेररकी
गूगि, टे मासेक और बेन एों ड डॉिर िक पहुोंिने का अनुमान है।
कोंपनी की सोंयुक्त ररप टि
• 2022 में भारत की इों टरनेट अर्नव्यिस्र्ा 155-175 यबयििन अमेररकी डॉिर हदगी।

र्डर्िटि अथिव्यिस्था के बारे में:


• िह आर्थिक गर्िर्िर्धय ,ों िायणल्कज्यक िेनदे न और पेर्ेिर इों टरै क्शन का यिश्वव्यापी नेटिकन है जद सूचना और सोंचार प्रौद्यदयगयकिदों
(आईसीटी) द्वारा सक्षम है ।
• इसे र्डर्िटि प्रौद्य र्गर्कय ों पर आधाररि अथिव्यिस्था के रूप में समझा जा सकता है ।

र्ारि में प्रमुख र्डर्िटि पहि:


• र्डर्िटि कनेम्भरर्िटी पहि:
 र्ारिनेट पररय िना: 2023 तक भारत के सभी गाोंिदों कद हाई-स्पीड िॉडबैंड से जदडना।
 इों टरनेट ऑि र्थोंग्स के र्िए उत्कृििा केंद्र (सीओई-आईटी): डदमेन क्षमता और निीन अनुप्रिदग बनाना।
 सामान्य सेिा केंद्र (सीएससीएस): सािनजयनक उपिदयगता सेिाओों की यडिीिरी।
 साइबर स्वच्छिा केंद्र: बॉटनेट सोंक्रमणदों से यसस्टम का पता िगाना और उसे सुरयक्षत करना।
• र्डर्िटि साक्षरिा और पहिान पहि:
 र्डर्िटि साक्षरिा अर्र्यान (र्िशा) कायिक्रम: घरदों में यडयजटि साक्षरता सुयनयित करना।
 आधार: भारत के यनिायसिदों के यिए यियर्ष्ट पहचान सोंख्या।

48
• उद्यर्मिा और व्यािसार्यक पहि:
• स्टाटि अप इों र्डया कायिक्रम: उद्ययमता और स्टाटन अप पाररल्कस्र्यतकी तोंत्र कद बढािा दे ना।
• र्डिीिॉकर: नागररकदों के यिए यडयजटि िॉिेट।
• र्डर्िटाइज इों र्डया प्लेटफॉमि: स्कैन यकए गए िा भौयतक दस्तािेज़दों कद यडयजटाइज़ करना।
• सेिाएाँ और अर्र्गम्यिा पहि:
 सुगम्य र्ारि अर्र्यान और म बाइि ऐप: यिकिाों ग िदगदों के यिए पहुोंच सुयनयित करना।
 एग्रीमाकेट ऐप: यकसानदों कद फसि मूल्य की जानकारी प्रदान करना।
 र्ीम (र्ारि इों टरिेस िॉर मनी): िूपीआई का उपिदग करके त्वररत और आसान भुगतान सक्षम करना।
• सुरक्षा और शासन पहि:
 अपराध और आपरार्धक टर ै र्कोंग नेटिकि और र्सस्टम (CCTNS): अपराि जाोंच के यिए राष्टरव्यापी टर ै यकोंग प्रणािी।
र्डर्िटि अथिव्यिस्था के िार्
• ई-कॉमसि में िृम्भद्ध: व्यािसायिक गयतयियििदों के यडयजटिीकरण ने यिकास, खरीदारी, यितरण और सेिाओों कद आसान, अयिक प्रयतस्पिी
और अयिक िाभदािक बना यदिा है ।
• व्यािसार्यक अिसर ों का र्िस्तार: र्डर्िटिीकरर् ने छ टी कोंपर्नय ों और व्यिसाय ों के र्िए अोंतरान ष्टरीि व्यापार में सयक्रि रूप से भाग
िेने के अिसर खदि यदए हैं ।
 छ टी कोंपर्नयााँ यिश्व स्तर पर िस्तुओों और सेिाओों की खरीद और यबक्री में सोंिग्न हद सकती हैं ।
 उिाहरर्: म्भफ्लपकाटि द्वारा समथि पहि।
• पारिर्शििा: इससे नकद िेनदे न में कमी आई है , यजससे पारदयर्नता बढी है और भ्रष्टाचार में कमी आई है । िह बेहतर टर ै र्कोंग, ऑर्डर्टों ग
और ििाबिे ही की र्ी अनुमर्ि िे िा है।
• र्िर्र्न्न सेिाओों का र्िस्तार: यडयजटि अर्नव्यिस्र्ा घरे िू और िैयश्वक स्तर पर सेिा क्षेि के पुनमूिल्ाोंकन और र्िस्तार क सक्षम बनािी
है।
 र्िर्कत्सा और शैर्क्षक सेिाओों जैसी सेिाओों के यिस्तार की सुयििा प्रदान करता है।
 उमोंग जैसे मदबाइि ऐप र्िर्र्न्न सरकारी सेिाओों के र्िए एक ही मोंि प्रिान करिे हैं।
• अन्य महत्वपूर्ि प्रर्ाि: स्र्ानीि स्तर पर नौकररिदों का सृिन और उत्पािकिा में िृम्भद्ध।
 स्थानीय र्ाषाओों में उत्पाद पहुों च और समािेयर्ता कद बढाते हैं ।
 नैसकॉम के अनुसार, िषि 2026 तक भारतीि आईटी उद्यदग द्वारा जनर्ल्कक्त की अनुमायनत आिश्यकता िगर्ग 95 िाख ह गी।
र्डर्िटि अथिव्यिस्था की सीमाएाँ :
• र्डर्िटि असमानिा: यडयजटि यिभाजन एक महत्वपूणन चुनौती बनी हुई है , खासकर ग्रामीण क्षेत्रदों में जहाों यडयजटि सेिाओों तक पहुोंच
सीयमत है।
• साइबर अपराध: प्रौद्यदयगकी पर यनभनरता के कारण साइबर अपराध में िृम्भद्ध हुई है, र्िसमें पहिान की ि री, िदखािडी और मनी
िॉल्क्रोंग र्ायमि है , जद व्यल्कक्तिदों और व्यिसािदों के यिए महत्वपूणन जदल्कखम पैदा करता है ।
 डे टा सुरक्षा: बडे डे टा सोंग्राहक डे टा उल्लोंघन ों और अनर्धकृि पहुोंि का ि म्भखम उठािे हैं , जद ग्राहकदों के यिश्वास और गदपनीिता
कद नुकसान पहुोंचाता है ।
• बेर िगारी: स्वचािन और प्रौद्यदयगकी प्रगयत के पररणामस्वरूप कुछ क्षेि ों में र िगार हार्न हुई है, र्िससे बेरदजगारी में िृल्कद्ध हुई है और
कािनबि अनुकूिन की आिश्यकता हुई है।
• र्निेश: यडयजटिीकरण के यिए प्रौद्यदयगकी बुयनिादी ढाों चे और सोंसािनदों में पिान प्त यनिेर् की आिश्यकता हदती है , जद सीयमत यििीि
क्षमताओों िािे छदटे व्यिसािदों के यिए चुनौतीपूणन हद सकता है।
• एकार्धकार: यडयजटि अर्नव्यिस्र्ा ने महत्वपूणन बाजार र्ल्कक्त िािी प्रमुख कोंपयनिदों के उद्भि, एकायिकार बनाने और कुछ क्षेत्रदों में प्रयतस्पिान
कद सीयमत करने की सुयििा प्रदान की है ।
• सोंर्ार्िि पयाििरर्ीय प्रर्ाि: िह इिेररॉयनक अपयर्ष्ट और ऊजान खपत में िृल्कद्ध में िदगदान दे ता है , यजसके पररणामस्वरूप भारी काबनन
फुटयप्रोंट और सोंभायित पिान िरणीि पररणाम हदते हैं ।
आगे की राह:
• र्ििीय समािेशन और सुरक्षा क बढ़ािा िे ना: यििीि समािेर्न और यडयजटि सेिाओों तक पहुों च में सुिार के यिए JAM र्टर र्नटी (िन
धन य िना, आधार, म बाइि) जैसी कनेल्करयिटी प्रगयत का िाभ उठाएों ।
49
• साइबर सुरक्षा पर ध्यान िे ना: यडयजटि अर्नव्यिस्र्ा में सुरक्षा सुयनयित करने और यडयजटि फोंड में चुनौयतिदों का समािान करने के यिए
साइबर सुरक्षा और सतकनता कद प्रार्यमकता दे ना।
• र्डर्िटि सािििर्नक बुर्नयािी ढाोंिे और कुशि कायिबि का र्िकास करें : यडयजटि कौर्ि पहि में यनिेर् करने के सार्-सार् मजबूत
यडयजटि सािनजयनक बुयनिादी ढाों चे की स्र्ापना करें ।
 यडयजटि बुयनिादी ढाों चे के िाभदों का पूरी तरह से िाभ उठाने के यिए यडयजटि रूप से साक्षर श्रयमकदों कद तैिार करने के यिए र्ैक्षयणक
सोंस्र्ानदों कद मजबूत करना आिश्यक है ।
प्र. 'यडयजटि इों यडिा' कािनक्रम यकसानदों कद कृयष उत्पादकता और
• िेनिे न सुरक्षा क मिबूि करें : िेनदे न सुरक्षा बढाने और
आि में सुिार करने में कैसे मदद कर सकता है ? सरकार ने इस सोंबोंि
उपिदगकतानओों कद सत्यापन यिोंडद प्रदान करने के यिए िन-टाइम
में क्ा कदम उठाए हैं ? (2015)
पासिडन (ओटीपी) जैसे प्रभािी उपाि िागू करें ।

3.9 र्ारि में र्गग अथि व्य िस्था


सोंिर्ि: हाि ही में, रािस्थान प्लेटफॉमि आधाररि र्गग िकिसि (पोंिीकरर् और कल्ार्) र्िधेयक, 2023 राजस्र्ान यििानसभा द्वारा पाररत
यकिा गिा र्ा।

रािस्थान प्लेटफॉमि आधाररि र्गग िकिसि (पोंिीकरर् और कल्ार्) र्िधेयक, 2023 की मुि र्िशेषिाएों
• र्गग िकिर क पररर्ार्षि करिा है: िह व्यल्कक्त जद काम करिा है या
कािन व्यिस्र्ा में भाग िेता है और पारों पररक र्नय क्ता-कमििारी सोंबोंध
के बाहर ऐसी गर्िर्िर्धय ों से कमािा है और जद एक अनुबोंि पर काम
करता है यजसके पररणामस्वरूप यनिान ररत यनिमदों और र्तों के आिार
पर भुगतान की एक यनयित दर यमिती है । ऐसे अनुबोंि में और सभी
पीस-रे ट कािन र्ायमि हैं।
• पोंिीकरर् और र्िर्शि आईडी: राज्य पोंजीकृत श्रयमकदों का एक
डे टाबेस बनाएगा और प्रत्येक कद एक अयद्वतीि आईडी प्रदान करे गा।
• रािस्थान प्लेटफॉमि आधाररि र्गग श्र्मक कल्ार् ब डि : इसमें दद
यसयिि सेिकदों के अिािा राज्य सरकार द्वारा नायमत यगग श्रयमकदों और
एग्रीगेटरदों में से प्रत्येक से दद सदस्य हदोंगे।
• रािस्थान प्लेटफॉमि आधाररि र्गग श्र्मक सामार्िक सुरक्षा और कल्ार् क ष: यह पोंजीकृत यगग श्रयमकदों के िाभ के यिए है ।
एग्रीगेटसि से कल्याण र्ुि यििा जाएगा।
• एग्रीगेटसि पर िुमािना: राज्य सरकार जुमान ना िगा सकती है जद पहिे उल्लोंघन के यिए 5 िाख रुपिे तक और बाद के उल्लोंघनदों के यिए
50 िाख रुपिे तक हद सकता है ।

र्गग इक नॉमी के बारे में:


• यगग इकॉनमी एक श्रम बाजार है जद पूणनकायिक स्र्ािी कमनचाररिदों के बजाि स्वतोंत्र श्रयमकदों और फ्रीिाों सरदों पर यनभनर करता है ।
• र्गग अथिव्यिस्था का िगीकरर्:
 प्लेटफॉमि-आधाररि: िे राइड-हेर्िोंग, िूड र्डिीिरी, ई-कॉमसि, ऑनिाइन फ्रीिाोंर्सोंग आयद जैसे काम खदजने और यनष्पायदत
करने के यिए ऑनिाइन ऐप िा यडयजटि प्लेटफॉमन का उपिदग करते हैं ।
 गैर-प्लेटफॉमि-आधाररि र्गग श्र्मक: िे पारों पररक यनिदक्ता-कमनचारी सोंबोंि के बाहर काम करते हैं , जैसे यनमान ण, घरे िू कािन, कृयष
आयद क्षेत्रदों में आकल्किक िेतन श्रयमक और स्व:रदजगार िािे श्रयमक।
• र्गग इकॉनमी के िार्:
 श्र्मक ों के र्िए: यगग अर्नव्यिस्र्ा अयिक िचीिापन, स्वाििता, आि के अिसर, कौर्ि यिकास और समािेर्न प्रदान कर सकती
है ।
 र्नय क्ताओों के र्िए: िह प्रयतभा के एक बडे और यियिि पूि तक पहुोंच, कम यनयित िागत, उच्च स्केिेयबयिटी और बेहतर ग्राहक
सोंतुयष्ट कद सक्षम कर सकता है ।
 ग्राहक ों के र्िए: िह अयिक यिकल्प, सुयििा, गुणििा और सामथ्न प्रदान कर सकता है ।

50
र्ारि में र्गग अथिव्यिस्था:
• सोंर्ािना: नीर्ि आय ग की एक ररपदटन के अनुसार , भारत में प्लेटफॉमि-आधाररि र्गग अथिव्यिस्था हाि के िषों में िेिी से बढ़ी है,
ि िनसाोंम्भिकीय िार्ाोंश, शहरीकरर्, र्डर्िटिीकरर् और उपर् क्ता माोंग िैसे कारक ों से प्रेररि है।

• प्लेटफॉमि आधाररि: 2020-21 िक र्ारि में िगभग 24 यमयििन प्लेटफॉमन-आिाररत यगग श्रयमक हैं , जद र्ारि में गैर-कृर्ष कायिबि
का िगभग 6.5% िा कुि कािनबि का 4% है।
 भारत में प्लेटफॉमन-आिाररत यगग अर्नव्यिस्र्ा 2029-30 िक 56 र्मर्ियन नई नौकररयााँ पैिा कर सकिी है, ि िोंबे समय में
र्ारि की िीडीपी में 1.3% िक बढ़ सकिी है।
• गैर-प्लेटफॉमि आधाररि: इन्वेस्ट इों यडिा के अनुसार, 2019-20 तक भारत में िगभग 120 यमयििन गैर-प्लेटफॉमन-आिाररत यगग श्रयमक
हैं , जद भारत में गैर-कृयष कािनबि का िगभग 32% िा कुि कािनबि का 20% है ।
 भारत में गैर-प्लेटफॉमि-आधाररि र्गग अथिव्यिस्था 2029-30 िक 34 र्मर्ियन नई नौकररयााँ पैिा कर सकिी है।
• आर्थिक य गिान: 2020 में, एस िैम ने अनुमान िगाया र्क र्ारि की र्गग अथिव्यिस्था 2023 िक 17% की चक्रिृल्कद्ध िायषनक िृल्कद्ध
दर से बढकर 455 यबयििन डॉिर हद जाएगी।

र्ारि में र्गग इकॉनमी की आिश्यकिा:


• र िगार के अिसर: नैसकॉम की एक ररप टि के अनुसार, 2025 तक यगग
इकॉनमी का भारत की िीडीपी में िगर्ग 1.25% य गिान ह ने की उम्मीि
है, र्िसमें 90 र्मर्ियन नौकररयाों पैिा करने की क्षमिा है।
• कम कुशि ि ग ों की खानपान की माोंग: ितनमान में, र्गग कायि का िगर्ग
47 प्रर्िशि मध्यम-कुर्ि नौकररिदों में है , िगभग 22 प्रयतर्त उच्च कुर्ि में
है , और िगभग 31 प्रयतर्त कम-कुर्ि नौकररिदों में है ।
• युिा िनसाोंम्भिकीय िार्ाोंश: र्गग अथिव्यिस्था में िुिाओों की भागीदारी में
2019-2022 के बीि 8 गुना िृम्भद्ध िे खी गई है।
 कारर्: िचीिापन और कािन-जीिन सोंतुिन।
• मर्हिा श्म बि: मयहिा यगग श्रयमकदों कद यगग अर्नव्यिस्र्ा की आि-सृजन
क्षमता, पसोंद और िचीिे कािन तौर-तरीकदों से िाभ हदता है।
 यगग इकॉनमी में मयहिाओों की भागीदारी 18% से बढ़कर 36% ह गई
है।
• सेिार्निृि व्यम्भक्तय ों की जरूरि ों क पूरा करना: अनुबोंि कािन द्वारा प्रदान
यकए जाने िािे िचीिेपन के कारण, कई िदग सेिायनिृयि के बाद अपने यिए
काम करना र्ुरू कर दे ते हैं ।
• िकनीकी व्यिधान: एआई, रदबदयटक्स और डे टा एनायियटक्स में तकनीकी प्रगयत ने यगग अर्नव्यिस्र्ा में यगग श्रयमकदों की उत्पादकता और
जीिन स्तर में सुिार यकिा है ।
51
• अोंर्िम-मीि र्डिीिरी क सुिभ, यकफािती और कुर्ि बनाकर क्राों यत िा दी गई है । टे िीकॉम ने काम कद गयतर्ीि बना यदिा, यजससे
कहीों भी सहिदग सोंभि हद सका।
• स्टाटि -अप सोंस्कृर्ि: यगग श्रयमक पारों पररक कमनचाररिदों के यिए एक िागत प्रभािी यिकल्प हैं , क्दोंयक उन्हें िाभ िा मुआिजे की आिश्यकता
के यबना पररिदजना-दर-पररिदजना के आिार पर काम पर रखा जा सकता है ।
• िाकि का स्तोंर्: यगग इकॉनमी ने िाखदों नौकररिाों पैदा करके और सामुदायिक कनेक्शन बनाए रखकर कदयिड-19 के दौरान िचीिापन
और क्षमता यदखाई है ।
र्ारि में र्गग अथिव्यिस्था क बढ़ािा िे ने के र्िए उठाए गए किम:
र्गग इक नॉमी से िुड़ी र्िोंिा: • केंद्रीय र्िधान: िेिन सोंर्हिा, 2019 के िहि यगग श्रयमकदों सयहत सभी
• िृम्भद्ध: इससे स्वैल्किक बेरदजगारी में िृल्कद्ध हुई है क्दोंयक कुछ सोंगयठत और असोंगयठत क्षेत्रदों कद एक सािनभौयमक न्यूनतम िे तन और
श्रयमक पारों पररक रदजगार की तुिना में र्गग कायि के ििीिेपन यनिनहन मजदू री (Floor wage) प्रदान यकिा जाना चायहए।
और स्वायििा क पसोंि करिे हैं।  सामार्िक सुरक्षा सोंर्हिा, 2020 में यगग श्रयमकदों कद एक नई
• नौकरी की असुरक्षा: भारत में यगग श्रयमकदों में अक्सर नौकरी व्यािसायिक श्रेणी के रूप में मान्यता प्रदान की गई है ।
की सुरक्षा का अर्ाि ह िा है, क् र्ों क िे आम िौर पर र्कसी  यगग श्रयमकदों कद सामायजक सुरक्षा िाभ दे ने के यिए समर्पिि
प्र िेर या असाइनमेंट के आधार पर िगे ह िे हैं , और आय सामार्िक सुरक्षा क ष।

में उिार-िढ़ाि ह िा है। • राज् सरकार की पहि: कनािटक सरकार ने र्गग श्र्मक ों के र्िए 4
िाख रुपये के िु घिटना और िीिन बीमा किर की घदषणा की है , यजसके
• सामान्यीकरर् का अर्ाि: भारत में कई यगग श्रयमक
यिए िह िायषनक प्रीयमिम की पूरी िागत िहन करे गी।
अनौपचाररक क्षेत्र में काम करते हैं , जद ऋर्, सामार्िक सुरक्षा
िार् और अन्य सोंसाधन ों िक पहुोंिने की उनकी क्षमिा क सीर्मि कर सकिा है।
• असमान सौिे बािी की शम्भक्त: यडयजटि प्लेटफॉमन पर श्रयमकदों कद सौदे बाजी की र्ल्कक्त की कमी के कारण उर्िि मुआििे और
कामकािी पररम्भस्थर्िय ों पर बाििीि करने के र्िए सोंघषि करना पड़ सकिा है, खासकर िब अन्य श्र्मक ों के एक बड़े समूह के
म्भखिाि प्रर्िस्पधाि ह ।
• प्रर्शक्षर् और अपम्भस्कर्िोंग: कई यगग श्रयमकदों में अपना काम प्रभािी ढों ग से करने के यिए आिश्यक कौर्ि की कमी हदती है । यगग
श्रयमकदों के पास अक्सर कौर्ि बढाने और कररिर में उन्नयत के सीयमत अिसर हदते हैं ।
• सामार्िक किोंक: भारत में कुछ िदगदों द्वारा यगग कािन कद अभी भी एक अस्र्ािी िा कम भुगतान िािे यिकल्प के रूप में दे खा जाता है ,
यजसके पररणामस्वरूप सामायजक किोंक हद सकता है और यगग श्रयमकदों द्वारा यकए गए काम कद मान्यता की कमी हद सकती है ।

आगे की राह:
• कौशि र्िकास: डर ाइयिोंग िा यचनाई जैसे अनौपचाररक क्षेत्र में श्रयमकदों कद यगग क्षेत्र में नौकररिाों िे ने में सक्षम बनाने के यिए पी प्लेटफॉमन
के नेतृत्व िािी अपल्कस्कयिोंग पहि आिश्यक है ।
• पयािप्त सुरक्षा: हािाँ यक यगग श्रयमकदों कद न्यूनतम िेतन यमिता है , िेयकन उन्हें श्रम कानूनदों के तहत सोंरयक्षत अन्य कमनचाररिदों की तरह कुछ
कानूनी सुरक्षा, नीयतिदों और अिसरदों की आिश्यकता हदती है।
 इससे यगग श्रयमकदों के अयिकारदों की रक्षा हद सकती है और श्रम यििाददों पर अोंकुर् िगाने में मदद यमि सकती है।
• र्िोंग पूिािग्रह क सोंब र्धि करना: अयिक मयहिाओों कद भूयमका यनभाने के यिए प्रदत्सायहत करना, प्लेटफामों कद बाजारदों का यिस्तार करने
और अयिक मयहिा ग्राहकदों कद आकयषनत करने में सक्षम बनाना।
 मयहिा प्लेटफॉमन यगग-िकनसन की भागीदारी कद सामान्य बनाने के यिए र्िोंग-समािेशी र्ाषा क अपनाएों , र्िससे प्लेटफॉमि बाजार
का र्िस्तार कर सकें और अर्धक मर्हिा ग्राहक ों क आकर्षिि कर सकें।
• नीयत आिदग ने सभी यगग और प्लेटफॉमन श्रयमकदों के यिए सामायजक सुरक्षा तक पूणन पहुोंच सुयनयित करने के यिए पाोंि-स्तरीय RAISE
दृर्िक र् की र्सिाररश की:
 न्यायसोंगि य िनाओों क र्डजाइन करने के यिए प्लेटफॉमन कािन की यियिि प्रकृयत कद पहचानना।
 नि न्मेषी र्ििप षर् िोंि के माध्यम से सामायजक सुरक्षा में िीमी िृल्कद्ध की अनुमयत दे ना।
 स्कीम तैिार करते समि प्लेटफॉमन के यियर्ष्ट यहतदों कद र्ायमि करना यजससे
रदजगार सृजन हद, प्लेटफॉमन व्यिसािदों और श्रयमकदों का कल्याण हद। प्र. भारत में यगग इकदनॉमी कद यकन चुनौयतिदों का सामना
 िागरूकिा अर्र्यान ों के माध्यम से िकनरदों कद सरकारी िदजनाओों और करना पडता है , और इसके सतत यिकास कद सुयनयित
कल्याणकारी कािनक्रमदों की सदस्यता िेने में सहािता करना। करने के यिए क्ा कदम उठाए जा सकते हैं ?
 यह सुयनयित करना यक सभी सरकारी िाभ श्र्मक ों क आसानी से उपिब्ध
ह।
52
• 'प्लेटफॉमािइजेशन' क प्र त्साहन िे ना यानी सरकार की र्पछिी स्टाटि अप इों र्डया पहि की ििि पर 'प्लेटफॉमि इों र्डया' नामक
कािनक्रम र्ुरू करके प्लेटफॉमन व्यिसािदों कद प्रदत्साहन दे ना।

3.10 र्ारि में खाद्य प्रसों स्क रर् उद्य ग


सोंिर्ि:
प्रधानमोंिी र्कसान सििा य िना (पीएमकेएसिाई) की प्रासोंयगक घटक िदजनाओों के यिए यकए गए मूल्याों कन अध्ययन ों से पता चिा है
यक इसकी प्रासोंयगक घटक िदजनाओों के माध्यम से समयर्नत पररिदजनाओों में पयािप्त प्रत्यक्ष/अप्रत्यक्ष र िगार के अिसर पैिा हुए हैं।

पीएमकेएसिाई के बारे में:


• पीएमकेएसिाई घटक िदजनाओों का एक व्यापक पैकेज है , यजसका उद्े श्य फामन गेट से ररटे ि आउटिेट तक कुर्ि आपूयतन श्रृोंखिा प्रबोंिन
के सार् आिुयनक बुयनिादी ढाोंचे का यनमान ण करना है।

खाद्य प्रसोंस्करर् उद्य ग:


• खाद्य प्रसोंस्करण उद्यदग एक उर्रिा हुआ उद्य ग है जद अर्नव्यिस्र्ा के
यियनमान ण और कृयष स्तोंभदों के बीच महत्वपूणन सोंबोंि प्रदान करता है ।
• खराब ह ने की सोंर्ािना क कम करना और शेल्फ िीिन क बढ़ाना:
िह कच्चे माि या कृर्ष उत्पाि ों क मानि उपर् ग के र्िए उपयुक्त
खाद्य पिाथों में पररियतनत करने के यिए यजम्मेदार है । िह खराब हदने िािी
सामयग्रिदों के प्रसोंस्करण और सोंरक्षण में महत्वपूणन भूयमका यनभाता है ,
र्िसके पररर्ामस्वरूप र्िस्ताररि शेल्फ िीिन के साथ पैक र्कए गए
खाद्य उत्पाि प्राप्त ह िे हैं , जद उन्हें यितरण और उपभदग के यिए
सुयििाजनक बनाते हैं ।

प्रमुख िािक:
• अर्धशेष कच्चा माि: कृयष उत्पादन का उच्च स्तर - बड़ा पशुधन आधार, िसि ों की र्िस्तृि र्िर्िधिा, अोंििे शीय िि र्नकाय और
एक िोंबी िटरे खा, जद समुिी उत्पादन बढाने में मदद करती है
• मिबूि घरे िू माोंग: खचन िदग्य आि में िृल्कद्ध के कारण िीिनशैिी और खान-पान की आिि ों में बदिाि। 2030 तक, भारतीि िायषनक
घरे िू खपत तीन गुना हदने की उम्मीद है , यजससे भारत दु यनिा का पाोंचिाों सबसे बडा उपभदक्ता बन जाएगा।
• र्नयािि के अिसर: भारतीि प्रसोंस्कृत भदजन की यिदे र्दों में माोंग है । यिदे र्ी उपभदक्ताओों की पसोंद में बदिाि और भारतीि प्रिायसिदों की
ओर से भारी माोंग है ।
 सरकार का अनुमान है यक भारतीि प्रसोंस्कृत खाद्य और सोंबोंयित यनिानत 2011-16 में 11.74 प्रयतर्त सीएजीआर की िृल्कद्ध के सार् 16.2
यबयििन अमेररकी डॉिर तक पहुों च गिा।
• सस्ते कायिबि की उपिब्धिा: इसका उपिदग घरे िू और यनिान त बाजार के यिए कम उत्पादन आिार स्र्ायपत करने के यिए प्रभािी ढों ग
से यकिा जा सकता है । अयिकाोंर् यिकयसत और यिकासर्ीि दे र्दों की िुिना में र्ारि में उत्पािन िागि िगर्ग 40 प्रर्िशि कम है ।
खाद्य प्रसोंस्करर् उद्य ग का महत्व:
• र् िन की बबाििी क कम करना: र् िन की बबाििी क कम करना: खाद्य प्रसोंस्करण उद्यदग भदजन की बबानदी कद कम करने और
खाद्य सुरक्षा में सुिार करने में महत्वपूणन भूयमका यनभा सकता है ।
• िनसोंिा क र् िन िे ना: उन्नत प्रसोंस्करण और सोंरक्षण तकनीकदों में यनिेर् करके, उद्यदग फसि के बाद के नुकसान कद कम करने
और पूरे िषन भदजन की ल्कस्र्र आपूयतन सुयनयित करने में मदद कर सकता है ।
• र िगार के अिसर: 2022 की एक सोंसदीि ररपदटन में बतािा गिा है यक 2017-18 में पोंजीकृत फैरर ी क्षेत्र में उत्पन्न नौकररिदों में एफपीआई
की यहस्सेदारी 12.38% र्ी। इस क्षेत्र में 2024 तक 9 यमयििन नौकररिाों पैदा हदने की उम्मीद है ।
• उद्यर्मिा: उयचत प्रयर्क्षण और कौर्ि यिकास कािनक्रमदों के सार्, व्यल्कक्तिदों कद उद्यमी बनने, स्र्ानीि आयर्नक यिकास में िदगदान दे ने के
सार्-सार् ग्रामीर्-शहरी प्रिास के मुद्दे क सोंब र्धि करने के र्िए सशक्त बनाया िा सकिा है।
• र्िकास में िेिी: भारतीि खाद्य उद्यदग 11% की सीएजीआर से यिस्तार कर रहा है और खाद्य प्रसोंस्करण क्षेत्र कुि खाद्य उद्यदग का 32%
यहस्सा है ।

53
र्ारि में खाद्य प्रसोंस्करर् उद्य ग की िुनौर्ियााँ:
• आपूर्िि पक्ष के मुद्दे:
• खोंर्डि बािार और खराब आपूर्िि श्ृोंखिा सोंबोंध: खोंयडत जदत, कम कृयष उत्पादकता, उच्च मौसमीता, खराब हदने की क्षमता और
मध्यस्र्ता के पररणामस्वरूप खोंयडत यिपणन िदग्य अयिर्ेष प्रसोंस्करण और यनिानत में बािा उत्पन्न करता है ।
✓ भारत में उत्पायदत भदजन का िगभग 40% हर साि खोंयडत भदजन और अप्रभािी आपूयतन श्रृोंखिा प्रणािी के कारण बबानद हद जाता
है ।
• माोंग पक्ष के मुद्दे:
 रे डी टू ईट भदजन के पदषण मूल्य के बारे में उपभदक्ताओों की नकारात्मक िारणा भारत में समग्र माोंग और यबक्री कद प्रभायित कर रही
है ।
• ढाोंिागि मुद्दे:
 कुशि िनशम्भक्त की कमी: एफपीआई के पास एक बडा असोंगयठत िगन है , यजनके पास आिुयनक तकनीक, औपचाररक प्रयर्क्षण और
सोंगयठत बाजार चैनिदों तक पहुोंच का अभाि है ।
 अपयािप्त रसि: पररिहन के पारों पररक तरीकदों पर यनभनरता, अपिान प्त प्रार्यमक प्रसोंस्करण, भोंडारण और यितरण सुयििाओों के
पररणामस्वरूप खेत से 30 प्रयतर्त से अयिक उपज का नुकसान हदता है ।
 कम िाोंड मूल् और िाकि: भारतीि िाों डदों की यिदे र्ी बाजारदों में बहुत कम िा कदई ताकत नहीों है , केिि भारतीि मूि के िदग ही
भारतीि खाद्य िाों डदों और उत्पाददों कद पहचानने में सक्षम हैं ।
• र्नयामक िािािरर् में कर्मयााँ:
 र्िर्ध की बहुिता के कारण यियर्ष्टताओों में यिरदिाभास, परस्पर यिरदिी दृयष्टकदण, समन्वि की कमी और प्रर्ासयनक दे री हदती है ।
सरकारी पहि:
• एिडीआई नीर्ि: खाद्य प्रसोंस्करण के यिए 100% एफडीआई की
अनुमयत।
• प्रधान मोंिी र्कसान सोंपिा य िना (पीएमकेएसिाई): िह खाद्य
प्रसोंस्करण और सोंरक्षण के यिए आिुयनक बुयनिादी ढाोंचे का यनमान ण करती
है , खाद्य प्रसोंस्करण एसएमई के यिए यिि तक पहुोंच में सुिार करती है
और इन उद्यमदों कद प्रयर्क्षण और तकनीकी सहािता प्रदान करती है ।
• पीएम सूक्ष्म खाद्य प्रसोंस्करर् उद्यम ों का औपिाररकीकरर्
(पीएमएिएमई) य िना: िह िदजना क्रेयडट यिोंक्ड सल्किडी के माध्यम
से दद िाख की मौयिक सहािता प्रदान करती है ।
• खाद्य प्रसोंस्करर् उद्य ग के र्िए उत्पािन से िुड़ी प्र त्साहन य िना
(पीएिआईएसएिपीआई): इस िदजना का उद्े श्य िैयश्वक खाद्य
यियनमान ण चैंयपिन के यनमान ण का समर्नन करना; खाद्य उत्पाददों के भारतीि
िाों डदों कद बढािा दे ना है ।
आगे की राह:
आपूर्िि पक्ष और बुर्नयािी ढाोंिे की बाधाओों क िू र करने के र्िए नीर्िगि पहि:
• बैकिडि र्िोंकेि: यकसानदों कद सीिे खाद्य प्रसोंस्करण कोंपयनिदों से जदडकर - माों ग कद आपूयतन से जदडकर टू टी हुई आपूयतन श्रृोंखिा कद ठीक
करना।
• र्निी क्षेि की र्ागीिारी: यनजी कोंपनी रसद और भोंडारण सुयििाओों में यनिेर् करके, यनिान त बाजारदों में भारतीि खाद्य उत्पाददों के िाोंड
मूल्य और ताकत कद बनाने और बढािा दे ने के यिए आिश्यक गयतयियििदों के यििपदषण में मदद कर सकते हैं ।
• प्रौद्य र्गकी उन्नयन: मौजूदा सुयििाओों का प्रौद्यदयगकी उन्निन और यनिेर्कदों कद राजकदषीि प्रदत्साहन प्रदान करके अनुसोंिान और यिकास,
पैकेयजोंग, खाद्य प्रसोंस्करण उपकरण यनमान ण जैसे सहािक उद्यदगदों के यिकास में यनिेर्।
• सोंस्थागि ऋर्: उच्च िागत प्रौद्यदयगकी अपनाने और पैमाने में िृल्कद्ध की सुयििा के यिए ऋण तक बेहतर पहुों च सक्षम करना।
• िागरूकिा और क्षमिा र्नमािर्: फसि क्षयत कद सीयमत करने और खेतदों में रसािनदों के अत्ययिक उपिदग कद रदकने के यिए नई तकनीकदों
के बारे में जागरूकता बढाना और छदटे यकसानदों (एसएचएफ) के बीच कुर्ि कृयष ज्ञान का प्रसार करना।

54
र्िर्नयामक सोंरिना क सुव्यिम्भस्थि करना:
• र्सोंगि र्िोंड िीयरें स: कई यिभागदों और कानूनदों कद एक ही यिोंडद के तहत िाकर मोंजूरी िेने में आने िािी बािाओों कद दू र करें ।
कुशि िनशम्भक्त की बढ़िी माोंग क पूरा करने के र्िए मानि
प्र. िद्ययप भारत में खाद्य प्रसोंस्करण उद्यदग ने िगातार यिकास का
सोंसाधन र्िकास:
अनुभि यकिा है , िेयकन अभी भी महत्वपूणन अप्रिुक्त क्षमता का
• व्यािसार्यक पाठ्यक्रम ,ों प्रसोंस्करर्, िथा कृयष और ऐसे सोंबद्ध
इस्तेमाि हदना बाकी है । यििेचना कीयजए। (250 र्ब्द)
क्षेत्रदों में सूिना प्रौद्य र्गकी का प्रय ग।
• सोंयुक्त सहय ग: उद्यदग के यिकास कद समर्नन दे ने के यिए सरकार और उद्यदग के बीच सहिदग आिश्यक है , यजससे न केिि क्षेत्र कद
बल्कि कृयष उद्यदग और समग्र अर्नव्यिस्र्ा कद भी िाभ हदगा।

3.11 र्ारि में से मीकों डरर र्िर्नमाि र्


सोंिर्ि: हाि ही में, िॉक्सकॉन टे क्न िॉिी ग्रुप ने िेिाोंिा र्िर्मटे ड के सार् अपने सोंिुक्त उद्यम से अपना समर्नन िापस िे यििा। इसमें
गुिराि में सेमीकोंडरर र्िर्नमािर् सोंयोंि स्र्ायपत करने की िदजना र्ी।
सेमीकोंडरर के बारे में
• िह एक ऐसी सामग्री है यजसमें एक कोंडरर और एक इन्सुिेटर के बीि र्िि् युि िािकिा ह िी है।
 इिेररॉयनक्स में, िह यकसी उपकरण में र्िि् युि धारा के प्रिाह क र्नयोंर्िि करिा है।
 सेमीकोंडरर के घटक: मुख्य रूप से यसयिकॉन, जमेयनिम, गैयििम आसेनाइड, आयद।
• उपय ग: कार, स्माटि ि न, र्िर्कत्सा उपकरर्, र्िमान और हर्थयार ों जैसे इिेररॉयनक उपकरणदों में इनका उपिदग यकिा जाता है ।
• उत्पािन: िह प्रयक्रिा अत्ययिक यियर्ष्ट और स्वि सुयििाओों में की जाती है , यजन्हें र्िप र्नमािर् सोंयोंि या फाउों डरी िा फैब कहा जाता है ।
 इसमें ि ट र्िथ ग्रार्िक और रासायर्नक प्रसोंस्करर् के कई िरर् शार्मि हैं , जैसे यक शुद्ध अधििािक सामग्री, आमिौर पर
र्सर्िकॉन से बने िेिर पर र्िि् युि गुर् ों का िमाि, र्नष्कासन, पैटर्निंग और सोंश धन।

र्ारि सेमीकोंडरर बािार


• इों र्डया इिेररॉर्नक्स एों ड सेमीकोंडरर एस र्सएशन (आईईएसए) की ररप टि के अनुसार, उपर् क्ता इिेररॉर्नक्स, िू रसोंिार और
आईटी हाडि िेयर िैसे क्षेि ों से महत्वपूर्ि माोंग के साथ, भारत का सेमीकोंडरर बाजार 2026 तक 64 यबयििन डॉिर तक पहुोंचने की
उम्मीद है ।
 2019 में दे र् का सेमीकोंडरर बाजार 22.7 र्बर्ियन डॉिर का था।

55
र्ारि में सेमीकोंडरर र्िर्नमािर् क बढ़ािा िे ने के िार् र्ारि में सेमीकोंडरर र्िर्नमािर् क बढ़ािा िे ने के र्िए र्िर्र्न्न पहि
• अथिव्यिस्था क बढ़ािा: सेमीकोंडरर उद्यदग रदजगार के • सेमीकोंडरर और र्डस्प्ले र्िर्नमािर् पाररम्भस्थर्िकी िोंि के र्िकास के
अिसर पैदा कर सकता है , यनिान त बढा सकता है और यिदे र्ी र्िए सोंश र्धि कायिक्रम: केंि सरकार ने 2021 में एक स्र्ािी
यनिेर् आकयषनत कर सकता है । सेमीकोंडरर और यडस्प्ले पाररल्कस्र्यतकी तोंत्र के यिकास के यिए 76,000
• िकनीकी उन्नर्ि: यह र्ारि क इिेररॉर्नक्स, डे टा कर ड़ रुपये के कायिक्रम की घ षर्ा की।
प्र सेर्सोंग, ऑट म र्टि आर्ि में अत्याधुर्नक िकनीक  प्र त्साहन: सेमीकोंडरर यनमानण सुयििा की स्र्ापना के यिए
यिकयसत करने और िैयश्वक बाजार में अपनी प्रयतस्पिानत्मकता पररिदजना िागत का 50% यििीि समर्नन, सभी प्रौद्यदयगकी नदड् स में
एक समान।
बढाने में मदद कर सकता है ।
 इसका उद्दे श्य उन कोंपर्नय ों /कोंसदयटन िा कद आकषनक प्रदत्साहन
• सामररक महत्व: सेमीकोंडरर उद्यदग का रक्षा, अोंिररक्ष और
सहािता प्रदान करना है जद यसयिकॉन सेमीकोंडरर फैि, यडस्प्ले
िू रसोंिार िैसे र्िर्र्न्न क्षेि ों में समाररक महत्व है। इस उद्यदग
फैि, कोंपाउों ड सेमीकोंडरर/यसयिकॉन फदटदयनक्स/सेंसर
कद बढािा दे ने से भारत कद इन महत्वपूर्ि क्षेि ों में (एमईएमएस सयहत) फैि, सेमीकोंडरर पैकेयजों ग
आत्मर्नर्िरिा हार्सि करने में मिि र्मि सकिी है। (एटीएमपी/ओएसएटी) और सेमीकोंडरर यडजाइन में िगे हुए हैं ।
• र्डर्िटि र्िर्ािन क सोंब र्धि करना: यह िनिा क • इों र्डया सेमीकोंडरर र्मशन (आईएसएम): िह र्डर्िटि इों र्डया
सस्ती और सुिर् िकनीक प्रदान करके भारत में यडयजटि कॉपोरे शन के र्ीिर एक र्िर्शि और स्विोंि र्बिनेस र्डिीिन है ,
यिभाजन कद पाटने में महत्वपूणन भूयमका यनभा सकता है । यजसका उद्े श्य इिेररॉयनक्स यियनमानण और यडजाइन के यिए एक िैयश्वक
• र्ारि द्वारा उपिब्ध अिसर: भारत सेमीकोंडरर उत्पाददों के केंि के रूप में भारत के उद्भि कद सक्षम करने के यिए एक जीिोंत
यिए एक यिर्ाि उपभदक्ता आिार प्रदान करता है , जद उद्यदग सेमीकोंडरर और यडस्प्ले इकदयसस्टम का यनमानण करना है ।
के यिए एक मजबूत माों ग चािक प्रदान करता है । • र्डजाइन र्िों क्ड इों सेंर्टि (डीएिआई) य िना: दे र् में यचप यडजाइयनोंग
में मूल् िर्धिि अनुसोंधान में बदिाि कद प्रदत्सायहत करना।
 र्ारि का प्रर्िर्ा पूि सेमीकोंडरर र्िकास क बढ़ािा
िे सकिा है और कौशि र्िकास और निािार पर
• महत्वपूर्ि और उर्रिी प्रौद्य र्गर्कय ों पर पहि (iCET): यह कृर्िम
बुम्भद्धमिा, क्वाोंटम कोंप्यूर्टों ग, सेमीकोंडरर और िायरिेस िू रसोंिार
महत्वपूर्ि ि र दे कर घरे िू यचप यडजाइन कौर्ि कद
जैसे क्षेत्रदों में महत्वपूर्ि और उर्रिी प्रौद्य र्गर्कय ों पर सहय ग के र्िए
प्रदत्सायहत कर सकता है ।
र्ारि और अमेररका द्वारा सहमत एक रूपरे खा है ।
 अनुर्ि: इों टेि और एनिीआईडीआईए जैसी यचप बनाने
िािी कोंपयनिदों की र्ारि में बड़ी सुर्िधाएों हैं ि र्डिाइन समस्याओों पर काम कर रही हैं।

र्ारि में सेमीकोंडरर र्िर्नमािर् में आने िािी िुनौर्ियााँ हैं:


• कुशि कायिबि की कमी: इों यडिन सेमीकोंडरर एसदयसएर्न की एक ररपदटन के अनुसार, र्ारि सेमीकोंडरर उद्य ग में िगर्ग
250,000 कुशि श्र्मक ों की कमी का सामना कर रहा है।
• बुर्नयािी ढााँिा: र्िर्नमािर् के र्िए र्बििी, पानी, िािािरर् और उपकरर् जैसे बुयनिादी ढाँचे की आिश्यकता हदती है , यजसे स्र्ायपत
करना महोंगा हद सकता है ।
• िर्टि र्िर्नयामक िािािरर्: केंि और राज्य सरकार के र्नयम ों और र्िर्नयम ों के बीि अोंिर र्िसे पार करना मुम्भिि ह सकिा
है।
 उिाहरर्: सेमीकोंडरर र्नमािर् के र्िए पयाििरर्ीय मोंिूरी जयटि हद सकती है ।
• पूोंिी गहन प्रर्क्रया: अपने उच्च जदल्कखम, िोंबी अिर्ध (आमिौर पर पाोंि िषि) और र्ुगिान समय और बड़े पूोंिी व्यय के कारर्,
सेमीकोंडरर यियनमान ण एक जयटि उद्यदग है यजसके यिए बडे पैमाने पर और यनरों तर यनिेर् की आिश्यकता हदती है ।
• प्रर्िस्पधाि: भारत के सेमीकोंडरर उद्यदग कद चीन, ताइिान और दयक्षण कदररिा जैसे िे श ों से कड़ी प्रर्िस्पधाि का सामना करना पड़िा
है, िहाों पहिे से ही अच्छी िरह से स्थार्पि उद्य ग हैं। 2020 में, िैयश्वक सेमीकोंडरर यबक्री में भारत की यहस्सेदारी र्सिि 0.3% थी, ि
िीव्र प्रर्िस्पधाि क उिागर करिी है।
 उिाहरर्: यचप्स अयियनिम के माध्यम से, अमेररकी सरकार सेमीकोंडरर िैब्स स्थार्पि करने के र्िए प्र त्साहन प्रिान कर रही
है ि अगिे 3-4 िषों में िािू ह सकिे हैं।
• आिात पर यनभनरता: आईएसए के अनुसार, र्ारि अपनी सेमीकोंडरर आिश्यकिाओों का िगर्ग 80% आयाि करिा है।
• र्ू-रािनीर्ि: ताइिान जिडमरूमध्य के आसपास चि रहे तनाि ने भारत कद भू-राजनीयतक खेि में उजागर कर यदिा है ।
 टीएसएमसी िैसी िाइिानी कोंपर्नय ों के पास सेमीकोंडरर यियनमान ण के यिए 60% बाजार यहस्सेदारी और अत्याधुर्नक र्िप्स के
र्िए 90% बािार र्हस्सेिारी है।

56
 इसी तरह, िीन ने सेमीकोंडरर यनमान ण में उपिदग यकए जाने िािे दद महत्वपूणन दु िनभ तत्वदों-गैयििम और जमेयनिम-पर यनिानत यनिोंत्रण
िगािा।

आगे की राह
• आत्मर्नर्िरिा क बढ़ािा िें : भारत सरकार कद र्निेशक ों और प्रौद्य र्गकी प्रिािाओों क आकर्षिि करने के यिए सेमीकोंडरर
पाररल्कस्र्यतकी तोंत्र के पदषण में घरे िू अोंिराि क सोंब र्धि करना िार्हए।
 इसे यचप यनमान ण के यिए आिश्यक यिर्ेषज्ञता और यसस्टम यिकयसत करने के यिए सेमीकोंडरसि की असेंबिी, पैकेर्िोंग और
परीक्षर् पर ध्यान केंर्द्रि करना िार्हए।
 कौशि र्िकास: सामग्री र्िज्ञान, र्सस्टम-ऑन-र्िप (एसओसी), र्सस्टम मॉडर्िोंग, इिेरर मैिेर्टक्स, र्िप यडजाइयनोंग आयद जैसे
क्षेत्रदों में कई तकनीयर्िनदों, इों जीयनिरदों और र्दिकतानओों के कौशि र्िकास और रर -म्भस्कर्िोंग की आिश्यकता है ।
• ऊिाि बुर्नयािी ढाोंिे क मिबूि
करना: र्ारि क यबजिी के 24*7
यिश्वसनीि स्रदत प्रदान करने के
यिए एक मिबूि ऊिाि बुर्नयािी
ढाोंिे की आिश्यकिा है।
• रर्नीर्िक साझेिारी का
र्नमािर्: भारत का सेमीकोंडरर
उद्यदग र्ू-रािनीर्िक िनाि ों से
र्नपटने और दु िनभ पृथ्वी िातुओों
जैसे आिश्यक उत्पाददों की आपूयतन
सुयनयित करने की अपनी क्षमता
पर यनभनर करे गा।
 उत्पािन क सोंर्ि बनाने के
र्िए र्ारि क सोंयुक्त राज्
अमेररका िैसे समान
र्ििारधारा िािे िे श ों के साथ
रणनीयतक गठबोंिन बनाना
चायहए।
• एबीसीडीई दृर्िक र् का पािन करना िैसा र्क इन्फ ग्रार्िक में र्िखाया गया है:

िघु समािार

3.12 यू र्निसि ि बे र्सक इनकम अर्धकार, और गारों टीकृि सामार्िक सुरक्षा पेंशन का
सोंिर्ि: अर्धकार।
हाि ही में, राजस्र्ान सरकार ने रािस्थान न्यूनिम गारों टी आय • काम की गारों टी: राज्य के सभी पररिारदों कद हर साि 125 यदनदों
र्िधेयक , 2023 पेर् यकिा, जद प्रभािी रूप से राज्य की सोंपूणन का गारों टीकृत रदजगार यमिता है ।
ििस्क आबादी कद गारों टीकृत मजदू री के सार् किर करने का • कमि र िगि के र्िए पेंशन: िृद्ध ,ों र्िकिाोंग ,ों र्िधिाओों
प्रिास करता है । और एकि मर्हिाओों क न्यूनतम 1,000 रुपये प्रर्ि माह
के बारे में: पेंशन र्मििी है। पेंशन में हर साि 15 प्रर्िशि की दर से
• यूर्निसिि बेर्सक इनकम: िह एक सामायजक कल्याण िृल्कद्ध की जाएगी।
प्रस्ताि है यजसमें सभी िाभायर्निदों कद यनियमत रूप से यबना र्तन 3.13 िै र्िक प्रर्िस्पधाि त्म किा सू ि काों क
भुगतान के बाद हस्ताोंतरण कद रखें भुगतान के रूप में एक
सोंिर्ि:
गारों टीकृि आय प्राप्त ह िी है।
इों टरनेशनि इों स्टीट्यूट िॉर मैनेिमेंट डे ििपमेंट (आईएमडी)
• किरे ि: िह यििेिक तीन श्रेयणिदों कद किर करता है : न्यूनिम
द्वारा जारी निीनतम यिश्व प्रयतस्पिानत्मकता रैं यकोंग में भारत 40िें
गारों टीकृि आय का अर्धकार, गारों टीकृि र िगार का
स्थान पर है।

57
िैर्िक प्रर्िस्पधाित्मकिा सूिकाोंक के बारे में: ि न ों के रूप में कायि करे गा और सर्ी एआई-सोंबोंर्धि
• आईएमडी यिश्व प्रयतस्पिान त्मकता िायषनकी (WCY), पहिी बार ड मेन के र्िए एक सिाहकार की र्ूर्मका र्नर्ाएगा।
1989 में प्रकार्शि हुई, िह एक व्यापक िायषनक ररपदटन है जद • एआई र्सस्टम क िगीकृि करना: उनके ि म्भखम ों के
यकसी दे र् की प्रयतस्पिानत्मकता के यिए िैयश्वक सोंदभन यबोंदु के आिार पर और र्िम्मेिार एआई के र्सद्धाोंि ों के अनुसार उन्हें
रूप में कािन करती है । यियनियमत करना।
• मुि मूल्ाोंकन मानिों ड र्िसमें 334 प्रर्िस्पधाित्मकिा • इिेररॉर्नक्स और सूिना प्रौद्य र्गकी मोंिािय (MeiTy) क
पैरामीटर शार्मि हैं AI के यिए प्रर्ासयनक मोंत्रािि हदना चायहए।
1. आयर्नक प्रदर्नन र्ारिीय िू रसोंिार र्नयामक प्रार्धकरर् (टर ाई) के बारे में
2. सरकारी दक्षता • इसकी स्र्ापना 1997 में र्ारिीय िू रसोंिार र्नयामक
3. व्यािसायिक दक्षता प्रार्धकरर् अर्धर्नयम, 1997 द्वारा की गई थी।
4. आिारभूत सोंरचना • टर ाई का कायि
 एक र्नष्पक्ष और पारिशी नीर्ि िािािरर् प्रिान
िैर्िक रैं र्कोंग: करना जद समान अिसर कद बढािा दे ता है और यनष्पक्ष
• िायषनक ररपदटन में सूचीबद्ध 64 अर्नव्यिस्र्ाओों में से , डे नमाकि, प्रयतस्पिान की सुयििा प्रदान करता है ।
आयरिैंड और म्भस्वट् िरिैंड ने शीषि िीन स्थान ों पर दािा • यह िू रसोंिार सेिाओों क र्नयोंर्िि करिा है र्िसमें
यकिा। दू रसोंचार सेिाओों के यिए टै ररफ का यनिानरण/सोंर्दिन र्ायमि
है जद पहिे केंि सरकार में यनयहत र्े।
3.14 र्ौग र्िक सों के ि टै ग
सोंिर्ि: 3.16 र्ििररि निीकरर्ीय ऊिाि
हाि ही में, चेन्नई में र्ौग र्िक सोंकेि रर्िस्टर ी ने उिर प्रदे र् के सोंिर्ि: हाि ही में, भारत की G20 प्रेसीडें सी ने अपनी अोंर्िम ऊिाि
सात अिग-अिग उत्पाददों कद टै ग यदए हैं । पररिििन कायि समूह की बैठक आय र्िि की, र्िसमें सिि
साि उत्पाि हैं:
र्िकास िक्ष्य 7 - सभी के यिए सस्ती, यिश्वसनीि, यटकाऊ और
• अमरदहा ढदिक, महदबा गौरा पत्थर हस्तयर्ल्प, मैनपुरी तारकर्ी,
आिुयनक ऊजान प्रदान करने के यिए यिकेंिीकृत निीकरणीि ऊजान
सोंभि हॉनन क्राफ्ट, बागपत हदम फयननयर्ोंग्स, बाराबोंकी हैं डिूम
(DRE) के उपिदग पर ध्यान केंयित यकिा गिा।
उत्पाद और कािपी हस्तयनयमनत कागज।

र्ौग र्िक सोंकेि टै ग के बारे में:


• िह उन उत्पाि ों पर उपय ग र्कया िाने िािा एक र्िन्ह है
र्िनकी एक र्िर्शि र्ौग र्िक उत्पर्ि ह िी है और उनमें
उस उत्पयि के कारण गुण िा प्रयतष्ठा हदती है ।
• औद्यदयगक सोंपयि की सुरक्षा के यिए पेररस कन्वेंशन के तहत
भौगदयिक सोंकेतदों कद बौम्भद्धक सोंपिा अर्धकार (आईपीआर)
के एक घटक के रूप में शार्मि र्कया गया है।
• भौगदयिक सोंकेत आमतौर पर कृर्ष उत्पाि ,ों खाद्य पिाथों,
िाइन और म्भस्पररट पेय, हस्तर्शल्प और औद्य र्गक उत्पाि ों
के र्िए उपय ग र्कए िािे हैं।
• पहिा उत्पाि यजसे िषि 2004-05 में जीआई टै ग यदिा गिा िह र्ििररि ऊिाि क्ा है?
िार्ििर्िोंग िाय है। • पररर्ाषा: यह पारों पररक पािर र्ग्रड से िुड़े र्बना एक
3.15 एआई के र्िए टर ाई की र्सिाररश र्िर्शि क्षेि के र्ीिर सौर और पिन ऊजान जैसे स्वि स्रदतदों
सोंिर्ि: हाि ही में, र्ारिीय िू रसोंिार र्नयामक प्रार्धकरर् से यबजिी उत्पादन कद सोंदयभनत करता है ।
(TRAI) ने सभी क्षेत्रदों में कृयत्रम बुल्कद्धमिा (AI) के यिकास कद • स्थानीयकृि र्बििी आपूर्िि: यग्रड-स्केि सौर और पिन ऊजान
यनिोंयत्रत करने के यिए यसफाररर्दों का एक सेट जारी यकिा। के यिपरीत, जद अोंतर-राज्य यबजिी पारे षण प्रणािी से जुडे हदते
र्सफाररश ों का महत्वपूर्ि अोंश: हैं , यितररत ऊजान समािान स्थानीयकृि र्बििी आपूर्िि प्रिान
• एक स्वतोंत्र िैिायनक प्रायिकरण, आर्टि र्िर्शयि इों टेर्ििेंस करिे हैं।
एों ड डे टा अथॉररटी ऑि इों र्डया (एआईडीएआई) की  इससे उत्पन्न यबजिी का स्थानीय स्तर पर उपर् ग र्कया
स्थापना करें , ि एक र्नयामक और अनुशोंसा र्नकाय िािा है, यजससे पारों पररक जीिाश्म ईोंिन-आिाररत यबजिी

58
पर यनभनरता कम हद जाती है और टर ाों सयमर्न हायन कम हद र्ारि में टमाटर का उत्पािन
जाती है । • र्ौग र्िक सघनिा: प्रमुख टमाटर उत्पादक राज्यदों में आों ध्र
प्रदे र्, मध्य प्रदे र्, कनानटक, ओयडर्ा और गुजरात र्ायमि हैं ,
3.17 मू ल् म्भस्थरीकरर् क ष जद कुि उत्पादन का 50% यहस्सा हैं ।
सोंिर्ि: • मौसमी िसिें: मई से यसतम्बर/अरू बर खरीफ की फसि,
निम्बर से माचन/अप्रैि तक रबी की फसि। कुछ क्षेत्र मानसून
सरकार ने टमाटर की कीमतदों में िृल्कद्ध कद सोंबदयित करने के यिए
के दौरान टमाटर उगाते हैं (उदाहरण के यिए, महाराष्टर और
मूल् म्भस्थरीकरर् क ष के िहि टमाटर की खरीि शुरू की है।
यहमाचि प्रदे र् का सदिन), जबयक आों ध्र प्रदे र् का मदनपल्ले
मूल् म्भस्थरीकरर् क ष (पीएसएि) के बारे में:
क्षेत्र ग्रीष्मकािीन उत्पादन में प्रमुख है ।
• स्थापना: इसकी स्र्ापना 2014-15 की अियि के दौरान कृर्ष, • उत्पािन रुझान: 2019-20 में 21.187 मीयटर क टन पर पहुों च
सहय ग और र्कसान कल्ार् र्िर्ाग (डीएसी एों ड गिा, यजसके बाद से यगरािट आ रही है - 2021-22 में 20.69
एिडब्ल्यू) के िहि की गई थी। मीयटर क टन, 2022-23 में 20.62 मीयटर क टन।
 1 अप्रैि 2016 कद इसे कृर्ष, सहय ग और र्कसान र्ारि में टमाटर की कीमि ों में िृम्भद्ध कई कारक ों से प्रे ररि है:
कल्ार् र्िर्ाग (डीएसी एों ड एिडब्ल्यू) से उपर् क्ता • िरम मौसम की म्भस्थर्ि: अप्रैि और मई में िू और उच्च
मामिे र्िर्ाग (DOCA) में स्थानाोंिररि कर र्िया गया तापमान के सार्-सार् दयक्षणी भारत और महाराष्टर में मानसून
था। की बाररर् में दे री के कारण टमाटर की फसि पर कीटदों का
• उद्दे श्य: प्याि, आिू और िाि ों जैसी महत्वपूणन कृयष- हमिा हुआ।

बागिानी िस्तुओों की कीमत में अल्कस्र्रता कद यनिोंयत्रत करना। • कम िार्र्म्भज्क प्राम्भप्त: यकसानदों कद जून से पहिे के महीनदों
में और िहाों तक यक यपछिे िषन में भी फसि की कम
• इस फोंड का उपिदग राज्य सरकारदों/केंि र्ायसत प्रदे र्दों (िूटी)
िायणल्कज्यक प्राल्कप्त का सामना करना पडा। पररणामस्वरूप,
और केंिीि एजेंयसिदों कद उनकी कािनर्ीि पूोंजी और अन्य खचों
कई यकसानदों ने अपनी फसिें कम कीमतदों पर बेच दीों िा
कद यििपदयषत करने के यिए ब्याि मुक्त ऋर् िे ने के र्िए टमाटर की खेती पूरी तरह से छदड दी।
र्कया िािा है, र्िससे िे ऐसी िस्तुओों की खरीि और • आपूर्िि में कमी: चरम मौसम की ल्कस्र्यत और कम कीमतदों के
र्ििरर् में खिि कर सकिे हैं। सोंिदजन के कारण, कई यकसानदों ने अपनी जद भी फसि र्ी
• इस फोंड का प्रबोंिन केंिीि रूप से मूल् म्भस्थरीकरर् र्नर्ध उसे कम दरदों पर बेच यदिा, यजससे टमाटर की आपूयतन में कमी
प्रबोंधन सर्मर्ि (पीएसएिएमसी) द्वारा र्कया िािा है। हद गई।
• िघु र्कसान कृर्ष व्यिसाय कोंस र्टि यम (एसएिएसी)
पीएसएफ कद केंिीि कॉपनस फोंड के रूप में बनाए रखता है ।
टमाटर सोंकट:
• भारतीि ररजिन बैंक ने टमाटर की कीमतदों में अल्कस्र्रता पर
प्रकार् डािा है ।

डे टा पॉइों ट् स

3.18 थ क मू ल् सू ि काों क मु द्रास्फीर्ि के रुझान • WPI डे टा: WPI िार्र्ज् और उद्य ग मोंिािय के िहि
सोंिर्ि: उद्य ग और आों िररक व्यापार सोंिधिन र्िर्ाग (DPIIT) के
भारत की र्दक कीमतें जून में िगािार िीसरे महीने अपस्फीर्ि आर्थिक सिाहकार कायाििय द्वारा प्रकायर्त यकिा जाता है।
की म्भस्थर्ि में रही ों , मूल्य सोंकुचन की दर मई में -3.48% से • आधार िषि: 2011-12
बढ़कर -4.12% ह गई।
• िह िगर्ग आठ िषों में थ क कीमि ों में सबसे िीव्र सोंकुिन
क िशाििा है।
थ क मूल् सूिकाोंक:
• पररर्ाषा: िह एक सोंकेतक है जद र्दक बाजार में र्दक में बेची
जाने िािी िस्तुओों की कीमि में औसि पररिििन र्नधािररि
करिा है। िह सूचकाों क खुदरा यिक्रेता तक पहुोंचने से पहिे
यियभन्न चरणदों में कमदयडटी की कीमतदों में बदिाि की गणना
करने में उपिदगी है । WPI की गर्ना केिि िस्तुओों के र्िए
की िािी है।

59
3.19 िषि की पहिी र्िमाही में र्नयाि ि में र्गरािट से बचता है यजससे अोंयतम उपभदक्ता पर कर का बदझ नहीों
बढता है।
सोंिर्ि:
• िह आपूयतन िाइन में यबक्री के प्रत्येक यबोंदु पर िस्तुओों और
• िािू र्िि िषि की पहिी र्िमाही में माि यनिानत में 15.1%
सेिाओों पर एकत्र यकिा जाता है।
की यगरािट आई है । िह िषि 2022-23 में 450 अरब डॉिर
• 101िाों सोंिैधार्नक सोंश धन अर्धर्नयम, 2016:
के व्यापाररक र्नयािि की ररकॉडि उपिम्भब्ध की पृष्ठर्ूर्म में
 िह सोंर्दिन अयियनिम केंि और राज्यदों ददनदों कद उत्पाद
है।
र्ुि, चुोंगी कर, सीमा र्ुि, सेिा कर, प्रिेर् कर,
• र्पछिे िीन महीन ों के रुझान: अप्रैि और मई में, िस्तुओों का
मनदरों जन कर इत्यायद का आकिन करने की अनुमयत दे ता
यनिान त क्रमशः 12.6% और 10.2% धीमा ह गया। हािाँयक,
है , जद सभी जीएसटी द्वारा प्रयतस्र्ायपत यकए जाते हैं , यजससे
िून में 22% की र्गरािट के साथ , िे 37 महीनदों में सबसे
िह एक अप्रत्यक्ष कर बन जाता है ।
अयिक यगर गए। यपछिे महीने 32.7 अरब डॉिर का कुि
• िीएसटी पररषि:
यनिान त अरू बर 2022 के बाि सबसे कम था।
 भारतीि सोंयििान के अनुिेद 279ए के अनुसार, जीएसटी
3.20 िस्तु एिों से िा कर के छह िषि का प्रर्ासन और सोंचािन करने के यिए राष्टरपयत द्वारा
सोंिर्ि: िस्तु एिों सेिा कर (जीएसटी) के रूप में आजादी के बाद से जीएसटी पररषद का गठन यकिा जाएगा।
भारत के सबसे पररितननकारी कर सुिार के इस साि जुिाई में छह  सोंघटन:
साि पूरे हद गए।  इसके अध्यक्ष भारत के केंिीि यिि मोंत्री हैं और राज्य
सरकारदों द्वारा नायमत मोंत्री इसके सदस्य हैं।
 मििान शम्भक्त: पररषद कद इस तरह से तैिार यकिा गिा
है यक केंि के पास 1/3 मतदान र्ल्कक्त हदगी और राज्यदों के
पास 2/3 हदगी।
 यनणनि 3/4 सिस्य ों के बहुमि से र्िये िािे हैं।

छह िषों में िीएसटी सोंग्रह डे टा:


िषि मार्सक औसि (कर ड़ में)

2017 ₹89,885

2022 ₹1.5 िाख कर ड़


िस्तु एिों सेिा कर (िीएसटी) के बारे में:
अप्रैि 2023 ₹1.87 िाख कर ड़
• उपर् ग कर: जीएसटी मूितः एक उपभदग कर है और अोंयतम
उपभदग यबोंदु पर िगािा जाता है । िीएसटी के र्िए करिािाओों का आधार:
• गोंिव्य र्सद्धाोंि: जीएसटी करािान में प्रिुक्त यसद्धाों त गोंतव्य
यसद्धाों त है। िषि करिािा आधार (िाख में)
• िेिी: िह मूल्यििनन पर िगािा जाता है और सेट-ऑफ प्रदान
2017 63.9
करता है ।
 कास्केर्डों ग इफेर से बिाि: पररणामस्वरूप, िह कर पर मौजूदा 140.0
पडने िािे कास्केयडों ग प्रभाि अर्िा कर पर िगने िािे कर

िार्िका में सों र्क्षप्त समािार

सेबी र्शकायि र्निारर् सेबी के स्कदसन प्लेटफॉमन के माध्यम से कोंपयनिदों और बाजार मध्यस्र्दों के ल्कखिाफ कुि 3,079 यर्काितदों का यनपटारा
प्रर्ािी (स्क र) यकिा गिा है ।
प्लेटिामि स्क सि प्लेटिामि के बारे में:
• िह सेबी की एक िेब आिाररत केंिीकृत यर्काित यनिारण प्रणािी है यजसे जून 2011 में िॉन्च यकिा गिा र्ा।

60
• स्कदसन यनिेर्कदों कद अपनी यर्काितें दजन करने और उन पर कारन िाई करने तर्ा ऐसी यर्काितदों के यनिारण
की ल्कस्र्यत कद उपरदक्त िेबसाइट से कहीों से भी ऑनिाइन टर ै क करने में सक्षम बनाता है ।
र्ारिीय प्रर्िर्ूर्ि और र्िर्नमय ब डि (सेबी) के बारे में: अप्रैि 1988 में एक कािनकारी यनकाि के रूप में
स्र्ायपत यकिा गिा और सेबी अयियनिम, 1992 के माध्यम से जनिरी 1992 में इसे िैधार्नक शम्भक्तयााँ िी गईों।

र्गफ्ट र्नफ्टी एसजीएक्स र्नफ्टी क र्गफ्ट र्नफ्टी के रूप में पुनः िाोंड र्कया गया है और $7.5 यबयििन के सभी डे ररिेयटि
अनुबोंि, जद पहिे यसोंगापुर में कारदबार करते र्े, भारत में स्र्ानाोंतररत हद गए हैं ।
र्गफ्ट र्नफ्टी के बारे में:
• र्सोंगापुर स्टॉक एक्सिेंि पर कारदबार यकए जाने िािे यनफ्टी 50 इों डेक्स पर आिाररत एक डे ररिेर्टि अनुबोंध
है।
 र्नफ्टी या र्नफ्टी 50 एक सूिकाोंक है र्िसमें शीषि 50 सूिीबद्ध भारतीि कोंपयनिाों र्ायमि हैं जद नेशनि
स्टॉक एक्सिेंि (एनएसई) पर कार बार करिी हैं।
• महत्व: इससे भारतीि बाजारदों कद िैयश्वक यनिेर्कदों के बीच अपनी पहुोंच बढाने में मदद यमिेगी, खासकर उन
िदगदों के बीच जद सीिे तौर पर भारतीि पूोंजी बाजार से नहीों जुडे हैं ।

अोंिराििरीय र्ििीय सेिा हाि ही में, के राजारमन कद अोंतरान ष्टरीि र्ििीय सेिा केंद्र प्रार्धकरर् (आईएिएससीए) का अगिा अध्यक्ष यनिुक्त
केंद्र प्रार्धकरर् यकिा गिा है ।
(आईएिएससीए) आईएिएससीए के बारे में
• अोंिराििरीय र्ििीय सेिा केंद्र प्रार्धकरर् अर्धर्नयम, 2019 के तहत स्र्ायपत एक िैधार्नक प्रार्धकरर् है।
• अर्धिे श: अोंतरान ष्टरीि यििीि सेिा केंिदों ('आईएफएससी') में र्ििीय उत्पाि ,ों र्ििीय सेिाओों और र्ििीय
सोंस्थान ों क र्िकर्सि और र्िर्नयर्मि करना।
 अोंतरान ष्टरीि यििीि सेिा केंिदों में व्यापार करने में आसानी कद बढािा दे ने और यिश्व स्तरीि यनिामक िातािरण
प्रदान करने के यिए समग्र दृर्िक र् के साथ एकीकृि र्नयामक के रूप में कािन करना।

र्टयर 2 बाोंड आईडीएफसी फस्टन बैंक ने कहा यक उसने घरे िू बाों ड बाजार में यटिर2 बाों ड से ₹1,500 करदड जुटाए हैं ।
र्टयर-2 बाोंड के बारे में:
• यटिर-2 बाों ड एक प्रकार का ऋर् साधन है, ि बैंकदों द्वारा अपने पररचािन के यिए पूोंिी िुटाने के र्िए िारी
र्कया िािा है।
• िे बैंक की र्टयर-2 पूोंिी का र्हस्सा ह िे हैं और यटिर-1 पूोंजी के अिीन हदते हैं ।
• बैंकदों कद आरबीआई की मोंजूरी िेने के बाद केस-दर-केस आिार पर यिदे र्ी मुिा यटिर -2 बाोंड भी जारी करने
की अनुमयत है ।
• र्टयर-2 बाोंड के िार्: िह पूोंजी जुटाने के मामिे में बैंकदों कद अयिक िचीिापन प्रदान करता है क्दोंयक इन्हें
बैंक द्वारा आिश्यकता पडने पर जारी और भुनािा जा सकता है ।

ग्रीडफ्लेशन दु यनिा भर में इस बात पर आम सहमयत बन रही है यक कॉपोरे ट िािि मुद्रास्फीर्ि क बढ़ा रहा है, जबयक
श्रयमकदों कद कम िेतन िृल्कद्ध और उच्च ब्याज दरदों से ददगुना दों यडत यकिा जाता है।
ग्रीडफ्लेशन के बारे में:
• इसका सीिा मििब है र्क कॉपोरे ट िािि मुद्रास्फीर्ि क बढ़ािा िे रहा है।
• िह उस ल्कस्र्यत कद दर्ानता है यजसमें कीमिें इसयिए नहीों बढ रही हैं क्दोंयक श्रयमकदों कद अयिक िेतन यमि रहा
है बल्कि इसर्िए क् र्ों क कोंपर्नयाों अर्धक िार् कमा रही हैं।
• ऐसा तब हदता है जब महामारी िा आपदा जैसा सोंकट व्यिसािदों के यिए असािारण मुनाफा कमाने के अिसर
में बिि िािा है।

61
यूर्नफाइड िॉर्िम्भस्टक्स प्रिानमोंत्री ने र्ारि के िॉर्िम्भस्टक्स क्षेि में बििाि में यूर्िप की र्ूर्मका की सराहना की।
इों टरफेस प्लेटफॉमि यूर्िप के बारे में:
(यूर्िप) • भारत के िॉयजल्कस्टक्स क्षेत्र में सुिार के यिए निीन दृयष्टकदण कद सक्षम करने के यिए यहतिारकदों के बीच डे टा
र्िर्नमय के र्िए एक एपीआई आिाररत मोंच है ।
• इसे 2022 में रािरीय िॉर्िम्भस्टक्स नीर्ि (एनएिपी) के एक भाग के रूप में िॉन्च यकिा गिा र्ा।
• िह कागो आिािाही की िास्तर्िक समय पर र्नगरानी प्रिान करे गा और दक्षता बढाएगा और भारत में
रसद की िागत कद कम करे गा।
• यह पररिहन क्षेि से सोंबोंर्धि सभी यडयजटि डे टा कद एक ही पदटन ि पर िाता है ।

स्टीि स्लैग र ड इस्पाि और ग्रामीर् र्िकास राज् मोंिी ने 'अपर्शि से धन' यमर्न कद प्राप्त करने में स्टीि स्लैग र ड
टे क्न िॉिी प्रौद्य र्गकी के महत्व पर प्रकार् डािा।
स्टीि स्लैग के बारे में:
• स्टीि स्लैग स्टीि यनमान ण प्रयक्रिा के दौरान उत्पन्न एक उप त्पाि है। िह एक ठ स अपर्शि पिाथि है जद
इस्पात सोंिोंत्रदों के आसपास बडी मात्रा में जमा हद जाता है ।
• स्टीि स्लैग र ड टे क्न िॉिी: इसे केंिीि सडक अनुसोंिान सोंस्र्ान द्वारा इस्पात मोंत्रािि और प्रमुख इस्पात
यियनमान ण कोंपयनिदों के सहिदग से यिकयसत यकिा गिा है ।
 िह अपयर्ष्ट स्टीि स्लैग के बडे पैमाने पर पुन: उपिदग कद सक्षम बनाता है , यजससे दे र् में उत्पन्न िगभग
19 यमयििन टन का प्रभािी ढों ग से यनपटान हदता है ।

साोंम्भिकी पर स्थायी साोंम्भिकी और कायिक्रम कायािन्वयन मोंिािय (MoSPI) ने आयर्नक साों ल्कख्यकी पर स्र्ािी सयमयत (SCES) कद
सर्मर्ि (SCoS) साोंम्भिकी पर स्थायी सर्मर्ि (SCoS) से बिि र्िया है।
साोंम्भिकी पर स्थायी सर्मर्ि (एससीओएस) के बारे में:
• सिेक्षर् ों की समीक्षा करना: एससीओएस कद रािरीय साोंम्भिकी कायाििय (एनएसओ) के तहत यकए गए
सर्ी सिेक्षर् ों की रूपरे खा और पररर्ाम ों की समीक्षा करने का काम सौोंपा गया है।
• डे टा गैप की पहिान: एससीओएस आर्धकाररक आों कड़ ों में डे टा गैप की पहिान करने के र्िए र्िम्मेिार
है। िह उन क्षेत्रदों का पता िगाएगा जहाों डे टा की कमी है और इन कयमिदों कद दू र करने के यिए रणनीयत तैिार
करे गा।
• प्रशासर्नक साोंम्भिकी की ख ि: सयमयत कद डे टा पररणामदों में सुिार के यिए प्रर्ासयनक साों ल्कख्यकी के उपिदग
का पता िगाने का दायित्व सौोंपा गिा है ।
• सिाहकार र्ूर्मका: िह सिेक्षण के यियभन्न पहिुओों पर मोंिािय क िकनीकी सिाह प्रदान करे गा , यजसमें
नमूनाकरर् फ्रेम, र्डिाइन, सिेक्षर् पद्धर्ि और पररर्ाम ों क अोंर्िम रूप िे ना शार्मि है।

िैंर्पयोंस 2.0 प टि ि सूक्ष्म, िघु और मध्यम उद्यम (एमएसएमई) मोंिािय ने 27 िून क अोंिराििरीय एमएसएमई र्ििस पर चैंयपिोंस
2.0 पदटन ि िॉन्च यकिा।
िैंर्पयोंस 2.0 प टि ि के बारे में:
• उद्दे श्य: उपय गकिाि के अनुकूि िरीके से एमएसएमई क सहायिा प्रिान करना, क् र्ों क यह यहों दी,
असयमिा, गुजराती, बोंगािी और कन्नड सयहत 11 र्ाषाओों में उपिब्ध हदगा।
• िक्ष्य: अपने क्लस्टर पररिदजनाओों कद कुर्ितापूिनक प्रबोंयित करने में एमएसएमई के सामने आने िािी
चुनौयतिदों का समािान करना।
• मदबाइि एल्कप्लकेर्न के माध्यम से र्िय टै र्गोंग से पररय िनाओों की िास्तर्िक समय पर र्नगरानी, टर ै र्कोंग
और मूल्ाोंकन ह सकेगा, र्िससे पारिर्शििा और प्रर्ािी सोंसाधन उपय ग सुर्नर्िि ह गा।

नोंिी प टि ि हाि ही में, केंिीि मत्स्य पािन, पशुपािन और डे यरी मोंिी ने NANDI (नई दिा और टीकाकरण प्रणािी के यिए
NOC अनुमददन) पदटन ि िॉन्च यकिा।
नोंिी प टि ि के बारे में:

62
• इसे केंिीि औषयि मानक यनिोंत्रण सोंगठन (सीडीएससीओ) के समन्वि से सीडीएसी के माध्यम से पर्ुपािन
और डे िरी यिभाग द्वारा र्िकर्सि र्कया गया है।
• उद्दे श्य: पर्ु यचयकत्सा उत्पाद प्रस्तािदों का मूल्याोंकन और जाोंच करने के यिए पारदयर्नता के सार् र्नयामक
अनुम िन प्रर्क्रया क सुर्िधािनक बनाना

र्िय क र्डों ग हाि ही में, जीएसटीएन ने जीएसटी के तहत फजी पोंजीकरण और फजी इनपुट टै क्स क्रेयडट के बारे में यचोंताओों कद
दू र करने के यिए सभी राज्यदों और केंि र्ायसत प्रदे र्दों के यिए र्िय क र्डों ग कायिक्षमिा क िाइि कर र्िया है ।
र्िय क र्डों ग के बारे में:
• र्िय क र्डों ग एक ऐसी प्रयक्रिा है जद यकसी स्र्ान के पते िा यििरण कद भौगदयिक यनदे र्ाों क में पररियतनत करती
है ।
• उद्दे श्य: जीएसटीएन ररकॉडन में पते के यििरण की सटीकता सुयनयित करना और पता स्र्ान और सत्यापन
प्रयक्रिा कद सुव्यिल्कस्र्त करना।
िस्तु एिों सेिा कर नेटिकि (िीएसटीएन):
• यह कोंपनी अर्धर्नयम, 2013 की िारा 8 के तहत पोंजीकृत एक गैर-िार्कारी कोंपनी है।
• िह एक आईटी पहि है जद भारत में करदाताओों कद तैिारी करने, ररटनन दाल्कखि करने, अप्रत्यक्ष कर दे नदाररिदों
का भुगतान करने और अन्य अनुपािन करने में मदद करने के यिए जीएसटी के यिए एक अप्रत्यक्ष कराधान
मोंि प्रिान करिी है।

उत्प्रेरक (UTPRERAK) हाि ही में, ऊिाि मोंिािय ने उद्यदग में स्वि प्रौद्यदयगयकिदों कद अपनाने में तेजी िाने के यिए एक समयपनत उत्कृष्टता
केंि की स्र्ापना की है ।
UTPRERAK (उन्नि िकनीकी प्रिशिन केंद्र) के बारे में:
• केंि कद उन्नि औद्य र्गक प्रौद्य र्गकी प्रिशिन केंद्र (एआईटीडीसी) भी कहा जाता है ।
• स्थार्पि: नई यदल्ली में ऊजान दक्षता ब्यूरद (बीईई)।
• अर्धिे श: उद्यदग के पेर्ेिरदों के बीच सिोिम प्रर्ाओों कद प्रसाररत करने के यिए एक प्रिशिनी सह सूिना केंद्र
और ज्ञान र्ोंडार के रूप में कायि करना।
• औद्य र्गक ऊिाि-कुशि प्रौद्य र्गर्कय ों के यिए अग्रणी सोंस्र्ान बनना।

GeM प टि ि कदििा मोंत्रािि ने GeM प टि ि के माध्यम से खरीद के यिए "सििश्ेष्ठ सहर्ार्गिा" पुरस्कार िीिा।
GeM प टि ि के बारे में:
• GeM एक िन-स्टॉप रािरीय सािििर्नक खरीि प टि ि है जद यियभन्न केंिीि और राज्य सरकार के यिभागदों,
सोंगठनदों और सािनजयनक क्षेत्र के उपक्रमदों (पीएसिू) के यिए सामान्य िस्तुओों और सेिाओों कद ऑनिाइन खरीदना
आसान बनाता है ।
• इसे सरकार के र्िए खरीि प्रर्क्रया में िक्षिा और पारिर्शििा बढाने के यिए 2016 में िॉन्च र्कया गया
था।
• न डि मोंिािय: िायणज्य और उद्यदग मोंत्रािि।
• अर्निायि खरीि: GeM पर दी जाने िािी िस्तुओों और सेिाओों के यिए, मोंिािय ों और केंद्रीय सािििर्नक
उद्यम द्वारा खरीि।

िस्ट एनिी टर ाोंर्िशन सोंिर्ि: दयक्षण अफ्रीका, इों डदनेयर्िा और यिितनाम के बाद सेनेगि JET-P समझौते पर हस्ताक्षर करने िािा चौर्ा
पाटि नरर्शप (JET-P) दे र् बन गिा है ।
िस्ट एनिी टर ाोंर्िशन पाटि नरर्शप:
• िॉन्च: इसे 2021 में ग्लासग , यूके में आय र्िि UNFCCC COP26 में िॉन्च र्कया गया था।
• उद्दे श्य: स्वि ऊजान की यदर्ा में आगे बढने में यिकयसत और यिकासर्ीि दे र्दों के बीच अोंतर कद पाटना।
 ऊजान क्षेत्र में उत्सििन क कम करना और क यिा िरर्-आउट में िेिी िाना।

63
समािार में सू ि काों क /ररप टि :

र्नयािि िैयारी हाि ही में, नीयत आिदग ने राज्यदों/केंिर्ायसत प्रदे र्दों के यिए 'र्नयािि िैयारी सूिकाोंक (ईपीआई) 2022' र्ीषनक
सूिकाोंक (ईपीआई) से ररप टि का िीसरा सोंस्करर् िारी र्कया।
2022 र्नयािि िैयारी सूिकाोंक (ईपीआई) 2022 के बारे में:
• िह र्ारि में राज् ों और केंद्रशार्सि प्रिे श ों की र्नयािि िैयाररय ों क मापिा है।
• उद्दे श्य: राज् ों और केंद्रशार्सि प्रिे श ों की िाकि और कमजदररिदों की पहचान करने के यिए र्नयािि-सोंबोंर्धि
मापिों ड ों का व्यापक र्िश्लेषर् करना।
 प्रर्िस्पधी सोंघिाि की र्ािना क बनाए रखने के यिए राज्यदों/केंिर्ायसत प्रदे र्दों के बीच सहकमी सीखने
कद प्रदत्सायहत करना।

ऊिाि सोंक्रमर् यिश्व आर्थिक मोंि ने अपने ऊजान सोंक्रमण सूचकाों क में र्ारि क र्िि स्तर पर 67िें स्थान पर रखा।
सूिकाोंक ऊिाि सोंक्रमर् सूिकाोंक के बारे में:
• इसे एक्सेंचर के सहिदग से यिश्व आयर्नक मोंच द्वारा प्रकायर्त यकिा जाता है ।
िैर्िक रैं र्कोंग और अिि कन:
• 120 िे श ों की सूिी में शीषि 5 िे श: स्वीडन, डे नमाकि, नॉिे, र्िनिैंड, म्भस्वट् िरिैंड

र्कसान सोंकट 2022 में, सेंटरि ररसिि इों स्टीट्यूट िॉर डर ाईिैंड एग्रीकल्िर (CRIDA) ने एक प्रारों यभक चेतािनी प्रणािी के
सूिकाोंक यिकास की र्ुरुआत की, यजसे "यकसान सोंकट सूचकाों क" के रूप में जाना जाता है ।
र्कसान सोंकट सूिकाोंक के बारे में:
• उद्दे श्य: फसि हायन/खराबी और आि सोंकट के रूप में कृयष सोंकट कद कम करना।
• सोंकट के यिए सूचकाों क में 0-1 का मान ह गा।
• 0-0.5 के बीच का मान 'कम सोंकट' क इों यगत करे गा, 0.5-0.7 'मध्यम' सोंकट क इों यगत करे गा और 0.7 से
ऊपर 'गोंर्ीर' सोंकट क इों यगत करे गा।
सेंटरि ररसिि इों स्टीट्यूट िॉर डर ाईिैंड एग्रीकल्िर (CRIDA) के बारे में:
• र्ारिीय कृर्ष अनुसोंधान पररषि (आईसीएआर) के तहत एक सोंस्र्ान है ।
• अर्धिे श: सोंसािन प्रबोंिन के माध्यम से िषाि आधाररि क्षेि ों में सुधार के र्िए अनुसोंधान करना।
• मुिािय: हैदराबाद

र्िि र्निेश ररप टि हाि ही में, व्यापार और र्िकास पर सोंयुक्त रािर सम्मेिन (UNCTAD) ने अपनी यिश्व यनिेर् ररपदटन 2023
2023 प्रकायर्त की।
र्ारि के बारे में र्िि र्निेश ररप टि 2023 के मुि र्नष्कषि:
• भारत 2022 में दु यनिा में ग्रीनफील्ड पररिदजनाओों में प्रत्यक्ष र्ििे शी र्निेश (एिडीआई) का िीसरा सबसे
बड़ा प्राप्तकिाि था।
• भारत में एफडीआई प्रिाह 2021 में 10 प्रर्िशि बढ़कर 44.7 र्बर्ियन डॉिर से बढ़कर 2022 में 49.3
र्बर्ियन डॉिर ह गया। िह केिि िूएस और िूके में एफडीआई प्रिाह से कम है।
• भारत 2022 में दु यनिा में अोंिराििरीय पररय िना र्िि का िू सरा सबसे बड़ा प्राप्तकिाि था।
अोंकटाड के बारे में:
• यह 1964 में UNGA द्वारा स्र्ायपत एक स्र्ािी अोंतरसरकारी यनकाि है।
• मुिािय: यजनेिा, ल्कस्वट् जरिैंड
• उद्दे श्य: यिकासर्ीि दे र्दों कद िैश्वीकृत अर्नव्यिस्र्ा के िाभदों कद यनष्पक्ष और प्रभािी तरीके से प्राप्त करने में
सहािता करना।

64
4. पयाि ि रर् और र्ू ग ि
4.1 िििायु र्ििप षर्
सोंिर्ि:
न्यू ग्ल बि िाइनेंर्सोंग पैर के र्शखर सम्मेिन ने जििािु र्ििप षर् के र्ििािास्पि मुद्दे क सोंब र्धि करने में प्रगयत की, िेयकन कई
महत्वपूणन मुद्े अनसुिझे हैं ।

िििायु र्ििप षर् क्ा है?


• सोंिुक्त राष्टर ने जििािु यिि कद सािनजयनक, यनजी और यििपदषण
के िैकल्कल्पक स्रदतदों से प्राप्त स्थानीय, रािरीय या अोंिररािरीय
र्ििप षर् के रूप में पररर्ार्षि र्कया है ि िििायु पररिििन
क सोंब र्धि करने िािे शमन और अनुकूिन कायों का
समथिन करना िाहिा है।
• अनुकूिन में सहायिा: िह सरि समािान प्रदान करके समुदािदों
कद जििािु पररितनन के प्रभािदों के अनुकूि बनने में भी मदद
करता है । उिाहरर् के र्िए, सहनशीि बीि र्कसान ों क सूखे
के िौरान र्ी र् िन उगाने में मिि कर सकिे हैं।
िििायु र्ििप षर् िोंि:
• जििािु यिि के प्राििान कद सुयििाजनक बनाने के यिए, िूएनएफसीसीसी ने यिकासर्ीि दे र् पायटन िदों कद यििीि सोंसािन प्रदान करने के
यिए एक यििीि तोंत्र की स्र्ापना की। यििीि तोंत्र क्दटद सामान्य िेर्कन र्िर्ेर्िि र्िम्मेिाररयाों और सोंबोंर्धि क्षमिाएों
प्रदटदकॉि और पेररस समझौते पर भी कािन करता है । (सीबीडीआर-आरसी):
• िैर्िक पयाििरर् सुर्िधा: 1994 में कन्वेंर्न के िागू हदने के • िििायु पररिििन पर सोंयुक्त रािर फ्रेमिकि कन्वेंशन (िूएनएफसीसीसी)
बाद से इसने यििीि तोंत्र की एक पररचािन इकाई के रूप में में एक ऐसा यसद्धाोंत है जद जििािु पररितनन कद सोंबदयित करने में अिग-
कािन यकिा है । अिग दे र्दों की यियभन्न क्षमताओों और अिग-अिग यजम्मेदाररिदों कद
• ग्रीन िाइमेट िोंड: सीओपी-16,2010 में पायटन िदों ने स्वीकार करता है ।
िीसीएि की स्थापना की और 2011 में इसे यििीि तोंत्र की • सीबीडीआर-आरसी का यसद्धाोंत 1992 की िूएनएफसीसीसी सोंयि में
यनयहत है , यजसे भाग िेने िािे दे र्दों द्वारा अनुमदयदत यकिा गिा र्ा।
एक पररचािन इकाई के रूप में भी नायमत यकिा।
• र्िशेष िििायु पररिििन क ष (एससीसीएि): इसकी
स्र्ापना 2001 में कन्वेंर्न के तहत यनम्नयिल्कखत से सोंबोंयित पररिदजनाओों
के यििपदषण के यिए की गई र्ी: अनुकूिन; प्रौद्य र्गकी हस्ताोंिरर्
और क्षमिा र्नमािर्; ऊिाि, पररिहन, उद्य ग, कृर्ष, िार्नकी और
अपर्शि प्रबोंधन; और आर्थिक र्िर्िधीकरर्।
• अल्प र्िकर्सि िे श र्नर्ध (एिडीसीएि): इसकी स्र्ापना राष्टरीि
अनुकूिन कािनक्रम कारन िाई (एनएपीए) की तैिारी और कािान न्विन में
अल्प यिकयसत दे र् पायटन िदों की सहािता के यिए कािन कािनक्रम का
समर्नन करने के यिए की गई र्ी।
• अनुकूिन क ष: अनुकूिन कदष की स्र्ापना 2001 में उन यिकासर्ीि
दे र्दों में व्यािहाररक अनुकूिन पहि के यिए िन उपिब्ध कराने के यिए
की गई र्ी जद क्दटद प्रदटदकॉि पर हस्ताक्षरकतान हैं और यिर्ेष रूप से
जििािु पररितनन के नकारात्मक प्रभािदों के प्रयत सोंिेदनर्ीि हैं ।

िििायु र्ििप षर् से िुड़ी िुनौर्ियााँ:


• र्िकर्सि िे श ों द्वारा प्रर्िबद्धिाओों क पूरा न करना: अमीर दे र्दों ने
ग्रीन क्लाइमेट फोंड के तहत 2009 कदपेनहे गन यर्खर सम्मेिन में की
गई िायषनक 100 यबयििन डॉिर की प्रयतबद्धता कद पूरा नहीों यकिा।

65
• गैर-ररयायिी ऋर्: जििािु यिि का एक बडा यहस्सा मुख्य रूप से ऋण के रूप में यितररत यकिा जाता है , यजसका एक बडा यहस्सा गैर -
ररिािती रहा है ।
• र्िषम धन आिोंटन: यिज्ञान और पिान िरण केंि की एक ररपदटन के अनुसार, जििािु पररिदजनाओों के यिए िन उिरी अमेररका, पर्िमी
यूर पीय सोंघ और पूिी एर्शया (मुि रूप से िीन) में र्ारी रूप से केंर्द्रि है।
• िोंर्डों ग और आिश्यकिाओों के बीि अोंिर: अपेक्षाओों
र्ारि द्वारा िििायु र्ििप षर् नीर्ियाों/पहि:
और प्रयतबद्धता के बीच बढती दू री। केपीएमजी की एक
• एनएएिसीसी: िह जििािु पररितनन के प्रयतकूि प्रभािदों कद कम करने
ररपदटन के अनुसार, जििािु र्मन और अनुकूिन पहि के
िािी ठदस अनुकूिन गयतयियििदों का समर्नन करने के यिए स्र्ायपत एक
यिए 2050 िक िैर्िक सकि घरे िू उत्पाि के 3-6% केंिीि क्षेत्र की िदजना है ।
की आिश्यकिा ह गी िेर्कन िििायु र्ििप षर् • रािरीय स्वच्छ ऊिाि क ष (एनसीईएि): इसे स्वि ऊजान प्रौद्यदयगयकिदों
आपूर्िि िैर्िक सकि घरे िू उत्पाि के 1% से कम है। के क्षेत्र में उद्यमर्ीिता उद्यमदों और अनुसोंिान में यनिेर् करने के यिए
• र्ििप षर् की ज्ािा ररप र्टिं ग: ऑक्सफैम क्लाइमेट बनािा गिा र्ा।
फाइनेंस र्ैडद ररपदटन 2023 से पता चिता है यक • प्रर्िपूरक िनर पर् (CAMPA) र्नर्ध: इसका उपिदग प्रयतपूरक
दानदाताओों ने 2020 में 83.3 यबयििन डॉिर का दािा िनरदपण कद बढाकर और िनदों की गुणििा में सुिार करके िन भूयम और
यकिा, िेयकन उनका खचन अयिकतम 24.5 यबयििन डॉिर पाररल्कस्र्यतकी तोंत्र सेिाओों के नुकसान की भरपाई के यिए यकिा जाता है ।
र्ा। $83.3 यबयििन का अनुमान बढा-चढाकर बतािा गिा • पहिा सॉिरे न ग्रीन बाोंड: भारत ने 80 यबयििन रुपिे मूल्य के अपने पहिे
है क्दोंयक इसमें जििािु उद्े श्यदों कद बढा-चढाकर पेर् सॉिरे न ग्रीन बाोंड की पहिी यकश्त जारी की।

करने िािी पररिदजनाएँ िा अोंयकत मूल्य पर ऋण र्ायमि


• आपिा प्रर्िर धी बुर्नयािी ढाोंिे के र्िए गठबोंधन, या सीडीआरआई:
िह अोंतर-सरकारी जििािु िचीिा बुयनिादी ढाोंचा कािनक्रमदों कद
हैं ।
यक्रिान्विन करता है ।
• र्िकास और िििायु र्ििप षर् की ि हरी िुनौिी:
यिकासर्ीि दे र्दों कद नुकसान और क्षयत की िागत कद सोंबदयित करते हुए यिकास और जििािु र्मन और अनुकूिन में एक सार् यनिेर्
करने की ददहरी चुनौती का सामना करना पडता है ।
• एिडीसी और एसआईडीएस के र्िए समथिन का अर्ाि: जििािु प्रभािदों के प्रयत अत्ययिक सोंिेदनर्ीिता के बािजूद, दु यनिा के सबसे
गरीब दे र्दों, यिर्ेष रूप से सबसे कम यिकयसत दे र्दों (एिडीसी) और छदटे द्वीप यिकासर्ीि राज्यदों (एसआईडीएस) कद पिानप्त समर्नन नहीों
यमि रहा है । इसके बजाि, उन्हें और गहरे कजन में िकेिा जा रहा है ।

आगे की राह:
• िािा पूरा करना: जीसीएफ के तहत यनम्न और मध्यम आि िािे दे र्दों में जििािु कारन िाई के यििपदषण के यिए 2020 तक सािाना 100
यबयििन अमेररकी डॉिर जुटाने का िक्ष्य िोंबे समि से िोंयबत है । िोंबे समि से चिे आ रहे िादे पर अमि करना जरूरी है ।'
• िी20 मोंि का उपय ग: भारत के बाद िाजीि और दयक्षण अफ्रीका के जी20 की अध्यक्षता सोंभािने के सार्, िह ग्लदबि साउर् यिश्व मोंच
पर उभर रहा है , और जी20 नुकसान और क्षयत के यिए यिि, तकनीकी यिर्ेषज्ञता और मानि सोंसािन जुटाने के यिए एक महत्वपूणन मोंच
हदगा।
• हार्न और क्षर्ि के आकिन के र्िए रूपरे खा: भारत कद जििािु पररितनन और आपदा जदल्कखम में कमी पर अपनी राष्टरीि और उप-
राष्टरीि कािन िदजनाओों के एक भाग के रूप में हायन और क्षयत के आकिन के यिए एक रूपरे खा बनाने कद प्रार्यमकता दे नी चायहए।
 मनरे गा जैसी सामायजक सुरक्षा िदजनाएों जद जमीनी स्तर पर जििािु -सहनर्ीिता बुयनिादी ढाों चे का यनमानण कर रही हैं , और यकसानदों
की सुरक्षा के यिए मौसम सूचकाों क-आिाररत फसि बीमा दर्ानती हैं यक कैसे नुकसान और क्षयत कद कम करना िदक कल्याण कािनक्रमदों
का यहस्सा बन सकता है।
• अनुिान आधाररि र्ििप षर् क बढ़ाना: जििािु कारन िाई के यिए अयिक अनुदान-आिाररत यििपदषण की समान रूप से तत्काि
आिश्यकता है , और जद िन दे ने का उन्हदोंने िादा यकिा है उसे उिार दे ने की यदर्ा में कम गयत है ।
• िििायु उिरिायी र्िकास बैंक: अब समि आ गिा है यक जििािु पररितनन कद बहुपक्षीि यिकास बैंकदों का एक प्रमुख पैरामीटर बनािा
जाए और कम काबनन पररिदजनाओों में यनिेर् बढािा जाए।
• ित्काि ऋर् राहि: एक दीघनकायिक िक्ष्य एक बहुपक्षीि ऋण कसरत प्रयक्रिा स्र्ायपत करना हदना चायहए जद दे र्दों कद यबगडते ऋण और
जििािु सोंकट के दु ष्चक्र कद तदडने में मदद कर सके।
• िििायु िक्ष्य ों के र्िए र्निी र्िि िुटाना: यनजी पूोंजी यििपदषण िैयश्वक यटकाऊ यििपदषण अोंतराि कद भरता है , यिर्ेष रूप से उभरते
बाजारदों और यिकासर्ीि अर्नव्यिस्र्ाओों में जहाों सरकारें क्षमता से यििर् हैं ।

66
● िििायु प्रूर्िोंग र्निेश: मैंग्रदि बहािी के माध्यम से तटीि क्षेत्रदों, प्र. जििािु यििपदषण की यदर्ा में आने िािी यियभन्न चु नौयतिदों पर चचान
बाढ से प्रभायित यनचिे इिाकदों और िषान आिाररत क्षेत्रदों के यिए कीयजए। जििािु यिि में आत्मयनभनरता हायसि करने के यिए आगे का
जििािु प्रूयफोंग यनिेर् यिकासर्ीि दे र्दों के यिए एक और रास्ता सुझाएों ।
प्रार्यमकता है ।
4.2 आपिा ि म्भखम में कमी

सोंिर्ि:
हाि ही में, सोंिुक्त राष्टर आपदा जदल्कखम न्यूनीकरण कािानिि
(िूएनडीआरआर) द्वारा आपदा जदल्कखम न्यूनीकरण (जीएआर) पर िैयश्वक
मूल्याोंकन ररपदटन जारी की गई।
आपिा ि म्भखम न्यूनीकरर् (डीआरआर) के बारे में
• िह आपदाओों के कारण कारकदों का यिश्लेषण करने और उन्हें कम
करने के व्यिल्कस्र्त प्रिासदों के माध्यम से आपिा ि म्भखम ों क कम
करने की अिधारर्ा और अभ्यास है।
• डीआरआर का उद्े श्य आपदा जदल्कखम के यनमानण कद रदकना, मौजूदा
जदल्कखम कद कम करना और आयर्नक, सामायजक, स्वास्थ्य और
पिानिरणीि िचीिेपन कद मजबूत करना है ।
 डीआरआर के उिाहरर्: खतरदों के जदल्कखम कद कम करना, िदगदों
और सोंपयि की सोंिेदनर्ीिता कद कम करना, भूयम और पिानिरण
का बुल्कद्धमान प्रबोंिन, और प्रयतकूि घटनाओों के यिए तैिाररिदों और
प्रारों यभक चेतािनी में सुिार करना।

ररप टि की मुि बािें


• बढ़िी प्रिृर्ि: इों टरगिननमेंटि पैनि ऑन क्लाइमेट चेंज (आईपीसीसी)
के अनुसार , यनकट अियि में ग्लदबि िायमिंग 1.5 यडग्री सेल्कल्सिस तक पहुोंचने से कई जििािु खतरदों में अपररहािन िृल्कद्ध हदगी और
पाररल्कस्र्यतक तोंत्र और मनुष्यदों के यिए कई जदल्कखम पैदा हदोंगे।
• कमज र समथिन: िचीिापन बढाने, जििािु पररितनन से यनपटने और सतत यिकास की यदर्ा में रास्ते बनाने की प्रयतज्ञाओों के बािजूद,
ितनमान सामायजक, राजनीयतक और आयर्नक यनणनिदों का यिपरीत प्रभाि पड रहा है ।
 आपिा ि म्भखम न्यूनीकरर् 2015-2030 के र्िए सेंडाई फ्रेमिकि की उपिल्कब्ध कद खतरे में डािता है , बल्कि पेररस समझौिे और
सिि र्िकास िक्ष्य ों (एसडीिी) की र्िशा में प्रगर्ि में र्ी बाधा डाििा है ।
• र्िि खिरे में: मानिीि पसोंद और जनसाोंल्कख्यकीि रुझान इस सोंभािना कद बढाते हैं यक आपदा फैि सकती है और सभी महाद्वीपदों कद तेजी
से प्रभायित कर सकती है।
 अोंतयननयहत जदल्कखम कारकदों के सोंपकन में, जैसे िािु प्रदू षण का उच्च स्तर, असुरयक्षत आिास िा स्वास्थ्य सेिाओों तक सीयमत पहुों च, मृत्यु
दर कद महत्वपूणन रूप से प्रभायित करते पाए गए।
• प्रर्ािीगि ि म्भखम: आपदा के प्रणािीगत प्रभाि सभी सोंकेतकदों में एसडीजी की उपिल्कब्धिदों कद पटरी से उतार दे ते हैं।
 उिाहरर्: 2020 में महामारी की आयर्नक और सामायजक िागत, जीिन के खदए हुए िषों में मापी गई, अन्य आपिाओों की औसि
िार्षिक िागि और 2000 से 2019 तक सभी महामाररिदों की कुि िागत से अयिक है।
• िििायु पररिििन और आपिाओों के प्रर्ाि के कारर् 2030 िक अर्िररक्त 37.6 र्मर्ियन ि ग ों के अत्ययिक गरीबी की ल्कस्र्यत में
रहने का अनुमान है ।
 सोंिुक्त राष्टर के अनुसार, िििायु पररिििन और आपिाओों का "सबसे खराब म्भस्थर्ि" पररदृश्य 2030 िक अर्िररक्त 100.7
र्मर्ियन ि ग ों क गरीबी में िकेि दे गा।
• आपिा की आर्थिक िागि: यपछिे तीन दर्कदों में आपदाओों से हदने िािी िायषनक प्रत्यक्ष आयर्नक हायन ददगुनी से भी अयिक हद गई है ।
 1990 के िशक में यह औसिन िगर्ग 70 र्बर्ियन डॉिर से बढकर 2010 में 170 यबयििन डॉिर से अयिक हद गिा है ।
• 1980 और 2018 के बीच सभी आपदा-सोंबोंिी नुकसान का केिि 40% बीमा यकिा गिा र्ा।

67
• कमि र ि ग ों पर प्रर्ाि: आपदा के व्यापक
आपिा ि म्भखम न्यूनीकरर् (िीएआर) पर सोंयुक्त रािर िैर्िक मूल्ाोंकन ररप टि के बारे
स्वास्थ्य और आयर्नक प्रभाि गरीब और हायर्ए पर में:
रहने िािे समुदािदों, यहों सा की यर्कार मयहिाओों और • िह आपदा जदल्कखम कद कम करने के यिश्वव्यापी प्रिासदों पर सोंयुक्त रािर की प्रमुख
पिनटन पर यनभनर छदटी अर्नव्यिस्र्ाओों के यिए बदतर ररप टि है ।
रहे हैं । • आपिा ि म्भखम न्यूनीकरर् (िीएआर) सोंयुक्त रािर आपिा ि म्भखम न्यूनीकरर्
• अिू रिशी स ि: चेताियनिदों के बािजूद, तैिारी कायाििय (UNDRR) द्वारा प्रकार्शि र्कया िािा है।
अपिानप्त है और दु यनिा भर में र्ासन प्रणायििाँ एक यूएनडीआरआर के बारे में
नई िास्तयिकता की ओर बढने के यिए सोंघषन कर रही • िह आपदा जदल्कखम में कमी के यिए सोंिुक्त राष्टर प्रणािी का केंि यबोंदु है और सेंडाई
हैं । फ्रेमिकन का सोंरक्षक है , इसके कािानन्विन, यनगरानी और प्रगयत की समीक्षा में दे र्दों और
समाजदों का समर्नन करता है ।
ररप टि से र्सफाररशें: आपिा ि म्भखम न्यूनीकरर् 2015-2030 पर सेंडाई फ्रेमिकि
• जदल्कखम के बारे में िारणा में बदिाि: खतरदों कद िह • िह उन उपािदों कद अपनाने पर केंयित है जद आपदा जदल्कखम के सभी आिामदों - खतरे ,
पहचानने की आिश्यकता पर प्रकार् डािा गिा है जदल्कखम, भेद्यता और मुकाबिा करने की क्षमता - कद सोंबदयित करते हैं तायक नए जदल्कखम
र्क ग्रह और मानि प्रर्ार्ियााँ अन्य न्यार्श्ि हैं, के यनमानण कद रदका जा सके, मौजूदा जदल्कखम कद कम यकिा जा सके और िचीिापन
और जदल्कखम ज्ञान प्रणायििदों कद अयिक िचीिा और बढािा जा सके।
स्विे शी और पारों पररक दृर्िक र् सर्हि र्िर्र्न्न सिि र्िकास िक्ष्य (एसडीिी)
र्िि-दृर्िक र् ों के र्िए खुिा बनाने की • िह 17 एसडीजी यनिानररत करता है और दे र्दों के बीच और भीतर सभी प्रकार की गरीबी,
आिश्यकिा है। भूख, असमानताओों कद समाप्त करने और पिानिरणीि यगरािट और जििािु पररितनन
से यनपटने के यिए एक व्यापक िैर्िक नीर्ि ढाोंिा प्रिान करिा है, िबर्क यह
• र्ासन कद पुनयिनन्यास करना और प्रभायित िदगदों के
सुर्नर्िि करिा है र्क "क ई र्ी पीछे न छूटे "।
परामर्न से यडजाइन तैिार करना।
• एसडीजी में कई सेंडाई फ्रेमिकि िक्ष्य ों के साथ-साथ िििायु पररिििन और
 साझा ज्ञान बनाने, पाश्वन सहिदग कद प्रदत्सायहत
म्भस्थरिा िक्ष्य र्ी शार्मि हैं।
करने और सीखने की गयत कद तेज करने के यिए
पेररस समझौिा
सामान्य र्ब्दाििी बनाएों और सभी यिषिदों में • िह िैयश्वक जििािु पररितनन अनुकूिन और ग्लदबि िायमिंग कद पूिन-औद्यदयगक स्तरदों से
ओपन एक्सेस डे टा प्रदान करें । 2 यडग्री सेल्कल्सिस से नीचे और अयिमानतः 1.5 यडग्री सेल्कल्सिस तक सीयमत करने के
• सख्त र्नगरानी: सरकार ों क यह सुर्नर्िि करने शमन िक्ष्य की र्िशा में कारि िाई करिा है।
की आिश्यकिा है यक असुरयक्षत स्र्ानदों, जैसे बाढ र्ारि का प्रयास: रािरीय आपिा ि म्भखम न्यूनीकरर् मोंि (एनपीडीआरआर), 2013
के मैदानदों, समुि के स्तर में िृल्कद्ध िािे क्षेत्रदों िा आग • इसका उद्े श्य सरकार, साोंसददों, महापौरदों, मीयडिा, अोंतरानष्टरीि सोंगठनदों, गैर सरकारी
िा अन्य खतरदों के अत्ययिक जदल्कखम िािे क्षेत्रदों में सोंगठनदों, स्र्ानीि समुदाि के प्रयतयनयििदों, िैज्ञायनक और र्ैक्षयणक सोंस्र्ानदों और कॉपोरे ट
यनमान ण कद रदकने, कम करने िा िचीिेपन कद व्यिसािदों आयद से भारत के आपदा जदल्कखम समुदाि की पूरी श्रृोंखिा कद एक सार् िाना
सुयनयित करने के यिए यनिम मौजूद हैं । . है ।

• र्निी क्षेि की र्ूर्मका - जदल्कखम कम करने की • िह अनुभिदों, यिचारदों कद साझा करने, अनुसोंिान और कारन िाई के यनष्कषों कद प्रस्तुत
करने और आपदा जदल्कखम न्यूनीकरण के क्षेत्र में आपसी सहिदग के अिसरदों का पता
कारन िाई में तेजी िाने और भयिष्य की आपदाओों से
िगाने में मदद करे गा।
हदने िािे नुकसान कद कम करने में यनजी क्षेत्र की
आपिा प्रर्िर धी बुर्नयािी ढाोंिे के र्िए गठबोंधन (सीडीआरआई)
प्रमुख भूयमका हद। • र्ारिीय प्रधान मोंिी द्वारा यसतोंबर, 2019 में न्यूिॉकन में िू एनडीआरआर के समथिन से
 उिाहरर्: बैंक और यििीि सोंस्र्ान जद सोंपयि सोंयुक्त रािर िििायु कारि िाई र्शखर सम्मेिन के दौरान िॉन्च यकिा गिा र्ा।
सुिार ऋण प्रदान करते हैं , उन्हें बोंिक के यिए • िह राष्टरीि सरकारदों, सोंिुक्त राष्टर एजेंयसिदों और कािनक्रमदों, बहुपक्षीि यिकास बैंकदों, यनजी
एक र्तन के रूप में यियर्ष्ट जदल्कखम कम करने के क्षेत्र और र्ैक्षयणक सोंस्र्ानदों की एक बहु-यहतिारक िैयश्वक साझेदारी है ।
उपािदों की आिश्यकता हद सकती है • उद्दे श्य: जििािु और आपदा जदल्कखमदों के प्रयत बुयनिादी ढाोंचा प्रणायििदों के िचीिेपन
• जदल्कखम की िास्तयिक िागतदों, यिर्ेष रूप से कद बढािा दे ना , यजससे सतत यिकास सुयनयित हद सके।
दीघनकायिक जदल्कखमदों कद ध्यान में रखने के यिए • िििमान सिस्यिा: 31 दे र्, 06 अोंतरानष्टरीि सोंगठन और 02 यनजी क्षेत्र के सोंगठन
यििीि प्रणायििदों कद यफर से तैिार करें , और जदल्कखम सीडीआरआई के सदस्य बन गए हैं।
में कमी कद प्रदत्सायहत करने के यिए यनिेर् और बीमा • मुिािय: नई यदल्ली, भारत।
प्रणायििदों कद यफर से तैिार करें ।
 उिाहरर्: हररि बाोंड िैसे र्ििीय उत्पाि ों क प्रदत्सायहत करने और ऐसे यनिेर् कद आसान बनाने की आिश्यकता है जद आपदा
जदल्कखम और जििािु पररितनन के प्रयत िचीिा हद।

68
• सॉिरे न बिट में पररिििन: राष्टरीि बजट कद जदल्कखम
और अयनयितता घटकदों कद र्ायमि करने के यिए
यिकयसत करने की आिश्यकता है , तायक यििीि
िदजनाकार अनुकूिी िदजना में अयिक कुर्ि बन सकें
और सोंकट की ल्कस्र्यतिदों में सोंसािनदों कद बेहतर ढों ग से
सोंचायित करने में सक्षम हद सकें।
• आपिा प्रूर्िोंग के र्िए प्रौद्य र्गकी: इन पहिुओों पर
मध्यम, दीघनकायिक पररदृश्य बनाने के यिए एआई
आिाररत मॉडयिोंग।
 यकसी मेगायसटी में जििािु के कारण हदने िािी
आपदा की ल्कस्र्यत में सोंभायित िागत की मात्रा
यनिान ररत करने के यिए इन्हें शहर आधाररि आर्थिक
मॉडि से ि ड़ा िा सकिा है।
● आपदा जदल्कखम न्यूनीकरण 2015-2030 के यिए
सेंडाई फ्रेमिकन के कािान न्विन में तेजी िाना और
2015 के बाद के अन्य एजेंडे, जैसे सिि र्िकास
के र्िए 2030 एिेंडा, िििायु पररिििन पर पेररस समझौिा, नया शहरी एिेंडा और मानििा के र्िए एिेंडा के साथ सोंरेम्भखि
करना।

प्र. सोंिुक्त राष्टर आपदा जदल्कखम न्यूनीकरण कािानिि (िूएनडीआरआर) ररपदटन द्वारा जारी आपदा जदल्कखम न्यूनीकरण (जीएआर) पर िैयश्वक
मूल्याोंकन ररपदटन के मुख्य यनष्कषों और यसफाररर्दों पर चचान कीयजए।

4.3 महासागर सों साधन और इसकी क्षमिा


सोंिर्ि:
इों टरनेर्नि सीबेड अर्ॉररटी (आईएसए) ने हाि ही में दद साि के यनिम कद बरकरार रखते हुए अोंतरराष्टरीि जि में खनन कािों के यिए तत्काि
अनुमयत दे ने के ल्कखिाफ फैसिा यकिा है , यजससे खनन गयतयियििदों कद 2024 में र्ुरू करने की सोंभािना है ।
पृष्ठर्ूर्म:
• ि साि का र्नयम: 2021 में, नाउरू
के प्रर्ाों त द्वीप ने UNCLOS में
अोंतयननयहत यनिम कद सयक्रि यकिा,
अोंतरान ष्टरीि सीबेड अर्ॉररटी (ISA) से 9
जुिाई, 2023 तक खनन यनिमदों कद
अपनाने का आग्रह यकिा, िा मौजूदा
यनिमदों के सार् खनन र्ुरू करने की
अनुमयत दी।
• गहरे समुद्र का ि हन: दद साि के
यनिम की समाल्कप्त के कारण, यनजी
सोंस्र्ाओों ने एक यनयदन ष्ट गहरे समुि
क्षेत्र में खयनजदों के ददहन के यिए
आईएसए में आिेदन करना र्ुरू कर
यदिा है । इस यनिम में गहरे समुि में
खनन कद गयत दे ने की क्षमता है।
गहरे समुद्र में खनन क र्नयोंर्िि करने िािे र्िर्नयम ों की आिश्यकिा:
• सोंयुक्त रािर सोंर्ध: सोंिुक्त राष्टर सोंयि के एक खोंड के यिए आईएसए कद गहरे समुि में ददहन कद यियनियमत करने की आिश्यकता हदती है।

69
• र्नयम ों के र्बना खनन: ियद आईएसए यनिमदों कद मोंजूरी दे ने SDG14: िह महासागरदों, समुिदों और समुिी सोंसािनदों के सोंरक्षण और सतत
में यिफि रहता है , तद अन्य दे र् और यनजी कोंपयनिाों अनोंयतम उपिदग के बारे में है ।
िाइसेंस के यिए आिेदन कर सकती हैं और कोंपयनिाों यबना
यकसी यियनिमन के खनन कर सकती हैं ।

महासागर सोंसाधन: र्ारि के प िीमेटेर्िक न ड्यूि अन्वेषर् अर्धकार:


• समुि के पानी और तिहटी में पाए जाने िािे जीयित और • एक अग्रणी यनिेर्क के रूप में, भारत कद पॉिीमेटेयिक नदड्यूल्स
यनजीि सोंसािनदों कद "समुद्री सोंसाधन" कहा िािा है। (पीएमएन) के ददहन के यिए सोंिुक्त राष्टर अोंतरानष्टरीि सीबेड प्रायिकरण द्वारा
• समुिी सोंसािन िे जीयित और यनजीि चीज़ें हैं जद समुि के मध्य यहों द महासागर बेयसन (सीआईओबी) में 75,000 िगन यकिदमीटर की
पानी और उसकी तिी में पाई जा सकती हैं । िे सोंसािन, यजनमें एक साइट आिोंयटत की गई है ।
समुिी जि भी र्ायमि है , समुिी सोंसािन कहिाते हैं । समुद्रयान र्मशन:
• इस यमर्न में तीन िदगदों कद समुि में 6,000 मीटर की गहराई तक िे जाने
खर्नि स्र ि: के यिए स्वदे र्ी रूप से यिकयसत एक मानििुक्त पनडु ब्बी, MATSYA
• समुद्री िि में घुिे खर्नि: कुछ खयनज कम मात्रा में मौजूद 6000 की मदद से गहरे समुि का पता िगाने का प्रस्ताि है ।
हदते हैं • मध्य यहों द महासागर में 6000 मीटर की गहराई से पॉिीमेटैयिक नदड्यूल्स
• समुिी जि के भीतर ही और यिर्ेष प्रयक्रिाओों का उपिदग के खनन के यिए एक एकीकृत खनन प्रणािी यिकयसत की जाएगी।
करके इसे यनकािा जा सकता है ।
• महाद्वीपीय शेल्फ और ढिान िमा: िे जमा उर्िे समुिी क्षेत्रदों में पाए जाते हैं , मुख्य रूप से महाद्वीपीि र्ेल्फ और ढिानदों पर। उदाहरण
के यिए: हीरा, मत्स्य पािन क्षेि, म िी।
• गहरे समुद्र िि पर ििछट: गहरे समुि में खयनज भोंडार कद पाँ च प्रकारदों में यिभायजत यकिा जा सकता है :
• पॉिीमेटेर्िक/मैंगनीि न ड्यूल्स: फेरदमैंगनीज (Fe-Mn) नदड्यूल्स िा मैंगनीज नदड्यूल्स में भारी मात्रा में िदहा और मैंगनीज हदते हैं ।
• समुद्री िि में बड़े पैमाने पर सल्फाइड: यियभन्न प्रकार के
खयनज हाइडर दर्मनि गयतयियि और कम सल्फर के माध्यम सोंयुक्त रािर समुद्री कानून पर कन्वेंशन (यूएनसीएिओएस), 1982:
से बनते हैं और कई िातुओों जैसे Cu, Zn, Fe, Au और Ag • समुद्री क्षेि: समुिी क्षेत्रदों कद पाोंच मुख्य क्षेत्रदों में यिभायजत करता है : उच्च समुद्र,
के यिहाज़ से समृद्ध हद सकते हैं । र्िर्शि आर्थिक क्षे ि (ईईिेड), सर्न्नर्हि क्षेि, प्रािे र्शक सागर और आों िररक
• िेर मैंगनीि क्रस्ट/क बाल्ट क्रस्ट: इन क्रस्ट में कदबाल्ट, िि।
REE, Nb, W, Bi, Ni, Mn, Te, और Ti जैसे तत्वदों की उच्च • समुद्री सोंप्रर्ु प्रार्धकरर्: िह एकमात्र अोंतरराष्टरीि समझौता है जद समुिी क्षेत्रदों
में सोंप्रभु अयिकार की नीोंि रखता है ।
साों िता हदती है ।
• आरईई-समृद्ध समुद्री र्मट्टी: इनमें मुख्य रूप से यसयिसस • अपिटीय शासन: िह तटीि दे र्दों और नायिकदों के बीच अपतटीि र्ासन के
यिए ढाोंचे के रूप में कािन करता है।
ओज़ हदता है , यजसमें अिीनस्र् यजओयियटक यमट्टी हदती है
• अर्धकार और िार्यत्व: तटीि राज्यदों के अपतटीि क्षेत्रदों के ज़दयनोंग के अिािा,
यजसमें बडी मात्रा में आरईई हदता है ।
िह पाँच सोंकेंयित क्षेत्रदों के अोंदर प्रत्येक राज्य के अयिकारदों और दायित्वदों पर
• िॉस्फ राइट् स: िे फॉस्फदरस से भरपूर तिछटी चट्टानें हैं , यिस्तृत यनदे र् प्रदान करता है ।
जद आमतौर पर काबोनेट हाइडर ॉल्कक्सि फ्लदरापैटाइट से बनी • िार् साझा करना: सोंयि के तहत, समुि ति और उसके खयनज सोंसािनदों कद
हदती हैं । इस तरह से प्रबोंयित यकिा जाना चायहए जद आयर्नक िाभदों कद साझा करने, समुिी
खर्नि सोंसाधन ों से िुड़ी िुनौर्ियााँ: िैज्ञायनक अनुसोंिान के यिए समर्न न और समुिी पिानिरण की रक्षा के माध्यम से
मानिता के यहतदों की रक्षा करे ।
• िायु प्रिू षर्: खनन के माध्यम से छदडे गए िािु कणदों के
अोंिराििरीय सीबेड प्रार्धकरर्:
साँ स िेने से श्वसन प्रणािी और एिजी के रदग उत्पन्न हद
● सोंयुक्त रािर र्नकाय: िह प्रायिकरण UNCLOS द्वारा स्र्ायपत तीन
सकते हैं । जैसे- सीसा, आसेयनक, कैडयमिम और अन्य अोंतरराष्टरीि सोंस्र्ानदों में से एक है ।
यिषैिे तत्व। ● सिि अन्वेषर्: आईएसए कद िह सुयनयित करना अयनिािन है यक समुिी
• िि प्रिू षर्: िातु की खदानदों िा कदििा खदानदों से अम्लीि पिानिरण की नाजुकता और पानी के नीचे की जैि यियििता कद ध्यान में
पानी के सतही जि में जाने िा जमीन के नीचे पानी के ररसाि रखते हुए समुिी ति से यनष्कषनण यटकाऊ और सोंचायित यकिा जाए।
के कारण पानी का पीएच बदि जाता है । • र्िर्नयमन: इसका प्रार्यमक कािन 'क्षेि' में पाए िाने िािे गहरे समुद्र
• ध्वर्न प्रिू षर्: खनन क्षेत्र में र्दर के सोंपकन में आने से यियभन्न िि के खर्नि ों की ख ि और ि हन क र्िर्नयर्मि करना है , यजसे
र्ारीररक और मानयसक स्वास्थ्य समस्याएों हदती हैं , जानिरदों कन्वेंर्न द्वारा राष्टरीि क्षेत्रायिकार की सीमाओों से परे , िानी बाहरी सीमाओों
की भिाई पर असर पडता है और तनाि का स्तर बढ जाता से परे समुि ति और उपमृदा के रूप में पररभायषत यकिा गिा है ।
है ।
70
• प्रकाश प्रिू षर्: खनन गयतयियििाँ प्राकृयतक रूप से अोंिेरे िातािरण में उज्ज्वि कृयत्रम प्रकार् िाती हैं , यजससे उन प्रजायतिदों पर काफी
प्रभाि पडता है यजन्हदोंने इन अोंिेरी पररल्कस्र्यतिदों में पनपने के यिए खुद कद अनुकूयित कर यििा है ।
• िैि र्िर्िधिा का नुकसान: खनन से यमट्टी के सूक्ष्मजीिदों से िेकर बडे स्तनिाररिदों तक की यियभन्न प्रकार की िनस्पयतिदों और जीिदों के
यनिास स्र्ान का बडे पैमाने पर नुकसान हदता है ।

आगे की राह:
• पयाििरर्ीय प्रर्ाि आकिन: यकसी भी गहरे समुि में खनन कािन र्ुरू करने से पहिे, व्यापक पिानिरणीि प्रभाि आकिन आिदयजत
यकिा जाना चायहए।
• सिि प्रथाएाँ : सिोिम उपिब्ध तकनीकदों कद अपनाना, अपयर्ष्ट उत्पादन कद कम करना और आिास बहािी के यिए रणनीयतिदों कद िागू
करने जैसी यटकाऊ प्रर्ाओों कद अपनाना।
• अोंिराििरीय सहय ग: गहरे समुि में खनन के यिए प्रभािी अोंतरान ष्टरीि यनिमदों और यदर्ायनदे र्दों के यिए दे र्दों के बीच सहिदगात्मक प्रिासदों
की आिश्यकता हदती है ।
• प्रौद्य र्गकी निािार: निीन प्रौद्यदयगयकिदों के यनरों तर अनुसोंिान और यिकास से अयिक कुर्ि और पिानिरण के अनुकूि खनन प्रयक्रिाएों
हद सकती हैं ।
• पारिर्शििा और ििाबिे ही: समुिी खयनज यनष्कषनण में र्ायमि कोंपयनिदों कद पूरे खनन जीिनचक्र में पारदयर्नता और जिाबदे ही कद
प्रार्यमकता दे नी चायहए।
• स्थानीय समुिाय ों का समािेश: स्वदे र्ी आबादी का पारों पररक ज्ञान और दृयष्टकदण अयिक यटकाऊ और साोंस्कृयतक रूप से सोंिेदनर्ीि
खनन प्रर्ाओों में िदगदान कर सकते हैं ।

4.4 िििायु स्माटि कृ र्ष (सीएसए)

सोंिर्ि:
सोंिुक्त अरब अमीरात (िूएई) िििायु-स्माटि कृर्ष के माध्यम से रे र्गस्तानी पररदृश्य क उत्पािक कृर्ष र्ूर्म में बििने के र्िए एक
पररिििनकारी यािा शुरू कर रहा है।

समािार पर और अर्धक:
• िूएई अपनी खाद्य सुरक्षा कद बढाने के यिए जििािु-िाटन कृयष में अग्रणी बनने का प्रिास कर रहा है।
• दे र् ने स्र्ानीि उपज कद बढािा दे ने और कृयष प्रौद्यदयगकी कद प्रदत्सायहत करने के यिए मैनबैट र्कसान बािार सर्हि र्िर्र्न्न पहि और
साझेिाररयाों शुरू की हैं।
 मैनबैट का अर्न है अोंकुर िा िह स्र्ान जहाँ पौिे उगते हैं ।
• िूएई ने 2051 तक िैयश्वक खाद्य सुरक्षा सूचकाोंक में अपनी रैं यकोंग में सुिार करने और सुरयक्षत और पिानप्त भदजन तक पहुोंच के माध्यम से
र्ून्य भूख हायसि करने के यिए महत्वाकाों क्षी िक्ष्य यनिान ररत यकए हैं ।

िििायु स्माटि कृर्ष (सीएसए) के बारे में:


● यिश्व बैंक के अनुसार, सीएसए पररदृश्यदों - फसि
भूयम, पर्ुिन, िन और मत्स्य पािन - के प्रबोंिन के
यिए एक एकीकृत दृयष्टकदण है जद खाद्य सुरक्षा और
जििािु पररितनन की परस्पर जुडी चुनौयतिदों का
समािान करता है ।
● इसे जििािु िचीिा कृयष के रूप में भी जाना जाता
है , िह जििािु पररितनन की नई िास्तयिकताओों के
तहत कृयष का यिकास है ।
● िििायु स्माटि कृर्ष:
 िसि प्रबोंधन: अोंतरफसि, फयििदों के सार् फसि
चक्र, सूखा, हिा और बाढ-सयहष्णु फसि यकिदों का
उपिदग करना, खाद बनाना, मल्क्चोंग करना और जैयिक उिनरक अपनाना।

71
 पशुधन प्रबोंधन: बेहतर भदजन रणनीयतिाँ , बारी-बारी से चराई, जानिरदों कद ल्कखिाने के यिए उपिुक्त फसिदों का उपिदग करना, और
बेहतर पर्ुिन स्वास्थ्य और पािन।
 मृिा और िि प्रबोंधन: सोंरक्षण कृयष, समदच्च रदपण, चेक बाोंि, जि भोंडारण, बेहतर यसोंचाई और कुर्ि जि उपिदग।
 कृर्ष िार्नकी: िािु अिरदिक के रूप में पेड िगाना, नाइटर दजन ल्कस्र्रीकरण करने िािे पेडदों का उपिदग करना और फिदों के बगीचे
र्ायमि करना।
 एकीकृि खाद्य ऊिाि प्रर्ार्ियााँ: बािदगैस, बेहतर स्टदि, सौर ऊजान और गुरुत्वाकषनण-आिाररत यसोंचाई कद िागू करना।
िििायु स्माटि कृर्ष के िार्:
• िििायु पररिििन के प्रर्ि ििीिापन: िह कृयष प्रणायििदों में िचीिापन पैदा करता है , यजससे यकसानदों कद सूखा, बाढ और अत्ययिक
तापमान जैसे जििािु पररितनन के प्रभािदों से यनपटने में सक्षम बनािा जा सकता है ।
• िसि उत्पािन: इससे िसि की पैिािार अर्धक ह सकिी है, र्कसान ों की आिीर्िका में सुधार, आय में िृम्भद्ध और बेहिर खाद्य
सुरक्षा में य गिान ह सकिा है।
• सिि सोंसाधन प्रबोंधन: िह पानी, यमट्टी और जैि यियििता सयहत प्राकृयतक सोंसािनदों के स्र्ािी प्रबोंिन पर केंयित है ।
 इससे पिान िरणीि यगरािट में कमी आती है और पाररल्कस्र्यतकी तोंत्र सेिाओों का सोंरक्षण हदता है।
• र्टकाऊ: सीएसए प्रर्ाएों , जैसे यक कृयष िायनकी और कम जुताई, कृयष क्षेत्र से ग्रीनहाउस गैस उत्सजनन कद कम करने में िदगदान कर सकती
हैं ।
• सामार्िक और आर्थिक िार्: िह मयहिा यकसानदों और कमजदर समुदािदों कद सर्क्त बनाकर समािेर्शिा और िैंर्गक समानिा क
बढ़ािा िे िा है।
• निीन प्रौद्य र्गर्कय ों क अपनाना: िह खेती में निाचार और उन्नत प्रौद्यदयगयकिदों कद अपनाने कद प्रेररत करता है , यजससे दक्षता और
उत्पादकता में िृल्कद्ध हदती है।
िििायु स्माटि कृर्ष के र्िए सरकारी पहि:
िििायु स्माटि कृर्ष की िुनौर्ियााँ: • नेशनि इन िेशन ऑन िाइमेट रे र्िर्िएों ट एग्रीकल्िर
• सोंसाधन ों िक सीर्मि पहुोंि: छदटे और सीमाों त यकसानदों कद (एनआईसीआरए): फसिदों, जानिरदों और मत्स्य पािन सयहत भारतीि
अक्सर सोंसािन बािाओों का सामना करना पडता है , जैसे यक कृयष की जििािु पररितननर्ीिता और पररितनन के प्रयत सहनर्ीिता कद
यिि, प्रौद्यदयगकी और जििािु-िाटन प्रर्ाओों के बारे में ज्ञान तक बढाना।
सीयमत पहुोंच। • सिि कृर्ष पर रािरीय र्मशन (एनएमएसए): यह िििायु पररिििन
• िकनीकी और ज्ञान अोंिराि: सीएसए प्रर्ाओों कद अपनाने के पर रािरीय कायि य िना (एनएपीसीसी) के तहत आठ यमर्नदों में से एक
यिए उपिुक्त प्रौद्यदयगयकिदों और जानकारी तक पहुोंच की है , जद उयचत िदजना बनाकर जििािु पररितनन से जुडे जदल्कखमदों के सोंदभन
आिश्यकता हदती है । में 'स्र्ािी कृयष' से सोंबोंयित मुद्दों का समािान करना चाहता है ।

 कई यकसानदों में ऐसी प्रर्ाओों कद प्रभािी ढों ग से िागू करने • िििायु पररिििन के र्िए रािरीय अनुकूिन क ष (एनएएिसीसी):
इसकी स्र्ापना 2015 में भारत के राज्य और केंि र्ायसत प्रदे र्दों के यिए
के यिए जागरूकता और तकनीकी जानकारी का अभाि है ।
जििािु पररितनन के अनुकूिन की िागत कद पूरा करने के यिए की गई
• सोंस्थागि बाधाएाँ : कमजदर सोंस्र्ागत समर्नन, अपयािप्त
र्ी, जद यिर्ेष रूप से जििािु पररितनन के प्रयतकूि प्रभािदों के प्रयत
नीर्ियाों और शासन सोंबोंधी मुद्दे जििािु-िाटन कृयष पहि कद सोंिेदनर्ीि हैं ।
आगे बढाने में बािा बन सकते हैं । • िििायु-स्माटि गाोंि (सीएसिी): सीएसिी पररिदजना 2011 में पयिम
• र्ििीय बाधाएाँ : यििीि सोंसािनदों की कमी यकसानदों की सीएसए अफ्रीका, पूिी अफ्रीका और दयक्षण एयर्िा के 15 जििािु -िाटन गाोंिदों के
प्रर्ाओों कद अपनाने की क्षमता में बािा बन सकती है , खासकर सार् र्ुरू की गई र्ी।
कम आि िािे क्षेत्रदों में। • उद्दे श्य: स्र्ानीि स्तर पर सीएसए कद िागू करना और बढािा दे ना,
िििायु स्माटि कृर्ष पद्धर्िय ों क बढ़ािा िे ने के र्िए उठाए यकसानदों की जििािु पररितनन के अनुकूि हदने की क्षमता कद बढाना।
िाने िािे किम
• छदटे पैमाने के यकसानदों के यिए प्रौद्य र्गर्कय ों िक पहुोंि सुर्नर्िि करना महत्वपूर्ि है।
• नीर्ि एकीकरर् क सक्षम बनाना: सरकारदों कद सीएसए का समर्नन करने िािी नीर्िय ों क र्डिाइन और कायािम्भन्वि करना िार्हए
और इसे रािरीय कृर्ष रर्नीर्िय ों और य िनाओों में शार्मि करना िार्हए। उिाहरर्: एनएिएसए।
• ज्ञान और क्षमिा र्नमािर् क बढ़ािा िे ना: प्रयर्क्षण कािनक्रमदों और यडयजटि प्रौद्यदयगकी के माध्यम से यकसानदों, यिस्तार कािनकतानओ,ों
र्दिकतान ओों और नीयत यनमानताओों के बीच ज्ञान-साझाकरण कद प्रदत्सायहत करना और क्षमताओों का यनमान ण करना।
• अनुसोंधान और र्िकास: कृयष पर जििािु पररितनन के प्रभािदों कद समझने और िििायु-ििीिी िसि र्कस्म ,ों निीन िसि
प्रर्ार्िय ों और िििायु सूिना सेिाओों क र्िकर्सि करने के र्िए अनुसोंधान एिों र्िकास में र्निेश करना।
72
• अनुकूिन और स्थानीय सोंिर्ि: सीएसए प्रर्ाओों कद स्र्ानीि सोंदभों और कृयष-पाररल्कस्र्यतक क्षेत्रदों के अनुरूप बनाने की आिश्यकता है।
िघु समािार

4.5 एम्बरग्रीस • न डि र्िर्ाग और मोंिािय: जि सोंसािन, नदी यिकास और


सोंिर्ि: गोंगा सोंरक्षण यिभाग, जि र्ल्कक्त मोंत्रािि।
हाि ही में, एम्बरग्रीस, र्िसे िैरिा हुआ स ना र्ी कहा िािा है , • कायािन्वयन एिेंसी: राष्टरीि स्वि गोंगा यमर्न (एनएमसीजी)
कैनरी द्वीप के तट पर बहकर आिे एक स्पमन व्हे ि के र्ि में पािा • कायिक्रम के स्तोंर्: सीिरे ज उपचार अिसोंरचना, नदी-सतह
गिा र्ा। की सफाई, िनीकरण, औद्यदयगक प्रिाह यनगरानी, नदी-तट
एम्बरग्रीस के बारे में: यिकास, जैि यियििता, सािनजयनक जागरूकता, गोंगा ग्राम।
• एम्बरग्रीस एक म मी पिाथि है ि सोंरर्क्षि स्पमि व्हेि के
4.7 कू न में िीिा की मृ त्यु िर
पाचन तोंत्र से उत्पन्न हदता है।
सोंिर्ि: दयक्षण अफ्रीका और नामीयबिा से स्र्ानाों तररत यकए गए 20
• इसमें मि की बहुि िेज गोंध ह िी है और िह हल्के पीिे रों ग
चीतदों में से, मध्य प्रिे श के कून रािरीय उद्यान से अब तक पाोंच
का पिाथि ह िा है।
चीतदों की मृत्यु की सूचना यमिी है ।
• उपय ग: इसका उपिदग ऐसे इि बनाने के र्िए र्कया िािा
है यजनमें कस्तूरी के नदट् स हदते हैं ।
• र्ारि में िैधार्नकिाएाँ : भारत में एम्बरग्रीस रखने और
व्यापार करने पर प्रर्िबोंध है ।
स्पमि व्हेि के बारे में
• पयाििास: िह दु यनिा भर में समर्ीतदष्ण और उष्णकयटबोंिीि
जि में पािा जाता है।
• सोंरक्षर् की म्भस्थर्ि
 आईयूसीएन: असुरयक्षत ल्कस्र्यत
 cites: पररयर्ष्ट I में
 िन्यिीि (सोंरक्षर्) अर्धर्नयम, 1972: अनुसूची 2 में

4.6 कछु ए: गों गा में सिाई एिें ट


सोंिर्ि:
नमायम गोंगे कािनक्रम के तहत, गोंगा कद साफ करने और िगभग समािार पर और अर्धक
2,600 यकमी नदी नेटिकन कद पुनजीयित करने के यिए कछु ओों कद • राष्टरीि बाघ सोंरक्षर् प्रार्धकरर् (एनटीसीए) के अनुसार,
िािा गिा है । सर्ी मौिें प्राकृर्िक कारर् ों से हुई हैं।
 एनटीसीए पिानिरण मोंत्रािि के तहत एक िैिायनक यनकाि
प्रमुख र्बोंिु
है , यजसे प्र िेर िीिा के कायािन्वयन का काम सौोंपा
• महत्व: कछु ए, सडे हुए िा आिे जिे हुए मानि र्रीर के सार्-
गया है।
सार् नदी में फेंके गए फूिदों कद साफ करने में भूयमका यनभाते
• प्र िेर िीिा: भारत सरकार ने चीतदों कद भारत िापस िाने के
हैं ।
यिए इसे िॉन्च यकिा है ।
 जैि रासाियनक ऑक्सीजन माों ग, मि कदिीफॉमन और
 इस पररिदजना के तहत, कुि 20 रे यडिद कॉिर िािे चीतदों
घुिनर्ीि ऑक्सीजन स्तर में सुिार करते है ।
कद नामीयबिा और दयक्षण अफ्रीका से कुनद नेर्नि पाकन,
नमार्म गोंगे कायिक्रम के बारे में: मध्य प्रदे र् में पहिी बार अोंतरमहाद्वीपीि जगोंिी जानिर का
• िह एक एकीकृत सोंरक्षण यमर्न है , यजसे 2014 में केंद्र स्र्ानाों तरण हुआ है ।
सरकार द्वारा 'फ्लैगर्शप प्र ग्राम' के रूप में अनुम र्िि एर्शयाई िीिा:
र्कया गया था। • सबसे िेज स्थिीय स्तनपायी: िह ज़मीन पर रहने िािा है
• उद्दे श्य: प्रदू षण का प्रभािी उन्मूिन करना। सबसे तेज़ स्तनपािी और र्बना पीछे हटने य ग्य पोंिे िािी
 राष्टरीि नदी गोंगा का सोंरक्षण एिों पुनजीिन। एकमाि र्बल्ली है। इसकी पकड इसे यकसी भी स्पदट्न स कार

73
(3 सेकोंड में 0- 100 यकमी/घोंटा) से भी तेज़ गयत दे ने में मदद
िैर्िक रुझान: • उष्णकर्टबोंधीय क्षेि ों में िन आिरर्
करती है ।
की हार्न: 2022 में 4.1 यमयििन
• र्ारि में म्भस्थर्ि: 1952 में, चीता कद आयिकाररक तौर पर हे रेिर भूयम नष्ट हद गई|
भारत से यििुप्त घदयषत कर यदिा गिा र्ा।
• िन हार्न से काबिन डाइऑक्साइड
• आईयूसीएन म्भस्थर्ि: एयर्िाई चीता कद आईयूसीएन रे ड उत्सििन: 2.7 यबयििन टन, भारत के
यिस्ट द्वारा गोंर्ीर रूप से िुप्तप्राय प्रिार्ि के रूप में
िायषनक जीिाश्म ईोंिन उत्सजनन के
सूिीबद्ध र्कया गया है ि केिि ईरान में िीर्िि है।
बराबर|
4.8 ग्ल बि र्गब्बन ने ट िकि (िीिीएन) • प्राथर्मक िन आिरर् हार्न में िृम्भद्ध:
2021 की तुिना में 2022 में 10%
सोंिर्ि: हाि ही में ग्ल बि र्गब्बन नेटिकि (िीिीएन) की पहिी
अयिक।
बैठक चीन के हाइकदउ में आिदयजत की गई।
• िैर्िक िन ों की कटाई िर: 2022 में
हिॉक र्गब्बन के बारे में:
2018-2020 बेसिाइन से 3.1% कम।
• र्गब्बन सभी िानरदों में सबसे छदटे और तेज़ हैं । • यपछिे 20 िषों में िृक्ष आिरण में कुि
• हूिॉक, यगब्बन िानर ों की 20 प्रिार्िय ों में से एक है और
यमिाकर 100 यमयििन हे रेिर का
र्ारि में पाया िाने िािा एकमाि िानर है।
र्ुद्ध नुकसान हुआ।
• पयाििास: िे एयर्िा के दयक्षणपूिी भाग में उष्णकर्टबोंधीय
• िाजीि और डे म क्रेर्टक ररपम्भिक
और उप ष्णकर्टबोंधीय िोंगि ों में रहिे हैं।
ऑफ काोंग : उच्चतम उष्णकयटबोंिीि
• िनसोंिा: हिॉक र्गब्बन की अनुमायनत जनसोंख्या 12,000 िन आिरण हायन िािे दे र्|
है ।
• दु यनिा 2030 तक िनदों की कटाई कद
• खिरे : बुयनिादी ढाोंचा पररिदजनाओों के यिए पेडदों की कटाई। समाप्त करने और 2030 तक 350
• सोंरक्षर् की म्भस्थर्ि:
यमयििन हे रेिर (एमएचए) खदए और
 भारतीि (िन्यजीि) सोंरक्षण अयियनिम 1972: अनुसूची 1। नष्ट हुए जोंगिदों कद बहाि करने जैसी
 आईयूसीएन म्भस्थर्ि: पयिमी हूिॉक यगब्बन िुप्तप्राि और अपनी अयिकाोंर् िन सोंबोंिी
पूिी हूिॉक सोंिेदनर्ीि ल्कस्तयर् में।
प्रयतबद्धताओों कद पूरा करने की राह पर
ग्ल बि र्गब्बन नेटिकि (िीिीएन) के बारे में: नहीों है ।
• दु यनिा भर में र्गब्बन प्रिार्िय ों और उनके आिास ों के
सोंरक्षर् और सुरक्षा के यिए यमिकर काम करने िािे र्ारि का रुझान • प्राथर्मक िन आिरर् क नुकसान:
र्दिकतान ओ,ों सोंरक्षणिायदिदों और सोंस्र्ानदों का एक अोंिररािरीय भारत ने 2021 और 2022 के बीच 43.9
सहय ग है। हजार हे रेिर आिन प्रार्यमक िन खद
• िह पहि इक िाउों डेशन ग्ल बि, हैनान इों स्टीट्यूट ऑि यदिा, जद इस अियि में दे र् के कुि िृक्ष
नेशनि पाकि, छदटे िानरदों पर आईिूसीएन प्रजायत जीिन रक्षा आिरण नुकसान का 17% है ।
आिदग अनुभाग (आईयूसीएन एसएसए), िूिॉर्िकि  िृक्ष आिरर् का नुकसान: 2021
स साइटी ऑि िोंिन आर्ि का एक सोंयुक्त प्रयास है। और 2022 के बीच भारत में कुि
िृक्ष आिरण का नुकसान 255,000
4.9 ग्ल बि िॉरे स्ट िॉि हे रेिर र्ा।

सोंिर्ि: र्िि सोंसाधन • िह एक िैयश्वक अनुसोंिान सोंगठन है जद


हाि ही में, िल्डन ररसदसेज इों स्टीट्यूट (डब्ल्यूआरआई) द्वारा ग्ल बि सोंस्थान दु यनिा की कुछ सबसे गोंभीर पिान िरणीि
िॉरे स्ट िॉि िॉन्च र्कया गया था। (डब्ल्यूआरआई) और सामायजक चुनौयतिदों का समािान
ग्ल बि फॉरे स्ट िॉि की मुि बािें के बारे में: खदजने की यदर्ा में काम करता है ।

74
4.10 काबि न कै प्िररों ग और र्ों डारर्  कैप्चर की गई काबनन डाइऑक्साइड कद िािुमोंडि में छदडे
जाने के बजाि भूयमगत रूप से सोंग्रहीत यकिा जा सकता है ,
सोंिर्ि:
जहाों िह जििािु पररितनन में िदगदान दे ता है ।
िैज्ञायनक अनुसोंिान की अपनी निीनतम समीक्षा में, इों टरगिननमेंटि
• काबिन डाइऑक्साइड र्नष्कासन (सीडीआर): काबनन कद
पैनि ऑन क्लाइमेट चेंज (आईपीसीसी) ने पािा यक उत्सजनन के
िािुमोंडि से बाहर यनकािा जाता है ।
यिए काबनन कैप्चर और भोंडारण की आिश्यकता हदगी यजसे खत्म
 सीडीआर िािुमोंडि में काबनन डाइऑक्साइड के स्तर कद
करना मुल्किि है ।
कम करता है , यजससे ग्रह ठों डा हदता है , जबयक जीिाश्म
काबिन कैप्िर और स्ट रे ि (सीसीएस) के बारे में: ईोंिन सोंिोंत्रदों और कारखानदों में सीसीएस गैस कद बाहर
• िह काबनन उत्सजनन कद कम करने का एक तरीका है , जद यनकिने से रदकता है ।
ग्लदबि िायमिंग से यनपटने में मदद करने में महत्वपूणन हद सकता
4.11 एों थ्र प सीन यु ग
है । सीसीएस प्रयक्रिा के तीन चरण:
सोंिर्ि: हाि ही में, िैज्ञार्नक ों ने कनाडा में क्रॉिडि झीि क
1. र्ोंडारर् के र्िए काबिन डाइऑक्साइड क कैप्िर
एों थ्रदपदसीन िुग के यिए ग्राउों ड-ज़ीरद के रूप में नायमत यकिा है।
करना: CO2 कद औद्यदयगक प्रयक्रिाओों में उत्पायदत अन्य
गैसदों से अिग यकिा जाता है , जैसे यक कदििा और एों थ्र प सीन युग के बारे में:
प्राकृयतक गैस से चिने िािे यबजिी उत्पादन सोंिोंत्रदों िा • िह ितनमान भूिैज्ञायनक समि अोंतराि कद दर्ानता है , यजसमें
स्टीि िा सीमेंट कारखानदों में। पृथ्वी का पाररल्कस्र्यतकी तोंत्र मानि प्रभाि के कारण आमूि-चूि
2. पररिहन: यफर CO2 कद सोंपीयडत यकिा जाता है और पररितननदों से गुजरा है , खासकर औद्यदयगक क्राों यत की र्ुरुआत
पाइपिाइनदों, सडक पररिहन िा जहाजदों के माध्यम से के बाद से।
भोंडारण के यिए एक साइट पर िे जािा जाता है । • अिाोंर्छि पररिििन: िह ग्लदबि िायमिंग, समुि के स्तर में िृल्कद्ध,
3. र्ोंडारर्: यफर CO2 कद स्र्ािी भोंडारण के यिए गहरी समुि के अम्लीकरण, बडे पैमाने पर यमट्टी के कटाि, घातक
भूयमगत चट्टानदों में डािा जाता है । गमी की िहरदों के आगमन, जीिमोंडि की यगरािट और
• सीसीएस का महत्व: िह यबजिी उत्पादन और उद्यदग में पिानिरण में अन्य हायनकारक पररितननदों से जुडा है ।
जीिाश्म ईोंिन के यनरों तर उपिदग की अनुमयत दे ता है जब तक • इसे पॉि क्रुटजेन और यूिीन स्ट एमिर (Paul Crutzen and
यक कम काबनन यिकल्प िागू नहीों यकिा जा सके। Eugene Stoermer)द्वारा गढ़ा गया है।
 1.5°C और 2°C जििािु िक्ष्यदों के यिए आिश्यक • युग की अनुमार्नि शुरुआि: एों थ्रदपदसीन ियकिंग ग्रुप
नकारात्मक CO2 उत्सजनन कद प्राप्त करना। (एडब्ल्यूजी) के सदस्यदों ने अनुमान िगािा है यक निा िुग 1950
और 1954 के बीच यकसी समि र्ुरू हुआ र्ा।

पृथ्वी के र्ूिैज्ञार्नक समय पैमाने के बारे में:


• िह कािानुक्रयमक डे यटों ग की एक प्रणािी है जद भूिैज्ञायनक
स्तरदों कद जदडती है ।
• इसका उपिदग भूिैज्ञायनकदों, जीिाश्म यिज्ञायनिदों और अन्य पृथ्वी
िैज्ञायनकदों द्वारा पृथ्वी के इर्िहास के िौरान घर्टि घटनाओों
के समय और सोंबोंध ों का िर्िन करने के र्िए र्कया िािा
है।
• इसे अमूतन समि की नायमत इकाइिदों में यिभायजत यकिा गिा है
- यजन्हें अियि के अिरदही क्रम में िगािा गिा है , अोंग्रेजी भाषा
में इसे अिग-अिग नामदों से जाना जाता है ।

4.12 पमाि फ्रॉस्ट र्िगिन: बटागाइका क्रे टर


सोंिर्ि: बटागाइका क्रेटर, दु यनिा का सबसे बडा पमानफ्रॉस्ट क्रेटर,
सोंबोंर्धि शब्दाििी
पमान फ्रॉस्ट के यपघिने के कारण खतरनाक दर से यिस्तार कर रहा
• काबिन कैप्िर, उपय ग और र्ोंडारर् (CCUS): िह एक ऐसी
है , जद ग्लदबि िायमिंग से उत्प्रेररत एक घटना है ।
प्रयक्रिा है जद औद्यदयगक प्रयक्रिाओों, यबजिी सोंिोंत्रदों और अन्य
बटागाइका क्रेटर के बारे में:
स्रदतदों से काबनन डाइऑक्साइड कद िािुमोंडि में छदडे जाने से
• स्थान: क्रेटर साखा गणराज्य में ल्कस्र्त है ।
पहिे एकत्र करती है ।
75
 साखा रूस का सबसे बडा गणराज्य है , जद आकनयटक  इसके र्पघिने से ग्लदबि िायमिंग में तेजी आएगी यजसके
महासागर के यकनारे रूसी सुदूर पूिन में ल्कस्र्त है पररणामस्वरूप र्ेष पमानफ्रॉस्ट के यपघिने में तेजी आएगी,
• इसका यनमान ण 1960 के दर्क में र्ुरू हुआ जब िन ों की िह एक ऐसी घटना है यजसे पमािफ्रॉस्ट काबिन िीडबैक
कटाई के कारर् र्ूर्मगि पमािफ्रॉस्ट र्पघि गया, यजससे िक्र के रूप में िाना िािा है।
भूयम डूबने िगी। • िायरस और बैरीररया का र्नकिना: िैज्ञायनकदों कद
पमािफ्रॉस्ट क्ा है? पमान फ्रॉस्ट में 400,000 साि से अयिक पुराने रदगाणु यमिे हैं ।
• िमे हुए क्षेि: पमानफ्रॉस्ट एक प्रकार की जमीन िा यमट्टी है जद  ऐसा माना िािा है र्क बुब र्नक प्लेग और िेिक िैसे
िगािार ि या अर्धक िषों िक र्हमाोंक र्बोंिु (0 यडग्री कई िायरस र्िन्ह न ों े मानििा क िस्त र्कया है, िे इस
सेल्कल्सिस िा 32 यडग्री फारे नहाइट) पर िा उससे नीचे जमी िमे हुए स्तर में र्नम्भिय पड़े हुए हैं।
रहती है । • पाररम्भस्थर्िक िोंि और उनकी िैि र्िर्िधिा क नुकसान:
• उच्च अक्षाोंश क्षेि: िह उन क्षेत्रदों में पािा जाता है जहाों तापमान साइबेररिाई पमानफ्रॉस्ट का यपघिना टुों डरा कद कीचड भरे
पूरे िषन िगातार ठों डा रहता है , आमतौर पर ध्रुि ों के पास उच्च पररदृश्य में बदि रहा है , यजससे स्र्ानीि िनस्पयतिाों मर रही हैं
अक्षाोंश क्षेि ों में, िैसे आकिर्टक और उपआकिर्टक क्षेि ों में। और इस पर भदजन करने िािे िन्यजीि भूखे मर रहे हैं ।
• र्ूस्खिन: पमानफ्रॉस्ट पर बने र्हरदों में भूस्खिन हद सकता है,
पमािफ्रॉस्ट र्पघिने का प्रर्ाि:
यजससे बुयनिादी ढाोंचे कद नुकसान हद सकता है , जैसे सडकदों
• ग्रीनहाउस गैस ों का र्नििहन: पमान फ्रॉस्ट उपमृिा में गीगाटन
का झुकना, घरदों का बोंट जाना और पाइपिाइनदों का बायित
मीथेन और काबिन डाइऑक्साइड रखिा है।
हदना।

76
5. फ कस आले ख
5.1 चरम मौसम घटना
संदभा:
िाल िी में, जुलाई 2023 के दौरान पृथ्वी के सभी सात मिाद्वीपों में असािािर् चरम मौसम की घटनाओं का अनुभव हुआ िै ।

चरम मौसम की घटनाओं के बारे में


• परिभाषा: जब मौसम की क्तस्थहत सामान्य
मौसम की तुलना में काफी बडा अंतर
क्रदखाती िै, त इसे चरम मौसम या
गंभीर मौसम किा जाता िै।
• प्रभाव: क्रवश्व मौसम क्रवज्ञान संगठन
(िब्ल्यूएमओ) के अनुसाि, चिम
मौसम, जलवायु औि पानी से संबंहित
घटनाओं के कािर् वैक्रश्वक स्तर पर
11,778 आपदाएाँ दजा की गईं, क्रजससे
1970 और 2021 के बीच द क्रमक्रलयन
से अक्रधक मौतें हुईं और 4.3 क्रट्क्रलयन
अमेररकी िॉलर का आक्रथाक नुकसान
हुआ।
 दु क्रनयाभर में दजा की गई 90 प्रक्रतशत से अक्रधक मौतें हवकासशील िे शों में हुईं।
 भाित में 1970 से 2021 के बीच 573 आपदाएाँ आईं क्रजनमें 1,38,377 लोगों की जान चली गई।

चरम मौसम की घटनाओं के प्रकार

घटनाएं क्रववरण उदािरण

िीटवेव • िीटवेव तब िोती िै जब हकसी स्थान पि तीन हिनों की • जून 2023 में यूर पीय अंतररक्ष एजेंसी के अनुसार,
अवहि में अक्रधकतम और न्यूनतम तापमान इटली के हसहसली औि साहडा हनया द्वीप अपने अब तक के
असामान्य रूप से गमा ि ता िै। सबसे गमा तापमान का अनुभव कि सकते िैं , अनुमाहनत
 इसे स्थान की स्थानीय जलवायु औि हपछले मौसम उच्चतम तापमान 48 हडग्री सेक्तल्सयस (118F) िोगा -
के संबंि में माना जाता िै । संभवतः यूिोप में अब तक का सबसे गमा तापमान िजा
• भाित में, गमी की लििें आम तौि पि माचा और जून के हकया गया िै ।
बीच ि ती िैं, औि कुछ िु लाभ मामलों में जुलाई तक भी • अप्रैल के मध्य में चीन, भाित, थाईलैंड, लाओस,
बढ़ जाती िैं । बां ग्लािे श औि एहशयाई मिाद्वीप के कई िे शों में लू चली
और तापमान 38 क्रिग्री से 45 क्रिग्री सेक्तियस के बीच
रिा। क्रजसने एक दजान से अक्रधक दे श ं में तापमान के
ररकॉिा त ड क्रदए।

जंगल की • जंगल की आग, क्रजसे जंगल, झाडी भी किा जाता िै, • कनािा: कनाडा इस समय अपने अब तक के सबसे
आग/झाडी की क जंगल, घास के मैदान, िश भूक्रम या टुं ि्ा जैसी खराब जंगल की आग के मौसम का सामना कर रिा
आग प्राकृक्रतक सेक्रटं ग में पौध ं के हकसी भी अहनयंहत्रत औि िै, क्रजसका मुख्य कािर् अत्यहिक शुष्क अवहि औि
गैि-हनिाा रित ििन या जलने के रूप में वहर्ात हकया जा उसके बाि लू चलना िै।
सकता िै , जो प्राकृहतक ईंिन का उपभोग किता िै औि

77
पयााविर्ीय परिक्तस्थहतयों के आिाि पि फैलता िै । (जैसे,
िवा, स्थलाकृहत)।

सूखा • सूखा प्राकृक्रतक जलवायु चि में एक लंबी शुष्क • िॉना ऑफ अफ्ीका सूखा, 2020-2023: िु हनया के
अवक्रध िै जो िु हनया में किीं भी िो सकती िै । सबसे गिीब क्षेत्रों में से एक, िॉना ऑफ अरीका में तीन
• इसकी हवशेषता वषाा की कमी िै , हजसके साल के सूखे के कािर् फसलें सूख गईं, पानी खत्म िो
परिर्ामस्वरूप पानी की कमी िोती िै । गया औि केन्या, सोमाहलया औि इहथयोहपया के बडे
हिस्सों में पशुिन भूखा मि गया।

क र्ल् स्नैप्स • कोल्ड स्नैप तापमान में बहुत अचानक क्रगरावट िै , जो • 2023 में, उत्तरी चीन के म िे शिर में तापमान शून्य
कोल्ड स्पेल (स्पेल का अथा िै समय की लंबाई) औि से 53 क्रिग्री सेक्तियस (शून्य से 63.4 हडग्री फ़ािे निाइट)
शीत लिि (एक लिि िीिे -िीिे आती िै औि टू टने तक तक हगि गया, जो िे श में अब तक िजा हकया गया सबसे
बढ़ती िै) के हवपिीत िै । कम तापमान िै ।
• भारतीय मौसम क्रवज्ञान क्रवभाग (IMD) शीत लिर
की घ षणा तब करता िै जब मैिानी इलाकों में हकसी
मौसम केंद्र पि न्यूनतम तापमान 10 क्रिग्री सेक्तियस
के बराबर या उससे कम ि ता िै और उस अवक्रध के
क्रलए सामान्य तापमान से 4.5 से 6.4 क्रिग्री सेक्तियस
कम ि ता िै।

ऊष्णकक्रटबंधीय • यि उष्णकहटबंिीय मिासागिों के ऊपि उत्पन्न िोने • उष्णकक्रटबंधीय तूफान 'मावर' ने मई के अंत औि जून
चिवात वाला एक तीव्र गक्रत से घूमने वाला तूफान िै जिां से की शुरुआत में गुआम, हफलीपींस, ताइवान औि जापान
यि हवकहसत िोने के हलए ऊजाा प्राप्त किता िै । को प्रभाहवत हकया।
• इसमें एक कम दबाव का केंद्र िै और बादल "आं ख" • इसे 2023 में क्रवश्व स्तर पर अब तक का सबसे
के आसपास नेत्रग लक की ओर बढते िैं , यि इस शक्तिशाली उष्णकक्रटबंधीय चिवात औि मई में िजा
प्रर्ाली का मध्य भाग िै जिां मौसम सामान्य रूप से हकया गया अब तक का सबसे मजबूत उििी गोलािा
शां त औि बािलों से मुि िोता िै । चिवात बताया गया िै ।
• आईपीसीस के अनुसाि, क्रपछले चार दशक ं में ये • चिवात म चा म्यांमार: यि इस क्षेत्र में आए अब तक
अक्रधक बार हुए िैं। के सबसे शक्तिशाली चिवातों में से एक साहबत हुआ।
 संयुि िाष्टर के अनुसाि, चिवात ने कुल क्रमलाकर
800,000 ल ग ं क प्रभाक्रवत क्रकया।

मूसलाधार • मूसलािाि बारिश, ऐसी हकसी भी मात्रा में िोने वाली • उत्तरी भारत में, लगाताि मानसूनी बारिश ने कम से कम
बाररश बाररश िै क्रजसे क्रवशेष रूप से भारी माना जाता िै। 90 लोगों की जान ले ली िै , हजससे क्षेत्र की पिले से िी
भीषर् गमी औि बढ़ गई िै ।

बाढ और • आकक्तिक बाढ़ से तात्पया उस बाढ़ की क्तस्थहत से िै जो • पाक्रकस्तान बाढ, 2022: रिकॉडा मानसूनी बारिश के
आकक्तस्मक बाढ बहुत कम समय यानी छि घंटे में घक्रटत ि ती िै। कािर् आई बाढ़ ने 2022 की गहमायों के िौिान लगभग
• भाित में, आकक्तिक बाढ़ अक्सर बादल फटने से 1,500 लोगों की जान ले ली, जबहक लाखों लोग साफ
जुडी ि ती िै - थ डे समय में अचानक, तीव्र वषाा। पानी औि भोजन की कमी से प्रभाहवत हुए।
• राष्ट््ीय बाढ आय ग के अनुसाि, भाित में लगभग 40 • िे श में अगस्त में सामान्य से तीन गुना अहिक बारिश हुई,
हमहलयन िे िेयि भूहम बाढ़ के हलए उिििायी िै औि हजससे यि 1961 के बाद से सबसे अक्रधक बाररश
औसतन 18.6 हमहलयन िे िेयि भूहम सालाना प्रभाहवत वाला अगस्त बन गया।
िोती िै ।

78
 अत्यक्रधक वषाा और कुल वषाा िोनों में वृक्तद् ने कुछ
क्षेत्रों में गंभीि बाढ़ की घटनाओं में वृक्तद् में योगिान
हिया िै ।

भूस्खलन • भूस्खलन प्राकृक्रतक आपदाएाँ िैं जो मुख्य रूप से • भाित को वैक्रश्वक स्तर पर शीषा पांच भूस्खलन-प्रवण
पिाडी इलाकों में िोती िैं जिााँ हमट्टी, चट्टान, भूहवज्ञान दे श ं में से एक माना जाता िै।
औि ढलान की अनुकूल परिक्तस्थहतयााँ िोती िैं । • िे श का लगभग 12.6% भौगोहलक भूहम क्षेत्र भूस्खलन की
• हकसी ढलान से चट्टान, पत्थर, क्रमट्टी या मलबे का दृहष्ट से संवेिनशील िै ।
अचानक क्तखसकना भूस्खलन किलाता िै । • लगभग 66.5% भूस्खलन उिि-पहिमी हिमालय से,
• इसके हटर गि किने वाले प्राकृहतक कािर्ों में भािी वषाा , लगभग 18.8% उिि-पूवी हिमालय से, औि लगभग
भूकंप, बफा का हपघलना औि बाढ़ के कािर् ढलानों का 14.7% पहिमी घाट से रिपोटा हकए जाते िैं ।
कम िोना शाहमल िै । • 2022 में, भाित के 10 िाज्यों में मानसून के िौिान कई
• मानवजक्रनत गक्रतक्रवक्रधयााँ जैसे उत्खनन, पिाहडयों औि हजलों में भूस्खलन से संबंहित 182 मौतें हुईं।
पेडों की कटाई, अत्यहिक बुहनयािी ढााँचे का हवकास
औि मवेहशयों द्वािा अत्यहिक चिाई।

वैक्रश्वक चरम मौसम की घटनाओं क प्रभाक्रवत करने वाले कारक:


• मानवजक्रनत कारक: मानवीय गहतहवहियााँ जैसे औद्योहगक गहतहवहियााँ , जीवाश्म ईंिन का जलना आहि िमािी वैहश्वक जलवायु में तेजी से
बिलाव ला ििे िैं ।
 ग्रीनिाउस गैस उत्सजान और भूक्रम उपय ग पररवतान िु हनया भि में चरम जलवायु पररवतान की घटनाओं में प्रमुख कािक िैं ।
 प्रभाव: ग्ल बल वाक्रमिंग के कारण बंगाल की खाडी और अरब सागर में चिवाती तूफान भी तेजी से बढ़ ििे िैं औि लंबी अवहि
तक अपनी तीव्रता बिकिाि िख ििे िैं ।
• ग्लोबल वाहमिंग: ग्लोबल वाहमिंग ग्रि के समग्र तापमान में दीघाकाक्रलक वृक्तद् िै।
 यि बहुत गमा हिनों औि िातों की संभावना को बढ़ाकि गमी की लिर ं की तीव्रता में य गदान कर सकता िै।
 गमा िवा वाष्पीकरण क बढाती िै, क्रजससे सूखे की क्तस्थक्रत क्रबगड सकती िै।
 अहिक सूखे से खेत और जंगल शुष्क ि जाते िैं क्रजनमें आग लगने का खतरा ि ता िै , औि बढ़ते तापमान का मतलब जंगल की
आग का मौसम लंबा ि ना िै।
 इससे वायुमंिल में जलवाष्प बढ जाता िै, हजससे बार-बार भारी बाररश और बफीले तूफान आ सकते िैं।
 इससे समुद्र का स्तर बढ जाता िै, क्रजससे समुद्री जल की मात्रा बढ जाती िै, साथ िी अहिक वषाा िोती िै , जो तटीय तूफानों के
िौिान हकनािे पि आ जाता िै ।
 समुद्री जल, अहिक वषाा के साथ, क्रवनाशकारी बाढ का कारण बन सकता िै।
 यि जलवायु पररवतान का कारण बनता िै, जो पृथ्वी पि जीवन के हलए गंभीि खतिा पैिा किता िै ।
• एल नीन , ला-नीना आक्रद सक्रित प्राकृक्रतक जलवायु पैटना का िु हनया भि में मौसम की क्तस्थहत पि िू िगामी प्रभाव पडता िै ।

एल नीन ला-नीना

• यि एक प्राकृहतक जलवायु चि िै जो िर तीन से सात साल में • यि भूमध्यिे खीय प्रशां त क्षेत्र में ठं डे समुद्री तापमान के साथ
ि ता िै, हजसमें मध्य औि पूवी उष्णकहटबंिीय प्रशांत क्षेत्र में समुद्र शीतलन प्रभाव का प्रक्रतक्रनक्रधत्व करता िै।
की सति का तापमान सामान्य से अहिक गमा िोता िै । • ला नीना घटनाओं के िौिान, व्यापाररक िवाएाँ सामान्य से भी
• यि सामान्य वायुमंिलीय पररसंचरण क बाक्रधत करता िै , अक्रधक तेज ि ती िैं, जो अहिक गमा पानी को एहशया की ओि
क्रजससे प्रशांत जेट स्ट् ीम अपनी तटस्थ क्तस्थहत से िहक्षर् की ओि िकेलती िैं ।
बढ़ने लगती िै । • दक्रक्षणी अमेररका में सूखा और प्रशांत उिि-पहिम औि
• इस बिलाव के साथ, उत्तरी अमेररका और कनािा के क्षेत्र कनाडा में भािी बारिश औि बाढ़ आती िै।
सामान्य से अक्रधक शुष्क और गमा ि गए िैं।

79
• उच्च वायुमंिलीय िवाएाँ : जेट स्टर ीम विााँ पाई जाती िै जिााँ पृथ्वी के ध्रुवों से आने वाली ठं डी िवा गमा उष्णकहटबंिीय िवा से हमलती िै ।
 ये िवाएाँ उििी गोलािा में पहिम से पूवा औि िहक्षर्ी गोलािा में पूवा से पहिम तक मौसम प्रणाली क जारी रखने और क्रनयंक्रत्रत करने
में मदद करती िैं।
 कभी-कभी ये िवाएाँ अपने साथ अक्रप्रय मौसम लेकर आती िैं क्रजससे बवंिर का क्रनमााण ि सकता िै।
भारत में चरम मौसम की घटनाएाँ
• भारत मौसम क्रवज्ञान क्रवभाग
(आईएमिी) के अनुसाि, भाित में 2022
में 365 क्रदन ं में से 314 क्रदन ं में चरम
मौसम की घटनाओं का अनुभव हुआ।
 मानव जीवन की िाक्रन: आईएमडी के
अनुसाि, भाित में 2022 में चिम मौसम
की घटनाओं के कािर् 2,227 मानव
मृत्यु दजा की गिी हैं ।
• भाित में, वषा 2023 में अभूतपूवा जलवायु
घटनाओं की एक श्रृंखला दे खी गई िै,
जो जलवायु परिवतान के कािर् िोने वाली
पिे शान किने वाली नई सामान्य क्तस्थहत का
संकेत िे ती िै ।
 तापमान: 2023 की शुरुआत भीषर्
तापमान के साथ हुई हजसने फरवरी में
123 साल पुराना ररकॉिा त ड क्रदया।
 चिवाती घटना: अरब सागर में
चिवात क्रबपरजॉय का हनमाार् हुआ,
जो 13 हिनों तक चला, जो 1977 के
बाद से सबसे लंबी अवक्रध का चिवात बन गया। छक्रव िेक्रिट: टीओआई
 भूस्खलन: क्रिमाचल प्रदे श राज्य आपातकालीन प्रक्रतक्रिया केंद्र के अनुसाि , 2023 में, 24 जून को हिमाचल प्रिे श में मानसून की
शुरुआत के बाि से, भूस्खलन, अचानक बाढ और बादल फटने जैसी बाररश से संबंक्रधत घटनाओं में अब तक 154 ल ग ं की
मौत ि चुकी िै।
 मानसून पररवतानशीलता: भारत मौसम क्रवज्ञान क्रवभाग (आईएमिी) के अनुसाि, जुलाई तक, उिि पहिम भाित में 59% अहिक
वषाा हुई; मध्य भाित में 4% अहिक; प्रायद्वीपीय भाित में 23% की कमी औि पूवी औि पूवोिि भाित में 17% की कमी।
• जंगल की आग: हपछले िो िशकों में, 2000 से 2020 तक, भाित में जंगल की आग की घटनाओं की आवृहि में 52% की वृक्तद् हुई िै।
2023 में, भारत में माचा की शुरुआत में उपग्रि द्वािा पता लगाई गई जंगल की आग में हपछले वषा की तुलना में लगभग 115% की वृक्तद्
दे खी गई।
• क्रवत्तीय क्रनक्रिताथा: जलवायु परिवतान के कािर् भाित को अपने सकल घरे लू उत्पाद का 3-5% का नुकसान ि रिा िै और यक्रद वाक्रमिंग
क 2 क्रिग्री सेक्तियस से नीचे सीक्रमत निी ं क्रकया गया त यि संख्या 10% तक बढ सकती िै।
• क्रवस्थापन: क्रजनेवा क्तस्थत आं तररक क्रवस्थापन क्रनगरानी केंद्र की एक रिपोटा के अनुसाि, प्राकृहतक आपिाओं, हवशेष रूप से भािी बाढ़
औि चिवातों के कािर् 2022 में भारत में लगभग 2.5 क्रमक्रलयन आं तररक क्रवस्थापन हुए।

भारत में मौसम की घटनाओं क प्रभाक्रवत करने वाले कारक


• जलवायु पररवतान: भारतीय प्रौद्य क्रगकी संस्थान, गांधीनगर की एक रिपोटा के अनुसाि, जलवायु पररवतान के कारण भक्रवष्य में भारत
में तीव्र मौसमी बाररश, भूस्खलन, बाढ और लू जैसी चिम मौसमी घटनाओं की आवृहि कई गुना बढ़ने का अनुमान िै ।
• मानसून: भारत की लगभग 80% वाहषाक वषाा जून से हसतंबि तक मानसून के मिीनों के िौिान िोती िै , हजससे भारी वषाा ि ती िै और
बाढ का खतरा बढ जाता िै।
 मानसून पैटना में बदलाव: इसकी हवशेषता लंबे समय तक कम वषाा की अवहि औि उसके बाि कुछ हिनों में तीव्र बारिश िोना िै ।

80
 उत्तर भारत में िाल िी में हुई भारी वषाा क कम दबाव वाली मौसम प्रणाली क्रजसे पक्रिमी क्रवक्ष भ और दक्रक्षण-पक्रिम मानसून
के रूप में जाना जाता िै, के बीच संपका के हलए हजम्मेिाि ठििाया जा सकता िै ।
• चिवात: उििी हिंि मिासागि में चिवातों का मानसून की शुरुआत पर सकारात्मक और नकारात्मक द न ं प्रभाव पडा िै।
 उदािरण: म चा चिवात ने अंडमान औि हनकोबाि द्वीप समूि में मानसून को समय पि पहुं चने में मिि की। दू सरी ओर, चिवात
क्रबपरजॉय के कािर् मुंबई में मानसून के आगमन में लगभग िो सप्ताि की िे िी हुई।
• मिासागर का गमा ि ना: मानसून का मागा तीन उष्णकहटबंिीय मिासागिों - भारतीय, अटलांक्रटक और प्रशांत से प्रभाक्रवत ि ता िै;
आकाक्रटक से 'वायुमंिलीय पुल'; औि समुद्री सुिंग के साथ-साथ िहक्षर्ी मिासागि से वायुमंडलीय पुल पि भी असि पडता िै ।
 भाित के पूवी समुद्र में समुद्री गमी के कारण उत्तर-पक्रिम में अत्यक्रधक बाररश ि रिी िै।
• अक्तस्थर श षण: मेगा सडक हवस्ताि परियोजना से लेकि कैस्केक्रिं ग पनक्रबजली पररय जनाओं के क्रनमााण तक, शििों के अहनयोहजत
हवस्ताि से लेकि अक्तस्थि पयाटन तक, भाितीय िाज्यों ने नाजुक पाररक्तस्थक्रतकी के बारे में चेतावक्रनय ं क नजरअंदाज कर क्रदया िै।
• क्रपघलते ग्लेक्रशयर: भारतीय हिमालयी िाज्यों को ग्लेहशयिों के हपघलने के कािर् बनने वाली अहतप्रवाहित हिमनि झीलों की चुनौहतयों का
सामना किना पडता िै ।
• वन ं की कटाई और क्रमट्टी का कटाव: जंगलों या घास के मैिानों जैसी वनस्पहत को िटाने से क्रमट्टी की प्राकृक्रतक जल अवश षण क्षमता
बाक्रधत ि सकती िै।
आगे की राि
• जलवायु न्याय: पयाा विर्ीय न्याय को केक्तित किना औि प्रत्येक समुिाय
में स्वच्छ ऊजाा तक पहुंच का हवस्ताि किना।
 िु हनया भि में कम आय वाले समुिायों औि िं गीन समुिायों, समस्या
के हलए कम से कम हजम्मेिाि लोगों औि स्थानों पि जलवायु परिवतान
के असंगत प्रभाव ं क पिचानता िै ।
 यि ऐसे समाधान ख जता िै ज जलवायु पररवतान के मूल कारण ं
का समाधान करते िैं औि सामाहजक, नस्लीय औि पयााविर्ीय
अन्याय की एक हवस्तृत श्रृंखला को संबोहित किते िैं ।
 जलवायु न्याय के छि स्तंभ िैं:
• क्रनमााण के क्रलए वैज्ञाक्रनक तरीके: पाररक्तस्थक्रतक नाजुक क्षेत्र में हनमाा र्
किते समय सभी वैज्ञाहनक तिीकों का उपयोग हकया जाना चाहिए।
 सतत भूक्रम प्रबंधन: हजम्मेिाि भूहम प्रबंिन प्रथाओं को प्रोत्साहित
किना, जैसे प्राकृहतक परिदृश्य को संिहक्षत किने के हलए अत्यक्रधक
उत्खनन और वन ं की कटाई से बचना और जल क्रनकासी प्रणाक्रलय ं सक्रित अच्छी तरि से क्रिजाइन और पयााप्त रूप से बनाए
गए बुक्रनयादी ढांचे में क्रनवेश करना।
 चिम सीमाओं के प्रभाव को कम किने के हलए शिर ं के क्रलए जलवायु अनुकूल बुक्रनयादी ढांचे क लागू करना।
• प्रारं क्रभक चेतावनी प्रणाक्रलय ं क मजबूत करना: बेितर क्रवश्व मौसम क्रवज्ञान संगठन (WMO):
पूवाानुमान तकनीक, बेितर बुक्रनयादी ढााँचा क्रिजाइन, और • यि संयुि राष्ट् (यूएन) की क्रवक्रशष्ट् एजेंसी िै।

क्रवनाशकारी आपदाओं के प्रभाव क कम करने के क्रलए • यि पृथ्वी के वायुमंडल की क्तस्थहत औि व्यविाि, मिासागिों के साथ इसके
सुिक्षा-प्रथम िशान को अपनाना। संपका, इससे पैिा िोने वाली जलवायु औि इसके परिर्ामस्वरूप जल
 समुदाय ं औि अहिकारियों को चेतावनी िे ने के हलए कुशल संसािनों के हवतिर् पि संयुि िाष्टर प्रर्ाली की आहिकारिक आवाज िै ।
और क्रवश्वसनीय पूवाानुमान क्रवक्रधयााँ । भारत मौसम क्रवज्ञान क्रवभाग (आईएमिी) के बारे में
 उपग्रि हचत्र, स्थानीयकृत डॉपलि िडाि, मौसम मॉडल औि • इसकी स्थापना 1875 में हुई थी।
डे टा हवश्लेषर् आिुहनक मौसम हवज्ञान तकनीक के • भाित सिकाि के पृथ्वी क्रवज्ञान मं त्रालय की एक एजेंसी िै ।
उिाििर् िैं ज वषाा पैटना की भक्रवष्यवाणी करने , बाढ़ • यि मौसम संबंिी हटप्पहर्यों, मौसम पूवाानुमान औि भूकंप हवज्ञान के हलए
संभाहवत स्थानों की पिचान किने औि समय पि चेतावनी क्रजम्मेदार प्रमुख एजेंसी िै।
िे ने में सिायता कि सकते िैं। • आईएमडी हवश्व मौसम हवज्ञान संगठन के छि क्षेत्रीय हवहशष्ट मौसम हवज्ञान
केंद्रों में से एक िै ।

81
• वनीकरण और मृदा संरक्षण: वनीकिर् को बढ़ावा िे ने औि उद्रण:
मौजूिा वनों की िक्षा किने से हमट्टी की प्राकृहतक जल अवशोषर् • “ग्लोबल वाहमिंग का युग समाप्त िो गया िै ; वैहश्वक उबाल का युग आ गया
क्षमता को बनाए िखने में मिि हमलती िै । िै " - संयुि राष्ट्् प्रमुख एं ट क्रनय गुटेरेस
• सिय गात्मक रणनीक्रतयााँ: चरम मौसम की घटनाओं के • “मौसम आपको इतनी जल्दी माि सकता िै । जीहवत ििने की पिली
कारण ि ने वाली समस्याओं के समािान के हलए सिकाि, प्राथहमकता चिम मौसम से सुिक्षा प्राप्त किना िै " - बेयर क्रग्रि
शैक्षहर्क संस्थानों, इं जीहनयरिं ग फमों, शििी योजनाकािों औि
स्थानीय समुिायों के बीच सियोग आवश्यक िै ।
• सामुदाक्रयक भागीदारी: इसे आपदा प्रबंधन में एक आवश्यक तत्व के रूप में मान्यता िी गई िै ताहक आपिा घटना में तेजी से वृक्तद् की
हवश्वव्यापी प्रवृहि को उलट हिया जा सके।
• स्थानीय समाधान: संवेिनशील क्षेत्रों की पिचान किने औि अनुकूहलत आपदा प्रबंधन रणनीक्रतय ं क क्रवकक्रसत करने के क्रलए स्थानीय
स्तर पर सूक्ष्म-काया य जनाओं की आवश्यकता।
• जागरूकता: सावाजक्रनक क्रशक्षा, क्रनकासी रणनीक्रतय ं और सामुदाक्रयक प्रक्रशक्षण कायािम ं के क्रलए अक्रभयान यि सुक्रनक्रित करते
िैं क्रक स्थानीय ल ग बाढ की क्तस्थक्रतय ं के क्रलए तैयार िैं औि खुि को बचाने औि प्रभावी प्रथम प्रहतहियाकताा बनने के हलए तुिंत काया
कि सकते िैं ।

प्र. भाित के प्रमुख शिि बाढ़ की क्तस्थहत के प्रहत संवेिनशील िोते जा ििे िैं । चचाा कीहजए। (2016)
प्र. चिम मौसम की घटनाओं के कािर्ों, परिर्ामों औि हनहिताथों पि चचाा कीहजए। िाष्टरीय औि अंतिाा ष्टरीय स्ति पि ऐसी घटनाओं से जुडे
जोक्तखमों को कम किने के हलए प्रभावी िर्नीहतयों का सुझाव िीहजए।

82
6. र्िज्ञान और प्रौद्य र्गकी
6.1 िीन म अनु क्र मर्
सोंिर्ि:
पहाडी यजिदों में पैर्दिॉजी सेिाओों कद बढािा दे ने के यिए, उिराखोंड के पौडी गढिाि यजिे के श्रीनगर मेयडकि कॉिेज में एक जीनदम अनुक्रमण
प्रिदगर्ािा खदिी गई।
िीन म और िीन म अनुक्रमर् के बारे में:
• जीनदम कद यकसी जीि के सभी जीन सयहत डीऑक्सीराइबदज न्यूल्कक्लक एयसड (डीएनए)
के पूणन सेट के रूप में पररभायषत यकिा गिा है ।
• िह यकसी जीि के जीनदम के सोंपूणन डीएनए अनुक्रम कद यनिान ररत करने की प्रयक्रिा है ।
• डीएनए में चार आिारदों - एडे र्नन (ए), साइट र्सन (सी), गुआर्नन (िी) और
थाइर्मन (टी) द्वारा र्नर्मिि एक डबि-स्टरैं डेड अर्ु ह िा है।
• मानि के आनुिोंर्शक र्िोंगरर्प्रोंट क र्डक ड करने के र्िए आिार जदडे के क्रम कद
समझने की प्रयक्रिा कद जीनदम अनुक्रमण कहा जाता है ।
िीन म अनुक्रमर् से िुड़े मुद्दे:
• ग पनीयिा: इसमें यकसी व्यल्कक्त के डीएनए का यिश्लेषण करना र्ायमि है , यजसमें
सोंिेदनर्ीि व्यल्कक्तगत जानकारी हदती है , यजससे गदपनीिता की रक्षा करते हुए अनुसोंिान
के यिए डे टा साझा करने में सोंतुिन बनाना चुनौतीपूणन हद जाता है ।
• सूर्िि सहमर्ि: व्यल्कक्तिदों से सूयचत सहमयत प्राप्त करने में चुनौयतिाँ हैं ,
खासकर ऐसे मामिदों में जहाों अनुक्रमण एक बडे र्दि अध्यिन के यहस्से के
रूप में यकिा जाता है।
• आनुिोंर्शक र्ेिर्ाि: िह रदग की सोंिेदनर्ीिता कद प्रकट कर सकता है ,
मनदिैज्ञायनक सोंकट पैदा कर सकता है और आनुिोंयर्क भेदभाि कद जन्म दे
सकता है ।
• मन िैज्ञार्नक प्रर्ाि: िह यकसी व्यल्कक्त की कुछ बीमाररिदों के प्रयत
सोंिेदनर्ीिता के बारे में जानकारी प्रकट कर सकता है जद मनदिैज्ञायनक
सोंकट का कारण बन सकती है।
• िागि: िह एक महों गी प्रयक्रिा है , और पररिदजना कद बडी सोंख्या में व्यल्कक्तिदों
से आनुिोंयर्क डे टा एकत्र करने और उसका यिश्लेषण करने के यिए काफी
िन की आिश्यकता हदती है।
 हािाँयक, इसमें िागत काफी कम हद गई है , यफर भी, सोंपूर्ि िीन म
अनुक्रमर् (डब्ल्यूिीएस) के यिए िह अभी भी $600 के आसपास है ।

आगे की राह:
• प्रर्शर्क्षि िनशम्भक्त: जीन डे टा व्याख्या में कुर्ि यचयकत्सकदों की सोंख्या
बढाना और जीनदम अनुक्रमण के यिए प्रिदगर्ािाओों की उपिब्धता का
यिस्तार करना।
 र्ारिीय SARS-CoV-2 िीन र्मक्स कोंस र्टि यम (INSACOG) जैसी
प्रिदगर्ािाएँ बनाई जा सकती हैं , िीन र्मक्स कोंस र्टि यम ने भारत में
SARS-CoV-2 के जीनदयमक पररितननदों की यनगरानी करने का बहुत
सराहनीि काम यकिा र्ा।
• डे टा की सुरक्षा: जीनदयमक डे टा की सुरक्षा के यिए, इसे एकत्र करने, सोंग्रहीत
करने और उपिदग करने िािे सोंगठनदों कद डे टा उल्लोंघनदों कद कम करना
चायहए और सािनजयनक यिश्वास बनाए रखना चायहए।

83
• नीर्ि र्नमािर्: व्यापक और प्रभािी नीयत जद अनुसोंिान यिषिदों की गदपनीिता सुरक्षा कद प्रार्यमकता दे ती है ।
• र्हिधारक की र्ागीिारी: यहतिारकदों कद र्ायमि करने िािा एक पारदर्ी ढाोंचा जद जीनदयमक डे टा सोंग्रह के उद्े श्य कद स्पष्ट रूप से
पररभायषत करता है और इसके डे टाबेस भोंडारण की अियि कद यनयदन ष्ट करता है ।
• नैर्िक और कानूनी र्ििार ों क सोंब र्धि करना: जीनदम अनुक्रमण नैयतक और कानूनी प्रश्न उठाता है , इन यचोंताओों कद सोंबदयित करना
और व्यल्कक्तिदों के आनुिोंयर्क डे टा के यजम्मेदार उपिदग और सुरक्षा कद सुयनयित करने के यिए स्पष्ट यदर्ायनदे र् स्र्ायपत करना आिश्यक
है ।
• आनुिोंर्शक परामशि: जीनदम अनुक्रमण से गुजरने िािे व्यल्कक्तिदों कद आनुिोंयर्क परामर्न सेिाएँ प्रदान करने से उन्हें प्रयक्रिा के यनयहतार्न
कद समझने में मदद यमि सकती है ।

िीन म अनुक्रमर् की र्िशा में प्रयास:


• अोंिराििरीय स्तर:
 मानि िीन म पररय िना: िह एक सािििर्नक रूप से र्िि प र्षि अोंिररािरीय सहय गात्मक अनुसोंधान पररिदजना है यजसका उद्े श्य
मानि डीएनए बनाने िािे रासाियनक आिार जदडे के अनुक्रम कद यनिानररत करना और मानि जीनदम के सभी जीनदों की पहचान और मानयचत्रण
करना है ।
• र्ारि स्तर:
 स्विे शी कायिक्रम
✓ िैज्ञार्नक एिों औद्य र्गक अनुसोंधान पररषि (CSIR) ने 2019 में IndiGen कािनक्रम र्ुरू यकिा।
✓ दे र् भर से यिए गए 1029 स्व-घदयषत स्वस्र् भारतीिदों की सोंपूणन जीनदम अनुक्रमण पूरी हद चुकी है ।
 िीन म इों र्डया प्र िेर:
✓ उद्दे श्य: सोंदभन जीनदम का यनमानण करने के यिए पूरे भारत में 10,000 आनुिोंयर्क नमूने एकत्र करना।
✓ इस पररिदजना का नेतृत्व बेंगिुरु ल्कस्र्त भारतीि यिज्ञान सोंस्र्ान के मल्कस्तष्क अनुसोंिान केंि द्वारा यकिा गिा है ।

प्र. यचयकत्सा, कृयष और बािदइों जीयनिररों ग जैसे क्षेत्रदों में जीनदम अनुक्रमण के सोंभायित अनुप्रिदगदों और िाभदों पर चचान करें ।

6.2 िों द्र यान-3


सोंिर्ि:
इसरद र्ारि के श्ीहररक टा में सिीश धिन अोंिररक्ष केंद्र से LVM3 रॉकेट का उपिदग करके िोंद्रयान 3 अोंिररक्ष यान िॉन्च करने के
र्िए िैयार है।

िोंद्रयान 3 के बारे में:


● चोंििान-3 र्ारि का िीसरा िोंद्र र्मशन है और
सार् ही िोंद्रमा की सिह पर र ब र्टक िैंडर की
सॉफ्ट िैंर्डों ग कराने का िू सरा प्रयास है। अगर
सॉफ्ट-िैंयडों ग सफि रही तद भारत यह उपिम्भब्ध
हार्सि करने िािा िु र्नया का िौथा िे श बन
िाएगा।
● 2019 िोंद्रयान-2 र्मशन का अनुिती यमर्न है , जद
आों यर्क रूप से यिफि हद गिा र्ा क्दोंयक इसके
िैंडर और र िर चोंिमा पर सॉफ्ट-िैंयडों ग नहीों कर
सके।
िोंद्रयान-3 के घटक:
• चोंििान-3 में एक स्विे शी िैंडर मॉड्यूि (एिएम), प्र पल्शन मॉड्यूि (पीएम) और एक र िर शार्मि है।
 इसमें मुख्य कायि एिएम क िॉन्च िाहन इों िेक्शन से अोंर्िम िोंद्र 100 र्कमी ग िाकार ध्रुिीय कक्षा िक िे िाना और एिएम
कद पीएम से अिग करना है ।

84
 िैंडर में एक र्नर्िि ि िोंद्र स्थि पर सॉफ्ट िैंर्डों ग
करने और र िर क िैनाि करने की क्षमता
हदगी जद अपनी गयतर्ीिता के दौरान चोंि सतह
का इन-सीटू रासाियनक यिश्लेषण करे गा।
 िैंडर और र िर के पास चोंि सतह पर प्रिदग
करने के यिए िैज्ञायनक पेिदड हैं ।
• र िर पेि ड: िेजर प्रेररत िेकडाउन स्पेररदस्कदप
(एिआईबीएस), अल्फा पायटन कि एक्स-रे
स्पेररदमीटर (एपीएक्सएस)
• िैंडर पेि ड: रे यडिद एनाटॉमी ऑफ मून बाउों ड
हाइपरसेंयसयटि आिनदस्फीिर एों ड एटमॉल्कस्फिर
(RAMBHA), चोंिा सरफेस र्मोयफयजकि
एक्सपेररमेंट (ChaSTE), इों स्टूमेंट फॉर िूनर यसल्किक
एल्करयिटी (ILSA), िेजर रे टर दररफ्लेरर ऐरे (LRA)।
• ऑर्बिटर: ऑयबनटर पेि ब्लू डॉट पर जीिन के सोंकेतदों
कद दे खने के यिए पृथ्वी पर ध्यान केंयित करे गा तायक
िह ऐसे एक्सदप्लैनेट (सौर मोंडि से परे ग्रह) की खदज
में सहािता कर सके जद जीिन का समर्नन कर सकते
हैं ।
िोंद्रयान-3 का महत्व:
• िर्क्षर्ी ध्रुि के पास पहिी सॉफ्ट िैंर्डों ग: िह
चोंिमा के दयक्षणी ध्रुि के पास सॉफ्ट-िैंयडों ग करने
िािा दु यनिा का पहिा यमर्न बन जाएगा।
• गहन अोंिररक्ष अन्वेषर्: भयिष्य के आटे यमस
अोंतररक्ष िात्री इन क्षेत्रदों से मुख्य नमूने और िाष्पर्ीि
पदार्न एकत्र करने में सक्षम हदोंगे। इसका गहरे
अोंतररक्ष अन्वेषण और अोंततः व्यािसायिक
गयतयियििदों के भयिष्य पर गहरा प्रभाि पड सकता
है ।
 भारत अब आटे र्मस समझौिे का एक हस्ताक्षरकिाि है , जद िैज्ञार्नक िन्द्रमा के िर्क्षर्ी ध्रुि का अन्वेषर् क् ों करना िाहिे हैं?
चोंिमा की खदज के यिए दु यनिा की अन्य प्रमुख अोंतररक्ष • अज्ञाि क्षेि: अपने ऊबड-खाबड िातािरण के कारण, चोंिमा के ध्रुिीि
एजेंयसिदों के सार् एक समझौता है । क्षेत्र अज्ञात रहे हैं ।
• अोंिररक्ष क्षेि में सहय ग: चोंििान-3 भारत के अोंतरान ष्टरीि सहिदग • पानी की उपम्भस्थर्ि: इस क्षेत्र में गहरे गड्दों में पिानप्त मात्रा में बफन के
के स्तर कद बढाएगा, यिर्ेष रूप से यिज्ञान, प्रौद्यदयगकी, निाचार अणुओों की उपल्कस्र्यत के सोंकेत हैं - भारत के 2008 चोंििान -1 यमर्न ने
और स्टाटन अप में। िह अरबदों डॉिर की क्षमता िािे उभरते क्षेत्र अपने दद उपकरणदों की मदद से चोंि सतह पर पानी की उपल्कस्र्यत का
सोंकेत यदिा र्ा।
"चोंिमा अर्नव्यिस्र्ा" के यिए भारत का यटकट हदगा।
• सौर मोंडि का अध्ययन: िहाों के बेहद ठों डे तापमान का मतिब है यक
• गगनयान र्मशन: एििीएम-3 का उपिदग करके यमर्न की
इस क्षेत्र में फोंसी कदई भी चीज यबना ज्यादा बदिाि के, समि के सार् जमी
सफिता आगामी गगनिान यमर्न के यिए िॉन्च िाहन की
रहे गी। इसयिए चोंिमा के उिरी और दयक्षणी ध्रुिदों की चट्टानें और यमट्टी
यिश्वसनीिता कद बढाएगी। प्रारों यभक सौर मोंडि के बारे में सुराग प्रदान कर सकती हैं ।
• अोंिररक्ष क्षेि क बढ़ािा: यह िे श के अोंिररक्ष उद्य ग, अोंिररक्ष
स्टाटि अप और अोंिररक्ष उद्यर्मय ों के यिकास कद बढािा दे ने में महत्वपूणन भूयमका यनभाएगा, यजससे निाचार, तकनीकी उन्नयत और रदजगार
सृजन के अिसर पैदा हदोंगे।
• र्निी र्ागीिारी क बढ़ािा िे ना: िह यनिेर्कदों का यिश्वास बढा सकता है और अोंतररक्ष प्रौद्यदयगकी में अयिक यनजी यनिेर् आकयषनत कर
सकता है ।

85
िोंद्रयान-3 िोंद्रमा की सिह िक कैसे पहुोंिेगा? क्रैश िैंर्डों ग क र कना/उर्िि सॉफ्ट िैंर्डों ग सुर्नर्िि करने के र्िए िैंडर पर
• िॉन्च िाहन: यह यमर्न र्ारि के LVM3 रॉकेट (दे र् का भारी उन्नि प्रौद्य र्गकी उपकरर्:
यिफ्ट िाहन जद िगभग 8 मीयटर क टन कद कम-पृथ्वी की कक्षा • अल्टीमीटर: ऊोंचाई मापने के यिए;
में रखने में सक्षम है) पर िॉन्च के सार् र्ुरू हदता है । • िेि र्समीटर: िेग मापने के यिए, और खतरे का पता िगाने और बचाि के
• कक्षा प्लेसमेंट: LVM3 अोंतररक्ष िान और एक सोंिग्न प्रणददन यिए कैमरे ;
मॉड्यूि कद ग्रह से िगभग 36,500 यकिदमीटर ऊपर एक • िेिर डॉपिर िेग मीटर: िैंयडों ग से पहिे चोंि इिाके का बेहतर आकिन
अपदजी, िा उच्च यबोंदु के सार् एक यिस्ताररत पृथ्वी कक्षा में करने के यिए एक निा िेजर सेंसर जदडा गिा है।
स्र्ायपत करे गा।
• प्र पल्शन मॉड्यूि: िह मॉड्यूि िैंडर और रदिर कद चोंिमा तक िे जाएगा। मॉड्यूि स्विों चोंिमा पर नहीों उतरता है और इसके बजाि
चोंिमा के चारदों ओर 100 यकमी x 100 यकमी की पायकिंग कक्षा में स्र्ायपत हद जाता है ।
• िैंर्डों ग स्थि: चोंिमा के दयक्षणी ध्रुि के पास िगभग 70 यडग्री सेल्कल्सिस पर ल्कस्र्त स्र्ान का चिन इसयिए यकिा गिा क्दोंयक िहाों कई क्रेटर
हैं जद स्र्ािी रूप से छािा में रहते हैं , और पानी, बिि और कीमिी खर्नि ों का र्ोंडार ह सकिे हैं।
• िैंडर और र िर मॉड्यूि: चोंिमा पर उतरने के यिए िैंडर और रदिर प्रदप्र्न मॉड्यूि से अिग हद जाएों गे ।
• क्रैश िैंर्डों ग क र कना: िैंडर में ऐसे इों जन हैं जद िोंद्रमा की गर्ि क धीमा कर िें गे, तायक िह चोंिमा पर क्रैर्-िैंयडों ग के बजाि धीरे से
उिर सके।
• िैंडर और र िर का टिडाउन: टचडाउन के बाद, िैंडर िैंर्डों ग स्थि पर म्भस्थर रहेगा, जबयक रदिर, ि पर्हय ों िािा एक छ टा, टर ॉिी
प्रकार का उपकरर् है, चोंिमा का पता िगाएगा।
• गर्िर्िर्ध अिर्ध: िैंडर और रदिर पृथ्वी के 14 यदनदों तक जीयित रहेंगे, ि एक िोंद्र र्ििस के अनुरूप है । चूँयक िैंडर और रदिर कद
यबजिी प्रदान करने िािे सौर पैनिदों कद सूिन के प्रकार् की आिश्यकता हदती है , िे एक चोंि यदिस, िानी पृथ्वी के 14 यदनदों तक जीयित
रहें गे।
र्नष्कषि:
• यह र्ारि के िोंद्र अन्वेषर् कायिक्रम में एक बडा कदम है , और िह यनयित रूप से चोंिमा के बारे में हमारी समझ में महत्वपूणन िदगदान
दे गा। र्ािद अयिक महत्वपूणन बात िह है यक िह र्ारि की क्षमिाओों क प्रिर्शिि करे गा और िैर्िक अोंिररक्ष समुिाय में इसकी
प्रर्िष्ठा बनाएगा, यजससे दे र् के अोंतररक्ष उद्यदग में अयिक यनिेर् आकयषनत करने में मदद यमिेगी।

6.3 र्ारि 6G एिायों स


सोंिर्ि:
हाि ही में, दू रसोंचार यिभाग (DoT) ने अगिी पीढी की िािरिेस टे क्नदिॉजी (6G) में निाचार और सहिदग कद बढािा दे ने के यिए भारत 6G
एिािोंस िॉन्च यकिा।
पृष्ठर्ूर्म: 1G से 6G िक र्िकास:
पीढ़ी र्ििरर्

1G
• 1970 के िशक के अोंि में पेर् यकिा गिा, यह कम ध्वर्न गुर्ििा और किरे ज के सार् िॉिस कॉि की पेर्कर् करता र्ा। कदई रदयमोंग समर्नन िा
डे टा सेिाएँ उपिब्ध नहीों र्ीों।

• 1991 में पेश र्कया गया, इसने एनािॉग से यडयजटि यसग्नि में पररितनन कद यचयित यकिा।
2G
• रदयमोंग की अनुमयत दी गई, िगभग 50 केबीपीएस पर एसएमएस और एमएमएस जैसी छदटी डे टा सेिाएों प्रदान की गईों।

3G
• 2001 में पेश र्कया गया, िह मदबाइि फदन पर तेज़ डे टा टर ाोंसयमर्न, मदबाइि इों टरनेट एक्सेस, ईमेि, िीयडिद कॉयिोंग, िेब िाउयजोंग और सोंगीत
िेकर आिा।

4G
• इसे 2010 के आसपास पेर् यकिा गिा, िह 3जी की तुिना में कम यििोंबता के सार् उच्च गयत, उच्च गुणििा िािी आिाज और डे टा सेिाएों प्रदान
करता र्ा।

5G • इसे 2020 में पेर् यकिा गिा, िह कम यििोंबता, तेज़ डाउनिदड गयत और इों टरनेट ऑफ यर्ोंग्स (IoT) के यिए बढी हुई कनेल्करयिटी प्रदान करता है।

• इसका अभी भी यिकास चि रहा है, 6G का िक्ष्य उच्च गयत, र्ून्य अोंतराि सोंचार, यिस्ताररत िास्तयिकता (XR) और मदबाइि हदिदग्राम जैसी उन्नत
सेिाओों और यनबानि दू रस्र् सोंचािन के सार् 5G की क्षमताओों में सुिार करना है।
6G • इसका अपेयक्षत व्यािसािीकरण 2030 के आसपास हद सकता है।
• उपय ग के मामिे: ररमदट-यनिोंयत्रत कारखाने, िगातार सोंचार करने िािी स्व-चायित कारें और मानि इों यििदों से सीिे इनपुट िेने िािे िाटन पहनने
िदग्य उपकरण।

86
6G की क्ा िरुरि है?
• अर्र्सरर्: 6G का िक्ष्य यडयजटि, भौयतक और मानिीि क्षेत्रदों कद एक सार् िाना है , यजससे उनके बीच सहज एकीकरण और बातचीत
सोंभि हद सके।
• बुम्भद्धमान अर्र्व्यम्भक्त: अयिक पररष्कृत और बुल्कद्धमान एल्कप्लकेर्नदों कद सक्षम करने के यिए कृयत्रम बुल्कद्धमिा और मर्ीन ियनिंग पर
आिाररत समािानदों कद बढाने के यिए 6G आिश्यक हदगा।
• र्ििसनीय प्रौद्य र्गकी: 6G का िक्ष्य डे टा गदपनीिता और साइबर सुरक्षा से सोंबोंयित यचोंताओों कद दू र करते हुए अयिक सुरयक्षत और
यिश्वसनीि प्रौद्यदयगकी का यनमानण करना है।
• र्िर्ािन क पाटना: 6G यकफािती दरदों पर ग्रामीण क्षेत्रदों में सिनव्यापी कनेल्करयिटी प्रदान करे गा, यडयजटि यिभाजन कद पाटे गा और सोंचार
नेटिकन तक समािेर्ी पहुोंच सुयनयित करे गा।
र्ारि 6G अिायोंस (B6GA):
• B6GA एक सहय गी मोंि है र्िसमें सािििर्नक और र्निी कोंपर्नयाों , यर्क्षायिद, अनुसोंिान सोंस्र्ान और मानक यिकास सोंगठन र्ायमि
हैं ।
• उद्दे श्य:
 उच्च प्रभाि िािे खुिे अनुसोंिान और यिकास (आर एों ड डी) पहि कद बढािा दे ना।
 भारतीि दू रसोंचार प्रौद्यदयगकी उत्पाददों और सेिाओों के यिए बाजार पहुोंच कद सुयििाजनक बनाना।
 प्रौद्यदयगकी स्वायमत्व और स्वदे र्ी यियनमान ण कद बढािा दे ना।
 प्रौद्यदयगकी सह-निाचार की सोंस्कृयत बनाना।
 6G तकनीक के यनमान ण से बौल्कद्धक सोंपदा (आईपी) के यिकास कद बढािा दे ना।
र्ारि में र्डर्िटि पाररम्भस्थर्िकी िोंि की म्भस्थर्ि:

िू रसोंिार बाजार: ● 1.2 यबयििन यडयजटि ग्राहकदों के सार् भारत यिश्व स्तर पर दू सरा सबसे बडा दू रसोंचार बाजार है ।
● यपछिे नौ िषों में, भारत की यडयजटि अर्नव्यिस्र्ा राष्टरीि अर्नव्यिस्र्ा की तुिना में 2.5 गुना तेजी से
बढी है ।

इों टरनेट और िॉडबैंड ● िॉडबैंड उपिदगकतान 60 यमयििन से बढकर 800 यमयििन हद गए हैं , जबयक इों टरनेट कनेक्शन 250
र्िस्तार: यमयििन से बढकर 850 यमयििन हद गए हैं ।
● सरकार और यनजी क्षेत्र ददनदों ने कनेल्करयिटी बढाने के यिए 2.5 यमयििन यकमी से अयिक ऑयप्टकि
फाइबर यबछािा है ।

र्डर्िटि कनेम्भरर्िटी: ● भारत कद दु यनिा भर में सबसे अयिक जुडे हुए िदकतोंत्र के रूप में मान्यता प्राप्त है ।

सरकारी पहि:
• र्ारिनेट पररय िना: ऑयप्टकि फाइबर के माध्यम से 2.5 िाख ग्राम पोंिायि ों (िीपी) कद कनेल्करयिटी प्रदान करना। इसे भारत िॉडबैंड
नेटिकन यियमटे ड (BBNL) द्वारा कािान ल्कन्वत यकिा जाता है ।
• केरि िाइबर ऑर्प्टकि नेटिकि (KFON): इसके तहत, केरि सरकार गरीबी रे खा से नीचे (बीपीएि) 20 िाख पररिारदों कद मुफ्त में
इों टरनेट कनेक्शन प्रदान करती है ।
• िू रसोंिार प्रौद्य र्गकी र्िकास र्नर्ध (TTDF) य िना: िूएसओएफ से िायषनक सोंग्रह का 5% प्रौद्यदयगयकिदों, उत्पाददों और सेिाओों के
अनुसोंिान और यिकास के यििपदषण के यिए उपिब्ध हदगा।
• र्डर्िटि पम्भिक इों फ्रास्टर क्चर (DPI) िा इों यडिा स्टै क: आईटी एक यडयजटि प्लेटफॉमन सोंग्रह (आिार, यडयजटि िॉकर, यडजीिात्रा,
िूपीआई) है यजसे यियभन्न सोंस्र्ाओों के बीच सहिदग के माध्यम से यिकयसत यकिा गिा है ।

िू रसोंिार क्षेि के सामने िुनौर्ियााँ:


• र्ििीय सोंघषि: कोंपयनिदों के यिए यपछिे िषन की तुिना में िषन 2017-18 के यिए सकि राजस्व में 15% से 20% की यगरािट आई है और
समग्र क्षेत्र के राजस्व में यगरािट आई है ।
• स्पेररम की कमी: उपिब्ध स्पेररम िूरदपीि दे र्दों की तुिना में 40% और चीन की तुिना में 50% से कम है ।

87
• िीव्र प्रर्िस्पधाि और टै ररि युद्ध: ररिािोंस यजिद के प्रिेर् से दू रसोंचार ऑपरे टरदों के बीच तीव्र प्रयतस्पिान बढ गई है , यजससे टै ररफ िुद्ध और
िाभ मायजनन कम हद गिा है ।
 इस यििीि तनाि के कारण इस क्षेत्र में और यनिेर् में दे री हुई है ।
• र्हरी-ग्रामीण यिभाजन: भारत में, पिान प्त टे िी घनत्व हायसि कर यििा गिा है , िेयकन शहरी (55.42%) और ग्रामीर् (44.58%) क्षेि ों
में पहुोंि के बीि एक बड़ी र्िसोंगर्ि है।
• नई प्रौद्य र्गर्कय ों से रािस्व में कमी: व्हाट् सएप, ओिा और उबर जैसे यियभन्न एल्कप्लकेर्नदों के उद्भि से दू रसोंचार क्षेत्र के यिए राजस्व में
कमी आई है ।
• उच्च िाइसेंस शुल्क: िूयनिसनि सयिनस िेिी (िूएसएि) सयहत िाइसेंस र्ुि, समािदयजत सकि राजस्व का आठ प्रयतर्त है , जद यिश्व में
सबसे अयिक अयिक है ।

आगे की राह:
• रािरीय र्डर्िटि सोंिार नीर्ि (एनडीसीपी) िागू करना: िह एक राष्टरीि यडयजटि यग्रड की स्र्ापना, रास्ते के अयिकारदों के यिए सहिदगी
तोंत्र और अनुमददन बािाओों कद हटाने पर जदर दे ती है ।
 राज्य सरकारदों कद एनडीसीपी के उद्े श्यदों का समर्नन करने के यिए पिान प्त दू रसोंचार बुयनिादी ढाों चे की स्र्ापना में सयक्रि रूप से भाग
िेना चायहए।
• ग्रामीर् कनेम्भरर्िटी में सुधार: यडयजटि यिभाजन कद पाटने के यिए ग्रामीण क्षेत्रदों में ऑयप्टकि फाइबर और यफक्स्ड-िाइन की पहुोंच
बढाना।
• व्यिसाय करने में आसानी: दू रसोंचार क्षेत्र में नई कोंपयनिदों के प्रिेर् और यनकास के यिए अनुकूि माहौि बनाना।
 आईटी प्रयतस्पिान , निाचार और यनिेर् कद प्रदत्सायहत करे गा, यजससे उपभदक्ताओों के यिए सेिाओों और सामथ्न में सुिार हदगा।
• दू रसोंचार ऑपरे टरदों पर यििीि बदझ कम करने के यिए िाइसेंस र्ुि कम करना।
• ऑपरे टरदों कद अपने पूोंजीगत व्यि (कैपेक्स) कद अनुकूयित करने की अनुमयत दे ने के यिए टे िीकॉम इों फ्रास्टर क्चर कद साझा करना, एक
महत्वपूणन यहस्से के रूप में, िगभग 40% से 60%, बुयनिादी ढाोंचे की स्र्ापना और प्रबोंिन के यिए उपिदग यकिा जाता है ।

5G और 6G के बीि अोंिर:
पहिू 5G 6G

स्पीड 10 जीबीपीएस तक 1 टीबीपीएस तक

िेटेंसी कम यििोंबता (10 एमएस) अल्टर ा-िद यििोंबता (<1 एमएस)

स्पेररम (प्रमुख 95 GHz से 3 THz (Terahartz) 5G की तुिना में उच्च आिृयि पर काम कर
24 गीगाहट्न ज़ से 100 गीगाहट्न ज़
आिृर्ि बैंड) सकता है

क्लाउड गेयमोंग, एिी/िीआर


उच्च-प्रदर्नन कोंप्यूयटों ग, ररमदट-यनिोंयत्रत कारखानदों, स्व-चायित कारदों, िाटन
अनुप्रय ग तकनीक, इों टरनेट ऑफ यर्ोंग्स
पहनने िदग्य िस्तुओों का समर्नन करे गा।
आयद कद अपनाने में तेजी िािा।

कनेम्भरर्िटी अरबदों उपकरणदों कद जदडना आभासी और भौयतक दु यनिा के बीच कनेल्करयिटी

6.4 ने श नि ररसिि िाउों डे शन र्िधे य क, 2023

सोंिर्ि:
हाि ही में, केंिीि मोंयत्रमोंडि ने नेशनि ररसिि िाउों डेशन (एनआरएि) र्िधेयक क मोंिूरी िे िी।
र्िधेयक की मुि र्िशेषिाएों :
• रािरीय अनुसोंधान िाउों डेशन: िह यििेिक दे र् में यियभन्न अनुसोंिान और यिकास (आरएों डडी) गयतयियििदों की दे खरे ख के यिए एनआरएफ
कद एक र्ीषन यनकाि के रूप में स्र्ायपत करे गा।

88
• र्िज्ञान और इों िीर्नयररों ग अनुसोंधान ब डि (एसईआरबी) क र्नरस्त करना: 2008 में सोंसद के एक अयियनिम द्वारा बनाए गए
एसईआरबी कद यनरस्त कर यदिा जाएगा और एनआरएफ में र्ायमि कर यदिा जाएगा।
नेशनि ररसिि िाउों डेशन (एनआरएि) के बारे में:
• एनआरएफ, रािरीय र्शक्षा नीर्ि (एनईपी), 2023
की र्सिाररश ों के अनुरूप भारत में िैज्ञार्नक
अनुसोंधान क उच्च स्तरीय रर्नीर्िक र्िशा
प्रिान करने िािा एक शीषि र्नकाय ह गा।
• प्रशासर्नक र्िर्ाग: यिज्ञान और प्रौद्यदयगकी
यिभाग (डीएसटी)।
• गिर्निंग ब डि (16 सिस्य), यजसमें पदे न अध्यक्ष
प्रिानमोंत्री हदते हैं।
• अनुमार्नि बिटीय पररव्यय: 50,000 करदड
• समयािर्ध: 2023 - 2028.
• एनआरएि के उद्दे श्य:
 रािरीय प्राथर्मकिाओों क सोंब र्धि करना:
स्वि ऊजान, यटकाऊ बुयनिादी ढाों चे, स्वास्थ्य
दे खभाि आयद सयहत राष्टरीि उद्े श्यदों में िदगदान
करने के यिए एस एों ड टी हस्तक्षेप के यिए
प्रार्यमकता िािे क्षेत्रदों की पहचान करना।
 अनुसोंधान क बढ़ािा िे ना: एनआरएफ
रचनात्मकता और आिदचनात्मक सदच कद
बढािा दे ते हुए मानयिकी, सामायजक यिज्ञान
और किा में अनुसोंिान कद यिि पदयषत और
बढािा दे गा।
र्िधेयक की आिश्यकिा:
• अनुसोंधान क सक्षम करने के र्िए: मौजूदा
कानूनदों ने र्निी अनुसोंधान सोंगठन ों के र्िए
एनआरएफ जैसे फोंयडों ग यनकाि में िदगदान करना
कयठन बना यदिा है ।
• खोंर्डि अनुसोंधान एिों र्िकास पाररम्भस्थर्िकी
िोंि: भारत में सरकार, सािनजयनक सोंस्र्ानदों और
यनजी सोंगठनदों के बीच तािमेि और समन्वि का
अभाि है ।
 इसके पररणामस्वरूप ददहरे प्रिास, सोंसािनदों
का अकुर्ि उपिदग और अोंतः यिषि अनुसोंिान
के अिसर चूक जाते हैं।
• अत्यर्धक नौकरशाही: भारत में यर्क्षा अत्ययिक
केंिीकृत है , यजसके पररणामस्वरूप अत्ययिक
सरकारी हस्तक्षेप हदता है जद िैज्ञायनक प्रगयत में बािा डािता है और हतदत्सायहत करता है ।
• कम श धकिाि िीव्रिा: चीन की तुिना में भारत में र्दिकतानओों की सोंख्या कम है (प्रर्ि र्मर्ियन 253 श धकिाि र्ारि में बनाम िीन
में 1,225)। इससे पेटेंट फाइयिोंग, िैज्ञायनक पयत्रकाओों में िदगदान और औद्यदयगक टर े डमाकन पर नकारात्मक प्रभाि पडता है ।
• पुराने पाठ्यक्रम और र्शक्षर् र्िर्धयाों भारतीि छात्रदों के बीच एक निीन मानयसकता कद बढािा दे ने में यिफि हैं क्दोंयक केिि 2.7%
कॉिेज ही पीएचडी करिाते हैं ।

89
• अनुसोंधान एिों र्िकास में सामार्िक-साोंस्कृर्िक िुनौर्ियााँ:
 प्रर्िर्ा पिायन: कुर्ि व्यल्कक्त सीयमत अिसरदों और अन्यत्र बेहतर जीिन ल्कस्र्यतिदों के कारण भारत छदड दे ते हैं ।
 समािेर्शिा का अर्ाि: सामायजक-साोंस्कृयतक बािाएँ , यिर्ेष रूप से मयहिाओों के यिए, अनुसोंिान एिों यिकास गयतयियििदों में भागीदारी
कद प्रयतबोंयित करती हैं। उदाहरण के यिए, 2018 में भारत में मयहिा र्दिकतान ओों की यहस्सेदारी केिि 18.7% थी।
• कम िोंर्डों ग: प्रमुख अर्नव्यिस्र्ाओों की तुिना में भारत का अनुसोंिान और यिकास (आरएों डडी) व्यि-जीडीपी अनुपात 0.7% बहुत कम है
और यिश्व औसत 1.8% से काफी नीचे है ।
 इसके अिािा आईआईटी और आईआईएससी जैसे सोंस्र्ानदों कद बडी मात्रा में ररसचन फोंड यमिता है , िेयकन राज्य यिश्वयिद्याििदों कद
बहुत कम, िगभग 10% ररसचन फोंड यमिता है ।

एनआरएि के िायिे और नुकसान:


नुकसान िायिा

• केंद्रीकृि शम्भक्त और निािार: एनआरएफ कुछ • िैज्ञायनक अनुसोंिान के यिए केंद्रीकृि र्िि प षर् िोंि, रािरीय
व्यल्कक्तिदों िा सोंस्र्ानदों में र्ल्कक्त केंयित कर सकता है , प्राथर्मकिाओों के साथ सोंरेखर् सुर्नर्िि करना और सभी स्र्ानदों और
सीयमत दृयष्टकदण के कारण निाचार और रचनात्मकता कद सोंस्र्ानदों के र्दिकतान ओों कद यिि पदषण सहािता के यिए प्रयतस्पिान करने
दबा सकता है । की अनुमयत दे ना।
• नौकरशाही िे री: इसके पररणामस्वरूप नौकरर्ाही • उद्य ग-अकािर्मक सहय ग: एनआरएफ अर्नव्यिस्र्ा के यिए
यनणनि िेने में दे री हद सकती है , यजससे अनुसोंिान प्रगयत अनुसोंिान प्रासोंयगकता सुयनयित करे गा और िायणल्कज्यक उत्पाददों और
और पररिदजना कािान न्विन में दे री हद सकती है । सेिाओों में यनष्कषों के अनुिाद की सुयििा प्रदान करे गा।
• भ्रिािार: एनआरएफ की भ्रष्टाचार के प्रयत सोंिेदनर्ीिता • सामार्िक िुनौर्िय ों क सोंब र्धि करना: एनआरएफ का उद्े श्य
के कारण िन का गित आिोंटन हद सकता है और सोंसािनदों नागररकदों के जीिन कद बेहतर बनाना और भारत की समग्र समृल्कद्ध में
कद अनुसोंिान से दू र यकिा जा सकता है । िदगदान दे ना है ।

अनुसोंधान एिों र्िकास क बढ़ािा िे ने का महत्व: सरकार द्वारा की गई पहि:


• उत्पािकिा और आर्थिक र्िकास क बढ़ािा िे ना: अनुसोंिान • र्िर्िर्टों ग एडिाों स्ड ज्वाइों ट ररसिि (िज्र) िैकल्टी य िना: अयनिासी
एिों यिकास नई प्रौद्यदयगयकिदों कद यिकयसत कर सकता है और भारतीिदों (एनआरआई) और प्रिासी भारतीि नागररकदों (ओसीआई) सयहत
मौजूदा प्रयक्रिाओों में सुिार कर सकता है, यजससे सोंसािन दक्षता यिदे र्ी िैज्ञायनकदों और यर्क्षायिददों कद भारत िाना।
में िृल्कद्ध हदगी और समग्र आयर्नक यिकास हदगा। • प्रर्ािशािी अनुसोंधान, निािार और प्रौद्य र्गकी:

• कम िागि िािे स्विे शी समाधान ों क बढ़ािा िे ना: अनुसोंिान  आईआईटी और आईआईएससी के बीच सहिदग के रूप में 2015 में
िॉन्च यकिा गिा।
एिों यिकास ियपुर िुट िैसे अनुरूप समाधान ों की अनुमर्ि
 उद्दे श्य: 10 चियनत प्रौद्यदयगकी डदमेन में इों जीयनिररों ग चुनौयतिदों का
िे िा है।
समािान प्रदान करना।
• सीखने के पररर्ाम ों क बढ़ाना: उच्च यर्क्षा सोंस्र्ानदों में एक
• अटि र्टों कररों ग िैब्स:
मजबूत अनुसोंिान सोंस्कृयत बेहतर यर्क्षण और सीखने के अनुभि
 यह नीर्ि आय ग के अटि इन िे शन र्मशन की पहि है ।
में िदगदान करती है । उदाहरण के यिए, हािनडन, स्टै नफदडन , • उद्दे श्य: िुिाओों में रचनात्मकता, यजज्ञासा और यडजाइन मानयसकता और
ऑक्सफदडन । कम्प्प्यूटेर्नि सदच जैसे कौर्ि कद बढािा दे ना।
• आयाि और रािक षीय घाटे क कम करना: अनुसोंिान एिों
यिकास स्वदे र्ी प्रौद्यदयगयकिदों कद यिकयसत करने में मदद करता है , जद आत्मयनभनरता प्राप्त करने और घरे िू उद्यदगदों कद बढािा दे ने में मदद
करता है ।
• िििायु पररिििन से मुकाबिा: कम िागत िािे सौर पैनिदों, इिेल्कररक िाहनदों (ईिी) और यियर्िम बैटरी में अनुसोंिान एिों यिकास से
ग्रीनहाउस गैस उत्सजनन कद कम करने और जििािु पररितनन से यनपटने में मदद यमिेगी।
आगे की राह:
• अनुसोंधान और र्िकास सोंबोंधी िोंर्डों ग क बढ़ािा िे ना: 2030 तक अपने अनुसोंधान और र्िकास यनिेर् कद सकि घरे िू उत्पाद के
न्यूनतम 2% तक बढाएों ।
 सीएसआईआर और डीएसटी जैसी एजेंयसिदों द्वारा स्वािि यिश्वयिद्याििदों और सोंस्र्ानदों कद प्रदान यकए जाने िािे अनुदान में कमी कद दू र
करने के यिए एनआरएफ की 50,000 करदड रुपिे की प्रयतबद्ध रायर् का उपिदग करें ।

90
• सरकारी-उद्य ग-अकािर्मक साझेिारी क बढ़ािा िे ना: सोंिुक्त अनुसोंिान एिों यिकास पररिदजनाओों के माध्यम से सािनजयनक सोंस्र्ानदों,
स्टाटन -अप और यनजी क्षेत्र के बीच सहिदग, उनकी यिर्ेषज्ञता और सोंसािनदों का िाभ उठाना।
 उिाहरर्: क िैम्भक्सन के र्िकास और उन्हें हररत प्रौद्यदयगयकिदों, रक्षा और इिेररॉयनक्स यियनमान ण जैसे अन्य क्षेत्रदों में िागू करने जैसे
सफि सहिदग।
• अनुसोंधान क्षमिा बढ़ाना: भारत की सॉफ्ट पािर का िाभ उठाते हुए दु यनिा भर की प्रयसद्ध प्रिदगर्ािाओों में भारतीि छात्रदों और िैज्ञायनकदों
के यिए प्रयर्क्षण के अिसरदों कद सुयििाजनक बनाना।
 भारत में पदस्ट-डॉररि अनुसोंिान कािन के यिए िुिा िैज्ञायनकदों कद आकयषनत करने के यिए बेहतर पाररश्रयमक और प्रदत्साहन प्रदान
करें ।
• आईपीआर अनुपािन सुर्नर्िि करना: यनिेर्कदों का यिश्वास बनाने और अनुसोंिान एिों यिकास में अयिक यनिेर् आकयषनत करने के यिए
राष्टरीि बौल्कद्धक सोंपदा अयिकार (आईपीआर) नीयत 2016 का सख्ती से पािन यकिा जाए।

6.5 र्क्रप्ट करें सी


सोंिर्ि:
हाि ही में गुरुग्राम में, अपराध और सुरक्षा पर G20 सम्मेिन हुआ, यजसमें र्क्रप्ट करें सी और डाकिनेट के युग में र्िर्ध और व्यिस्था में
सुधार के र्िए महत्वपूर्ि प्रगर्ि करने के िरीक ों पर र्ििार-र्िमशि र्कया गया।
अपराध और सुरक्षा पर G20 सम्मेिन के बारे में:
• िह यर्खर सम्मेिन G-20 प्रेसीडें सी
की एक नई और अनूठी पहि है । G-
20 में साइबर सुरक्षा पर िह पहिा
सम्मेिन है ।
• उद्दे श्य: इसका उद्े श्य साइबर
अपराध से उत्पन्न िुनौर्िय ों का
समाधान करना है और एनएिटी,
एआई और मेटािसि जैसी नई
प्रौद्यदयगयकिदों के िुग में सुरक्षा।
• सम्मेिन की मुि झिर्कयााँ:
इों टरनेट गिनेंस - र्डर्िटि
पम्भिक इों फ्रास्टर क्चर, र्िस्ताररि
िास्तर्िकिा, मेटािसि,
आर्टि र्िर्शयि इों टेर्ििेंस, डाकि
नेट और र्क्रप्ट करें सी िुनौर्ियाों, और सूिना और सोंिार प्रौद्य र्गर्कय ों के आपरार्धक उपय ग सर्हि यियभन्न यिषिदों पर चचान की
गई।
र्क्रप्ट करें सी क्ा है?
• यक्रप्टदकरें सी एक बैंक-स्विोंि र्डर्िटि मुद्रा है ि र्कसी िीसरे पक्ष के हस्तक्षेप या सौिे की र्नगरानी करने िािे केंद्रीय प्रार्धकरर्
के र्बना यडयजटि बहीखाता में िेनदे न कद ररकॉडन और सत्यायपत करने के यिए िॉकिेन नामक र्िकेन्द्रीकृि िकनीक का उपय ग
करिी है।
• सुरर्क्षि और गुमनाम र्क्रप्ट िेनिे न: िेनदे न यक्रप्टदग्राफी के उपिदग से यकिा जाता है , जद सूचना और सोंचार कद सुरयक्षत करने के यिए
कदड का उपिदग करता है , और िेनदे न की सुरक्षा और गुमनामी सुयनयित करने के यिए पैसे भेजने और प्राप्त करने के यिए एक िचुनअि
िॉिेट का उपिदग करता है ।
• यक्रप्टद करें सी के कुछ उदाहरण र्बटकॉइन, एथेरम, ररपि और िाइटकॉइन हैं।

र्क्रप्ट करें सी के िार्:


• र्िकेंद्रीकरर्: इसका स्वार्मत्व र्कसी एक र्ििीय या सरकारी इकाई के पास नही ों है। यह धन के एकार्धकार क खत्म करिा है
और सुर्नर्िि करिा है र्क र्क्रप्ट करें सी का मूल् केंिीि बैंक िा प्रायिकरण द्वारा ति नहीों यकिा जाता है।

91
• कम िेनिे न शुल्क: यक्रप्टदकरें सी में िेनदे न का र्ुि बहुत मामूिी और कभी-कभी र्ून्य हदता है । ऐसा इसयिए है क्दोंयक इस प्रयक्रिा में
VISA और PayPal िैसे िीसरे पक्ष और मध्यस्थ समाप्त ह िािे हैं।
• उच्च िक्षिा: यक्रप्टदकरें सी का उपिदग करके सीमा पार िेनदे न कद यिदे र्ी मुिा प्रयक्रिाओों कद चुनौती यदए यबना तेज यकिा जाता है , यजससे
उनकी दक्षता बढती है और उनकी िागत कम हदती है ।
• पहुोंि क्षमिा: यक्रप्टदकरें सी यििीि सेिाओों की पहुोंच कद बढाती है क्दोंयक िे यिकेंिीकृत नेटिकन पर काम करती हैं और इों टरनेट कनेक्शन
और यक्रप्टद िॉिेट िािा कदई भी व्यल्कक्त उन तक पहुोंच सकता है ।
• पारिर्शििा: जबयक यक्रप्टदकरें सी िेनदे न गुमनाम हैं , यबटकॉइन और एर्ेररिम ब्लॉकचेन जैसे सािनजयनक ब्लॉकचेन िहीखाता पर दजन डे टा
सािनजयनक रूप से यकसी के भी दे खने के यिए उपिब्ध है।
• मुद्रास्फीर्ि से सुरक्षा: अपनी सीयमत आपूयतन के कारण, यक्रप्टदकरें सी कद अक्सर मुिास्फीयत से बचाने के एक तरीके के रूप में दे खा जाता
है । यक्रप्टदकरें सी मुिास्फीयत के ल्कखिाफ सुरक्षा के रूप में कािन करते हुए आपूयतन कद सीयमत करने के तोंत्र का उपिदग करती है।
र्क्रप्ट करें सी के उपय ग से िुड़ी िुनौर्ियााँ: र्ारि में र्क्रप्ट करें सी र्िर्नयमन:
• अम्भस्थरिा: यक्रप्टदकरें सी की कीमतदों में अक्सर उतार-चढाि हद • र्ुगिान र्िर्नयमन: भारत में भुगतान माध्यम के रूप में यक्रप्टदकरें सी
सकता है। जबयक इस अल्कस्र्रता से त्वररत िाभ हद सकता है , िह कद यकसी केंिीि प्रायिकरण द्वारा यियनियमत नहीों यकिा जाता है ।
यनिेर्कदों के यिए महत्वपूणन यििीि नुकसान भी पैदा कर सकता • र्क्रप्ट टै क्स: यििीि िषन 2022-23 के केंिीि बजट में, सरकार ने
है । उदाहरण के यिए, हाि ही में 2021 में यबटकॉइन का मूल्य आभासी मुिा िा यक्रप्टदकरें सी पररसोंपयििदों के हस्ताोंतरण पर 30% कर
30% यगर गिा। कटौती िागू करने की अपनी िदजना की घदषणा की।
• र्िर्नयमन का अर्ाि: यक्रप्टदकरें सी बाजार में यनिोंत्रण और • उच्चिम न्यायािय का सकिु िर: इस सकुनिर ने आरबीआई द्वारा
यियनिमन की अनुपल्कस्र्यत से यक्रप्टदक्ूरेंसी घदटािे और बाजार में यक्रप्टदकरें सी पर िगाए गए प्रयतबोंि कद हटा यदिा, यजसने बैंकदों और
यििीि सोंस्र्ानदों कद यक्रप्टद पररसोंपयििदों में िेनदे न में िगे िदगदों कद बैंयकोंग
हे रफेर का खतरा बढ जाता है ।
सेिाओों तक पहुों च प्रदान करने से प्रयतबोंयित कर यदिा र्ा।
• र्छपी हुई समानाोंिर अथिव्यिस्था: यक्रप्टदकरें सी एक पूणन
यिकयसत डाकिनेट में र्िकर्सि ह गई है, ि एक यछपी हुई समानाोंतर ऑनिाइन दु यनिा में डे टा कद रूट और एल्कन्क्रप्ट कर रही है , जद
मानक िाउज़रदों के माध्यम से पहुों च िदग्य नहीों है और खदज इों जन द्वारा अनुक्रयमत नहीों है।
• अिैध गर्िर्िर्धयााँ: डर ग्स, आग्नेिास्त्र और गदिा-बारूद, है यकोंग टू ि और सेिाओों जैसी अिैि पेर्कर्दों के यिए बाज़ारदों का यिकास। सोंिुक्त
राष्टर की एक ररपदटन में कहा गिा है यक िगभग 20% गोंभीर आपरायिक हमिदों में अब यक्रप्टदकरें सी फोंयडों ग, यक्रप्टद टदकन र्ायमि हैं ।
• सुरक्षा ि म्भखम: यक्रप्टदकरें सी पाररल्कस्र्यतकी तोंत्र में सुरक्षा चूक और है यकोंग के प्रिास बढ रहे हैं । उदाहरण के यिए, र्बनेंस िॉकिेन
नेटिकि, र र्नन नेटिकि आर्ि पर हमिे ।
• उत्पािन की अत्यर्धक िागि: इसके प्रकार के आिार पर, यक्रप्टदकरें सी के यिए यबजिी और अन्य सोंसािनदों की बडी आपूयतन की
आिश्यकता हद सकती है यजसके पररणामस्वरूप स्र्ानीि प्रदू षण, र्दर और अन्य पररणाम हद सकते हैं । उदाहरण के यिए, स्र्िदों के पास
रहने िािे समुदािदों के यिए ग्रीनहाउस गैस उत्सजनन में िृल्कद्ध।
• र्क्रप्ट करें सी और डाकिनेट का अर्र्सरर्: यक्रप्टदकरें सी और डाकननेट ने बडे पैमाने पर यक्रप्टदकरें सी चदरी िा सोंचािन की जाोंच करना
कयठन बना यदिा है ।

आगे की राह:
• सीमाओों के पार सोंयुक्त सहय ग: साइबर खिरे के पररदृश्य की प्रकृयत राष्टरीि सीमाओों के पार फैि गई है , यजससे राष्टरदों, सोंगठनदों और
अन्य यहतिारकदों द्वारा िास्तयिक समि में सहिदग और जानकारी साझा करना आिश्यक हद गिा है ।
• उभरती प्रौद्यदयगयकिदों के कारण उभरते खतरदों से यनपटने के यिए कोंप्यूटर आपातकािीन प्रयतयक्रिा टीमदों (सीईआरटी) कद मजबूत यकिा
जाना चायहए।
 प्रभािी 'र्र्िष्यिार्ी-र्निारक-सुरक्षात्मक और पुनप्रािम्भप्त' कारन िाई के यिए 24x7 साइबर सुरक्षा िोंि हदना चायहए।
• साइबर अपराध के प्रर्ि सोंिेिनशीििा और र्क्रप्ट करें सी की र्ागीिारी, साइटर े न जैसी पहि, जद एक खुिा ऑनिाइन पाठ्यक्रम है,
कद यक्रप्टद चदरी और डाकन नेट के बारे में ज्ञान कद और एकीकृत करना चायहए।
• डाकिनेट ड मेन र्िशेषज्ञिा: डाकननेट नेटिकन, सेिाओों, यक्रप्टदकरें सी जाोंच और सूचना एकत्र करने के यिए नीयत समर्नन और पररचािन
यिर्ेषज्ञता की आिश्यकता हदती है ।
• अोंिराििरीय ढााँिा: इस सम्मेिन ने इों टरनेट गिनेंस, यडयजटि सािनजयनक सामान और यडयजटि सािनजयनक बुयनिादी ढाोंचे की सुरक्षा, यडयजटि
स्वायमत्व से सोंबोंयित कानूनी और यनिामक मुद्े, एआई का यजम्मेदार उपिदग और डाकन नेट के यिए अोंतरान ष्टरीि सहिदग ढाँचे के यिए एक
सुरयक्षत और कुर्ि अोंतरान ष्टरीि ढाँ चे कद बढािा दे ने की आिश्यकता पर प्रकार् डािा।
92
6.6 िािि र्ाषा मॉडि-िै ट िीपीटी
सोंिर्ि:
ितनमान में, ChatGPT और अन्य समान "जेनरे यटि कृयत्रम बुल्कद्धमिा ' (gAI) जैसे बाडन , यचनयचिा, PaLM और LaMDA पर महत्वपूणन ध्यान
यदिा जा रहा है ।
िेनरे र्टि एआई क्ा है?
• जेनरे यटि एआई एक व्यापक र्ब्द है जद कृयत्रम बुल्कद्धमिा मॉडि कद सोंदयभनत करता है यजसमें कोंटें ट उत्पन्न करने की क्षमिा ह िी है।
जेनरे यटि एआई टे क्स्ट, कदड, यचत्र, िीयडिद और सोंगीत उत्पन्न कर सकता है । जेनरे यटि एआई के उदाहरणदों में र्मडिनी, डीएएिएि-ई
और िैटिीपीटी शार्मि हैं।

िैटिीपीटी के बारे में:


• चैटजीपीटी एक बड़े र्ाषा मॉडि (एिएिएम) का एक उिाहरर् है जद एक प्रकार का टर ाोंसिामिर-आधाररि न्यूरि नेटिकि है जद
र्ब्ददों के अनुक्रम में अगिे र्ब्द की भयिष्यिाणी करने में बहुत अिा है । चैटजीपीटी जेनरे यटि टे क्स्ट एआई का एक िदकयप्रि उदाहरण
है ।

एक बड़ा र्ाषा मॉडि क्ा है?


• एक बडा भाषा मॉडि (एिएिएम) एक प्रकार का कृयत्रम बुल्कद्धमिा
मॉडि है यजसे बडी मात्रा में यिल्कखत मानि भाषा और पाठ्य डे टा कद
पहिानने, उत्पन्न करने, अनुिाि करने और/या साराोंर्शि करने के
यिए गहन र्शक्षर् एल्ग ररिम के माध्यम से प्रर्शर्क्षि र्कया गया
है।
• िाउों डेशन मॉडि: बडे भाषा मॉडि (एिएिएम) िाउों डेशन मॉडि
हैं जद प्राकृर्िक र्ाषा प्रसोंस्करर् (एनएिपी) और प्राकृर्िक र्ाषा
र्नमािर् (एनएििी) कािों में गहन यर्क्षण का उपिदग करते हैं । उन्हें
भाषा की जयटिता और जुडाि सीखने में मदद करने के उद्े श्य से, बडे
भाषा मॉडि कद बडी मात्रा में डे टा पर पूिन-प्रयर्यक्षत यकिा जाता है ।
बड़े र्ाषा मॉडि का महत्व:
• मानि िैसा कोंटें ट बनाना: बडे भाषा मॉडि, यजन्हें सैकडदों अरब
मापदों डदों के सार् इों टरनेट-स्केि डे टासेट पर प्रयर्यक्षत यकिा जाता है , ने अब मानि िैसा कोंटें ट उत्पन्न करने के र्िए एआई मॉडि की
क्षमिा क अनिॉक कर र्िया है।
र्ारि में बड़े र्ाषा मॉडि:
• मानि रिनात्मकिा क बढ़ाना और उत्पािकिा में सुधार
• र्ार्षर्ी: िह अनुसूयचत भारतीि भाषाओों में बडे डे टासेट बनाने के यिए
करना: िह मॉडि यिश्वसनीि तरीके से पढ, यिख, कदड, यचत्र
भारत का AI आिाररत भाषा अनुिाद मोंच है ।
बना और बना सकते हैं और मानि रिनात्मकिा क बढ़ा
• इों र्डक टर ाोंसर्िटरे ट: इों यडकएक्सयिट रदमन से मूि यियप रूपाोंतरण और
सकिे हैं और दु यनिा की सबसे कयठन समस्याओों कद हि इसके यिपरीत के यिए एक टर ाोंसफामनर-आिाररत बहुभाषी यिप्योंतरण मॉडि
करने के यिए उद्यदगदों में उत्पादकता में सुिार कर सकते हैं। है ।
• र्ाषा अनुिाि: भाषाओों के बीच पाठ का अनुिाद करने के • इों र्डक नेिुरि िैंग्वेि िनरे शन: इों यडकबाटन एक बहुभाषी, अनुक्रम-से-
यिए बडे भाषा मॉडि का उपिदग यकिा जा सकता है । इससे अनुक्रम पूिन-प्रयर्यक्षत मॉडि है यजसका उपिदग भारतीि भाषाओों के यिए
अोंतर-साोंस्कृयतक सोंचार कद सुयििाजनक बनाना और भाषा प्राकृयतक भाषा यनमानण एल्कप्लकेर्नदों के यनमानण के यिए यकिा जा सकता है ।
सोंबोंिी बािाओों कद तदडना आसान हद सकता है ।
• बढ़ी हुई िक्षिा: एिएिएम मानि भाषा कद समझ सकते हैं , यजससे िे नीरस िा श्रम-गहन कािों कद पूरा करने के यिए आदर्न बन जाते हैं ।
• सोंकेि: िह एक एआई प्रणािी का यनमान ण कर सकता है जद यबना यकसी अयतररक्त प्रदग्रायमोंग के सरि सोंकेतदों के आिार पर मानि-स्तरीि
पाठ उत्पन्न करने में सक्षम है , यजससे केिि बुयनिादी इनपुट मापदों डदों का उपिदग करके सोंपूणन िेख, िहाों तक यक यकताबें बनाना सोंभि हद
जाता है ।

बड़े र्ाषा मॉडि से िुड़ी िुनौर्ियााँ:


• र्िशाि बुर्नयािी ढााँिा िागि: एिएिएम कद बनाए रखने और यिकयसत करने के यिए महत्वपूणन पूोंजी यनिेर्, तकनीकी यिर्ेषज्ञता और
बडे पैमाने पर बुयनिादी ढाँचा आिश्यक है ।

93
• आिश्यक डे टा का पैमाना: कोंपयनिाों अपने बडे भाषा मॉडि कद प्रयर्यक्षत करने के यिए पिान प्त बडे डे टासेट तक पहुोंच पाने के यिए सोंघषन
करती हैं ।
• िकनीकी र्िशेषज्ञिा: उनके पैमाने के कारण, बडे भाषा मॉडि कद प्रयर्यक्षत करना और तैनात करना बहुत कयठन है और इसके यिए
गहन यर्क्षण िकनफ़्िद, टर ाों सफामनर और यितररत सॉफ़्टिेिर और हाडन िेिर की मजबूत समझ की आिश्यकता हदती है ।
• सोंिर्ि र्िोंड : प्रत्येक बडे भाषा मॉडि में केिि एक यनयित मात्रा में मेमदरी हदती है , इसयिए िह इनपुट के रूप में केिि एक र्नर्िि
सोंिा में ट कन स्वीकार कर सकिा है।
• र्ििसनीयिा और पूिािग्रह: भाषा मॉडि की क्षमताएों पाठ्य प्रयर्क्षण डे टा तक सीयमत हैं और इस प्रकार, इसमें गिि िानकारी, िार्ि,
िैंर्गक पूिािग्रह आर्ि शार्मि ह सकिे हैं।
• कौशि की कमी: इन मॉडिदों में यिर्ेषज्ञता के सार् प्रयतभा की कमी के कारण इन्हें प्रभािी ढों ग से िागू करना और उपिदग करना मुल्किि
हद जाता है।
• साोंस्कृर्िक पूिािग्रह: िे मॉडि मुख्य रूप से अोंग्रेजी पर ध्यान केंयित करते हैं िेयकन भारत की समृद्ध भाषाई यियििता अयिक समािेर्ी
दृयष्टकदण की माों ग करती है ।
• नौकरी का नुकसान: एआई के बढते उपिदग ने नौकरी बाजार के भयिष्य कद िेकर यचोंता बढा दी है । एआई दु यनिा भर में 300 यमयििन
पूणनकायिक नौकररिदों की जगह िे सकता है , यजससे िैयश्वक कािनबि का िगभग 20% प्रभायित हदगा।

आगे की राह:
• नैर्िक र्नर्हिाथों पर र्ििार: जबयक एिएिएम जबरदस्त क्षमता प्रदान करता है , इसके पररयनिदजन से जुडे नैयतक यनयहतार्ों और
चुनौयतिदों पर यिचार करना आिश्यक है ।
• पारिर्शििा और ििाबिे ही सुर्नर्िि करना: एआई भाषा मॉडि के रूप में, एिएिएम उस डे टा में पूिान ग्रहदों और सीमाओों कद दर्ान ता है
यजस पर इसे प्रयर्यक्षत यकिा गिा र्ा। यकसी भी पूिानग्रह कद कम करना और इसके उपिदग में पारदयर्नता और जिाबदे ही सुयनयित करना
महत्वपूणन है ।
• र्िम्मेिार उपय ग: सोंभायित जदल्कखमदों कद कम करते हुए िाभ कद अयिकतम करने के यिए निाचार और यजम्मेदार तैनाती के बीच सोंतुिन
बनाना सिोपरर है ।
• मानि-केंर्द्रि दृर्िक र्: जबयक एिएिएम मानि-जैसी बातचीत का अनुकरण कर सकता है , इसकी सीमाओों कद पहचानना और िह
सुयनयित करना आिश्यक है यक मानि यनरीक्षण और यनणनि इसके अनुप्रिदगदों में एकीकृत हैं।
• कौशि र्िकास: उद्यमदों कद िह सुयनयित करने के यिए प्रयर्क्षण और यिकास कािनक्रमदों में यनिेर् करना चायहए यक उनकी टीमदों के पास
इन मॉडिदों का प्रभािी ढों ग से उपिदग करने का कौर्ि है ।
• र्ारि र्िर्शि एिएिएम: भारत के यिए भारतीि भाषाओों के अनुरूप अपना स्विों का एिएिएम यिकयसत करना एक तत्काि रणनीयतक
आिश्यकता है ।

िघु समािार

6.7 क्वासर • िासर की चमक िैज्ञायनकदों कद िह्ाोंड के र्ुरुआती चरणदों का


अध्यिन करने की अनुमयत दे ती है , क्दोंयक उन्हें दू री से दे खा जा
सोंिर्ि: िैज्ञायनकदों ने िासर नामक ब्लैक हदि के एक क्रूर िगन का
सकता है ।
अििदकन यकिा , जद प्रारों यभक िह्ाोंड में "समय के िैिाि" क
प्रिर्शिि करिा है , र्िससे पिा िििा है र्क उस समय आि 6.8 डाकि पै ट नि
की िुिना में केिि पाोंििाों र्हस्सा ही िेिी से बीििा था। सोंिर्ि: केंि ने ई-कॉमसन कोंपयनिदों से कहा है यक िे अपने प्लेटफॉमन
क्वासर के बारे में: पर "डाकन पैटनन" का उपिदग न करें जद ग्राहकदों कद िदखा दे सकते
• िासर आकार्गोंगाओों के केंि में पाए जाने िािे अत्योंत सयक्रि हैं िा उनकी पसोंद में हेरफेर कर सकते हैं ।
सुपरमैयसि ब्लैक हदि हैं । डाकि पैटनि के बारे में:
• िे हमारे सूिन से िाखदों-करदडदों गुना अयिक यिर्ाि हैं । • इसे भ्रामक पैटनि के रूप में भी जाना जाता है ।
• िे ब्लैक हदि अपने मजबूत गुरुत्वाकषनण ल्कखोंचाि के सार् पदार्न • िह िह र्ब्द है यजसका उपिदग उन तरीकदों का िणनन करने के
कद खीोंचते हैं और उच्च-ऊजान कण जेट सयहत तीव्र यियकरण यिए यकिा जाता है यजनसे िेबसाइटें िा ऐप्स अपने उपिदगकतान
उत्सयजनत करते हैं । कद यनम्नयिल्कखत करने कद मजबूर करते हैं -
• िे पदार्न की एक चमकती हुई यडस्क से यघरे हुए हैं ।

94
 ऐसे कािन करना यजन्हें उपिदगकतान करने का इरादा नहीों 6.10 नॉन-िों र्गबि ट कन
रखते हैं िा अन्यर्ा नहीों करें गे
सोंिर्ि: हाि ही में, गृह मोंिािय (एमएिए) ने हररिाणा के गुरुग्राम
 उपिदगकतान के ऐसे व्यिहार कद हतदत्सायहत करना जद
में 'एनएिटी (नॉन-िोंर्गबि ट कन) एआई और मेटािसि के
कोंपयनिदों के यिए फािदे मोंद नहीों है ।
िुग में अपराि और सुरक्षा' पर जी20 सम्मेिन आिदयजत यकिा।
• यह शब्द 2010 में हैरी र्ििुि ने गढ़ा था ि एक उपिदगकतान
अनुभि (िूएक्स) यडजाइनर हैं , नॉन-िोंर्गबि ट कन के बारे में:
• उिाहरर्: • नॉन-फोंयगबि टदकन, यजन्हें अक्सर एनएफटी के रूप में जाना
 कष्टप्रद यिज्ञापन जद उपिदगकतान की स्क्रीन पर पॉप अप जाता है , िॉकिेन-आधाररि ट कन हैं ि प्रत्येक किा,
हदता रहता है , और उपिदगकतान इसे हटाने के यिए क्रॉस र्डर्िटि सामग्री या मीर्डया जैसी एक अनूठी सोंपयि का
माकन 'X' नहीों ढू ों ढ पाता क्दोंयक यनर्ान नदयटस करने (िा प्रयतयनयित्व करते हैं ।
ल्कक्लक/टै प करने) के यिए बहुत छदटा है । • स्वार्मत्व और प्रामार्र्किा का एक अपररिििनीय
 जब उपिदगकतान छदटे 'X' पर ल्कक्लक/टै प करने का प्रिास र्डर्िटि प्रमार्पि माना जा सकता है , चाहे िह यडयजटि हद
करते हैं , तद कभी-कभी उपिदगकतान इसके बजाि यिज्ञापन िा भौयतक।
पर टै प कर दे ते हैं । • एनएफटी कद यमोंयटों ग नामक प्रयक्रिा के माध्यम से बनािा जाता
है , यजसमें एनएफटी की जानकारी ब्लॉकचेन पर दजन की जाती
6.9 र्िशाि मीटरिे ि रे र्डय टे िीस्क प
है ।
सोंिर्ि: एक बडी सफिता में, खगदियिददों की एक अोंतरराष्टरीि टीम नॉन-िोंगीर्बर्िटी क्ा है?
ने पल्सर अिि कन ों का उपय ग करके गुरुत्वाकषिर् िरों ग ों की • नॉन-फोंगीयबयिटी यकसी पररसोंपर्ि या िस्तु की अनूठी और
उपम्भस्थर्ि की पुर्ि करने िािे िैज्ञार्नक प्रमार् की घ षर्ा की। र्िर्शि प्रकृर्ि क सोंिर्र्िि करिी है ि नही ों ह सकिी है , इसे
एक ही प्रकार की दू सरी िस्तु के सार् एक-से-एक आिार पर
पल्सर के बारे में:
आसानी से आदान-प्रदान यकिा जा सकता है ।
• पल्सर िेिी से घूमने िािे न्यूटरॉन िारे हैं जद सेकोंड से िेकर
• उिाहरर् के र्िए, एक हिाई जहाज का यटकट अयद्वतीि हदता है
यमिीसेकोंड तक के यनियमत अोंतराि पर यियकरण के स्पोंदन
- िह एक यियर्ष्ट उडान पर, एक यियर्ष्ट समि पर, एक यियर्ष्ट
कद यिस्फदयटत करते हैं । सीट यनयदन ष्ट करता है ।
• पल्सर एक िह्ाों डीि प्रकार्स्तोंभ की तरह है क्दोंयक िह रे र्डय
र्करर्ें उत्सर्ििि करिा है जद एक बोंदरगाह प्रकार्स्तोंभ के 6.11 र्ारि में मधु मे ह मे र्िटस (डीएम) और
समान यनियमत रूप से पृथ्वी पर चमकती हैं । िपे र्िक (टीबी)।
र्िशाि मीटरिेि रे र्डय टे िीस्क प (िीएमआरटी):
सोंिर्ि: र्ारि ि गोंर्ीर महामाररय ों - टाइप 2 मधुमेह (उिि
• जीएमआरटी एक कम आिृर्ि िािा रे र्डय टे िीस्क प है जद
मधुमेह मेर्िटस, डीएम) और िपेर्िक (टीबी) के ददहरे बदझ का
आस-पास के सौर मोंडि से िेकर अििदकन िदग्य िह्ाोंड के
सामना कर रहा है।
यकनारे तक यियभन्न रे यडिद खगदिभौयतकीि समस्याओों की जाोंच
डीएम और टीबी के बीि सोंबोंध:
करने में मदद करता है ।
सोंबोंध र्ििरर्
• न डि एिेंसी: यह परमार्ु ऊिाि र्िर्ाग (डीएई) की एक
डीएम और श्वसन डीएम से श्वसन सोंक्रमण यिकयसत हदने
पररिदजना है , जद टाटा इों स्टीट्यूट ऑि िोंडामेंटि ररसिि
सोंक्रमण का खतरा बढ जाता है ।
(टीआईएिआर) के िहि सोंिार्िि ह िी है।
टीबी के यिए जदल्कखम डीएम एक प्रमुख जदल्कखम कारक है
• आिृर्ि:
कारक के रूप में जद टीबी की घटनाओों और गोंभीरता
1. जीएमआरटी 100 मेगाहट्न ज-1,500 मेगाहट्न ज की आिृयि
डीएम कद बढाता है।
बैंडयिड् र् के भीतर काम करने िािी एक अनूठी सुयििा है ।
डीएम और टीबी सह-सोंक्रमण का
2. उपरदक्त आिृयि रें ज में अपने सोंिेदनर्ीि अििदकनदों कद डीएम और टीबी सह-
रदयगिदों में टीबी उपचार के पररणामदों
पूरा करने के यिए जीएमआरटी कद यिर्ेष सुरक्षा प्राप्त है । सोंक्रमण
पर प्रयतकूि प्रभाि पडता है ।
• िू रबीन ों की सोंिा: यह 45 मीटर व्यास के िीस पूर्ििः
2012 में चेन्नई में तपेयदक इकाइिदों में
ििाने य ग्य परििर्यक रे र्डय िू रबीन ों की एक श्ृोंखिा है।
टीबी रदयगिदों में डीएम यकए गए एक अध्यिन में िह पािा
• र्ारि की िीएमआरटी दु यनिा की छह बडी दू रबीनदों में से एक
और प्री-डाियबटीज गिा यक टीबी से पीयडत िदगदों में डीएम
र्ी यजसने गुरुत्वाकषनण तरों गदों की उपल्कस्र्यत की पुयष्ट करने िािे
की व्यापकता की व्यापकता 25.3% थी और
साक्ष्य प्रदान करने में महत्वपूणन भूयमका यनभाई।
24.5% व्यल्कक्त प्री-डाियबयटक र्े।

95
टाइप 2 मधुमेह के बारे में: मल्कस्तष्क सोंक्रमण हदता है यजसे प्राथर्मक अमीर्बक
• टाइप 2 मिुमेह इस बात कद प्रभायित करता है यक र्रीर ऊजान के मेर्नोंग एन्सेििाइर्टस (पीएएम) के रूप में िाना िािा है।
यिए र्कनरा (ग्लूकदज) का उपिदग कैसे करता है ।  िह एक व्यल्कक्त से दू सरे व्यल्कक्त में नहीों फैिता है ।
• िह र्रीर कद इों सुयिन का ठीक से उपिदग करने से रदकता है , • उपिार: अमेररका ल्कस्र्त रदग यनिोंत्रण केंि (सीडीसी) दिाओों के
यजसका इिाज न करने पर रक्त र्कनरा का स्तर उच्च हद सकता सोंिदजन के सार् उपचार की यसफाररर् करता है , यजसमें अक्सर
है । एम्प्फदटे ररयसन बी, एयज़थ्रदमाइयसन, फ्लुकदनाज़दि, ररफैल्किन,
• टीबी के बारे में: टीबी माइक बैरीररयम ट्यूबरकुि र्सस यमल्टे फदयसन और डे क्सामेर्ासदन र्ायमि हदते हैं ।
नामक िीिार्ु के कारण हदता है , जद िगभग 200 सदस्यदों िािे
माइकदबैरीररिासी पररिार से सोंबोंयित है । 6.14 बाह्य अों ि ररक्ष मामि ों के र्िए सों यु क्त रािर
कायाि ि य (यू ए नओओएसए)
6.12 म जे क िायरस
सोंिर्ि:
सोंिर्ि: कूकोंबर म जेक िायरस (सीएमिी) और टॉमेट म जेक
सोंिुक्त राष्टर ने भारतीि मूि की उपग्रह यिर्ेषज्ञ आरिी ह ल्ला-
िायरस (टीओएमिी) ने क्रमर्ः कनानटक और महाराष्टर में फसिदों
मैनी क बाह्य अोंतररक्ष मामिदों के सोंिुक्त राष्टर कािान िि का
कद प्रभायित यकिा है ।
यनदे र्क यनिुक्त यकिा है ।
सीएमिी और टीओएमिी के बारे में:
• ददनदों पौिदों के रदगजनकदों के नाम समान हैं और िे फसिदों कद बाह्य अोंिररक्ष मामि ों के र्िए सोंयुक्त रािर कायाििय के बारे में:
समान नुकसान पहुोंचाते हैं , िेयकन िे अिग-अिग िािरि • बाह्य अोंतररक्ष मामिदों का कािानिि सोंिुक्त राष्टर का कािानिि है
पररिारदों से सोंबोंयित हैं , और अिग-अिग तरीके से फैिते हैं । जद बाह्य अोंतररक्ष के र्ाों यतपूणन उपिदग में अोंतरान ष्टरीि सहिदग कद
• ियद समि पर उयचत उपचार न यकिा जाए तद ददनदों िािरस बढािा दे ने के यिए यजम्मेदार है।
िगभग 100% िसि हार्न का कारर् बन सकिे हैं। • िह कािानिि यििना ल्कस्र्त सोंिुक्त राष्टर कािानिि का यहस्सा है।
• ToMV: यह र्िगािर्िररडे पररिार से सोंबोंयित है और तोंबाकू • बाह्य अोंिररक्ष के शाोंर्िपूर्ि उपय ग पर सर्मर्ि: िह महासभा
मदज़ेक िािरस (टीएमिी) से यनकटता से सोंबोंयित है । की एकमात्र सयमयत के र्िए सर्ििािय के रूप में कायि
 ToMV मेजबानदों में टमाटर, तम्बाकू, यमचन और कुछ करिी है ि र्िशेष रूप से उन मुद्द ों से र्नपटिी है।
सजािटी पौिे र्ायमि हैं । • सयमयत की ि उप-सर्मर्ियााँ हैं:
• सीएमिी: इसकी पहचान 1934 में खीरे में हुई र्ी, यजससे इस 1. िैज्ञायनक और तकनीकी उपसयमयत।
िािरस कद इसका नाम यमिा। 2. कानूनी उपसयमयत।
 सीएमिी में एक बहुत बडा मेजबान पूि है यजसमें ककडी, • कायि:
तरबूज, बैंगन, टमाटर, गाजर, सिाद, अजिाइन, िौकी  महासयचि की ओर से, बाह्य अोंतररक्ष मामिदों का कािानिि
पररिार के सदस्य और कुछ सजािटी पौिे र्ायमि हैं । बाह्य अोंिररक्ष में प्रक्षेर्पि िस्तुओों के रर्िस्टर का
रखरखाि करिा है और रयजस्टर में दजन की गई जानकारी
6.13 र्िमाग खाने िािा अमीबा
कद अपनी िेबसाइट के माध्यम से प्रसाररत करता है।
सोंिर्ि: हाि ही में, केरि के अिाप्पुझा र्ििे में एक 15 िषीि
6.15 सागर सों प कि
िडके की नेगिेररया िाउिेरी या "र्िमाग खाने िािे अमीबा"
सोंिर्ि: बोंदरगाह, जहाजरानी और जिमागन मोंत्री ने हाि ही में
के कारण हदने िािे एक दु िनभ सोंक्रमण के कारण मृत्यु हद गई।
'सागर सोंपकि' नामक स्विे शी र्डिरें र्शयि ग्ल बि नेर्िगेशन
नेगिेररया िाउिेरी के बारे में:
सैटेिाइट र्सस्टम (डीिीएनएसएस) का उि् घाटन र्कया।
• नेगिेररिा फाउिेरी, यजसे आमतौर पर "यदमाग खाने िािा
अमीबा" के नाम से जाना जाता है , एक एकि-कदयर्का िािा सागर सोंपकि के बारे में:
जीि है । • िह एक स्र्िीि आिाररत सोंिद्धन न प्रणािी है जद ग्लदबि
• िह झीिदों, गमन झरनदों और ल्कस्वयमोंग पूि जैसे गमन मीठे पानी के नेयिगेर्न सैटेिाइट यसस्टम (जीएनएसएस) में त्रुयटिदों और
िातािरण में पािा जाता है । अर्ुल्कद्धिदों कद ठीक करती है और अयिक सटीक ल्कस्र्यत की
• िह इतना छदटा है यक इसे केिि माइक्रदस्कदप से ही दे खा जा जानकारी प्रदान करती है ।
सकता है । • उद्दे श्य:
• िैिाि: िह नाक के माध्यम से र्रीर में प्रिेर् करता है और  सुरक्षा सुर्नर्िि करना: नायिकदों कद सुरयक्षत नेयिगेर्न में
मल्कस्तष्क तक जाता है , यजससे गोंभीर और आमतौर पर घातक मदद करना और बोंदरगाह और बोंदरगाह क्षेत्रदों में टकराि,
ग्राउों यडों ग और दु घनटनाओों के जदल्कखम कद कम करना।

96
 सहायक रे र्डय र्सिि: समुिी नेयिगेर्न के यिए जहाजदों • ओजेम्भिक िेनेररक ििा सेमाग्लूटाइड का एक िाोंड नाम
कद रे यडिद सहािता प्रदान करने के यिए िाइटहाउस और है यजसे रक्त र्कनरा के स्तर कद यनिोंयत्रत करके टाइप -2 मिुमेह
िाइटयर्प महायनदे र्क (डीजीएिएि) की सहािता करना के इिाज के यिए एफडीए द्वारा अनुमदयदत यकिा गिा है ।
र्डिरें र्शयि िीएनएसएस क्ा है? • इसे इों जेक्शन द्वारा यदिा जाता है िा मौल्कखक रूप से यििा जा
• िीएनएसएस की कायिप्रर्ािी: एक ल्कस्र्यत स्र्ायपत करने के यिए, सकता है ।
जीएनएसएस ररसीिर कम से कम चार उपग्रहदों से समि सोंकेतदों का • ओज़ेल्किक िीएिपी-1 नामक हामोन के प्रर्ाि की नकि
उपिदग करते हैं , और पृथ्वी पर यसग्नि के पारगमन के दौरान करके काम करिा है, जद र्रीर में स्वाभायिक रूप से उत्पन्न
यकसी भी सोंख्या में त्रुयटिाों िा दे री हद सकती है ।
हदता है ।
• सटीक म्भस्थर्ि: डीजीएनएसएस जीएनएसएस का एक सोंिद्धन न है
 िीएिपी-1 इों सुयिन कद छदडने की प्रयक्रिा कद उिेयजत
यजसे जीएनएसएस प्रणािी में इन त्रुयटिदों और अर्ुल्कद्धिदों कद ठीक
करके, िकृत से ग्लूकदज के यडस्चाजन कद िीमा करके रक्त
करने के यिए यिकयसत यकिा गिा र्ा, यजससे अयिक सटीक ल्कस्र्यत
की जानकारी यमि सके। इस सुिार जानकारी तक पहुों च अोंतर पैदा शकिरा के स्तर क र्िर्नयर्मि करने में मदद करता है ।
करती है जीपीएस और जीएनएसएस ररसीिर अन्य ररसीिरदों की • र्िोंिा: ओजेम्भिक िेस दिा का दु ष्प्रभाि है जद चेहरे की उम्र
तुिना में अयिक सटीक हदते हैं । बढने, त्वचा में ढीिापन का कारण बनता है ।

6.19 पीबीडब्ल्यू आरएस1: गे हों की नई र्कस्म


6.16 एस्पाटे म
सोंिर्ि: गैर-िीनी स्वीटनर एस्पाटे म के स्वास्थ्य प्रर्ाि ों का सोंिर्ि: पोंजाब कृयष यिश्वयिद्यािि ने पदषण सुरक्षा के यिए PBW
आकिन इों टरनेशनि एिेंसी िॉर ररसिि ऑन कैंसर RS1 गेहूों यकि यिकयसत की है।
(आईएआरसी), र्िि स्वास्थ्य सोंगठन (डब्ल्यूएिओ) और खाद्य पीबीडब्ल्यू आरएस1 गेहों की र्कस्म के बारे में:
और कृर्ष सोंगठन (एिएओ) की खाद्य एर्डर्टव्स की सोंयुक्त • नई यकि कद पीबीडब्ल्यू आरएस1 कहा जाता है , यजसमें RS
र्िशेषज्ञ सर्मर्ि द्वारा िारी र्कया गया है। प्रर्िर धी स्टािि का सोंर्क्षप्त रूप है।
• इस यकि कद पादप प्रजनन और आनुिोंयर्की यिभाग के प्रमुख
एस्पाटे म के बारे में:
डॉ. िीएस सदहू के नेतृत्व में गेहूों प्रजनकदों की एक टीम द्वारा 10
• ख ि: इसकी खदज 1965 में अमेररकी रसािनज्ञ िेम्स श्लैटर
िषों की अियि में यिकयसत यकिा गिा है।
ने की थी।
• उच्च एमाइिदज़ और प्रयतरदिी स्टाचन िह सुयनयित करते हैं यक
• अनुमददन: 1974 में एफडीए द्वारा अनुमदयदत, इसका उपिदग
ग्लूकदज रक्तप्रिाह में िीरे -िीरे जारी हद, यजससे ग्लूकदज के स्तर
1980 के दर्क से यियभन्न खाद्य और पेय उत्पाि ों में र्कया
में तत्काि िृल्कद्ध कद रदका जा सके और मिुमेह और हृदि रदगदों
िािा है, इसने अपनी शून्य कैि री और कड़िे स्वाि की
का खतरा कम हद सके।
कमी के कारण िदकयप्रिता हायसि की।
• इसमें कुि स्टाचन सामग्री अन्य गेहूों यकिदों के समान है िेयकन
• उपय ग: इसका उपिदग पके हुए और यडब्बाबोंद खाद्य पदार्ों,
इसमें 30.3% प्रयतरदिी स्टाचन है, जबयक अन्य यकिदों में केिि
पाउडर पेि यमश्रण, कैंडी और पुयडों ग का स्वाद बढाने के यिए
7.5-10% है ।
यकिा जाता है । अन्य उपिदगदों में आहार पेय, च्यूइोंग गम,
र्िोंिाएों :
र्ििेर्टन, आइसक्रीम, डे यरी उत्पाि िैसे िही, नाश्ता
• गेहूों की अन्य यकिदों की तुिना में इसकी उत्पादकता कम है ,
अनाि, टू थपेस्ट और खाोंसी की बूोंि ों और िबाने य ग्य
औसत अनाज उपज 43.18 ल्किोंटि प्रयत हे रेिर है , जद पोंजाब
र्िटार्मन िैसी ििाओों में मीठा करने िािे एिेंट शार्मि
की औसत उपज से कम है ।
हैं।
फायिे :
6.18 ओजे म्भिक औषर्ध • टाइप-2 मिुमेह और हृदि रदगदों के खतरदों कद कम करना।
सोंिर्ि: मिुमेह की दिा ओज़ेल्किक, िजन घटाने की अपनी क्षमता • इसका स्वाद और अहसास सामान्य गेहूों जैसा है , जद इसे एक
के कारण तेजी से िदकयप्रि हद रही है । व्यिहािन यिकल्प बनाता है
ओजेम्भिक ििा के बारे में: • "पीिा रतुआ" के प्रयत पूरी तरह से प्रयतरदिी और भूरे रतुआ
किक रदगदों के यिए "मध्यम रूप से प्रयतरदिी"।

97
िार्िका में सों र्क्षप्त समािार

अोंिररक्ष में काबिन • CH3+ अणु, यजसे र्मथाइि केशन के रूप में र्ी िाना िािा है, क पहिी बार िेम्स िेब स्पेस टे िीस्क प
अर्ु (JWST) द्वारा अोंिररक्ष में ख िा गया है।
• CH3+ हाइडर दजन के सार् अकुर्ि रूप से प्रयतयक्रिा करता है , जबयक अन्य अणुओों के सार् आसानी से
प्रयतयक्रिा करता है , यजससे अयिक जयटि काबनन-आिाररत अणुओों के यनमान ण में आसानी हदती है ।

हाइडर िन ईोंधन सेि • हाइडर दजन चायित बसदों का िैज्ञायनक परीक्षण प्रिदग यदल्ली और फरीदाबाद के बीच यकिा जाएगा।
• हाइडर दजन ईोंिन सेि एक इिेररदकेयमकि सेि है जद हाइडर िन की रासायर्नक ऊिाि क र्बििी में
पररिर्ििि करिा है।

स्माटि बैंडेि • "साइों स एडिाों सेज" में एक अध्यिन प्रकायर्त यकिा गिा र्ा यजसमें एक उों गिी यजतनी बडी पहनने िदग्य,
िािरिेस, िाों यत्रक रूप से िचीिी िाटन पट्टी का उपिदग करके पुराने घािदों के उपचार में तेजी िाने में मदद
करने की पेर्कर् की गई र्ी।
• िाटन बैंडेज एक उन्नि र्िर्कत्सा डर े र्सोंग है यजसमें घाि की उपचार प्रयक्रिा कद बढाने के यिए यियभन्न तकनीकदों
कद र्ायमि यकिा गिा है ।

यूम्भिड र्मशन • िूल्कक्लड एक िूरदपीि यमर्न है , यजसे नासा के रदमन स्पेस टे िीस्कदप के सहिदग से ईएसए द्वारा यनयमनत और
सोंचायित यकिा जाता है ।
• इसका िक्ष्य 'अोंिेरे' िािे िह्ाों ड की सोंरचना और यिकास का पता िगाना है।

डिेन मस्कुिर िर्मिनाडु के डॉररदों की एक टीम ने िापान के िैज्ञायनकदों के सार् यमिकर डीएमडी के यिए रदग-यनिारक
र्डस्टर ॉिी (डीएमडी) उपचार यिकयसत यकिा है ।
डिेन मस्कुिर र्डस्टर ॉिी (डीएमडी) के बारे में:
• डीएमडी एक आनुिोंर्शक र्िकार है यजसकी यिर्ेषता माोंसपेर्शय ों की बढ़िी र्गरािट और कमजदरी है।
• कारर्: िह डीएमडी जीन के एक्स क्रदमदसदम में पाए जाने िािे डािस्टर दयफन जीन में उत्पररितनन के कारण
यकसी भी कािानत्मक यडस्टर दयफन का उत्पादन करने में यिफि हदने के कारण हदता है ।
• यडस्टर दयफन एक प्रदटीन है जद माों सपेयर्िदों की कदयर्काओों में सोंरचनात्मक अखोंडता बनाए रखता है और
उत्पररितनन के अिीन हदता है जद िा तद यिरासत में यमि सकता है िा रदगाणु सोंचरण के दौरान अनािास उत्पन्न
हद सकता है ।
• इिाि: जीन र्ेरेपी, एक्सॉन-ल्कस्कयपोंग और रदग सोंर्दयित करने िािे एजेंट (स्टे रॉिड जैसी सूजनरदिी दिाएों )।

िाउड स्ट रे ि 2023 के सिेक्षण से पता चिा है यक भारत में 68% व्यिसाि अपने क्लाउड-स्टदर यकए गए 40% से अयिक डे टा
कद सोंिेदनर्ीि मानते हैं ।
िाउड स्ट रे ि के बारे में:
• क्लाउड स्टदरे ज ऑि-साइट सििर पर र्डर्िटि डे टा सोंग्रहीि करने की एक र्िर्ध है।
• िाउड स्ट रे ि की आिश्यकिा:
 कोंपयनिाों अपने डे टा केंिदों के सोंचािन की िागत और रखरखाि से बचने के यिए इसका उपिदग करती हैं।
 क्लाउड स्टदरे ज भौयतक सुरक्षा, एल्कन्क्रप्र्न और एक्सेस प्रबोंिन जैसी स्केिेयबयिटी और सुरक्षा सुयििाएँ प्रदान
करता है ।

थ्रेड्स मेटा, र्िसे पहिे िेसबुक के नाम से िाना िािा था, ने हाि ही में यिटर कद टक्कर दे ने के यिए थ्रेड्स नामक
एक निा ऐप यिकयसत यकिा है।
• थ्रेड्स इों स्टाग्राम का एक म्भस्पन-ऑि ऐप है जद दृश्य सामग्री के बजाि बातचीत पर ध्यान केंयित करता है ।
सोंर्ार्िि र्िटर र्िकल्प:
• िूस्की: िह एक यिटर क्लदन है जद यिटर के सह-सोंस्र्ापक और पूिन सीईओ जैक डदसी द्वारा समयर्नत है ।
• मास्ट डॉन: "एों टी-यिटर" प्लेटफॉमन के रूप में जाना जाने िािा मास्टदडॉन एक यिकेन्द्रीकृत सदर्ि नेटियकिंग
अनुभि प्रदान करता है ।
• सबस्टै क न ट् स: एक िदकयप्रि न्यूज़िेटर प्लेटफॉमन सबस्टै क ने यिटर के सोंभायित यिकल्प के रूप में सबस्टै क
नदट् स पेर् यकिा है ।

98
अनुकोंपा उपय ग हाि ही में, है दराबाद के एक अस्पताि में, आईसीिू में व्यापक दिा प्रयतरदिी जीिाणु सोंक्रमण से पीयडत एक मरीज
प्र ट कॉि का इिाज 'अनुकोंपा उपिदग प्रदटदकॉि' के तहत एक दिा सेफेपाइम-ज़ाइडबैरम के सार् यकिा गिा र्ा, यजसका
चरण 3 परीक्षण चि रहा है ।
अनुकोंपा उपय ग प्र ट कॉि क्ा है?
• र्कसी गोंर्ीर िीिन-घािक बीमारी या म्भस्थर्ि िािे र गी के र्िए नैदायनक परीक्षणदों के बाहर इिाज के यिए
एक िाोंि र्िर्कत्सा उत्पाि (ििा, िैर्िक, या र्िर्कत्सा उपकरर्) तक पहुोंच प्राप्त करने का एक सोंभायित
मागन है , जब कदई तुिनीि िा सोंतदषजनक िैकल्कल्पक यचयकत्सा यिकल्प उपिब्ध नहीों हदते हैं ।

िेप्ट स्पाइर र्सस मानसून से पहिे िेप्टदस्पािरदयसस के मामिदों में िृल्कद्ध से यचोंयतत, मुोंबई में नागररक स्वास्थ्य यिभाग ने सकारात्मक
परीक्षण करने िािे 300 से अयिक रदयगिदों का अध्यिन करने का यनणनि यििा है ।
िेप्ट स्पायर र्सस के बारे में:
• यह िानिर ों में ह ने िािा एक सोंक्रामक रदग है िेयकन कभी-कभी कुछ पिान िरणीि पररल्कस्र्यतिदों में िह मनुष्यदों
में भी फैि जाता है ।
• इसके कारर्: िेप्ट स्पाइरा इों टरर गन्स या िेप्ट स्पाइरा नामक जीिाणु।
• र ग के िाहक: जोंगिी िा घरे िू जानिर, यजनमें कृोंतक, मिेर्ी, सूअर और कुिे, जि भैंस, बकरी, घदडे और
भेड र्ायमि हैं ।

रािरीय अोंग एिों राष्टरीि अोंग और ऊतक प्रत्यारदपण सोंगठन (एनओटीटीओ) ने व्यापार के यिए अोंगदों की पेर्कर् और प्रचार करने
ऊिक प्रत्यार पर् िािी यनजी िेबसाइटदों और सदर्ि मीयडिा पदस्ट के ल्कखिाफ चेतािनी जारी की है ।
सोंगठन रािरीय अोंग और ऊिक प्रत्यार पर् सोंगठन (NOTTO) के बारे में:
(NOTTO) • िह स्वास्थ्य सेिा महायनदे र्ािि, स्वास्थ्य और पररिार कल्याण मोंत्रािि के तहत स्र्ायपत एक राष्टरीि स्तर का
सोंगठन है ।
• िह दे र् में अोंगदों और ऊतकदों की खरीद और यितरण तर्ा अोंगदों और ऊतकदों के दान और प्रत्यारदपण की रयजस्टर ी
के यिए समन्वि और नेटियकिंग की अल्कखि भारतीि गयतयियििदों के यिए र्ीषन केंि के रूप में कािन करता है ।

99
7. रक्षा और सु र क्षा
7.1 र्ारि में नशीिी ििाओों का खिरा
सोंिर्ि:
नारकदयटक्स कोंटर दि ब्यूरद (एनसीबी) ने बड़े पैमाने पर 1,44,000 र्कि ग्राम अिैध ििाओों का र्नपटान करके नशीिी ििाओों के खिरे
से र्नपटने में एक महत्वपूर्ि किम उठाया है।
र्ारि में नशीिी ििाओों के खिरे के पीछे कारक:
• र्ौग र्िक कारर्:
• ग डन र्क्रसेंट और ग डन टर ाइएों गि: इन ददनदों के बीच
ल्कस्र्त भारत की भौगदयिक ल्कस्र्यत इसे हेर इन और अफीम
के पररिहन के र्िए एक आिशि मागि बनािी है।
 गदल्डन टर ाइएों गि मॉयफनन और हे रदइन के उत्पादन के
यिए कुख्यात है जबयक गदल्डन यक्रसेंट अफीम का
उत्पादन करता है
• समुद्र से र्नकटिा: भारत कई दे र्दों के सार् सीमा साझा
करता है जद प्रमुख दिा उत्पादक और पारगमन यबोंदु हैं ,
यजनमें अफगायनस्तान और म्याोंमार र्ायमि हैं ।
• र्छद्रपूर्ि सीमाएाँ : िह अोंतरान ष्टरीि, भूयम और समुिी सीमाओों
के माध्यम से दे र् में घुसपैठ करता है , यजसका केंि यबोंदु
पायकस्तान के सार् पयिमी अोंतरान ष्टरीि सीमा है। छर्ि स्र ि: इों र्डया टु डे
 अरब सागर और बोंगाि की खाड़ी में समुिी मागों के माध्यम से मादक पदार्ों की तस्करी, भारत में तस्करी की गई कुि अिैि दिाओों
का िगभग 70% यहस्सा है ।
• िकनीकी कारर्:
• र्डर्िटि उपकरर् और डर न: अब नए र्डर्िटि उपकरर् ों और डर न का उपिदग करके इन सीमाओों के पार दिाओों की तस्करी की
जा रही है । इसके अिािा, डर ग यगरदह अब ििाओों क र्छपाने और उन्हें र्ििररि करने के र्िए क ररयर, पैकेि और मेि का उपय ग
करिे हैं।
• डाकििेब: कदररिर िा डाक सेिाओों का बढता उपिदग सीिे तौर पर भारत में डाकन िेब गयतयियि में िृल्कद्ध से जुडा हुआ है यजसका उपिदग
अिैि पदार्ों कद खरीदने के यिए यकिा जाता है ।
• सोंगर्ठि अपराध:
 नशीिी ििाओों के िस्कर ,ों सोंगर्ठि आपरार्धक नेटिकि और आिोंकिार्िय ों के बीि साोंठगाोंठ: आतोंकिादी स्र्ायपत तस्करी मागों
का फािदा उठाते हैं , हर्थयार ों और र्िस्फ टक ों के साथ सीमाओों में घुसपैठ करने के र्िए अच्छी िरह से मिबूि आपरार्धक
समूह ों के साथ सहय ग करिे हैं, र्िससे सुरक्षा िुनौर्ियाों बढ़ िािी हैं।
• भ्रिािार: र्िर्ध प्रितनन और अन्य सरकारी अयिकाररिदों के बीच भ्रष्टाचार ने नर्ीिी दिाओों की तस्करी कद और बढािा यदिा है ।
• सामार्िक आर्थिक कारर्:
 र्शक्षा का अर्ाि: िुिाओों की आकाों क्षाएों , जद यर्क्षा की यनम्न गुणििा के कारण पूरी नहीों हद पातीों, उन्हें यनरार् और नर्ीिी दिाओों के
प्रयत सोंिेदनर्ीि बना दे ती हैं ।
 िु ष्िक्र: गरीब पररिारदों के नर्े के आदी िदग, अपनी दै यनक खुराक के खचन कद पूरा करने के यिए नर्ीिी दिाओों की तस्करी में फोंस
जाते हैं ।
• र्सोंथेर्टक ििाओों और प्रीकसिर रसायन ों की िस्करी: भारत बहुत सारी यसोंर्ेयटक दिाओों और प्रीकसनर रसािनदों का यनमान ण करता है
यजनकी दे र् से तस्करी की जाती है ।
• खराब र्िर्ध प्रिििन: भारत की र्िर्ध प्रितनन एजेंयसिदों में कमििाररय ों की कमी है, र्िशेष प्रर्शक्षर् और उपकरर् ों की कमी है , और
अक्सर डर ग यर्पमेंट का पता िगाने िा उस पर रदक िगाने में असमर्न हैं ।

100
र्ारि में नशीिी ििाओों के िु रुपय ग का प्रर्ाि:
• हर्थयार ों की िस्करी और आिोंकिािी गर्िर्िर्धय ों का र्ििप षर्: नर्ीिी दिाओों के तस्करदों द्वारा उपिदग की जाने िािी दे र् की
अोंतरान ष्टरीि सीमाओों का उपिदग हयर्िारदों के सार्-सार् आतोंकिायदिदों की दे र् में तस्करी के यिए यकिा जाता है ।
• र्िर्ध और व्यिस्था के मुद्दे: नर्ीिे पदार्ों और दिाओों की व्यापक उपिब्धता घरे िू माोंग में िृल्कद्ध में िदगदान दे ती है , यजससे अव्यिल्कस्र्त
व्यिहार हदता है और समाज में र्िर्ध और व्यिस्र्ा की समस्याएों पैदा हदती हैं ।
• आर्थिक प्रर्ाि: नर्ीिी दिाओों के दु रुपिदग से हदने िािे बडे आयर्नक नुकसान में उत्पादन में हदने िािी हायन और नर्ीिी दिाओों के आदी
िदगदों की दे खभाि और पुनिान स के यिए सोंसािनदों का उपिदग र्ायमि है ।
• सोंगर्ठि अपराध की सुर्िधा: मादक पदार्ों की तस्करी, मानि तस्करी और नाको आतोंकिाद जैसे अन्य सोंगयठत आपरायिक उद्यदगदों कद
सुयििा प्रदान करती है , जद सभी िदगदों, हयर्िारदों और प्रयतबोंयित िस्तु ओों की तस्करी के यिए समान नेटिकन और मागों का उपिदग करते हैं ।
• िनसाोंम्भिकीय िार्ाोंश का नुकसान: मादक िव्यदों के सेिन से यहों सा और अपराि में िृल्कद्ध हदती है , व्यल्कक्तगत उत्पादकता और क्षमता
ख़राब हदती है ।
• मन िैज्ञार्नक प्रर्ाि: िह पररिारदों के भीतर सोंघषों कद जन्म दे ता है , यजससे इसके सभी सदस्यदों कद अत्ययिक भािनात्मक परे र्ानी हदती
है ।

नशीिी ििाओों के खिरे पर सरकार की प्रर्िर्क्रया: गृह मोंत्रािि तीन अिग-अिग तरीकदों से नर्ीिी दिाओों पर कारन िाई कर रहा है :
• सोंस्र्ागत सोंरचनाओों कद मजबूत बनाना
• नर्ीिे पदार्ों के यनिोंत्रण से सोंबोंयित सभी एजेंयसिदों कद सर्क्त बनाना और उनके बीच समन्वि कद मजबूत करना एिों
• मोंत्रािि द्वारा जागरूकता अयभिान चिािा जा रहा है ।

अन्य उपाय नीिे सूिीबद्ध हैं -


• र्िधायी:
 नारक र्टक डर ग्स और साइक टर र्पक पिाथि अर्धर्नयम, 1985: िह यकसी भी नर्ीिे पदार्न िा मन:प्रभािी पदार्न के
उत्पादन/यियनमान ण/खेती, कब्जे, यबक्री, खरीद, पररिहन, भोंडारण और उपभदग पर प्रयतबोंि िगाता है ।
 नारक र्टक डर ग्स और साइक टर र्पक पिाथि के अिैध व्यापार की र कथाम अर्धर्नयम, 1988: अयियनिम के तहत, यकसी व्यल्कक्त
के यिए यकसी भी नर्ीिे पदार्न िा मन:प्रभािी पदार्न का उत्पादन/यनमान ण/खेती करना, रखना, बेचना, खरीदना, पररिहन करना, भोंडारण
करना अर्िा उपभदग करना गैरकानूनी है ।
• सोंस्थागि उपाय: नारक र्टक्स कोंटर ि ब्यूर (एनसीबी) के बारे में:
 नारक र्टक्स कोंटर ि ब्यूर (एनसीबी): िह अिैि नर्ीिी • एनसीबी भारत की मुख्य कानून प्रितनन और खुयफिा एजेंसी है जद मादक
दिाओों के यनिोंत्रण में समन्वि के सार्-सार् दे र् में नर्ीिी पदार्ों की तस्करी और अिैि पदार्ों के दु रुपिदग से िडने के यिए
यजम्मेदार है ।
दिाओों की तस्करी के मामिदों कद सोंकयित करने के यिए
• िह नारकदयटक डर ग्स और साइकदटर दयपक पदार्न अयियनिम, (1985) के
यजम्मेदार है।
पूणन कािानन्विन कद सक्षम बनाता है ।
• सामार्िक न्याय और अर्धकाररिा मोंिािय: नशीिी ििा
की माोंग में कमी के यिए नदडि मोंत्रािि ने यनम्नयिल्कखत कदम उठाए हैं :
 2018-2025 के र्िए नशीिी ििाओों की माोंग में कमी के र्िए रािरीय कायि य िना (एनएपीडीडीआर): इसका उद्े श्य नर्ीिी
दिाओों के दु रुपिदग के प्रयतकूि पररणामदों कद कम करना है ।
 शराब और मािक द्रव्य ों के सेिन की र कथाम के र्िए य िना: नर्ेयडिदों के
यिए एकीकृत पुनिान स केंि की स्र्ापना/सोंचािन के यिए स्वैल्किक सोंगठनदों और
अन्य पात्र एजेंयसिदों कद यििीि सहािता दी जाती है ।
• अन्य र्निारक सरकारी उपाय:
 गश्त और र्नगरानी में िृम्भद्ध: ज्ञात नर्ीिी दिाओों के मागों पर गहन यनिारक और
यनषेि प्रिास और आिात और यनिान त यबोंदुओों पर सख्त यनगरानी और प्रितनन।
 सीमा सुरक्षा एिेंर्सय ों क सशक्त बनाना: सीमा सुरक्षा एजेंयसिदों कद नारक र्टक
डर ग्स और साइक टर र्पक पिाथि (एनडीपीएस) अर्धर्नयम के िहि कारि िाई
करने का अर्धकार र्िया गया है।
 नशा मुक्त र्ारि अर्र्यान: इसमें नर्ीिी दिाओों की कमी की यदर्ा में एक
व्यापक दृयष्टकदण र्ायमि है।
101
 नाको क -ऑर्डि नेशन सेंटर (एनसीओआरडी): िह सभी डर ग र्िर्ध प्रितनन एजेंयसिदों और अन्य यहतिारकदों द्वारा ठदस कारन िाई के
यिए एक साझा मोंच प्रदान करता है ।
आगे की राह:
• खुर्िया िानकारी साझा करना: सूचना साझा करने के यिए एक प्रणािी यिकयसत करने की तत्काि आिश्यकता है जद प्रितनन एजेंयसिदों
कद यितरकदों कद आपूयतन के स्रदत से जदडने में मदद करे गी।
• आपूर्िि के स्र ि पर ध्यान िे ना: प्रितनन एजेंयसिदों कद यितरकदों और यिक्रेताओों पर ितनमान फदकस के बजाि मुख्य रूप से नेटिकि पर
गहराई से नजर रखने और उत्पािक ों और आपूर्ििकिािओों पर मुकिमा ििाने पर ध्यान केंर्द्रि करना िार्हए।
• र्िशेष कौशि, समर्पिि इकाइयााँ: नर्ीिी दिाओों से यनपटने िािे राज्य पुयिस सोंगठनदों के भीतर यिर्ेष कौर्ि यिकयसत करने की
आिश्यकता है ।
• नशा मुम्भक्त केंद्र ों और र्शर्िर ों की स्थापना: सरकारी एजेंयसिाों प्रभायित िुिाओों की मदद के यिए र्ििेिार नशा मुम्भक्त केंद्र और र्शर्िर
स्थार्पि कर सकिी हैं। पुनिानस और दे खभाि के बाद परामर्न से कई िदगदों की जान बचाई जा सकती है ।
• व्यापक दृर्िक र्: उपिार, पुनिािस और नुकसान में कमी के यिए व्यापक दृयष्टकदण की खदज करना।
• नागररक समाि की र्ूर्मका: िह जीिन में स्वस्र् यिकल्प चुनने और नर्ीिी दिाओों से दू र रहने के यिए छात्रदों, िुिा क्लबदों आयद के यिए
जागरूकता कािनक्रम चिा सकता है ।
िघु समािार

7.2 सस्पें श न ऑि ऑपरे शों स (एसओओ) • रक्षा अयिग्रहण पररषद (डीएसी) की अध्यक्षता रक्षा मोंत्री
राजनार् यसोंह ने की।
सोंिर्ि: हाि ही में, मयणपुर के मुख्यमोंत्री ने कहा यक केंि पहाडी
क्षेत्र में कुकी यििदही समूहदों के सार् सस्पेंर्न ऑफ ऑपरे र्ोंस रािेि समुद्री र्िमान के बारे में
(एसओओ) समझौते के कािानन्विन कद सुयनयित करे गा। • यह सोंबोंर्धि सहायक उपकरर्, हयर्िार, यसम्युिेटर, स्पेिर,
सस्पेंशन ऑि ऑपरे शोंस (एसओओ) के बारे में: दस्तािेज़ीकरण, चािक दि प्रयर्क्षण और िॉयजल्कस्टक समर्नन
• मर्र्पुर में उग्रिादी समूहदों के सार् राजनीयतक बातचीत र्ुरू के सार् आएगा।
करने के उद्े श्य से 2008 में सस्पेंर्न ऑफ ऑपरे र्ोंस  फ्राों सीसी सरकार से इसकी खरीद अोंिर-सरकारी
(एसओओ) समझौते पर मुहर िगाई र्ी। समझौिे (आईिीए) के आधार पर हुई है।
• मयणपुर में 30 कुकी यििदही समूहदों में से 25 भारत सरकार और
राज्य के सार् यत्रपक्षीि सोंचािन यनिोंबन (एसओओ) के तहत
पक्षकार हैं ।
सस्पेंशन ऑि ऑपरे शोंस (एसओओ) के िहि मुि र्बोंिु:
• राज्य और केंिीि बिदों सयहत सुरक्षा बिदों कद कदई अयभिान
र्ुरू नहीों करना है , न ही भूयमगत समूह ऐसा कर सकते हैं ।
• यििदही हस्ताक्षरकतान भारत के सोंयििान, भूयम र्िर्ध और
मयणपुर की क्षेत्रीि अखोंडता का पािन करें गे।
• उन्हें सभी प्रकार के अत्याचार, जबरन िसूिी आयद करने से
प्रयतबोंयित यकिा गिा है ।
• उग्रिादी कैडरदों कद सरकार द्वारा यचल्कन्हत यनयदन ष्ट यर्यिरदों में
सीयमत यकिा जाना है ।
• हयर्िारदों कद डबि-िॉयकोंग यसस्टम के तहत एक सुरयक्षत कमरे
में जमा यकिा जाता है। समूहदों कद केिि अपने यर्यिरदों की
सुरक्षा और अपने नेताओों की सुरक्षा के यिए हयर्िार यदए जाते
हैं ।  26 रािेि-एम िड़ाकू र्िमान ों में 22 र्सोंगि सीटर िेट
और िार र्िन-सीटर टर े नर र्ायमि हैं ।
7.3 र्ारिीय नौसे ना के र्िए रािे ि  भारतीि िािु सेना अप्रैि 2016 में हस्ताक्षररि €7.87
र्बर्ियन के सौिे के तहत खरीदे गए र्ारिीय
सोंिर्ि: हाि ही में, रक्षा अर्धग्रहर् पररषि (डीएसी) ने भारतीि आिश्यकिाओों के र्िए अनुकूर्िि 36 रािेि िेट का
नौसेना के यिए 26 रािेि समुद्री र्िमान और िीन अर्िररक्त सोंिािन करिी है।
स्कॉपीन पनडु म्भब्बय ों की खरीद के प्रस्तािदों कद मोंजूरी दी हैं ।

102
स्कॉपीन पनडु म्भब्बय ों के बारे में • इस पर 120 से अर्धक िे श ों ने हस्ताक्षर र्कये हैं। अमेररका,
• इन पनडु ल्कब्बिदों कद अोंडर बाि श्रेणी (under Buy (Indian) रूस और िूक्रेन ने इस पर हस्ताक्षर नहीों यकिे हैं ।
category) के तहत खरीदा जाएगा और इसका यनमान ण मझगाोंि • सोंिुक्त राष्टर के महासयचि कन्वेंर्न के सरों क्षक हैं ।
डॉक यर्पयबल्डसन यियमटे ड (एमडीएि) द्वारा यकिा जाएगा।
• एमडीएि पहिे से ही नौसेना समूह से प्रौद्यदयगकी हस्ताों तरण 7.5 सू ि ना सु र क्षा प्रथाओों पर र्िशार्निे श:
के तहत प्र िेर-75 के अोंतगनत छह स्कॉपीन श्ेर्ी की सीईआरटी-इन
पनडु म्भब्बय ों का र्नमािर् कर रहा है, ि अरू बर 2005 में सोंिर्ि: हाि ही में, भारतीि कोंप्यूटर आपातकािीन प्रयतयक्रिा टीम
हस्ताक्षररि 3.75 र्बर्ियन डॉिर का सौिा था। (CERT-In) ने सुरयक्षत और यिश्वसनीि इों टरनेट एिों सरकारी
रक्षा अर्धग्रहर् पररषि (डीएसी) के बारे में सोंस्र्ाओों के यिए "सूचना सुरक्षा प्रर्ाओों पर यदर्ायनदे र्" जारी
• तीनदों सेनाओों (र्ि सेना, नौसेना और िािु सेना) और भारतीि यकए।
तटरक्षक बि के यिए नई नीयतिदों और पूोंजी अयिग्रहण पर
यनणनि िेने के यिए रक्षा मों िािय में सिोच्च यनणनि िेने िािी CERT-In के बारे में:
सोंस्र्ा है । • CERT-In कोंप्यूटर सुरक्षा सोंबोंिी घटनाओों के घयटत हदने पर
• रक्षा मोंत्री इस पररषि के अध्यक्ष हैं। प्रयतयक्रिा दे ने के यिए रािरीय न डि एिेंसी है ।
• इसका गठन 2001 में कारर्गि यु द्ध (1999) के बाि 'रािरीय • अर्धिे श:
सुरक्षा प्रर्ािी में सुधार' पर मोंर्िय ों के समूह की  साइबर घटनाओों पर सूचना का सोंग्रहण, यिश्लेषण और
र्सिाररश ों के बाि र्कया गया था। प्रसार करना।
 साइबर सुरक्षा घटनाओों का पूिाननुमान और अिटन दे ना।
7.4 िस्टर यु द्ध सामग्री
 साइबर सुरक्षा घटनाओों से यनपटने के यिए आपातकािीन
सोंिर्ि: सोंिुक्त राज्य अमेररका ने अपनी सेना कद रूसी सेना कद
उपाि,
पीछे िकेिने में मदद करने के यिए िूक्रेन कद क्लस्टर िुद्ध सामग्री
 साइबर घटना प्रयतयक्रिा गयतयियििदों का समन्वि करना।
भेजने का यनणनि यििा है ।
सूिना सुरक्षा प्रथाओों पर र्िशार्निे श ों के बारे में
िस्टर युद्ध सामग्री के बारे में:
• क्लस्टर िुद्ध सामग्री एक बम है जद हिा में खुिता है और एक • उद्दे श्य: अपने उपिदगकतानओों के यिए एक खुिा, सुरयक्षत और
यिश्वसनीि और जिाबदे ह इों टरनेट सुयनयित करना।
यिस्तृत क्षेत्र में छ टे "बम" छ ड़िा है।
• बमदों कद एक ही समि में कई िक्ष्यदों पर हमिा करके टैं कदों और • आिश्यकिा: भारत के यडयजटि पररदृश्य में जबरदस्त िृल्कद्ध
दे खी गई है , 80 कर ड़ से अर्धक र्ारिीय (र्डर्िटि
उपकरणदों के सार्-सार् सैयनकदों कद नष्ट करने के यिए यडज़ाइन
नागररक) सयक्रि रूप से इों टरनेट और साइबरस्पेस का
यकिा गिा है ।
उपिदग कर रहे हैं ।
• किस्टर युद्ध सामग्री में डूड िर बहुि अर्धक रही है ।
 डूड रे ट (Dud rate): इसका मतिब है यक हजारदों छदटे • प्रय ज्िा:
गैर-यिस्फदयटत बम यबना यिस्फदट के रह सकते हैं और
 भारत सरकार (व्यिसाि का आिोंटन) यनिम, 1961 की
पहिी अनुसूची में यनयदन ष्ट सभी मोंत्रािि, यिभाग, सयचिािि
दर्कदों बाद भी िदगदों कद मार और अपोंग कर सकते हैं ।
और कािानिि, उनके सोंिग्न और अिीनस्र् कािान ििदों के
• रें ि: 15 से 20 मीि (24 से 32 यकिदमीटर) दू र िक्ष्य पर हमिा।
िस्टर युद्ध सामग्री पर कन्वेंशन:
सार्।
• इसे 2008 में अपनाया गया, 2010 में िागू यकिा गिा  सािनजयनक क्षेत्र के उद्यम
• िह क्लस्टर हयर्िारदों के सर्ी प्रकार के उपय ग, उत्पािन, • मुि सूिना सुरक्षा अर्धकारी की र्नयुम्भक्त: सरकारी
स्थानाोंिरर् और र्ोंडारर् पर प्रर्िबोंध िगािा है। सोंगठनदों कद आईटी सोंचािन टीम से स्वतोंत्र, एक समयपनत
• िह पीयडत सहािता, दू यषत स्र्िदों की सफाई, जदल्कखम कम साइबर सुरक्षा टीम के सार्-सार् एक मुख्य सूचना सुरक्षा
करने की यर्क्षा और भोंडार नष्ट करने का समर्नन करने के यिए अयिकारी (सीआईएसओ) की यनिुल्कक्त करनी चायहए।
एक रूपरे खा स्र्ायपत करता है ।

समािार में अभ्यास

अभ्यास प्रर्िर्ागी कायिक्रम का स्थान

र्िमेक्स 23 (JIMEX) जापान-भारत समुिी अभ्यास यिर्ाखापिनम (आों ध्र प्रदे र्)

साल्वेक्स (SALVEX) भारतीि नौसेना - अमेररकी नौसेना (आईएन - िूएसएन) कदल्कच्च (केरि)

103
8. समाि और सामार्िक न्याय
8.1 इच्छामृ त्यु
सोंिर्ि:
हाि ही में एक 62 िषीय मर्हिा ने अपनी सोंपयि से अिैि कब्जा हटाने में यिफि रहने पर इिामृत्यु की अनुमयत माोंगने के यिए िररष्ठ
प्रर्ासयनक अयिकाररिदों से सोंपकन यकिा।

के बारे में:
• व्युत्पर्ि: 'यूथेनेर्सया' र्ब्द दद ग्रीक र्ब्ददों(eu और
thanatos) से बना है । 'ईयू' का अथि है 'अच्छा' और
'थानाट स' का अथि है 'मृत्यु'
• इिामृत्यु उस प्रर्ा कद सोंदयभनत करता है यजसके तहत
कदई व्यल्कक्त यकसी िाइिाि म्भस्थर्ि या असहनीय
पीड़ा से राहि पाने के र्िए िानबूझकर अपना
िीिन समाप्त कर िेिा है।
• इिामृत्यु सहािता प्राप्त मृत्यु की श्ेर्ी में आिी है,
यजसमें सहािता प्राप्त आत्महत्या भी र्ायमि है ।
 इच्छामृत्यु तब हदती है जब कदई व्यल्कक्त यकसी अन्य
व्यल्कक्त के जीिन कद ददन रयहत तरीके से समाप्त कर
दे ता है , िबर्क सहायिा प्राप्त आत्महत्या के तहत
एक यचयकत्सक आमतौर पर घातक इों जेक्शन द्वारा
अपने जीिन कद समाप्त करने में रदगी की सहािता
करता है ।
इच्छामृत्यु के प्रकार:
• सर्क्रय इच्छामृत्यु: इसमें र्कसी व्यम्भक्त के िीिन क
पदार्ों िा बाहरी बि के साथ समाप्त करने के र्िए
सर्क्रय हस्तक्षेप शार्मि है, िैसे र्क घािक इों िेक्शन
िे ना।
• र्नम्भिय इच्छामृत्यु: िह जीिन समर्नन िा उपचार बोंद
करने कद सोंदयभनत करता है जद एक असाध्य रूप से
बीमार व्यल्कक्त कद जीयित रखने के यिए आिश्यक है।
• स्वैम्भच्छक इच्छामृत्यु: िह रदगी की सहमयत से हदता है ।
• अनैम्भच्छक इच्छामृत्यु: िह रदगी की सहमयत के यबना
हदता है ।

इच्छामृत्यु पर र्ारिीय न्यायपार्िका:


• अरुर्ा रामिन्द्र शानबाग बनाम र्ारि सोंघ (2011): इस मामिे में, सिोच्च न्यािािि ने माना र्ा यक असािारण पररल्कस्र्यतिदों में यनल्किि
इिामृत्यु की अनुमयत दी जा सकती है ।
• कॉमन कॉज बनाम र्ारि सोंघ (2018): उच्चतम न्यािािि ने असाध्य रूप से बीमार र र्गय ों की र्िर्िोंग र्िि क मान्यिा िे िे हुए
र्नम्भिय इच्छामृत्यु की अनुमर्ि िी, ि िडिा र्नम्भष्क्रयिा िधी अिस्था में जा सकते र्े, और इस प्रयक्रिा कद यियनियमत करने के यिए
यदर्ायनदे र् जारी यकए।
 र्िर्िोंग र्िि: यियिोंग यिि एक कानूनी दस्तािेज है जद उस प्रकार की र्िर्कत्सा िे खर्ाि क र्नर्िि ि करिा है जद कदई व्यल्कक्त अपनी
इिाओों कद बताने में असमर्न हदने की ल्कस्र्यत में करता है िा नहीों चाहता है ।
 इसे अर्ग्रम र्निे श के रूप में र्ी िाना िािा है।

104
• 2013 में उच्चिम न्यायािय ने गररमा के सार् मरने के अयिकार कद और अयिक सुिभ बनाने के यिए यदर्ायनदे र्दों में सोंर्दिन यकिा।
प्रथागि धार्मिक प्रथाएाँ :
• प्राय पिेशन (र्ाल्कब्दक रूप से उपिास के माध्यम से मरने
का सोंकल्प) र्होंिू धमि में एक प्रथा है जद एक ऐसे व्यम्भक्त
की उपिास द्वारा आत्महत्या क िशाििी है यजसकी कदई
इिा िा महत्वाकाों क्षा नहीों बची है , और जीिन में कदई
यजम्मेदाररिाों नहीों बची हैं ।
• इसी तरह की एक प्रर्ा िैन धमि में र्ी मौिूि है, र्िसे
सोंथारा कहा जाता है।
• िर्मि सोंस्कृर्ि में इसे िािम्भिरुिि और बौद्ध धमि में
स कुर्शनबुत्सु कहा िािा है ।
िैर्िक रुझान:
• नीिरिैंड, िक्जमबगि, बेम्भियम: िे दे र् "असहनीि
पीडा" का सामना करने िािे ऐसे यकसी भी व्यल्कक्त के यिए
इच्छामृत्यु और सहायिा प्राप्त आत्महत्या ि न ों की
अनुमर्ि िे िे हैं, यजसमें सुिार की कदई सोंभािना नहीों है ।
• म्भस्वट् जरिैंड: ल्कस्वट् ज़रिैंड ने इिामृत्यु पर प्रयतबोंि िगा
यदिा है िेयकन डॉरर िा यचयकत्सक की उपल्कस्र्यत में मरने
की अनुमयत दे ता है ।
छर्ि क्रेर्डट: र्होंिुस्तान टाइम्स
• कनाडा: कनाडा ने घदषणा की र्ी यक माचन 2023 तक मानयसक रूप से बीमार रदयगिदों के यिए इिामृत्यु और सहािता प्राप्त मृत्यु की
अनुमयत दी जाएगी; हािाँ यक, इस यनणनि की व्यापक रूप से आिदचना की गई है , और इस कदम में दे री हद सकती है।
• सोंयुक्त राज् अमेररका: सोंिुक्त राज्य अमेररका के यियभन्न राज्यदों में अिग-अिग र्िर्ध हैं । िार्शोंगटन, ओरे गॉन और म ट
ों ाना जैसे कुछ
राज्यदों में इिामृत्यु की अनुमयत है ।
• यूनाइटे ड र्कोंगडम: िूनाइटे ड यकोंगडम इसे अिैि और हत्या के बराबर मानता है।

इच्छामृत्यु के पक्ष में िकि इच्छामृत्यु के र्िरुद्ध िकि

• स्वायििा और व्यम्भक्तगि पसोंि: िह उन व्यल्कक्तिदों कद • िीिन की पर्िििा: कई िमन और नैयतक प्रणायििाँ जीिन कद पयित्र
अपने जीिन और मृत्यु पर यनिोंत्रण रखने की अनुमयत दे ता है मानती हैं और ऐसा माना जाता है यक जानबूझकर अपना जीिन समाप्त
जद िाइिाज बीमारी िा असहनीि ददन से पीयडत हैं । करना िा दू सरे की मृत्यु में सहािता करना नैयतक रूप से गित है ।
• कि कम करना: िह उन व्यल्कक्तिदों के यिए एक दिािु • ऐसी यचोंताएों व्यक्त की गई हैं यक ियद सहािता प्राप्त आत्महत्या अयिक
यिकल्प प्रदान करता है जद अत्ययिक र्ारीररक िा व्यापक रूप से उपिब्ध हद जाती है , तद कमजदर आबादी, जैसे यक
भािनात्मक ददन का अनुभि कर रहे हैं यजसे प्रभािी ढों ग से यिकिाों ग िा मानयसक बीमाररिदों िािे िदगदों कद जबरदस्ती िा
प्रबोंयित नहीों यकिा जा सकता है । िह िोंबे समि तक हदने दु व्यनिहार का खतरा हद सकता है ।
िािी पीडा कद रदकते हुए, मानिीि और सम्मानजनक तरीके • र्िर्कत्सा नैर्िकिा: यचयकत्सा पेर्ा "क ई नुकसान न करें " के
से उनकी पीडा कद समाप्त करने का एक सािन प्रदान करता र्सद्धाोंि पर आधाररि है। आत्महत्या में सहािता करना इस यसद्धाों त
है । के यिरदिाभासी के रूप में दे खा जा सकता है , क्दोंयक इसमें
• िीिन की गुर्ििा: िह उन व्यल्कक्तिदों के यिए एक यिकल्प जानबूझकर मौत का कारण बनना िा जल्दबाजी करना र्ायमि है , जद
प्रदान कर सकता है जद महसूस करते हैं यक उनके जीिन की उपचारकतान और दे खभाि करने िािदों के रूप में स्वास्थ्य सेिा
गुणििा असहनीि हद तक खराब हद गई है । प्रदाताओों की पारों पररक भूयमका के ल्कखिाफ जाता है।
• सोंसाधन आिोंटन: िह सोंभायित रूप से पररिारदों और • प्रशामक िे खर्ाि (Palliative Care) के र्िकल्प: आिदचकदों का
स्वास्थ्य दे खभाि प्रणायििदों पर यििीि बदझ कद कम कर तकन है यक इिामृत्यु का यिकल्प प्रदान करने के बजाि, प्रर्ामक
सकता है । दे खभाि सेिाओों में सुिार पर अयिक जदर यदिा जाना चायहए।

105
सोंस्थागि र्सफाररशें:
• र्ारि के र्िर्ध आय ग ने अपनी 196िी ों ररप टि में आत्महत्या बनाम इच्छामृत्यु:
यसफाररर् की र्ी यक असाध्य रूप से बीमार रदयगिदों, जद • आत्महत्या और इिामृत्यु िैिाररक रूप से अिग हैं । चाहे चदट पहुों चाकर, नर्ा
यचयकत्सा उपचार, कृयत्रम पदषण िा हाइडर े र्न से इनकार करके िा यकसी भी तरीके से, एक व्यल्कक्त ने आत्महत्या कर िी। इस प्रकार,
आत्महत्या करने िािे व्यल्कक्त की ओर से जानबूझकर यकिा गिा कृत्य है ।
करते हैं , कद र्ारिीय िों ड सोंर्हिा की धारा 309 से बिाने
के र्िए एक र्िर्ध बनाया िाना िार्हए।
• दू सरी ओर, इिामृत्यु में यकसी अन्य व्यल्कक्त के अल्कस्तत्व कद रदकने के यिए उपाि
करना र्ायमि हदता है ।
आगे की राह: क्ा र्ारिीय सोंर्िधान के अनुच्छेि 21 के िहि िीने के अर्धकार में मरने का
• सािििर्नक प्रििन और र्शक्षा: सािनजयनक यर्क्षा अर्धकार र्ी शार्मि है?
अयभिान, बहस और मोंच जागरूकता बढाने, समझ कद • पी रर्थनम बनाम र्ारि सोंघ (1994): न्यािपायिका ने इस बात पर बहस की
बढािा दे ने और जीिन के अोंत के यिकल्पदों से सोंबोंयित यक आत्महत्या के प्रिास (आईपीसी की िारा 309) के यिए सजा सही र्ी िा
यचोंताओों कद दू र करने में मदद कर सकते हैं । गित।

• कानूनी ढाोंिा: एक व्यापक कानूनी ढाों चा स्र्ायपत करना  इस मामिे में उच्चतम न्यािािि ने कहा यक मरने की आजादी जीने की
आजादी के अोंतगनत आती है । इस प्रकार, आईपीसी की धारा 309 क
आिश्यक है जद इिामृत्यु तक पहुों च की र्तों कद रे खाों यकत
सोंिैिायनक रूप से अमान्य माना गिा।
करता है ।
• र्ियान कौर बनाम पोंिाब राज् (1996): उच्चतम न्यािािि(एससी) ने कहा
• सुरक्षा उपाय और सोंरक्षर्: इिामृत्यु से सोंबोंयित यकसी
यक सोंयििान में यनयहत जीिन का अयिकार मरने के अयिकार कद अयनिािन नहीों
भी र्िर्ध में कमजदर व्यल्कक्तिदों की सुरक्षा के यिए मजबूत करता है क्दोंयक आत्महत्या िा यकसी के मरने का यिकल्प चुनना यकसी की
सुरक्षा उपाि र्ायमि हदने चायहए। यजोंदगी कद खत्म करने का एक अप्राकृयतक तरीका है ।
• र्िर्कत्सा नैर्िकिा और व्यािसार्यक र्िशार्निे श: • उच्चतम न्यािािि (एससी) ने िारा 309 की िैिता िापस िे िी और आत्महत्या
स्वास्थ्य दे खभाि यचयकत्सकदों के यिए यदर्ायनदे र् और के प्रिास कद यफर से अपराि बना यदिा।
प्रदटदकॉि यिकयसत करने के यिए नीयत यनमान ताओों,
यचयकत्सा पेर्ेिरदों और नैयतकता सयमयतिदों के बीच सहिदग आिश्यक है ।
• प्रशामक िे खर्ाि: प्रर्ामक दे खभाि यिकल्पदों कद बढाना प्र . भारतीि न्यािपायिका इिामृत्यु के मुद्े कद कैसे दे खती और सोंबदयित करती
इिामृत्यु पर यिचार करने िािदों के यिए िैकल्कल्पक यिकल्प है , और भारत में इिामृत्यु के सोंबोंि में प्रमुख कानूनी यिचार और यनणनि क्ा हैं ?
प्रदान कर सकता है ।

8.2 र्िि िनसों िा र्ििस और र्ारि की िनसों िा नीर्ि


सोंिर्ि:
हाि ही में 11 िुिाई क र्िि िनसोंिा र्ििस मनािा गिा।
उद्दे श्य: िनसोंिा र्नयोंिर् की िात्कार्िकिा की ओर िनिा का ध्यान आकयषनत करना और उन्हें प्राकृयतक पिानिरण के सार् सद्भाि और
एकता कद यफर से स्र्ायपत करने के यिए यमिकर काम करने के यिए प्रदत्सायहत करना।
र्ारि की िनसोंिा प्रिृर्ि का र्िश्लेषर्
• सोंयुक्त रािर िनसोंिा क ष (यूएनएिपीए) की एक नई ररपदटन के अनुसार, अप्रैि 2023 में भारत 1.4 अरब से अर्धक आबािी के
साथ िीन क पछाड़कर िु र्नया का सबसे अर्धक आबािी िािा िे श बन गया है।
 र्ारि की िगभग 68% आबादी 15-64 साि की श्रेणी में है , और 26 प्रयतर्त 10-24 साि के समूह में है , जद भारत कद िु र्नया के
सबसे युिा िे श ों में से एक बनािा है।
 बुिुगि आबािी का प्रयतर्त 1991 में 6.8% से बढकर 2016 में 9.2% हद गिा है।
• िनसोंिा िृम्भद्ध की धीमी िर के कारक: गभनयनरदिक तरीकदों का बढता उपिदग, गभनिारण में अोंतर, स्वास्थ्य दे खभाि तक पहुोंच और
पररिार यनिदजन कद प्रदत्साहन, के अिािा यर्क्षा और आि में िृल्कद्ध ने जनसोंख्या िृल्कद्ध दर कद िीमी करने में िदगदान यदिा है
• र्ारि में प्रिनन िर में र्गरािट: र्ारि सरकार द्वारा िार्षिक रूप से आय र्िि नमूना पोंिीकरर् प्रर्ािी सिेक्षर् के अनुसार , 2011
से 2020 तक जन्म दर में िगातार यगरािट आई है ।
 पाोंििें और निीनिम रािरीय पररिार स्वास्थ्य सिेक्षर् के आों कडदों के अनुसार, 2019-21 के यिए, भारत की प्रजनन दर र्गरकर 2.0
ह गई है, अथिा ि िन्म प्रर्ि र्ििार्हि मर्हिा।
 िह 2.1 के प्रर्िस्थापन स्तर, से नीिे है, प्रिनन क्षमिा का अर्िा िह दर यजस पर एक पीढी खुद कद प्रयतस्र्ायपत करती है ।
• हािाँयक, दे र् में िदगदों की सोंख्या अभी भी अगिे कुछ दर्कदों तक बढने की उम्मीद है , जद 2064 िक 1.7 र्बर्ियन के अपने िरम पर
पहुोंि िाएगी।
106
उच्च िनसोंिा से सोंबोंर्धि र्िोंिाएाँ :
• प्रर्िकूि आर्थिक पररर्ाम: बडे पैमाने पर गरीबी, बढती
असमानता और व्यापक बेरदजगारी और अल्परदजगार।
 सिि र्िकास एक िुनौिी बन िािा है क्दोंयक
अत्ययिक सोंसािनदों का ददहन, पिान िरण का ह्रास और
यनम्नीकरण सभी सीिे तौर पर अयिक जनसोंख्या से
सोंबोंयित हैं ।
• शासन िुनौिी: जीिन प्रत्यार्ा में िृल्कद्ध और इसके
पररणामस्वरूप बुजुगन आबादी में िृल्कद्ध के कारण,
स्वास्थ्य िे खर्ाि और सामार्िक सुरक्षा िागि में
िृम्भद्ध हुई है।
• नकारात्मक सामार्िक पररर्ाम:
 श्म बि में मर्हिाओों की कम र्ागीिारी: यिश्व बैंक
के आों कडदों से पता चिता है यक भारत में मयहिा श्रम
भागीदारी 2005 में 32 प्रयतर्त से यगरकर 2021 में
19 प्रयतर्त हद गई है ।
 गर्ि र्नर धक ों का कम उपय ग: िगभग आिे
भारतीि अभी भी गभनयनरदिक के आिुयनक तरीकदों
का उपिदग नहीों करते हैं । गभनयनरदिक उपिदग का
बदझ मर्हिाओों पर असमान रूप से पड़िा है,
जबयक कोंडदम का उपिदग अस्वीकािन रूप से 10
प्रयतर्त से भी कम है ।
• अयिक जनसोंख्या के कारण िािीय अल्पसोंिक ों और प्रिार्सय ों की पहिान ख ने का डर र्ी उत्पन्न ह सकिा है।
 िह साोंप्रिार्यकिा क बढ़ािा िे सकिा है और िमन और मूि स्र्ान के आिार पर सामायजक ताने-बाने में दरार पैदा कर सकता है ।
• पयाििरर् और स्वास्थ्य क खिरा: अयिक जनसोंख्या और उसके पररणामस्वरूप पिानिरणीि क्षयत के कारण, सोंक्रामक रदग यिकयसत
हदते हैं और आसानी से फैिते हैं । जैसे:कदयिड-19
 कुपदषण और कई बीमाररिाँ र्नम्न िीिन स्तर के कारर् ह िी हैं।
• प्रिनन क्षमिा में क्षेिीय र्र्न्निाएाँ : उिर प्रदे र् और यबहार जैसे बडे जनसोंख्या िािे राज्यदों में प्रजनन दर क्रमर्ः 2.35 और 2.98 है , जद
राष्टरीि औसत से अयिक है ।
र्ारि में िनसोंिा क र्नयोंर्िि करने के र्िए उठाए गए किम:
• प्रथम पोंििषीय य िना: 1952 में भारत जनसोंख्या यनिोंत्रण कािनक्रम िागू करने िािा यिश्व का पहिा दे र् बना। इसमें प्राकृयतक पररिार
यनिदजन उपकरणदों के उपिदग पर जदर यदिा गिा।
रािरीय िनसोंिा नीर्ि:

रािरीय िनसोंिा नीर्ि, 1976 रािरीय िनसोंिा नीर्ि, 2000


• यििाह की आिु िडकी 15-18 िषि और िडकदों के यिए 18 से • भारत की प्रजनन और बाि स्वास्थ्य आिश्यकताओों कद पूरा
बढाकर 21 िषन यकिा। करने और 2010 तक टीएफआर हायसि करने के यिए र्िर्र्न्न
• राष्टरीि सोंसद में प्रयतयनयित्व के सार्-सार् राज्यदों कद केंिीि रर्नीर्िय ों के िक्ष्य ों और प्राथर्मकिाओों क आगे बढ़ाने के
सहािता के आिोंटन, करदों के हस्ताों तरण आयद के प्रिदजनदों के यिए यिए एक नीयतगत ढाोंचा प्रदान यकिा।
िषि 2001 िक िनसोंिा के आों कड़ ों क 1971 के स्तर पर • उद्दे श्य: मातृ स्वास्थ्य, बाि उिरजीयिता और गभनयनरदिक से
म्भस्थर रखें। सोंबोंयित यियभन्न मुद्दों का समािान करना, सार् ही प्रजनन स्वास्थ्य
• िड़र्कय ों की र्शक्षा पर अयिक ध्यान। दे खभाि कद सभी के यिए अयिक सुिभ और यकफािती बनाना।
• पररिार यनिदजन कािनक्रम में सरकार के सभी मोंत्राििदों/यिभागदों • उद्दे श्य
की भागीदारी।  अस्थायी उद्दे श्य: जन्म यनिोंत्रण उपकरणदों का प्राििान एक
• नसबोंिी हेिु आर्थिक मुआििे में िृल्कद्ध। अस्र्ािी िक्ष्य के रूप में र्ायमि यकिा गिा र्ा। इसमें
107
• स्र्ानीि स्तर पर िदगदों का प्रयतयनयित्व करने िािे यियभन्न सोंगठनदों स्वास्थ्य-सुरक्षा ढाोंचे का यिकास और स्वास्थ्य दे खभाि
और यनकािदों के यिए प्र त्साहन के रूप में समूह पुरस्कार ों की कयमनिदों की भती भी र्ायमि र्ी।
स्र्ापना।  मध्यािर्ध उद्दे श्य: 2010 तक कुि प्रजनन दर (टीएफआर)
• र्दि पर अयिक ध्यान। कद घटाकर 2.1 करना र्ा, जद प्रयतस्र्ापन स्तर र्ा।
• पररिार यनिदजन की स्वीकािनता बढाने के यिए, यिर्ेष रूप से  िीघिकार्िक उद्दे श्य: िक्ष्य 2045 तक जनसोंख्या
ग्रामीण क्षेत्रदों में, प्रेरक मीर्डया का अयिक से अयिक उपिदग। ल्कस्र्रीकरण प्राप्त करना है ।
 आर्थिक, सामार्िक और पयाििरर्ीय र्िकास और सुरक्षा
के मामिे में सामोंजस्यपूणन हद।
रािरीय िनसोंिा नीर्ि 2000: प्रमुख उपिम्भब्धयााँ
• कुि प्रिनन िर 2005 में 2.9 से घटकर 2019 में 2.0 (एसआरएस) हद गई है ।
• 36 में से 28 राज्य/केंिर्ायसत प्रदे र् पहिे ही 2.1 या उससे कम का प्रर्िस्थापन स्तर प्रिनन क्षमिा हार्सि कर िुके हैं ।
• 2005 से 2018 (एसआरएस) तक अर्दयित जन्म दर 23.8 से घटकर 20.0 हद गई है ।

• िनसोंिा र्नयोंिर् र्िधेयक, 2019


 इसने प्रर्ि ि ड़े के र्िए ि बच्च ों की नीर्ि का प्रस्ताि रखा और इसका उद्े श्य र्ैयक्षक िाभ, मुफ्त स्वास्थ्य सेिा, बेहतर रदजगार के
अिसर, गृह ऋण और कर कटौती के माध्यम से इसे अपनाने के यिए प्रदत्सायहत करना र्ा।
 2022 में यह र्बि िापस िे र्िया गया: बच्चदों की सोंख्या कद यनिोंयत्रत और यियनियमत करने की नीयत अनुिेद 16 (सािनजयनक रदजगार
के मामिदों में समान अिसर) और अनुच्छेि 21 (िीिन और स्विोंििा की सुरक्षा) िैसे सोंिैधार्नक अर्धकार ों का उल्लोंघन करिी
है।

िनसोंिा क र्नयोंर्िि करने की सरकारी य िना


• गर्ि र्नर धक ों की ह म र्डिीिरी य िना: इसका उद्े श्य गभन यनरदिकदों की घर-घर यडिीिरी के यिए आर्ा कािनकतानओों कद सोंगयठत
करना है ।
• िन्म के समय अोंिर सुर्नर्िि करना य िना: आर्ा कािनकतान नियििायहत जदडदों कद परामर्न प्रदान करती हैं।
• र्मशन पररिार र्िकास: इसका उद्े श्य कुछ उच्च प्रजनन क्षमता िािे यजिदों में गभन यनरदिकदों और पररिार यनिदजन सेिाओों तक पहुोंच
बढाना है ।
• म्भिर्नकि आउटरीि टीम य िना: िह दू र-दराज के क्षेत्रदों में मान्यता प्राप्त सोंगठनदों की मदबाइि टीमदों के माध्यम से पररिार यनिदजन
सेिाएों प्रदान करती है ।
• रािरीय पररिार र्नय िन क्षर्िपूर्िि य िना: िह नसबोंदी के बाद मृत्यु, जयटिता और यिफिता के मामिे में अपने ग्राहकदों का बीमा करती
है ।
अन्य िे श ों के िनसोंिा र्नयोंिर् कानून
राज् सरकार की र्ूर्मका • िीन: इसने पररिारदों के यिए एक-बच्चा नीयत(one child policy)
• असम: असम की िनसोंिा और मर्हिा सशम्भक्तकरर् स्र्ायपत करने का बीडा उठािा है ।
नीर्ि, 2017 • जनसोंख्या के आकार कद उपिब्ध सोंसािनदों से यमिाने के िक्ष्य के सार्
 केिि दद बच्चदों िािे उम्मीदिार ही सरकारी रदजगार के पररिार र्नय िन अर्र्यान शुरू र्कया।
यिए पात्र हदोंगे और मौजूदा सरकारी कमनचाररिदों कद दद • उच्च मृत्यु दर के कारण िनसोंिा में र्गरािट िे खी िा रही है, ि
बच्चदों िािे पररिार के मानदों ड का पािन करने का यनदे र् सोंर्ििः ििाओों और शराब िैसे कारक ों के कारर् है।
यदिा गिा र्ा • सामार्िक प्रगर्ि और र्िकास पर 1969 की घ षर्ा का अनुच्छेि
• उिर प्रिे श का र्िर्ध आय ग, 2021 एक प्रस्ताि िेकर 22: िह सुयनयित करता है यक जदडदों कद स्वतोंत्र रूप से और यजम्मेदारी
आिा है , जहाों दद से अयिक बच्चे रखने िािे यकसी भी व्यल्कक्त से िह चुनने का अयिकार है यक उनके यकतने बच्चे हदोंगे।
कद सरकारी सल्किडी प्राप्त करने से रदक यदिा जाएगा।

आगे की राह
• सरकार की र्ूर्मका: सरकार कद अयिक आबादी िािे राज्यदों में उच्च प्रिनन क्षमिा िािे र्िि ों पर ध्यान केंर्द्रि करना िार्हए।
 भारत सरकार ने 146 उच्च-प्रजनन िािे यजिदों की पहचान की है , जद अयिकाों र् उिर भारतीि राज्यदों यबहार, झारखोंड, मध्य प्रदे र्,
राजस्र्ान और उिर प्रदे र् में हैं।

108
• मर्हिाओों का सशम्भक्तकरर्: अयिक िैंयगक समानता िािे समाजदों में प्रजनन दर कम हदती है और जनसाोंल्कख्यकीि सोंकेतक बेहतर हदते
हैं ।
 एनएफएचएस 5 बताता है यक जद िडयकिाों अयिक समि तक पढाई करती हैं उनके बच्चे कम हदते हैं और िे रदजगार भी अपनाती हैं ।
 जैसे-जैसे मयहिाएँ अयिक सर्क्त हदती जाती हैं , उन्हें समाज के सदस्यदों के रूप में बेहतर महत्व यदिा जाता है , यजससे एक िडके की
आिश्यकता कम हद जाती है । हररिाणा में बेटी बिाओ, बेटी पढ़ाओ अर्र्यान की सिििा इसका उदाहरण है ।
• क्षमिा र्िकास: भारत की कुि जनसोंख्या का दद-यतहाई यहस्सा 15 से 64 िषि की आयु के बीि है।
 ई- र्शक्षा, कौशि र्िकास और अिसर पैिा करना, यिर्ेष रूप से िोंयचत िगों के िुिाओों और मयहिाओों के यिए, अगिे 20 िषों में
अपने िाभ के यिए जनसाोंल्कख्यकीि िाभाों र् का उपिदग करने में दे र् की कुोंजी हदगी।
• िर्क्षर्ी राज् ों की सिििा का अनुकरर्: पाों च दयक्षणी राज्य प्रजनन क्षमता कद कम करने में सफि रही, िह पारों पररक ज्ञान कद खाररज
करता है यक जनसोंख्या ल्कस्र्रीकरण के यिए साक्षरिा, र्शक्षा और र्िकास की आिश्यकिा है।
 िर्क्षर्ी सरकार ों ने सर्क्रय रूप से पररिारदों कद केिि दद बच्चे पैदा करने के यिए प्रदत्सायहत यकिा, यजसके बाद नसबोंदी की गई।
 पुरुष नसबोंिी, जद मयहिा नसबोंदी से कहीों अयिक सुरयक्षत है , कद राष्टरीि और राज्य नीयतिदों द्वारा बढािा यदिा जाना चायहए।
• पररिार र्नय िन व्यय बढ़ाएाँ : ियद पररिार यनिदजन नीयतिदों कद सयक्रि रूप से प्रार्यमकता दी जाए तद भारत का प्रयत व्यल्कक्त सकि घरे िू
उत्पाद 2031 तक अयतररक्त 13% बढ सकता है ।
 इससे 2.9 र्मर्ियन र्शशु मृत्यु और 1.2 र्मर्ियन मािृ मृत्यु क र का िा सकता है और पररिारदों कद प्रसि और बच्चे के अस्पताि
में भती हदने पर अपनी जेब से हदने िािे स्वास्थ्य व्यि से 77,600 करदड रुपिे (20%) की बचत हद सकती है ।

8.3 र्ारि में प्रशामक िे खर्ाि क सु दृ ढ़ बनाना


सोंिर्ि:
हाि ही में, केंि सरकार ने दे र् की आबादी बढने के कारण ऐसी बीमाररिदों के बढते बदझ कद स्वीकार करते हुए गैर-सोंिारी र ग ों (एनसीडी)
के प्रबोंधन के र्िए पररिािन र्िशार्निे श िारी र्कए हैं।
• िह एनसीडी से प्रभायित व्यल्कक्तिदों के यिए उपर्ामक दे खभाि सेिाओों की आिश्यकता पर जदर दे ता है ।
प्रशामक िे खर्ाि के बारे में:
• प्रर्ामक दे खभाि यचयकत्सा की एक र्िशेष शाखा है ि िीिन की गुर्ििा बढ़ाने और िीिन सीर्मि करने िािी बीमाररय ों िािे
व्यल्कक्तिदों की पीडा कद कम करने पर केंयित है ।
• दृर्िक र्:
 व्यापक दृर्िक र्: मरीि ों की शारीररक, मन िैज्ञार्नक, आध्याम्भत्मक और सामार्िक आिश्यकिाओों क सोंबदयित करके उनकी
भिाई में सुिार करें ।
 आनुपार्िक कारि िाई: जीिन की गुणििा कद प्रार्यमकता दे ते हुए अयत-यचयकत्साकरण और पररिारदों पर यििीि बदझ कद रदकें।
 परामशि सहायिा: रदगी की मृत्यु के बाद र्दक सहािता सयहत पररिारदों और दे खभाि करने िािदों कद सहािता प्रदान करता है ।
• प्रय ज्िा: हृदि खराब हदना, गुदे खराब हदना, कुछ तोंयत्रका सोंबोंिी रदग, कैंसर, आयद।
र्ारि में प्रशामक िे खर्ाि की आिश्यकिा:
• गैर-सोंिारी र ग ों का ब झ: कैंसर, मिुमेह, उच्च रक्तचाप और श्वसन रदगदों जैसी बीमाररिदों के प्रसार में भारी िृल्कद्ध दे खी गई है।
• उच्च कैंसर घटनाएाँ : भारत में हर साि िगभग 1.4 यमयििन िदगदों में कैंसर का पता चिता है , इसयिए प्रर्ामक दे खभाि सेिाओों की पिानप्त
आिश्यकता है ।
• ग्रामीर् स्वास्थ्य िे खर्ाि आिश्यकिाओों क सोंब र्धि करना: भारत की दद-यतहाई आबादी ग्रामीण क्षेत्रदों में रहती है , जहाों उपर्ामक
दे खभाि सयहत स्वास्थ्य सेिाओों तक पहुों च सीयमत हद सकती है।
• िु र्नया र्र में बुिुगों की आबािी में िृम्भद्ध: WHO के अनुसार, 2030 तक, 6 में से 1 व्यल्कक्त की उम्र 60 िा उससे अयिक हदने पर, प्रर्ामक
दे खभाि सेिाओों की महत्वपूणन माों ग हदगी।
 इस समि 60 साि और उससे अयिक उम्र की आबादी का यहस्सा 2020 में 1 अरब से बढकर 1.4 अरब हद जाएगा।
प्रशामक िे खर्ाि के र्िए मौिूिा कायिक्रम में िुनौर्ियााँ:
• गैर-सोंिारी र ग ों की र कथाम और र्नयोंिर् के र्िए रािरीय कायिक्रम (एनपी-एनसीडी):
 एनपी-एनसीडी क गैर-सोंचारी रदगदों (एनसीडी) के बदझ से यनपटने के यिए 2010 में िॉन्च यकिा गिा र्ा।

109
 इसे मूि रूप से एनपीसीडीसीएस कहा िािा है, इसका उद्दे श्य कैंसर, मधुमेह, हृिय र ग और स्टर क िैसी पुरानी बीमाररिदों के
यिए प्रदत्साहन, यनिारक और उपचारात्मक दे खभाि प्रदान करना है ।
 एक आदर्न पररदृश्य में, उन्नत चरण की बीमाररिदों के यिए उपर्ामक दे खभाि कद उपचारात्मक दे खभाि की जगह िे िेनी चायहए।
सोंश र्धि एनपी-एनसीडी र्िशार्निे श ों में कर्मयााँ:
प्रशामक िे खर्ाि की अन्य िुनौर्ियााँ:
• गैर-कैंसर बीमाररय ों पर सीर्मि ि कस:
• सीर्मि पहुोंि: िह बडे पैमाने पर र्हरी क्षेत्रदों में तृतीिक स्वास्थ्य सुयििाओों
 यदर्ायनदे र् मुख्य रूप से कैंसर के सोंबोंध में उपशामक
पर उपिब्ध है ।
िे खर्ाि का उल्लेख करिे हैं , गैर-कैंसर बीमाररिदों में
 सेिाओों की इस यिषम उपिब्धता के कारण, दे र् में अनुमायनत 7-10
उपर्ामक दे खभाि की उच्च आिश्यकता कद नजरअोंदाज र्मर्ियन ि ग ों में से केिि 1-2% ि ग ों िक ही इसकी पहुोंि है,
करते हैं। र्िन्हें इसकी आिश्यकिा है।
 2020 में प्रशामक िे खर्ाि के िैर्िक एटिस ने कैंसर • ख़राब कायाि न्वयन: बुिुगों की स्वास्थ्य िे खर्ाि के र्िए रािरीय
से परे प्रर्ामक दे खभाि के व्यापक दािरे पर जदर यदिा। कायिक्रम जैसे सरकारी कािनक्रमदों की उपल्कस्र्यत के बािजूद , उपर्ामक
• पुरानी और िु बिि करने िािी म्भस्थर्िय ों का बर्हष्कार: दे खभाि तक पहुों च अपिानप्त है ।
 र्पछिे पररिािन र्िशार्निे श ों (2013-2017) में, पुरानी • गैर सरकारी सोंगठन ों की र्ूर्मका: पैयििम इों यडिा, करुणाश्रि और
और दु बनि करने िािी ल्कस्र्यतिदों कद प्रर्ामक दे खभाि के कैनसपदटन जैसे गैर सरकारी सोंगठन इस अोंतर कद भरने की कदयर्र् करते
अोंतगनत र्ायमि यकिा गिा र्ा। हैं िेयकन सरकार की िुिना में सोंसाधन की कमी के कारर् उन्हें
सीमाओों का सामना करना पड़िा है।
 सोंर्दयित यदर्ायनदे र् इन ल्कस्र्यतिदों कद र्ायमि करने में
• उपेर्क्षि बाि र्िर्कत्सा प्रशामक िे खर्ाि: ये यदर्ायनदे र् मुख्य रूप से
यिफि हैं , यजससे उपर्ामक दे खभाि का दािरा सीयमत हद
कैंसर पर ध्यान केंयित करते हैं , यियभन्न पुरानी बीमाररिदों िािे बच्चदों के यिए
गिा है ।
प्रर्ामक दे खभाि की तत्काि आिश्यकता कद नजरअोंदाज करते हैं।
• घर-आधाररि िे खर्ाि पर ि र का अर्ाि:
 यपछिे यदर्ायनदे र्दों में घर-आिाररत उपर्ामक दे खभाि सेिाओों के महत्व कद स्वीकार यकिा गिा र्ा।
 सोंर्दयित यदर्ायनदे र्दों में घर-आिाररत दे खभाि की अनदे खी की गई है , यजसमें उपर्ामक दे खभाि यितरण केिि यजिा अस्पताि से
र्ुरू हदता है ।
• प्राथर्मक स्वास्थ्य िे खर्ाि स्तर पर अपयािप्त र्ििरर्:
 सोंर्दयित यदर्ायनदे र्दों में प्रर्ामक दे खभाि सेिा यितरण स्वास्थ्य और कल्याण केंि और उप-केंि स्तरदों कद दरयकनार करते हुए केिि
यजिा अस्पताि से र्ुरू हदता है।
गैर-सोंिारी र ग (एनसीडी) के बारे में:
• एनपीपीसी के कायािन्वयन में कर्मयााँ:
• गैर-सोंचारी रदग (एनसीडी), यजन्हें पुरानी बीमाररिदों के रूप में भी जाना
 िे यदर्ायनदे र् गैर-सोंचारी रदगदों (एनसीडी) के यिए जाता है , िोंबी अियि के हदते हैं और आनुिोंयर्क, र्ारीररक, पिानिरणीि
सेिाओों के अयभसरण कद बढािा दे ने के यिए 11 और व्यिहार सोंबोंिी कारकदों के सोंिदजन का पररणाम हदते हैं ।
कािनक्रमदों के यिोंकेज पर प्रकार् डािते हैं । • िार प्रमुख एनसीडी हैं : हृदि रदग (सीिीडी), कैंसर, पुरानी श्वसन रदग
 इन कािनक्रमदों में से एक है नेशनि प्र ग्राम िॉर (सीआरडी) और मिुमेह जद चार व्यिहाररक जदल्कखम कारकदों कद साझा
पेर्िएर्टि केयर (एनपीपीसी)। करते हैं - अस्वास्थ्यकर आहार, र्ारीररक गयतयियि की कमी, और तोंबाकू
✓ एनपीपीसी की घदषणा 2012 में प्रर्ामक दे खभाि और र्राब का उपिदग।
सेिाओों कद बढाने के उद्े श्य से की गई र्ी। र्ारि में एनसीडी की म्भस्थर्ि:
 हािाँयक, समयपनत बजट और जागरूकता की कमी के • र्ारिीय र्िर्कत्सा अनुसोंधान पररषि (आईसीएमआर) के एक
कारण कािनक्रम कद चुनौयतिदों का सामना करना पडा है । अध्यिन में अनुमान िगािा गिा है यक र्ारि में एनसीडी के कारण हदने
िािी मौतदों का अनुपात 1990 में 37.9% से बढकर 2016 में 61.8% हद
आगे की राह: गिा है ।
• र्िि स्वास्थ्य सर्ा सोंकल्प: राष्टरीि स्वास्थ्य नीयतिदों में • र्िि स्वास्थ्य सोंगठन के अनुसार, 2019 में र्ारि में एनसीडी के
प्रर्ामक दे खभाि कद एकीकृत करके प्रर्ामक दे खभाि पर कारण 60.46 िाख से अयिक िदगदों की मृत्यु हद गई।
यिश्व स्वास्थ्य सभा 2014 कद िागू यकिा।
• स्वास्थ्य िे खर्ाि के बुर्नयािी ढाोंिे क मिबूि करना: जमीनी स्तर पर मजबूत उपर्ामक दे खभाि सेिाओों के यनमान ण में यनिेर् करने
की आिश्यकता है , यजससे िह सुयनयित हद सके यक सभी जरूरतमोंद व्यल्कक्तिदों के यिए गुणििापूणन दे खभाि सुिभ हद।
• प्रर्शक्षर् और र्शक्षा: उपर्ामक दे खभाि में स्वास्थ्य दे खभाि पेर्ेिरदों कद यिर्ेष प्रयर्क्षण कािनक्रम प्रदान करने से प्रयर्यक्षत कयमनिदों की
कमी कद दू र करने और सेिाओों की यडिीिरी बढाने में मदद यमिेगी।

110
• आिश्यक ििाओों िक पहुोंि: ददन प्रबोंिन के यिए ओयपओइड (नर्ीिे पदार्न) और अन्य आिश्यक दिाओों तक बेहतर पहुों च की सुयििा
प्रदान करना प्रभािी उपर्ामक दे खभाि के यिए महत्वपूणन है ।
• सहय ग और एकीकरर्: स्वास्थ्य दे खभाि पेर्ेिरदों, यिर्ेषज्ञदों और उपर्ामक दे खभाि टीमदों के बीच सहिदग कद मजबूत करने से रदगी
दे खभाि के यिए एक समल्कन्वत दृयष्टकदण और समर्नन की यनरों तरता सुयनयित हद सकती है ।
• सािििर्नक िागरूकिा और स्वीकृर्ि: मृत्यु कद जीिन के स्वाभायिक यहस्से के रूप में स्वीकार करने और जागरूकता कद बढािा दे ने से
उपर्ामक दे खभाि के यिए अयिक दिािु और सहािक िातािरण कद बढािा दे ने में मदद यमि सकती है ।
• पुनिािस और सामार्िक समथिन: रदयगिदों और उनके पररिारदों की समग्र आिश्यकताओों कद पहचानते हुए, उपर्ामक दे खभाि कािनक्रमदों
के भीतर पुनिान स और सामायजक सहािता सेिाओों कद एकीकृत करने से समग्र कल्याण में िृल्कद्ध हद सकती है ।

8.4 स शि स्टॉक एक्सिें ि

सोंिर्ि:
हाि ही में, सेबी ने बीएसई पर स शि स्टॉक एक्सिेंि (एसएसई) क एक अिग खोंड के रूप में पेश करने के यिए अपनी मोंजूरी दे दी
है । अब से, िगर्ग 20 र्िर्र्न्न सामार्िक सोंगठन ों ने नेशनि
र्ारि में सामार्िक उद्यम ों द्वारा धन िुटाना:
स्टॉक एक्सिेंि (एनएसई) और बॉम्बे स्टॉक एक्सिेंि
• ितनमान में, भारत में सामायजक-यिकास क्षेत्र कद कॉपोरे ट सामायजक
(बीएसई) के एसएसई प्लेटिॉमि के िहि पोंिीकरर् कराया
यजम्मेदारी (सीएसआर), परदपकार, सरकारी फोंयडों ग और खुदरा दान जैसे
है। कई स्रदतदों से िन प्राप्त हदता है ।
समािार के बारे में अर्धक िानकारी: • एक एसएसई फोंयडों ग, उपिदग, प्रभाि-यनमानण, माप, प्रकटीकरण और
• पूोंिी िुटाने के यिए सामार्िक उद्यम ों और स्वैम्भच्छक ररपदयटिं ग के समान ढाोंचे के सार् र्िर्र्न्न प्लेटिामों में सामोंिस्य िाने
का प्रयास करे गा।
सोंगठन ों की क्षमिा बढ़ाने के उद्े श्य से , भारत के यिि मोंत्री
ने 2019 में भारत-आिाररत एसएसई स्र्ायपत करने की िदजना की घदषणा की।

स शि स्टॉक एक्सिेंि (एसएसई) क्ा है:


• एसएसई मौिूिा स्टॉक एक्सिेंि के र्ीिर एक अिग
खोंड के रूप में कायि करिा है और सामार्िक उद्यम ों
कद अपने तोंत्र के माध्यम से जनता से िन जुटाने में मदद
करता है ।
• भारत-एसएसई एनएसई और बीएसई के िहि एक
अिग खोंड के रूप में कायि कर रहा है।
• एसएसई केिि उन प्रर्िर्ूर्िय ों क सू िीबद्ध करे गा
ि 'गैर-िार्कारी' या 'िार्कारी' 'सामार्िक
उद्यम 'ों के र्िए धन िुटािी हैं।
• इसका िक्ष्य पूोंिी बािार क िनिा के करीब िे िाना
और समािेर्ी यिकास और यििीि समािेर्न से सोंबोंयित
यियभन्न सामायजक कल्याण उद्े श्यदों कद पूरा करना है।
गैर-िार्कारी सोंगठन एसएसई पर धन िुटा सकिा है:
• सोंस्र्ागत यनिेर्कदों और गैर-सोंस्र्ागत यनिेर्कदों कद िीर कूपन िीर र्प्रोंर्सपि इों स्टूमेंट जारी करना। ZCZP बाोंड पारों पररक बाों ड से यभन्न
हदते हैं क्दोंयक इसमें पररपिता पर र्ून्य कूपन और कदई मूि भुगतान नहीों हदता है।
• सेबी द्वारा यनयदन ष्ट म्यूचुअि फोंड िदजनाओों के माध्यम से दान;
• सेबी द्वारा समि-समि पर यनयदन ष्ट कदई अन्य सािन।
सामार्िक उद्यम (एसई) के रूप में मान्यिा प्राप्त करने की पाििा:
• सामार्िक इरािा: कदई भी गैर-िाभकारी सोंगठन (एनपीओ) िा िाभकारी सामायजक उद्यम (एफपीएसई) स्थार्पि करिा है , एक
सामायजक उद्यम (एसई) जद इसे पोंजीकृत हदने िा सूचीबद्ध हदने के िदग्य बना दे गा। उसे सामायजक मोंर्ा की प्रिानता रूप में मान्यता दी
जाएगी,
• पोंिीकरर्: िह अयनिािन है यक जारी करने की सुयििा के यिए एनपीओ एसएसई के सार् पोंजीकृत हद।

111
• र्नर्िि कायिकाि: उपकरण का एक
यियर्ष्ट कािनकाि हदना चायहए और इसे
केिि एक यियर्ष्ट पररिदजना िा गयतयियि
के यिए जारी यकिा जा सकता है ।
• र्िशेषज्ञिा: इसे अतीत में इसी तरह की
पररिदजनाओों में अपने प्रदर्नन के माध्यम
से अपेयक्षत यिर्ेषज्ञता प्रदयर्नत करनी
चायहए।
• सेबी के आईसीडीआर (पूोंिी और
प्रकटीकरर् आिश्यकिाओों का मुद्दा)
र्िर्नयम, 2018 के यियनिमदों के तहत
सूचीबद्ध मानदों ड िह कहते हैं यक उद्यमदों
कद हदना चायहए
 भूख, गरीबी, कुपदषण और असमानता कद यमटाने के यिए
सेिा करना;
 यर्क्षा, रदजगार िदग्यता, समानता, मयहिाओों और
LGBTQIA+ समुदािदों के सर्ल्कक्तकरण कद बढािा दे ना;
 पिानिरणीि ल्कस्र्रता की यदर्ा में कािन करना;
 अन्य बातदों के अिािा, राष्टरीि यिरासत और किा का सोंरक्षण
िा यडयजटि यिभाजन कद पाटना।
 उनकी कम से कम 67% गयतयियििाँ बताए गए उद्े श्य कद
प्राप्त करने के यिए यनदे यर्त हदनी चायहए।
एसएसई सामार्िक उद्यम ों की कैसे मिि करे गा? उद्यम ों क एसएसई के रूप में मान्यिा नही ों िी िानी िार्हए:
• पूोंिी िक पहुोंिने में आने िािी बाधाओों क िू र करना: • कॉपोरे ट फाउों डेर्न, राजनीयतक िा िायमनक सोंगठन िा गयतयियििाँ, पेर्ेिर
एसएसई उन सामायजक उद्यमदों कद बहुत आिश्यक पूोंजी प्रदान िा व्यापार सोंघ, बुयनिादी ढाँचा और आिास कोंपयनिाँ (यकफािती आिास
करे गा जद अक्सर पारों पररक फोंयडों ग स्रदतदों तक पहुोंचने के यिए कद छदडकर) कद एसई के रूप में पहचाना नहीों जाएगा।

सोंघषन करते हैं । • ियद एनपीओ अपनी फोंयडों ग के 50% से अयिक के यिए यनगमदों पर यनभनर
हैं तद उन्हें अिदग्य माना जाएगा।
• सामार्िक व्यिसाय ों का समथिन करना: एसएसई सामायजक
उद्यमदों की दृश्यता बढाएगा, सामायजक रूप से प्रभािर्ािी उद्यमदों
सिोिम िैर्िक िै र्िक प्रथाएाँ :
का समर्नन करने में रुयच रखने िािे खुदरा और सोंस्र्ागत
• िूके का स शि स्टॉक एक्सिेंि सामायजक रूप से प्रभािर्ािी कोंपयनिदों
यनिेर्कदों ददनदों कद आकयषनत करे गा। की एक यनदे यर्का के रूप में कािन करता है , जद सोंभायित यनिेर्कदों कद
• र्निेशक आधार का र्िस्तार: एसएसई िगातार फोंयडों ग, उपिदग, दृश्यता प्रदान करता है ।
प्रभाि सृजन, माप, पारदयर्नता और ररपदयटिं ग प्रयक्रिाओों के सार् • कनाडा का स शि िेंिर कने क्शन एक "यिश्वसनीि कनेरर" के रूप
कई प्लेटफामों कद एकजुट करने का प्रिास करे गा। में कािन करता है , जद सामायजक व्यिसािदों कद इिु क प्रभािर्ािी
• र्निेशक ों के र्हि ों की सुरक्षा: िह सुयनयित करता है यक िायषनक यनिेर्कदों और सेिा प्रदाताओों से जदडता है ।
आिार पर सामायजक प्रभाि के अयनिािन प्रकटीकरण द्वारा
यनिेर्कदों के यहतदों की सुरक्षा की जाती है ।
• एक समानाोंिर सामार्िक अथिव्यिस्था का र्नमािर्: एसएसई नैयतक यनिेर्कदों कद अपने मूल्यदों के अनुरूप व्यिसािदों में यनिेर् करने की
अनुमयत दे ते हैं , यजससे एक समानाों तर सामायजक अर्नव्यिस्र्ा का यनमान ण हदता है ।
• िे ने की सोंस्कृर्ि का प्रसार: एसएससी से सामायजक पूोंजी के यनमान ण में अग्रणी भूयमका यनभाने की उम्मीद की जाती है । िह भारत में दान
दे ने की सोंस्कृयत का यनमान ण करे गा, यिर्ेष रूप से उन िदगदों कद प्रेररत करे गा जद दान दे ने में अयनिु क हैं।

112
र्नष्कषि:
• भारत में सदर्ि स्टॉक एक्सचेंज की स्र्ापना सामार्िक र्िि में एक पररिििनकारी र्िकास प्रिीक है, ि सामायजक व्यिसािदों कद
सर्क्त बनाता है और अत्ययिक आिश्यक पूोंजी कद आकयषनत करते हुए महत्वपूणन चुनौयतिदों का समािान करने के यिए यनिेर्कदों कद
प्रभायित करता है ।
• इस सोंबोंि में सरकार कद SSEs के यिए एक सक्षम िातािरण बनाने और भारत में सामायजक यिि कद बढािा दे ने के यिए SAMBHAV:
िार्कारी और प्रर्ािशािी उद्यम ों के र्िए सामार्िक रूप से सोंरेम्भखि बािार प्रिान करना िार्हए।

8.5 घरे िू पे य िि, सिाई िथा स्वच्छिा (िॉश) 2000-2022 पर प्रगर्ि


सोंिर्ि:
UNICEF और WHO, द्वारा जारी एक नई ररपदटन , "घरे िू पेयिि, सिाई और स्वच्छिा पर प्रगर्ि (WASH) 2000-2022: र्िोंग पर र्िशेष
ि कस" 2000 से 2022 की अियि के यिए घरदों में WASH के यिए अद्यतन राष्टरीि, क्षेत्रीि और िैयश्वक अनुमान प्रस्तुत करती है ।

WASH (िि, सिाई और स्वच्छिा) के बारे में:


• जि, सफाई और स्विता (WASH) एक र्ब्द है जद स्वि पानी, उयचत सफाई और अिी स्विता प्रर्ाओों तक पहुोंच कद बढािा दे ने के
उद्े श्य से पहि और प्रर्ाओों के एक समूह कद सोंदयभनत करता है ।
ररप टि से मुि र्नष्कषि:
• िि आपूर्िि का अर्ाि: यिश्व स्तर पर, 1.8 र्बर्ियन ि ग पररसर में जि आपूयतन के यबना घरदों में रहते हैं ।
• र्निी स्थान की कमी: उपिब्ध डे टा िािे 51 दे र्दों में, सबसे गरीब घरदों में मयहिाओों और यकर्दर िडयकिदों और यिकिाों ग िदगदों के पास
कपडे िदने और कपडे बदिने के यिए यनजी स्र्ान की कमी हदने की सबसे अयिक सोंभािना है।
• िीिन की हार्न: अपिानप्त जि, स्विता और साफ-सफाई के कारण हर साि 1.4 यमयििन िदगदों की जान चिी जाती है।
• मर्हिाओों पर असोंगि प्रर्ाि: 15 िषन और उससे अयिक उम्र की मयहिाएों और िडयकिाों मुख्य रूप से 10 में से 7 घरदों में जि सोंग्रह के
यिए यजम्मेदार हैं , जबयक 10 में से 3 घरदों में उनके पुरुष सहकमी यजम्मेदार हैं ।
 र्शक्षा का नुकसान: घरे िू कामदों पर खचन यकिा गिा अयतररक्त समि िडयकिदों की माध्ययमक यिद्यािि की पढाई पूरी करने और
रदजगार पाने की सोंभािनाओों कद भी सीयमत कर सकता है ।
 िीिन की खराब गुर्ििा: मयहिाएों और िडयकिाों इसे इकट्ठा करने के यिए िों बी िात्राएों करती हैं , काम और आराम के यिए समि
बबानद करती हैं , और खुद कद र्ारीररक चदट और खतरदों के जदल्कखम में डािती हैं ।
 गरीबी के िु ष्िक्र: घर में असुरयक्षत पानी, र्ौचािि और िि एिों स्वच्छिा का अर्धकार:
हार् िदना िडयकिदों से उनकी क्षमता छीन िेता है , उनकी • सोंिुक्त राष्टर ने िदगदों के स्वि और उपिुक्त र्ौचाििदों के अयिकार कद
भिाई से समझौता करता है और गरीबी के चक्र कद कािम सुदृढ करने के यिए आयिकाररक तौर पर पानी और स्वच्छिा िक पहुोंि
रखता है । क एक मानि अर्धकार घ र्षि र्कया है।
 स्वास्थ्य ि म्भखम: अपिान प्त WASH सेिाएँ मयहिाओों और • न केिि बेहतर स्वास्थ्य पररणामदों के यिए, बल्कि समग्र मानि गररमा के
िडयकिदों के यिए स्वास्थ्य जदल्कखम बढाती हैं और उनकी यिए िि, सिाई और स्वच्छिा (WASH) के महत्व कद स्र्ायपत यकिा।
मायसक िमन कद सुरयक्षत और यनजी तौर पर प्रबोंयित करने
की क्षमता कद सीयमत करती हैं।
 बढ़ी हुई कमि ररयााँ: उन्हें अयतररक्त स्वास्थ्य जदल्कखमदों का सामना करना पडता है क्दोंयक जब उन्हें पानी िाने के यिए घर से बाहर
जाना पडता है तद उन्हें उत्पीडन, यहों सा और चदट िगने का खतरा हदता है ।

र्ारि में स्वास्थ्य और स्वच्छिा की म्भस्थर्ि:


• खराब पहुोंि: 2020 तक, 165 र्मर्ियन से अर्धक र्ारिीय ों के पास साफ पानी तक पहुोंच नहीों र्ी, 600 यमयििन पानी की कमी िािे
क्षेत्रदों में रहते र्े, और 63% घरदों में पानी नहीों र्ा।
• पानी की कमी िािे क्षेि: भारत में पानी की कमी िािे क्षेत्र जद सरकार और नगर यनगम के पानी के टैं करदों पर यनभनर हैं , उन्हें प्रयत व्यल्कक्त
प्रयत यदन अयिकतम 25 िीटर अर्िा पाोंच िदगदों के पररिार के यिए 125 िीटर पानी यमिता है । इसके यिपरीत, औसत और उच्च आि िािे
पररिार प्रर्ि व्यम्भक्त प्रर्ि र्िन 200L-600L का उपय ग करिे हैं।
• िि प्रिू षर्: भारत का िगभग 70% ताज़ा पानी दू यषत है ।
• स्टों र्टों ग और िेम्भस्टोंग: अस्वास्थ्यकर आजीयिका के जदल्कखमदों कद कम करने के प्रिासदों के बािजूद, भारत में अभी भी खराब स्विता के
कारण पाोंच साि से कम उम्र के बच्चदों में स्टों र्टों ग और िेम्भस्टों ग की िर अर्धक है।
113
 एनएफएचएस-5 (2019-2021) डे टा के अनुसार, भारत में सिाई और स्वच्छिा में सुधार के र्िए र्ारि द्वारा िििमान
िेल्कस्टोंग का कुि प्रसार 18.7% और स्टों यटों ग का 35.5% हदने का उपाय:
अनुमान है । • प्रमुख कायिक्रम: जि जीिन यमर्न (जेजेएम) और स्वि भारत
• डे टा र्िसोंगर्ियााँ: हािाँ यक भारत कद 2019 में खुिे में र्ौच-मुक्त यमर्न (एसबीएम) का िक्ष्य भारत में हर घर कद सुरयक्षत और पिानप्त
(ODF) घदयषत यकिा गिा र्ा, कई ररपदटों से पता चिता है यक ODF पेिजि उपिब्ध कराना है ।
क्षेत्रदों में खुिे में र्ौच जारी है , जहाँ बडी सोंख्या में यनल्किि र्ौचािि • अटि र्ूिि य िना: इसमें ग्राम पोंचाित स्तर पर जि उपिदग सोंघदों
हैं । (डब्ल्यूिूए) में 33 प्रयतर्त मयहिा सदस्यदों की आिश्यकता है ।
• मर्हिा-केंर्द्रि मुद्दे: • एसपीएम-र्निास: अोंतरराष्टरीि ख्यायत प्राप्त पेिजि और स्विता
 ज्ञान की कमी: जमीनी ररपदटें मायसक िमन स्विता ज्ञान की के यिए एक र्ीषन सोंस्र्ान, र्ैक्षयणक गयतयियििाँ, अनुसोंिान कािन
आयद करता है ।
कमी का सोंकेत दे ती हैं । भिे ही आर्ा कािनकतान जागरूकता
फैिाना जारी रख रही हैं, िेयकन ऐसे मामिे हैं जहाों गाोंि ों में 80
• मार्सक धमि स्वच्छिा प्रबोंधन (एमएिएम): यकर्दररिदों कद अपनी
यर्क्षा जारी रखने और मायसक िमन से जुडी िजननाओों कद समाप्त
प्रर्िशि मर्हिाएों और िड़र्कयाों अस्वास्थ्यकर प्रथाओों का
करने में सक्षम बनाने के यिए पेिजि और स्विता मोंत्रािि द्वारा
पािन कर रही हैं।
जारी एमएचएम यदर्ायनदे र्।
 कायिस्थि पर स्वच्छिा प्रबोंधन की कमी: अन्य बािाओों के • स्वच्छ र्ारि स्वच्छ र्िद्यािय (एसबीएसिी) कायिक्रम: सरकारी
अिािा, शहरी कायिस्थि ों में स्वच्छ, अिग मर्हिा शौिािय ों प्रार्यमक और माध्ययमक यिद्याििदों में िडयकिदों और िडकदों के यिए
की कमी मयहिाओों कद काम करने से हतदत्सायहत करती है। यिोंग-पृर्क र्ौचािि सुयििाएों प्रदान करना।
 र्िोंग टर ै र्कोंग प्रर्ािी का अर्ाि: भिे ही WASH नीयतिाों अपने
यडजाइन और िदजना में यिोंग-सोंिेदनर्ीि िा यिोंग-पररितननकारी हैं , यफर भी िे नीयत के यिोंग आिाररत प्रभाि कद मापने में यिफि हैं।

र्ारि के र्िए आगे का रास्ता:


• बुर्नयािी िॉश सुर्िधाएों : बुयनिादी स्विता सुयििाओों और स्वि पानी तक पहुोंच में सुिार करना, और सािनजयनक स्र्ानदों पर पिान प्त
मायसक िमन स्विता सुयििाएों प्रदान करना।
• स्वास्थ्य िे खर्ाि पर ध्यान: प्रयर्यक्षत स्वास्थ्य सेिा प्रदाताओों की सोंख्या बढाना, आिश्यक दिाओों और आपूयतन की उपिब्धता और सामथ्न
में सुिार करना।
• मर्हिाओों के नेिृत्व में पररिििन: समुदािदों की भूयमका, यिर्ेष रूप से मयहिाओों और िडयकिदों कद सफाई और स्विता कािनक्रमदों कद
यडजाइन और कािान ल्कन्वत करने में मदद करनी चायहए।
• िागरूकिा और सोंिेिीकरर् अर्र्यान: समुदािदों में स्विता सोंबोंिी जागरूकता और सोंिेदीकरण अयभिान की यनियमतता कद बढाने
की जरूरत है तायक घरे िू स्तर पर भी व्यिहाररक बदिाि हद।
• स्कूि पाठ्यक्रम: स्कूि पाठ्यक्रम में िॉर् के महत्व कद र्ायमि करना िह सुयनयित करने का एक तरीका है यक यर्क्षक, छात्र और प्रर्ासक
इस मुद्े पर ध्यान केंयित करते रहें ।

िघु समािार

8.6 स्वच्छ सिे क्ष र् 2023


सोंिर्ि:
स्वि सिेक्षण 2023 के तहत र्हरदों की साफ-सफाई का जमीनी स्तर पर आकिन र्ुरू हद गिा है ।
स्वच्छ सिेक्षर् के बारे में:
• 2016 से आय र्िि स्वि सिेक्षण, दु यनिा का सबसे बडा र्हरी स्विता और सफाई सिेक्षण है और शहरी- स्वच्छ र्ारि र्मशन का
र्हस्सा है।
• न डि मोंिािय: आिास और र्हरी मामिदों का मोंत्रािि (MoHUA)
• प्राथर्मक िक्ष्य:
 बडे पैमाने पर नागररक भागीदारी कद प्रदत्सायहत करना और
 कस्दों और र्हरदों कद रहने के यिए बेहतर स्र्ान बनाने की यदर्ा में यमिकर काम करने के महत्व के बारे में जागरूकता पैदा करना।
• स्वि सिेक्षण 2023 रैं र्कोंग का आठिाों िार्षिक सोंस्करर् है।
• 2023 की थीम: किरा मुक्त शहर ों के र्िए अपर्शि से धन।
114
• िह मूल्याोंकन यपछिे सोंस्करणदों के 3 चरणदों की तुिना में 4 चरणदों में आिदयजत यकिा जाना है । सिेक्षण में 3आर - ररड्यूस, ररसाइकि
और ररयूि के र्सद्धाोंि क प्राथर्मकिा िी िाएगी।
• मूल्ाोंकन पैरामीटर:
1. सािनजयनक स्र्ानदों एिों र्ौचाििदों की साफ-सफाई
2. यनिायसिदों से प्रयतयक्रिा का सोंग्रह
3. अपयर्ष्ट सोंग्रहण, पृर्क्करण और प्रसोंस्करण में नगर पायिकाओों का प्रदर्नन।

स्वच्छ र्ारि र्मशन के बारे में:


• िह खुिे में र्ौच कद खत्म करने और ठदस अपयर्ष्ट प्रबोंिन में सुिार के यिए 2014 में भारत सरकार द्वारा र्ुरू यकिा गिा एक दे र्व्यापी अयभिान है ।
• यह 2009 में शुरू र्कये गये र्नमिि र्ारि अर्र्यान का पुनगनयठत सोंस्करण है ।
• स्वच्छ र्ारि र्मशन का पहिा िरर् अरू बर 2019 तक चिा।
• िरर् 2 क 2020-21 और 2024-25 के बीच िागू यकिा जा रहा है ।
• यमर्न का िक्ष्य र्ौचाििदों के यनमानण के माध्यम से 2 अरू बर 2019 , महात्मा गाोंिी की 150िीों जिोंती तक भारत कद "खुिे में र्ौच मुक्त" (ओडीएफ)
प्राप्त करना है ।
• यमर्न कद दद भागदों में यिभायजत यकिा गिा है :
1. ग्रामीर् क्षेि ों में "एसबीएम - ग्रामीण " कद जि र्ल्कक्त मोंत्रािि के माध्यम से यििपदयषत और मॉयनटर यकिा जाता है ।
• शहरी क्षेि ों में "एसबीएम - र्हरी" की दे खरे ख आिास और र्हरी मामिदों के मोंत्रािि द्वारा की जाती है ।

िार्िका में सों र्क्षप्त समािार

प्रिशिन ग्रेर्डों ग सूिकाोंक र्शक्षा मोंिािय ने राज्यदों /केंिर्ायसत प्रदे र्दों के यिए प्रदर्नन ग्रेयडों ग इों डेक्स (पीजीआई) 2.0 पर ररपदटन जारी की
2.0 है ।
प्रिशिन ग्रेर्डों ग सूिकाोंक के बारे में:
• इसे स्कूिी यर्क्षा और साक्षरता यिभाग, यर्क्षा मोंत्रािि जारी करता है ।
• उद्दे श्य: व्यापक यिश्लेषण के यिए एक सूचकाों क बनाकर राज्य/केंि र्ायसत प्रदे र् स्तर पर स्कूिी यर्क्षा
प्रणािी के प्रदर्नन का आकिन करना।
• पहिी बार िारी: िषन 2017-18 के यिए
पीिीआई 2.0
• नई पीजीआई सोंरचना में 73 सोंकेिक शार्मि हैं, जद यडयजटि पहि और यर्क्षक-यर्क्षा कद र्ायमि करने
के अिािा गुर्ात्मक मूल्ाोंकन पर अयिक ध्यान केंयित करते हैं ।

बच्चे और सशस्त्र सोंघषि: सोंिुक्त राष्टर ने उनकी "बेहतर सुरक्षा के यिए सरकार द्वारा उठाए गए कदमदों" का हिािा दे ते हुए अपनी िायषनक
सोंयुक्त रािर ररप टि ररपदटन -'बच्चे और सर्स्त्र सोंघषन' से भारत कद हटा यदिा है ।
'बच्चे और सशस्त्र सोंघषि' के मुि र्नष्कषि: सोंयुक्त रािर ररप टि :
• यिश्व स्तर पर 2022 में, बच्चे सर्स्त्र सोंघषन से असोंगत रूप से प्रभायित हदते रहे ।
• सोंिुक्त राष्टर द्वारा आतोंकिादी समूहदों के रूप में नायमत िदगदों सयहत सर्स्त्र समूहदों के सार् िास्तयिक िा
कयर्त जुडाि के यिए 2,496 बच्चदों कद यहरासत में यििा गिा र्ा।

115
9. किा एिों सों स्कृ र्ि
9.1 पु रािशे ष और किा खिाना अर्धर्नयम
सोंिर्ि:
हाि ही में, सोंिुक्त राज्य अमेररका ने तस्करी की गई 105 पुरािर्ेष िस्तुएों भारत कद सौोंपीों।
समािार में:
• किाकृयतिाँ भारत में अपनी उत्पयि के सोंदभन में व्यापक भौगदयिक यिस्तार का
प्रयतयनयित्व करती हैं - पूिी भारत से 47, दयक्षणी भारत से 27, मध्य भारत से 22,
उिरी भारत से 6 और पयिमी भारत से 3।
• िे प्राचीन िस्तुएँ टे राक टा, पत्थर, धािु और िकड़ी से बनी हैं - ि िू सरी-
िीसरी शिाब्दी से िेकर 18िी ों-19िी ों शिाब्दी िक की अियि की हैं ।
• उिाहरर् के र्िए: पहिी र्ताब्दी ईसा पूिन की टे राक टा यक्षी पर्ट्टका; 9िीों
सदी का िाि बिुआ पत्थर का नृत्य करिा हुआ गर्ेश; और 10िीों र्ताब्दी
का कुबेर।
पुरािशेष क्ा हैं?
• पुरातनता र्कसी र्ी िस्तु या किा के कायि क सोंिर्र्िि करिी है ि र्पछिे युग के यिज्ञान, किा, सायहत्य, िमन, रीयत-ररिाजदों, नैयतकता
िा राजनीयत कद दर्ान ती है ।
• इसमें र्सिे, मूर्िियाों, पेंर्टों ग, पुरािेख, पृथक िस्तुएों शार्मि ह सकिी हैं।
• पुरािशेष और किा खिाना अर्धर्नयम, 1972 (एएटीए) के अनुसार, "प्राचीन िस्तु" एक ऐसी िस्तु है जद कम से कम 100 िषि पुरानी
है।
 ियद िह यकसी िैज्ञायनक, ऐयतहायसक, सायहल्कत्यक िा सौोंदिन मूल्य की पाोंडुर्िर्प िा अयभिेख है , तद िह कम से कम 75 िषि पुराना
ह ना िार्हए।

किा खिाना:
• एक किा खजाना, पुरातनता के अिािा, किा का एक मानिीि कािन है , यजसे किाकार की मृत्यु के बाद उसके किात्मक मूल्य के यिए
केंि द्वारा एक खजाना घदयषत यकिा जाता है ।
 उिाहरर् के र्िए: चदि काों स्य

र्ारिीय र्िरासि से सोंबोंर्धि सोंिैधार्नक, र्िधायी और अोंिराििरीय प्रािधान:


• सोंिैधार्नक:
 र्िषय ों का िगीकरर्: भारत में, सोंयििान की अनुसूिी साि में भारतीि यिरासत से सोंबोंयित यनम्नयिल्कखत प्राििान र्ायमि हैं :

सोंघ सूिी (आइटम 67) राज् सूिी (आइटम 12) समििी सूिी (आइटम 40)

• प्राचीन और ऐयतहायसक िारक • राज्य द्वारा यनिोंयत्रत िा यििपदयषत पुस्तकािि, सोंग्रहािि • राष्टरीि महत्व के [सोंसद द्वारा िा
और अयभिेख, और पुराताल्कत्वक और अन्य समान सोंस्र्ान; सोंसद द्वारा अर्िा उसके द्वारा उसके द्वारा बनाए गए र्िर्ध के
स्र्ि और अिर्ेष, [सोंसद द्वारा िा बनाए गए र्िर्ध के तहत राष्टरीि महत्व के घदयषत यकए गए तहत घदयषत] के अिािा अन्य
उसके द्वारा बनाए गए र्िर्ध के िारकदों और अयभिेखदों के अिािा अन्य प्राचीन और पुराताल्कत्वक स्र्ि और
तहत घदयषत] राष्टरीि महत्व के हैं । ऐयतहायसक िारक और अयभिेख। अिर्ेष।

• राज् के नीर्ि र्निे शक र्सद्धाोंि: अनुच्छेि 49 राज्य पर प्रत्येक िारक िा स्र्ान िा किात्मक िा ऐयतहायसक रुयच की िस्तु की रक्षा करने
का दायित्व डािता है , जद सोंसद द्वारा िा उसके तहत राष्टरीि महत्व के घदयषत र्िर्ध के तहत घदयषत यकिा गिा है ।
• मौर्िक कििव्य: सोंयििान के अनुच्छेि 51ए में कहा गिा है यक हमारी सोंस्कृयत की समृद्ध यिरासत कद महत्व दे ना और सोंरयक्षत करना
भारत के प्रत्येक नागररक का कतनव्य हदगा।
• र्िधायी:
 1947: यबना िाइसेंस के पुरािर्ेषदों के यनिानत कद रदकने के यिए पुरािर्ेष (यनिान त यनिोंत्रण) अयियनिम पाररत यकिा गिा र्ा।
116
 1958: प्रािीन स्मारक ों और पुरािाम्भत्वक स्र्िदों कद यिनार् और दु रुपिदग से बचाने के यिए प्राचीन िारक और पुरातत्व स्र्ि और
अिर्ेष अयियनिम िागू यकिा गिा र्ा।
 1972: पुरािर्ेष और किा खजाना अयियनिम, 1972 (एएटीए), 1 अप्रैि 1976 से िागू यकिा गिा।
 िह िाइसेंस भारतीि पुरातत्व सिेक्षण (एएसआई) द्वारा यदिा जाता है ।
• र्िरासि से सोंबोंर्धि अोंिराििरीय अर्र्समय ों के प्रर्ि र्ारि के िार्यत्व:
 साों स्कृयतक सोंपयि के स्वायमत्व के अिैि आिात, यनिान त और हस्ताोंतरण कद प्रर्िबोंर्धि करने और र कने के सािनदों पर कन्वेंर्न ,
1977
 अमूिि साोंस्कृर्िक र्िरासि की सुरक्षा के यिए कन्वेंर्न , 2005
 साों स्कृयतक अयभव्यल्कक्तिदों की यियििता के सोंरक्षण और सोंििनन पर कन्वेंर्न, 2006

पुरािशेष ों क र्ारि िापस िाने का व्यिस्था: पुरािशेष और किा खिाना अर्धर्नयम, 1972 (एएटीए) की मुि
र्िशेषिाएों :
पुरािशेष ों की िीन श्ेर्र्याों हैं यजन्हें भारत से बाहर िे जािा गिा है :
• पुरािशेष ों और किा खिान ों की पररर्ाषा: अयियनिम पररभायषत
• आजािी से पहिे र्ारि से बाहर िे िाये गये पुरािशेष;
करता है यक पुरािर्ेषदों और किा खजानदों का गठन क्ा है अर्िा यकस
• ि आजािी के बाि से मािि 1976 िक यार्न AATA िागू ह ने
चीज का बना हदता है ।
से पहिे िक िे िाए गए थे;
• र्नयािि का र्िर्नयमन: अयियनिम पुरािर्ेषदों और किा खजानदों के यनिानत
• ऐसे पुरािशेष र्िन्हें अप्रैि 1976 के बाि िे श से बाहर िे पर प्रयतबोंि िगाता है जब तक यक िह सरकार द्वारा जारी िैि िाइसेंस के
िाया गया। तहत नहीों यकिा जाता है ।
• पहिी ि श्ेर्र्य ों की िस्तुओों के यिए, यद्वपक्षीि िा अोंतरराष्टरीि • पोंिीकरर् की प्रर्क्रया: एएटीए की धारा 14(3) के िहि , "प्रत्येक
मोंचदों पर अनुरदि करना हदगा। व्यल्कक्त जद यकसी पुरािर्ेष का मायिक है , यनिोंत्रण करता है िा उसके कब्जे
 उिाहरर् के र्िए, महाराष्टर सरकार ने 2022 में घदषणा की में है " कद पोंजीकरण अयिकारी के समक्ष ऐसी पुरािर्ेषदों कद पोंजीकृत करना
यक िह िोंदन से छत्रपयत यर्िाजी महाराज की तििार िापस हदगा "और ऐसे पोंजीकरण के प्रतीक के रूप में एक प्रमाण पत्र प्राप्त करना
िाने के यिए काम कर रही है । हदगा।"

• िू सरी और िीसरी श्ेर्ी की पुरािर्ेषदों कद स्वायमत्व के प्रमाण • पुरािशेष ों की र्बक्री: पुरािर्ेषदों कद केिि िाइसेंस प्राप्त व्यल्कक्त ही बेच
सकता है ।
के सार् और िूनेस्कद सम्मेिन की मदद से यद्वपक्षीि रूप से मुद्ा
• िों ड: केिि केंद्र सरकार द्वारा र्बक्री के र्िए र्नर्िि ि पुरािर्ेषदों कद बेचना
उठाकर आसानी से पुनः प्राप्त यकिा जा सकता है ।
और िाइसेंस की समाल्कप्त के समि यकसी के कब्जे में मौजूद सभी
पुरािशेष ों के सोंरक्षर् में िुनौर्ियााँ: पुरािर्ेषदों की घदषणा करने में यिफिता अपराि है ।
• रर्नीर्ि का अर्ाि: सीएजी की ररपदटन के अनुसार, एएसआई • छह महीने की िेि िा जुमानना िा ददनदों दों ड का प्राििान है ।
के पास अपने अयिदे र् कद पूरा करने के यिए रणनीयत िा रदड-
मैप (दीघनकायिक/मध्यम अियि) का अभाि है । सोंरक्षण गयतयियििाँ
तदर्न/िायषनक आिार पर की जा रही र्ीों।
• स्विे श िापसी के मुद्दे: यिदे र्दों से चदरी हुई पुरािर्ेषदों कद पुनः प्राप्त करना
एक िोंबी और जयटि प्रयक्रिा हद सकती है , यजसमें रािनर्यक बाििीि
और कानूनी िुनौर्ियााँ शार्मि हैं ।
• अिैध िस्करी और िूटपाट: भारत साोंस्कृयतक यिरासत में समृद्ध है , जद
इसे पुरािर्ेषदों की अिैि तस्करी और िूटपाट का िक्ष्य बनाता है ।
 ग्ल बि िाइनेंर्शयि इों टीर्ग्रटी का अनुमान है यक पेंयटों ग, मूयतनिदों
और अन्य किाकृयतिदों के अिैि व्यापार से सािाना 6 अरब डॉिर की
आि हदती है ।
• अपयािप्त र्िर्ध प्रिििन:
 राष्टरीि अपराि ररकॉडन ब्यूरद ने बतािा यक 2008 और 2012 के बीि,
िे श र्र में 3,676 एएसआई-सोंरर्क्षि स्मारक ों से 4,408 िस्तुएों
ि री ह गईों, िेयकन केिि 1,493 क ही पुर्िस पकड़ सकी।
• िाििीिाशाही: िोंबी और जयटि नौकरर्ाही प्रयक्रिाएों पुरािर्ेषदों के उयचत दस्तािेज़ीकरण, सोंरक्षण और परररक्षण में बािा बन सकती हैं।
• अर्िक्रमर् और शहरीकरर्: तेजी से हद रहे र्हरीकरण और यिकास पररिदजनाओों के कारण ऐयतहायसक स्र्िदों और पुराताल्कत्वक स्र्िदों
पर कभी-कभी अयतक्रमण हद जाता है िा उन्हें नष्ट कर यदिा जाता है ।
117
• अल्पर्नर्धकरर् और सोंसाधन सोंबोंधी बाधाएाँ : पुरािर्ेषदों के सोंरक्षण और प्रबोंिन के यिए महत्वपूणन यििीि सोंसािनदों और कुर्ि कयमनिदों
की आिश्यकता हदती है , जद हमेर्ा उपिब्ध नहीों हद सकते हैं ।
 2017-18 के बाद, एएसआई के कुि व्यि में िृल्कद्ध और यिरासत सोंरक्षण गयतयियििदों पर इसका व्यि (कुि व्यि का 40 प्रयतर्त) मध्यम
र्ा।
 सीएजी के अनुसार, उत्खनन और अन्वेषण गयतयियििदों पर एएसआई का खचन अभी भी एक प्रर्िशि से र्ी कम था।
• िििायु और पयाििरर्ीय कारक: मौसम और प्रदू षण जैसे प्राकृयतक तत्व, पुरािर्ेषदों के सोंरक्षण के यिए खतरा पैदा कर सकते हैं ।
आगे की राह: रािरीय स्मारक और पुरािशेष र्मशन (NMMA):
• कानूनी ढाोंिे क मिबूि करना: पुरािर्ेषदों की अिैि तस्करी • राष्टरीि िारक और पुरािर्ेष यमर्न (NMMA) कद िे श के स्मारक ों और
और यनिान त से यनपटने के यिए मजबूत र्िर्ध बनाना और िागू पुरािर्ेषदों का दस्तािेजीकरण करने के यिए 2007 में िॉन्च यकिा गिा र्ा।
करना। • इससे सर्ी पु रािशेष ों का एक डे टाबेस बनाने में मदद यमिेगी और
• बेहिर र्िर्ध प्रिििन: पुरािर्ेषदों की तस्करी की पहचान करने िदखािडी िािे िेनदे न िा चदरी के मामिे में इसे आसानी से सोंदयभनत यकिा
और उससे यनपटने के यिए र्िर्ध प्रितनन अयिकाररिदों के यिए जा सकता है ।

यिर्ेष प्रयर्क्षण यदिाने में यनिेर् करें । • इसे र्ुरुआत में 2007 से 2012 तक 5 साि की अियि के यिए स्र्ायपत
यकिा गिा र्ा। इसे 2017 तक 5 साि के यिए बढा यदिा गिा और बाद में
• र्डर्िटि िस्तािेजीकरर्: पुरािर्ेषदों का यिस्तृत ररकॉडन
भारतीि पुरातत्व सिेक्षण (एएसआई) में यििि कर यदिा गिा।
बनाने के यिए 3डी स्कैयनोंग और यडयजटि दस्तािेज़ीकरण जैसी
आिुयनक तकनीक का उपिदग करें । िह अनुसोंिान, सोंरक्षण और प्रत्याितनन प्रिासदों में सहािता करता है ।
• बेहिर पुराित्व सिेक्षर्: उन स्र्िदों की पहचान और दस्तािेजीकरण
प्र. भारत कद अपने पुरािर्ेषदों और किा खजानदों कद सोंरयक्षत करने
करने के यिए व्यिल्कस्र्त और यनियमत पुराताल्कत्वक सिेक्षण करें , जद में यकन-यकन चुनौयतिदों का सामना करना पडता है और दे र् ने इन
िूटपाट िा अयतक्रमण के प्रयत सोंिेदनर्ीि हद सकते हैं । चुनौयतिदों से यनपटने के यिए क्ा उपाि यकए हैं ? इसके अयतररक्त,
• र्िरासि सोंरक्षर् क्षेि: महत्वपूणन यिरासत स्र्िदों के आसपास यियर्ष्ट भारत अोंतरराष्टरीि स्तर पर अपनी साोंस्कृयतक यिरासत की सुरक्षा के
क्षेत्र स्र्ायपत करें जहाों साोंस्कृयतक यिरासत की रक्षा के यिए यिकास और प्रयत अपनी प्रयतबद्धता कैसे प्रदयर्नत करता है ?
यनमान ण कद सख्ती से यियनियमत यकिा जाता है।

िार्िका में सों र्क्षप्त समािार

रुद्रर्गरर पहाड़ी रुियगरर पहाडी, मेसदयियर्क काि के प्रागैयतहायसक र्ैि यचत्रदों और काकिीय राििोंश की उत्कृि किाकृर्ि
के सोंय िन का अनािरर् करिी है।
रुद्रर्गरर पहाड़ी के बारे में:
• स्थान: रुियगरर पहाडी आों ध्र प्रिे श के गुोंटूर र्ििे में पूिी घाट के बीि म्भस्थि है ।
• मेस र्िर्थक युग के िौरान िदगदों के रहने के यिए िाटन र के रूप में काम करते र्े , और िे उस िुग के
चमकदार र्ैि यचत्रदों के गिाह हैं ।

र्सिे हुए िहाि र्नमािर् सोंस्कृत मोंत्रािि और र्ारिीय नौसेना ने जहाज यनमान ण के प्राचीन यसिे हुए तरीकदों कद पुनजीयित करने के
र्िर्ध (टों काई र्िर्ध) यिए एक समझौता ज्ञापन पर हस्ताक्षर यकए।
र्सिे हुए िहाि र्नमािर् र्िर्ध (टों काई र्िर्ध) के बारे में
• िह जहाज यनमान ण की 2000 साि पुरानी तकनीक है , जहाों जहाजदों का यनमान ण कीि ों का उपय ग करने
के बिाय िकड़ी के िख्त ों क एक साथ र्सिाई करके र्कया िािा है , यजससे िचीिापन और स्र्ायित्व
यमिता है , यजससे उन्हें उथिे और सैंडबार से नुकसान ह ने की सोंर्ािना कम ह िािी है।

नमिा किा प्रिानमोंत्री कौर्ि यिकास िदजना के तहत, कश्मीर के नमिा र्शल्प क सिििापूििक पुनिीर्िि र्कया
िा रहा है और यिटे न कद यनिानत के यिए नमिा किा उत्पाि ों का पहिा बैच भेज यदिा गिा है ।
नमिा किा के बारे में:

118
• कपड़ा: नमदा िेल्टे ड गिीिे हैं जद ऊनी रे श ों क पानी, साबुन और िबाि के साथ र्मिाकर और
यफर पररणामी कपडे पर कढाई करके बनाए जाते हैं । नमदा बनाने के यिए कम गुणििा िािे ऊन कद
र्दडी मात्रा में कपास के सार् यमिाकर उपिदग यकिा जाता है।
• िशि किररों ग और गद्दा: इन्हें कश्मीरी घरदों में प्रभािी और सस्ते फर्न किररों ग और गद्े के रूप में बडे
पैमाने पर उपिदग यकिा जाता है ।
• प्रकार: िे आम तौर पर दद प्रकार के हदते हैं, सािे और कढ़ाई िािे। पहिे, ऊनी िागे का उपिदग कढाई
के यिए यकिा जाता र्ा, िेयकन अब ऐक्रेयिक िागे (कैर्मेिन) का उपिदग यकिा जाता है ।

िम्बानी कढ़ाई किा िीसरी िी20 सोंस्कृर्ि कायि समूह (सीडब्ल्यूिी) की बैठक के यहस्से के रूप में, 'थ्रेड्स ऑि यूर्नटी'
शीषिक से िम्बानी िस्तुओों के सबसे बडे प्रदर्नन के यिए यगनीज यिश्व ररकॉडन बनािा गिा र्ा।
िम्बानी कढ़ाई के बारे में:
• िह रों गीन िागदों, दपनण-कािन और यसिाई पैटनन की यिर्ेषता िािे िस्त्र अिोंकरण का एक जयटि रूप है ।
• कनान टक के कई गाोंिदों में इसका अभ्यास र्कया िािा है।
• िकनीक: इसमें फेंके गए कपडे के छदटे -छदटे टु कडदों कद एक सार् जदडकर एक सुोंदर कपडा तैिार यकिा
जाता है ।

िैर्िक शाोंर्ि सूिकाोंक • जीपीआई ने र्ाों यत के स्तर के अनुसार 163 स्विोंि राज् ों और क्षेि ों क रैं क प्रिान र्कया।
2023 • िह इों स्टीट्यूट फॉर इकदनॉयमक्स एों ड पीस (आईईपी) द्वारा जारी यकिा गिा।
• िैयश्वक र्ाोंयत का औसत स्तर 0.42 प्रयतर्त यगर गिा।
• आइसिैंड 2008 से दु यनिा का सबसे शाोंर्िपूर्ि िे श है।
• रैं यकोंग में भारत 126िें स्थान पर कार्बि है।

हुि र्ििस यियटर् औपयनिेयर्क अयिकाररिदों के ल्कखिाफ िडाई में आयदिायसिदों के बयिदान कद िाद करते हुए, झारखोंड
30 िून क 'हुि र्ििस' के रूप में मनािा है।
हुि र्ििस के बारे में:
• 'हुि' का र्ाल्कब्दक अर्न है- क्राोंर्ि ि 1855 में र्ुरू हुई।
• नेिा: इसका नेतृत्व चार भाईिदों- र्सि् धु, कान्फ्ह, िााँि और र्ैरि ने र्कया
• र्िद्र ह का क्षेि: यििदह िार्मन-ए-क ह क्षेि में हुआ।
 'दायमन-ए-कदह' का अर्न है 'पहार्ड़य ों का र्कनारा'। िह क्षेत्र राजमहि पहायडिदों के आसपास ितनमान
झारखोंड में पडता है ।

मीरा पैर्बस भारतीि सेना की स्पीिर कदर ने मयणपुर में मयहिा कािनकतान ओों पर राज्य में चि रहे दों गदों और अर्ाों यत के बीच
मागों में बािा डािने और सुरक्षा कािों में हस्तक्षेप करने का आरदप िगािा।
मर्र्पुर की मीरा पैर्बस के बारे में:
• मीरा पैयबस, यजन्हें इमास या मर्र्पुर की मािा के नाम से र्ी िाना िािा है, मैतेई मयहिाएों हैं जद मयणपुर
में एक र्ल्कक्तर्ािी नैयतक र्ल्कक्त का प्रयतयनयित्व करती हैं ।

रािरीय समुद्री र्िरासि केंिीि बोंदरगाह, जहाजरानी और जिमागन मोंत्री ने गुजरात के गाोंिीनगर में राष्टरीि समुिी यिरासत पररसर, िदर्ि
पररसर की पररिदजना प्रयक्रिा की समीक्षा की।
रािरीय समुद्री र्िरासि पररसर (एनएिएमसी) के बारे में:
• न डि मोंिािय: बोंदरगाह, जहाजरानी और जिमागन मोंत्रािि।
• उद्दे श्य: हमारे दे र् के मजबूत समुिी इयतहास और जीिोंत तटीि परों परा कद उजागर करना।
• अनुमार्नि िागि: रु. 4500 करदड
• िोंर्डों ग स्र ि: सािनजयनक और यनजी सोंस्र्ान, सोंगठन और सीएसआर।

119
10. नीर्िशास्त्र (एर्थक्स)
छाि ों के र्िए न ट: िूपीएससी ने अपने पेपर 4 के पाठ्यक्रम में सािनजयनक/यसयिि सेिा मूल्यदों और सािनजयनक नैयतकता कद र्ायमि यकिा
है ।
प्रशासन: नैयतक मागनदर्नन के स्रदत के रूप में र्िर्ध, यनिम, यियनिम और यििेक;
खेि के यनिम बनाम खेि की भािना के यिषि कद नैयतकता अनुभाग में र्ायमि यकिा गिा है तायक छात्रदों कद यिषि के पठन कद िागू करने
और अपने स्विों के उदाहरण बनाने के यिए यिषिदों कद चुनने के बारे में मागनदर्नन यकिा जा सके। इससे अोंततः जीएस 4 पेपर में बेहतर उिर
और उच्च अोंक प्राप्त हदोंगे।

10.1 खे ि के र्नयम बनाम खे ि की र्ािना


प्रसोंग:
हाि ही में खत्म हुई एर्ेज टे स्ट सीरीज के दू सरे टे स्ट में जॉनी बेिरस्टद (इों ल्कग्लर् के प्लेिर) के आउट हदने पर यक्रकेट फैंस में आक्रदर् फैि गिा
और िदगदों ने इस कदम कद खेि की भािना के यिपरीत बताते हुए इसकी यनोंदा की।

पृष्ठर्ूर्म:
िह 52िाों ओिर र्ा और इों ग्लैंड 193/6 रन बनाकर खेि रहा र्ा। जॉनी बेिरस्टद
ने गेंद छदडने का फैसिा यकिा और गेंद यिकेटकीपर एिेक्स केरी के पास चिी
गई। इसी िक्त जॉनी बेिरस्टद की नजर क्रीज़ अोंदर अपने पैरदों पर पडी और
यफर स्टर ाइक बदिने के यिए आगे बढे उन्हदोंने सदचा यक ओिर पूरा हद चुका है।
िेयकन, अचानक, एिेक्स कैरी ने जॉनी बेिरस्टद कद स्टों प करने का फैसिा यकिा
िह यबिुि स्पष्ट र्ा यक जॉनी बेिरस्टद यपच की दू सरे छदर की ओर जा रहे र्े।
यनणनि तीसरे अोंपािर के पास गिा, जद जॉनी बेिरस्टद कद आउट हदने की घदषणा
कर यदिा।

पररर्ाम
इसके पररणामस्वरूप उपल्कस्र्त अोंग्रेजी प्रर्ोंसकदों ने इसकी आिदचना की और घटना के बारे में सदर्ि मीयडिा पर तीखी चचान हुई। बात इतनी
बढ गई यक ददनदों दे र्दों के नेता सदर्ि मीयडिा पर एक-दू सरे कद टर दि करने िगे।
जॉनी बेिरस्टद की बखानस्तगी से सोंबोंयित मौजूदा यििाद इस बात पर आिाररत है यक जब खेि के यनिम खे ि की भािना के अनुरूप नहीों हदों तद
क्ा करना सही हदगा
र्नयम क्ा कहिा है ?
आउट ह ने के पक्ष में िकि:( खेि के र्नयम > खेि की र्ािना): • एमसीसी के डे ड बॉि कानून के अनुसार, कानून 20.1.1.1 कहता है ,यक
• समानिा: कानूनदों का पािन सभी ल्कखिायडिदों के सार् समान "गेंद तब डे ड हद जाती है जब िह यिकेटकीपर िा गेंदबाज के हार्दों में पहुों च
व्यिहार कद बढािा दे ता है ; एक सफि प्रयतिदयगता कद समान जाती है ।"
अिसर प्रदान करना चायहए; उपरदक्त घटना के मामिे में, • कानून 20.1.2 में िह भी कहा गिा है , "गेंद कद डे ड माना जाएगा जब
ऑस्टर े यििाई बल्लेबाज भी समान कानूनदों से बोंिे हैं । गेंदबाज और अोंयतम अोंपािर कद िह स्पष्ट हद जाता है यक ग्राउों ड पक्ष और
यिकेट पर ददनदों बल्लेबाजदों ने इसे खेिने से मना कर यदिा है ।
• न्याय: इसमें यकसी व्यल्कक्त कद िह दे ना र्ायमि है यजसका उसे हक
है ; यनष्पक्षता, यनष्पक्ष खेि के अिीन नहीों हदनी चायहए। (fairness should not be subservient to fair play)। ऑस्टर े यििाई ल्कखिाडी ने
ददनदों पक्षदों के यिए उपिब्ध यनिमदों का इस्तेमाि यकिा और बल्लेबाजदों की आत्मसोंतुयष्ट उनके नुकसान का कारण नहीों हदनी चायहए।
• िस्तुर्नष्ठिा: यनिम स्वाभायिक रूप से िस्तुयनष्ठ हदते हैं , यजसका अर्न है यकसी की भािनाओों िा व्यल्कक्तगत पूिान ग्रहदों की परिाह यकए यबना,
केिि तथ्दों के आिार पर यनणनि िेना। यनिम बल्लेबाजदों कद यकसी भी तरह का अनुयचत िाभ पहुों चाने से रदकते हैं और इनका पािन यकिा
जाना चायहए।
• अस्पििा समाप्त करना : यनिमदों की अनुपल्कस्र्यत से अराजकता और भ्रम पैदा हद सकता है , जद खेि की व्यािसायिकता कद खत्म कर दे ता
है । व्यािसायिकता के उच्चतम मानकदों का पािन करना अोंतरराष्टरीि ल्कखिायडिदों की यजम्मेदारी हदनी चायहए।
• र्डटरे न्स िाना : यनिम बनाने की मूि िारणा िह है यक सजा दे ने से गित कािन करने का डर पैदा हदता है और सजा की गोंभीरता यनरदिात्मक
प्रभाि के सीिे आनुपायतक हदती है ।

120
• व्यिहार में एकरूपिा: आउट दे ने का दों ड भेदभािपूणन नहीों है क्दोंयक िह अन्य बल्लेबाजदों कद ऐसा करने से रदकता है और खेि कद यनष्पक्ष
तरीके से सोंचायित करने में मदद करता है।
• र्र सा और र्ििसनीयिा: दर्नकदों का भरदसा खेि के पेर्ेिर मानकदों पर आिाररत है ; यनिमदों का पािन यिश्वसनीिता कद बढािा दे ता है
और दर्नकदों की सोंख्या में िृल्कद्ध करता है ।
• उदाहरण के यिए, समान पररल्कस्र्यतिदों में ल्कखिायडिदों का पक्ष िेने से अोंपािरदों और मेजबान दे र्दों के बीच भ्रष्टाचार और अपयित्र साों ठगाों ठ हद
सकती है , यजससे खेि की यिश्वसनीिता कम हद सकती है ।
आउट ह ने के र्िरुद्ध िकि (खेि की र्ािना > खेि के र्नयम)
र्िर्ध/र्नयम की कारि िाई नैर्िकिा युक्त ह ने िार्हए:
अयिकाों र् र्िर्ध खेि की भािना कद बढािा दे ने के यिए हैं जबयक
खेि खेिना; ियद इस मूि मान्यता से असोंगत है , तद र्िर्ध के
अल्कस्तत्व पर ही प्रश्न उठता है ।
र्िर्ध/र्नयम सीर्मि हैं यह व्यापक नही ों ह सकिे : सभी
ल्कस्र्यतिदों कद र्िर्ध के दािरे में िाना असोंभि है ; इसयिए ऐसी
ल्कस्र्यतिदों में खेि की भािना ही मागनदर्नक हदनी चायहए।
उदाहरण के यिए, इस तरह से बखान स्त करना मुक्केबाजी में रे फरी
के कदम उठाने पर िेक मारने के समान है । ऐसे क्षण हदते हैं जब
ल्कखिायडिदों से ग्लैडीएटदररिि प्रयतिदयगता के बािजूद सिनतापूणन,
सम्मानजनक तरीके से व्यिहार करने की उम्मीद की जाती है।
• व्यम्भक्तय ों की र्ूर्मका: यकसी र्िर्ध कद यक्रिाल्कन्वत करने,
व्याख्या करने और िागू करने में , एक यनयित स्तर के
आदर्निाद की आिश्यकता हदती है यजसे कदई र्िर्ध आश्वस्त
नहीों कर सकता है ।
• ितनमान पररदृश्य में, ऑस्टर े यििाई कप्तान यनणनि के सार् आगे
बढे , जबयक 2011 में इसी तरह की ल्कस्र्यत में, एमएस िदनी ने
इिान बेि कद आउट यदए जाने के बाद िापस बुिा यििा र्ा।
• म्भखिाड़ी की ईमानिारी: ऐसे बल्लेबाज कद रन आउट करने
का यनणनि जद कदई अनुयचत िाभ नहीों िे रहा है , ल्कखिाडी की
ईमानदारी पर सिाि उठाता है; सत्ययनष्ठा का अर्न है यक तब
भी सही काम करना जब पररणाम आपके अनुरूप नहीों हद रहे
हदों।
• बल्लेबाज कद आउट करने के यिए और िाों यछत पररणाम प्राप्त
करने के यिए कानूनदों कद तदड-मरदड कर पेर् करने में एक
यनयित स्तर की क्षमता र्ायमि हदती है ; इस पर सिाि उठािा
जा सकता है ।
• अोंि बनाम साधन: खेि की भािना में खेि खेिने का आनोंद र्ायमि है यजसमें खेि का र्ुद्ध और सरि आनोंद भी र्ायमि है । यकसी भी
कीमत पर जीतना "खेिने की खुर्ी" की मूि अििारणा कद नष्ट कर दे ता है।
• केिि अक्षरश:, आत्मा से नही ों: खेि के यनिमदों का पािन केिि न्यूनतम अपेयक्षत मानकदों कद यनिान ररत कर सकता है। इससे मात्रात्मक
आिाम कम हद जाते हैं और केिि प्रयक्रिात्मक अनुपािन हदता है ।
क्ा र्कया िाए?
• ऐसी ल्कस्र्यतिदों में जहाों खेि के यनिमदों और खेि की भािना के बीच यिसोंगयतिाों हदती हैं , व्यल्कक्त का यििेक महत्वपूणन हद जाता है । िह उन
नैयतक आिामदों कद पहचानने में मदद करता है यजन्हें कारन िाई के सबसे उपिुक्त तरीके के यिए अपनाने की आिश्यकता है ।
र्ििेक क्ा है?
• यििेक दद चीजदों का िणनन करता है - एक व्यल्कक्त जद मानता है िह सही है और एक व्यल्कक्त ऐसे यनणनि िेता है यक क्ा सही है । िे अोंदर की
आिाज से अयिक, हमारा यििेक एक 'नैयतक मजबूती' है ।
• िह िह मानदों ड बन जाता है यजसका उपिदग हम िह यनिान ररत करने के यिए करते हैं यक हमारे यिश्वास और आदर्ों हमें सूयचत करे यक
हमारे कािन नैयतक हैं िा नहीों।

121
र्ििेक की प्रासोंर्गकिा:
• िह यकसी व्यल्कक्त कद यििेकपूणन और व्यािहाररक यिचार के सार् यनिानररत यनिमदों की व्याख्या
करने में मदद करता है ।
 उदाहरण के यिए, ऐसी ल्कस्र्यत जहाों कदई बल्लेबाज अनुयचत िाभ उठा रहा है , उसे उन
मामिदों से अिग यकिा जा सकता है जहाों कदई दु भानिनापूणन इरादा नहीों है ।
• िह पुराने कानूनदों की पहचान करने में मदद करता है और उनके सोंर्दिन के यिए मामिा तैिार
करता है ।
 उदाहरण के यिए, ितनमान पररदृश्य में, जब बल्लेबाज कदई अनुयचत िाभ नहीों चाह रहा हद,
तद तीसरा अोंपािर यनणनि िे सकता है और कप्तान कद अपनी अपीि िापस िेने का सुझाि
दे सकता है ।
• खेि की भािना के मूल्यदों कद बनाए रखने में यनिमदों की भूयमका का पूरक है।
 यनिमदों में उन टीमदों के यिए यनष्पक्ष खेि अोंक भी र्ायमि हद सकते हैं जद खेि कद उसकी
िास्तयिक भािना से खेिते हैं ।
 यििेक व्यल्कक्त की यििेकर्ल्कक्त पर यनभनर करता है और उसे बढाता भी है । इसयिए, िह
सुयनयित करना महत्वपूणन हद जाता है यक र्ल्कक्त, प्रभाि अर्िा अयिकार की ल्कस्र्यत का आनोंद
कौन उठा रहा है ।
 आिश्यक है यक सिा में बैठे िदग अपनी यजम्मेदारी के अनुरूप अपने यििेक कद यर्यक्षत
करें ।
• ददनदों टीमदों के कप्तान न्यूनतम सामान्य मानकदों कद ति करने के यिए श्रृोंखिा से पहिे यमि सकते
हैं यजनका ददनदों टीमें खेि के दौरान पािन करें गी।
र्नष्कषि:
ऐसे मामिदों में जहाों व्यल्कक्तगत यििेक यियि के सार् टकराि में है , यियि िागू हदना चायहए, िेयकन
ऐसे मामिदों में जहाों यियि नैयतकता की कसौटी पर यिफि रहता है , ऐसे कानूनदों कद बदिना यनणनि
यनमान ताओों पर यनभनर है ।

122
11. समािार ों में व्यम्भक्तत्व
ि कमान्य • हाि ही में प्रिानमोंत्री ने िदकमान्य यतिक कद उनकी जिोंती पर श्रद्धाों जयि दी।
र्ििक • 'िदकमान्य' यिर्ेषण का अर्न है प्रिा का र्प्रय।
• समािार पि: केसरी ("द िािन"), द मराठा।
• सार्हम्भत्यक कृर्ियााँ: गीता रहस्य, 'ि आकिर्टक ह म इन ि िेिाज'।
• महात्मा गाों िी ने उन्हें "आधुर्नक र्ारि का र्नमाििा" कहा, और जिाहरिाि नेहरू ने उन्हें "र्ारिीय क्राोंर्ि का
िनक" कहा।

िोंद्रशेखर • प्रिानमोंत्री ने स्वतोंत्रता सेनानी चन्द्रर्ेखर आज़ाद कद उनकी जिोंती पर श्रद्धाोंजयि अयपनत की।
आिाि • आजाि बनना: 15 साि की उम्र में मदहनदास के. गाोंिी के असहिदग आों ददिन (1920-22) में भाग िेने के दौरान
पुयिस द्वारा पकडे जाने पर, उन्हदोंने अपना नाम आजाि ("मुक्त") और अपना पता "िेि" बिाया।
• सोंगठन एिों य गिान: र्होंिुस्तान ररपम्भिकन एस र्सएशन के सदस्य। िह काक री षड्योंि (1925) और एक
र्िर्टश पुर्िस अर्धकारी की हत्या (1928) में शार्मि थे।

रॉबटि • के बारे में: िह एक अमेररकी सैद्धाोंर्िक र्ौर्िक र्िज्ञानी और र्िज्ञान प्रशासक र्े यजन्हें परमाणु बम के यिकास के
ओपेनहाइमर दौरान उन्नि अध्ययन सोंस्थान के र्निे शक और िॉस एिाम स प्रय गशािा के र्निे शक (1943-45) के रूप में
सेिा दे ने के यिए जाना जाता है।
• मैनहट्टन पररय िना: मैनहट्टन पररिदजना र्द्विीय र्िि युद्ध के िौरान एक कायाित्मक परमार्ु हर्थयार र्िकर्सि
करने के अमेररकी नेिृत्व िािे प्रयास का क ड नाम था।

श्यामा प्रसाि प्रिानमोंत्री ने डॉ. श्यामा प्रसाद मुखजी कद उनकी जिोंती (6 जुिाई) पर श्रद्धाों जयि अयपनत की है ।
मुखिी • िन्म: 6 जुिाई, 1901 किकिा में।
• रािनीर्िक कैररयर और नेिृत्व र्ूर्मकाएाँ :
 1944: 1944 में िे र्होंिू महासर्ा के अध्यक्ष बने।
 1951: 1951 में मुखजी ने र्ारिीय िनसोंघ की स्थापना की।
• र्िखी गई पुस्तकें:
1. भारतीि सोंघषन का एक चरण
2. एकीकृत भारत का सोंकल्प
3. भारत कद बचाने की समझदार सिाह
4. जागद यहोंदुस्तान

अल्लूरी राष्टरपयत ने स्वतोंत्रता सेनानी अल्लूरी सीतरामम राजू कद उनकी 125िी ों ियोंिी पर श्द्धाोंिर्ि िी।
सीिाराम रािू • िन्म और प्रारों र्र्क िीिन:
 4 जुिाई, 1897 कद िििमान आों ध्र प्रिे श में जन्म।
 यियटर् र्ासन के अिीन दे र्िायसिदों के कष्टदों से िे अत्योंत व्ययर्त र्े।
• स्विोंििा सेनानी के रूप में र्ूर्मका:
 इन्हें मन्यम िीरुडु (िोंगि य द्धा) के रूप में भी जाना जाता है , उन्हदोंने हजारदों गरीब आयदिायसिदों कद एकजुट
यकिा और उनमें स्वतोंत्रता की भािना जगाई।
 इन्ह न ों े िमनकारी मद्रास िन अर्धर्नयम, 1882 के यिरुद्ध िडाई िडी।
• मृत्यु: राम राजू कद 7 मई, 1924 कद अोंग्रेजदों ने पकड यििा और र्हीद कर यदिा। उनके अिर्ेषदों कद र्िशाखापिनम
के कृष्णािे िी पेटा में ििनाया गया है।

र्मर्हर र् ि • हाि ही में, हररिाणा में गुििर समुिाय ने यमयहर भदज कद गुजनर प्रयतहार सम्राट (राजा) बताते हुए उनकी एक मूयतन का
अनािरण करने की िदजना बनाई र्ी।

123
• गुििर और रािपूि ददनदों का दािा है यक राजा यमयहर भदज उनके पूिनज हैं , यजसके कारण सोंघषन हुआ है ।
• शासक एिों राििोंश: र्मर्हर र् ि 9िीों र्ताब्दी में गुििर प्रर्िहार िोंश के र्ासक र्े।
• रािधानी: कन्नौज.
• शीषिक: यिष्णु के सम्मान में िराह और प्रभास।
• ग्वार्ियर प्रशम्भस्त: यमयहर भदज की उपिल्कब्धिदों का यििरण दे ने िािा यर्िािेख

रानी िु गािििी हाि ही में मध्य प्रदे र् में सरकार ने छह यदिसीि रानी दु गानिती गौरि िात्रा र्ुरू की है .
• िन्म: 1524 में मह बा के िोंिेि राििोंश में - िह क्षेत्र ितनमान उिर प्रदे र् के अोंतगनत आता र्ा।
• उनका यििाह गढ-कटों गा राज्य के गदोंड राजा सोंग्राम र्ाह के पुत्र दिपत र्ाह से हुआ र्ा
• अपने र्ासनकाि के दौरान, दु गान िती ने पडदसी माििा के सुल्तान बाज बहादु र से िडाई की, जद अोंततः अकबर से
हार गिा।
• अकबर के दरबारी इयतहासकार अबुि ििि , र्िन्ह न ों े अकबरनामा र्िखा था , ने दु गानिती कद सुोंदरता, अनुग्रह
और मदान ना साहस और बहादु री का सोंिदजन बतािा।

124
Offline / Online

IDMP
INTEGRATED DAILY MCQ s + MAINS ANSWER WRITING PROGRAM
Prelims + Mains Test Series 2024

In a 4-Phased Well Planned Schedule to run as

168 171
Prelims Tests 1 2 Mains Tests

CSAT NCERT
Module 3 4 Module

Answer + Essay 14 PRAHAAR


Writing Sessions 5 6 Hard Copies

Regular Expert
Mentorship 7 8 Evaluation
of All Tests

English Medium- Old Rajinder Nagar | �ह�ी मा�म - मुख़ज� नगर


12. समािार ों में स्थान

कच्चार्ििू द्वीप
स्थान: यह पाक ििडमरूमध्य में एक द्वीप है , ि रामेिरम से 14
समुद्री मीि िू र है , और िोंबे समय से र्ारि और श्ीिोंका के बीच
यििाद का यिषि रहा है।

यूक्रेन का ओडे सा बोंिरगाह


हाि ही में रूस ने यूक्रेन के ओडे सा बोंिरगाह पर यमसाइिदों से हमिा
यकिा र्ा.
ओडे सा समुद्री बोंिरगाह के बारे में:
• ओडे सा कािा सागर पर यूक्रेन का सबसे बड़ा बोंिरगाह है।
• िह र्िि का 10% गेहाँ , 12 से 17% मिा और र्िि का आधा
सूरिमुखी िेि र्नयािि करिा है।

क्रीर्मया र्िि
हाि ही में, क्रीयमिा पुि कद तब नुकसान हुआ जब उसका एक यहस्सा
उडा यदिा गिा।
क्रीर्मया र्िि के बारे में:
• केचन यिज, र्िसे क्रीर्मया र्िि के नाम से र्ी िाना िािा है , रूस
के पररिहन नेटिकि और क्रीर्मया प्रायद्वीप के बीच एकमात्र सीिा
यिोंक है ।
• केचन स्टर े ट के ऊपर से गुजरने िािे पुि का उि् घाटन 2018 में र्कया
गया था।
• केचन जिडमरूमध्य पूिी िूरदप में ल्कस्र्त है , िह एक जिमागन के रूप
में कािन करता है जद कािा सागर क आज ि सागर से ि ड़िा है।
छर्ि स्र ि: NYPOST

125
इराक
इराक में र्ारि की सरकारी कोंपनी ओएनिीसी र्ििे श र्िर्मटे ड
(ओिीएि) हाइडर दकाबनन अन्वेषण ब्लॉक का सोंचािन यफर से र्ुरू करने
पर यिचार कर रही है , जद 2003 से अप्रत्यायर्त ल्कस्र्यत में है।
इराक के बारे में:
• स्थान: इराक पयिमी एयर्िा में ल्कस्र्त है ।
• सीमाएाँ : इराक की सीमाएँ कई दे र्दों के सार् िगती हैं :
 उिर में तुकी
 पूिन में ईरान
 दयक्षण पूिन में फारस की खाडी और कुिैत
 दयक्षण में सऊदी अरब
 दयक्षण पयिम में जॉडन न
 पयिम में सीररिा
• नर्ियााँ: दद प्रमुख नयदिाँ , टाइर्ग्रस और यूफ्रेट् स, इराक से हदकर
बहती हैं । िे र्त अि-अरब में फारस की खाडी के पास यमिती हैं ।

कास पठार (Kaas Plateau)


कास पठार में एक मौसमी झीि से ििछट के एक नए अध्यिन ने
प्रारों यभक-मध्य-हदिदसीन, िगभग 8664 िषन ईसा पूिन के दौरान कम िषान
के सार् शुष्क और िनािपूर्ि म्भस्थर्िय ों की ओर र्ारिीय ग्रीष्मकािीन
मानसून में एक बडे बदिाि का सोंकेत यदिा है ।
कास पठार के बारे में
• स्थान: कास पत्थर महारािर के सिारा र्ििे में ज्वािामुखीय
िट्टान ों से बना एक पठार है।
• कास पठार पर्िमी घाट के नाम से एक यूनेस्क र्िि प्राकृर्िक
र्िरासि स्थि है।
• कास का पठार िरम म्भस्थर्िय ों के एक प्राकृर्िक िक्र का अनुर्ि
करिा है।
 िह मानसून की बाररर् में गीिा िियुक्त ठों डा, बहुत र्ुष्क, बोंजर
छर्ि स्र ि: ररसििगेट
गमन ग्रीष्म (45 र्डग्री सेम्भल्सयस) और शुष्क सर्िि य ों (5 र्डग्री
सेम्भल्सयस) में बिि िािा है।

स ि मन द्वीप
सदिदमन द्वीप और चीन के नेताओों ने सोंबोंिदों का यिस्तार करने का िादा
यकिा, यजससे दयक्षण प्रर्ाोंत क्षेत्र में चीन के प्रभाि कद िेकर अमेररका और
ऑस्टर े यििा में बेचैनी बढ गई है ।
स ि मन द्वीप के बारे में:
• स्थान: दयक्षण-पयिमी प्रर्ाोंत महासागर।
• रािधानी: हदयनिारा
• जििािु उष्णकर्टबोंधीय समुद्री है - िानी, गमन और आिन िेयकन ठों डी
हिाओों और प्रचुर मात्रा में, साि भर हदने िािी िषान से राहत यमिती
है ।

126
पैंग ग
ों त्स झीि
चीन पैंगदोंग त्सद के पार उिर और दयक्षण तटदों कद जदडने िािा एक पुि
जल्द ही तैिार करने जा रहा है ।
पैंग ग
ों त्स झीि के बारे में:
• स्थान: पैंगदोंग त्सद एक एों ड रर्हक झीि (र्ूर्म से र्घरी हुई) है जद
आों यर्क रूप से भारत के िद्ाख क्षेत्र में और आों यर्क रूप से यतब्बत
में है । इसका कुि क्षेत्रफि 600 िगन यकमी से अयिक है ।
• इसका यनमान ण टे र्थस र्िय र्सोंिाइन से हुआ है।
• झीि का िगभग दद-यतहाई यहस्सा चीन द्वारा यनिोंयत्रत है, जबयक
िगभग 45 यकमी यहस्सा भारतीि यनिोंत्रण में है ।
• एिएसी, उिर-दयक्षण की ओर चिती हुई, झीि के पयिमी यहस्से कद
काटती है , पूिन-पयिम की ओर सोंरेल्कखत हदती है ।

127
13. ख़बर ों में य िनाएों

य िनाएों मोंिािय के बारे में/सुर्िधाएाँ

राज् ों में अर्िशमन गृह मोंत्रािि उद्दे श्य: राज्यदों में अयग्नर्मन सेिाओों का यिस्तार और आिुयनकीकरण करना।
सेिाओों के र्िस्तार और िोंर्डों ग: बिट: 5,000 करदड रुपिे. राज्य सरकारें अपने बजट से कुि पररिदजना
आधुर्नकीकरर् की िागत का 25% (एनईएच राज्यदों के यिए 10%) िदगदान करती हैं ।
य िना

उिर पूिी क्षेि के र्िए कािान न्विन हे तु कृयष उिर पूिी क्षेि के र्िए र्मशन िैर्िक मूल् श्ृोंखिा र्िकास (MOVCDNER) के
र्मशन िैर्िक मूल् एिों यकसान कल्याण चरण-III की प्रगयत की समीक्षा के यिए एक बैठक आिदयजत की गई र्ी।
श्ृोंखिा र्िकास मोंत्रािि • उद्दे श्य: उिर पूिी राज् ों में र्ुरू से अोंत तक िैर्िक मूल् श्ृोंखिा र्िकर्सि
करना।
• िह एक केंद्रीय क्षेि की य िना है, ि रािरीय सिि कृर्ष र्मशन
(एनएमएसए) के िहि एक उप-र्मशन है।
• िार्ाथी राज्:
 अरुणाचि प्रदे र्, असम, मयणपुर, मेघािि, यमजदरम, नागािैंड, यसल्कक्कम और
यत्रपुरा

अटि िय अभ्युिय सामायजक न्याि और • र्पछिी य िना का समामेिन: िररष्ठ नागररक ों के र्िए रािरीय कायि य िना
य िना अयिकाररता मोंत्रािि (NAPSrc) क निा रूप यदिा गिा, 2021 में इसका नाम बदिकर अटि ििद
अभ्युदि िदजना (AVYAY) कर यदिा गिा।
• िह एक व्यापक िदजना है यजसमें िररष्ठ नागररक ों के र्िए एकीकृि कायिक्रम
और रािरीय िय श्ी य िना शार्मि है।
• उद्दे श्य: आश्य, र् िन, र्िर्कत्सा िे खर्ाि और मन रों िन के अिसर जैसी
बुयनिादी सुयििाएों प्रदान करके िररष्ठ नागररकदों के जीिन की गुणििा में सुिार
करना।

समथि कपडा मोंत्रािि समथि कपडा क्षेत्र में एक माोंग आिाररत और प्लेसमेंट-उन्मुख अम्ब्रेिा कौर्ि
कािनक्रम है ।
उद्दे श्य: किाई और बुनाई क छदडकर, कपडा की सोंपूणन मूल्य श्रृोंखिा कद किर
करते हुए, सोंगर्ठि कपड़ा और सोंबोंर्धि क्षेि ों में र िगार पैिा करने में उद्यदग के
प्रिासदों कद प्रदत्सायहत करना और पूरक बनना।

नाबार्िग बिात्कार मयहिा एिों बाि यिकास उद्दे श्य: उन मामिदों में पीयडतदों कद यचयकत्सा, यििीि और बुयनिादी ढाोंचा सहािता
पीर्ड़ि ों की सहायिा के मोंत्रािि प्रदान करना जहाों िौन उत्पीडन के पररणामस्वरूप गभनिारण हदता है।
र्िए य िना

पशुधन क्षेि के र्िए ऋर् मत्स्य पािन, पर्ुपािन उद्दे श्य: सोंपायश्वनक सुरक्षा की आिश्यकता के यबना पर्ुिन क्षेत्र में सूक्ष्म, िघु और
गारों टी य िना और डे िरी मोंत्रािि मध्यम उद्यमदों (एमएसएमई) कद ऋण प्रिाह की सुयििा प्रदान करना।
िर्क्षि िार्ाथी: िह िदजना पर्ुिन क्षेत्र में पहिी पीढ़ी के उद्यर्मय ों और समाि
के िोंर्िि िगों का समथिन करिी है।

पीएम-प्रर्ाम य िना रसािन एिों उिनरक PM-PRANAM का अर्न है (धरिी मािा की पुनस्थािपना, िागरूकिा, सृिन,
मोंत्रािि प षर् और सुधार के र्िए प्रधान मोंिी कायिक्रम)।

128
उद्दे श्य: राज्यदों कद िैकल्कल्पक, गैर-रासाियनक उिनरकदों का उपिदग करने के यिए
प्रदत्सायहत करना।
इसकी पहिी बार घ षर्ा 2023-24 के बिट में ₹ 3,70,128.7 कर ड़ के कुि
पररव्यि के सार् की गई र्ी।
समय सीमा: 2022-23 से 2024-25

हररि ऋर् य िना पिानिरण, िन और उद्दे श्य: हररत ऋण के रूप में अयतररक्त प्रदत्साहन प्रदान करके िनीकरण कािनक्रम,
जििािु पररितनन जि सोंरक्षण जैसी पिानिरण-अनुकूि गयतयियििदों कद र्ुरू करने के यिए एक बािार-
मोंत्रािि आधाररि िोंि बनाना।

पूोंिी र्निेश 2023-24' यिि मोंत्रािि इस िदजना के तहत, राज्यदों कद यििीि िषन 2023-24 के दौरान कुि ₹1.3 िाख करदड
य िना के र्िए राज् ों तक के 50-िषीि ब्याज-मुक्त ऋण के रूप में यिर्ेष सहािता प्रदान की जा रही है ।
क र्िशेष सहायिा केंद्रीय बिट 2023-24 में की गई र्ी।

सेिा र् ि य िना सोंस्कृयत मोंत्रािि • उद्दे श्य: िदजना के तहत धमािथि धार्मिक सोंस्थान ों द्वारा यियर्ष्ट कच्चे खाद्य पदार्ों
की खरीद पर भुगतान यकए गए केंद्रीय माि और सेिा कर (सीिीएसटी) और
एकीकृि माि और सेिा कर (आईिीएसटी) के केंद्र सरकार के र्हस्से की
प्रर्िपूर्िि भारत सरकार द्वारा यििीि सहािता के रूप में की जाएगी।
• िह उन सोंस्र्ानदों कद प्रदान यकिा जाता है जद िनिा/र्क्त ों क र्बना र्कसी
र्ेिर्ाि के र् िन/प्रसाि/िोंगर (सामुिार्यक रस ई)/र्ोंडारा र्नः शुल्क
प्रिान करिे हैं।
• यह एक केंद्रीय क्षेि की य िना है।

अर्ग्रम प्रार्धकरर् िायणज्य एिों उद्यदग र्ििे श व्यापार महार्निे शािय (डीिीएिटी) यिदे र् व्यापार नीयत के तहत अर्ग्रम
य िना मोंत्रािि प्रार्धकरर् य िना िागू करता है ।
• िह इनपुट के शुल्क-मुक्त आयाि की अनुमर्ि िे िा है, ि र्नयािि उत्पाि में
र्ौर्िक रूप से शार्मि ह िे हैं।
• र्कसी र्ी इनपुट के अिािा, यनिानत उत्पाददों के उत्पादन की प्रयक्रिा में
उपभदग/उपिदग की जाने िािी पैकेयजोंग सामग्री, ईोंिन, तेि और उत्प्रेरक की भी
अनुमयत है ।
• इनपुट की पाििा: इनपुट-आउटपुट मानदों डदों के आिार पर सेरर-यियर्ष्ट
मानदों ड सयमयतिदों द्वारा यनिानररत की जाती है ।

स्वामी र्निेश क ष यिि मोंयत्र • यकफािती और मध्य-आि आिास के यिए यिर्ेष यिोंडद स्वामी यनिेर् कदषI एक
सामार्िक प्रर्ाि क ष है जद यिर्ेष रूप से तनािग्रस्त और रुकी हुई आिासीि
पररिदजनाओों कद पूरा करने के यिए बनािा गिा है ।
• िॉन्च र्कया गया: 2019
• प्रबोंर्धि: स्टे ट बैंक समूह की कोंपनी एसबीआईकैप िेंचसन यियमटे ड।
• सोंकटग्रस्त पररय िनाओों के र्िए अोंर्िम उपाय का ऋर्िािा: फोंड पहिी
बार डे ििपसन, समस्याग्रस्त पररिदजनाओों िािे स्र्ायपत डे ििपसन, रुकी हुई
पररिदजनाओों, ग्राहक यर्काितदों और एनपीए खातदों के खराब टर ै क ररकॉडन िािे
डे ििपसन पर यिचार करे गा।

129
14. र्िर्िध
14.1 र्ििे श में पहिा आईआईटी कैं पस
• दे र् के बाहर पहिा र्ारिीय प्रौद्य र्गकी सोंस्थान (आईआईटी) पररसर िोंिार्नया में स्थार्पि र्कया िाएगा।
• प्रिेर् मानदों ड आईआईटी-मिास द्वारा ति यकिा जाएगा, और यडग्री सोंस्र्ान द्वारा प्रदान की जाएगी।
• अपिटीय पररसर के र्िए शैक्षर्र्क रर्नीर्िय ों की य िना बनाने के यिए यजम्मेदार हदगा।
• रािरीय र्शक्षा नीर्ि (एनईपी) 2020 के अनुरूप भी है , जद यसफाररर् करती है यक "उच्च प्रदर्नन करने िािे भारतीि यिश्वयिद्याििदों कद
अन्य दे र्दों में पररसर स्र्ायपत करने के यिए प्रदत्सायहत यकिा जाएगा"।
समािार में पु र स्कार

पुरस्कार के बारे में

पेन र्पोंटर पुरस्कार • यियटर् बाि िेखक और पफॉनमनन्स कर्ि माइकि र सेन क प्रयतयष्ठत PEN यपोंटर पुरस्कार 2023 से
सम्मायनत यकिा गिा है।
• स्र्ायपत: 2009
• िह न बेि पुरस्कार र्ििेिा नाटककार हेर ड र्पोंटर की याि में र्िया िािा है।
• िार्षिक पुरस्कार के र्िए मानिों ड: रािरीयिा िूके, आिरिैंड िा राष्टरमोंडि दे र्दों से हदनी चायहए।

रािरीय र्ूर्िज्ञान पुरस्कार • भारत के राष्टरपयत द्वारा राष्टरीि भूयिज्ञान पुरस्कार-2022 प्रदान यकिे गिे।
• द्वारा स्थार्पि: खान मोंत्रािि, भारत सरकार
• उद्दे श्य: भूयिज्ञान के यियभन्न क्षेत्रदों में असािारण उपिल्कब्धिदों और उत्कृष्ट िदगदान के यिए व्यल्कक्तिदों और
टीमदों कद सम्मायनत करना।



130
TEST YOURSELF

रािव्यिस्था एिों शासन


1. यनम्नयिल्कखत में से यकस दे र् ने िैिायहक बिात्कार कद अपराि (b) केिि दद
घदयषत नहीों यकिा है ? (c) सभी तीन
1. घाना 2. भारत (d) उपरदक्त में से कदई भी नहीों
3. इों डदनेयर्िा 4. जॉडन न उिर- 1 -(d), 2 - (c) , 3 - (b), 4 - (a), 5 - (c)
5. नाइजीररिा
नीचे यदए गए कूट का उपिदग करके सही उिर चुनें: अों ि ररािरीय सों बों ध
(a) केिि 1, 3 और 4 (b) केिि 2, 4 और 5
(c) केिि 3 और 5 (d) 1, 2, 3, 4 और 5 1. यनम्नयिल्कखत में से कौन 'र्ोंघाई सहिदग सोंगठन (एससीओ)'
का सदस्य है /हैं ?
2. 'यनिम 267' के सोंदभन में, यनम्नयिल्कखत कर्नदों पर यिचार करें : 1. कजायकस्तान 2. चीन
1. राज्यसभा का कदई भी सदस्य यनिम 267 के तहत यकसी 3. यकयगनस्तान 4. ईरान
भी यिषि पर चचान के यिए सभापयत कद नदयटस जारी कर 5. तायजयकस्तान 6. उज़्बेयकस्तान
सकता है । नीचे यदए गए कूट का उपिदग करके सही उिर चुनें:
2. यनिम राज्यसभा सदस्य कद सभापयत की मोंजूरी से सदन (a) केिि 1, 3 और 4
के पूिन-यनिान ररत एजेंडे कद यनिोंयबत करने की यिर्ेष (b) केिि 2, 4 और 5
र्ल्कक्त दे ता है । (c) केिि 1, 4, 5 और 6
उपरदक्त कर्नदों में से कौन सा/से सही है /हैं ? (d) उपरदक्त सभी
(a) केिि 1 (b) केिि 2
(c) 1 और 2 ददनदों (d) न तद 1 और न ही 2 2. 'उिरी अटिाोंयटक सोंयि सोंगठन(नाटद)' के सोंदभन में,
यनम्नयिल्कखत कर्नदों पर यिचार करें :
3. 'चुप रहने का अयिकार' यनयहत है : 1. यफनिैंड नाटद का 31िाों सदस्य बना।
(a) अनुिेद 19 (b) अनुिेद 20 2. इस्ताोंबुि सहिदग पहि (ICI) नाटद का एक गठबोंिन है ।
(c) अनुिेद 21 (d) अनुिेद 25 उपरदक्त कर्नदों में से कौन सा/से सही है /हैं ?
(a) केिि 1 (b) केिि 2
4. 'NeSDA पदटन ि' यनम्नयिल्कखत में से यकस क्षेत्र से सोंबोंयित है ? (c) 1 और 2 ददनदों (d) न तद 1 और न ही 2
(a) ई-गिनेंस (b) अोंतररक्ष अन्वेषण
(c) खेि (d) स्वास्थ्य दे खभाि 3. यनम्नयिल्कखत में से कौन सा दे र् 'अोंतरान ष्टरीि उिर दयक्षण
व्यापार गयििारे ' से जुडा है ?
5. 'भारत में आरक्षण' के सोंदभन में यनम्नयिल्कखत में से कौन सा/से 1. अज़रबैजान 2. बेिारूस
कर्न सही है /हैं ? 3. बुल्गाररिा 4. आमेयनिा
1. भारतीि राष्टरीि कानूनी सेिा प्रायिकरण (एनएएिएसए) 5. भारत
बनाम भारत सोंघ (2014) मामिे में , सुप्रीम कदटन ने नीचे यदए गए कूट का उपिदग करके सही उिर चुनें:
फैसिा सुनािा यक टर ाों सजेंडर व्यल्कक्तिदों कद आरक्षण का (a) केिि 1, 3 और 4 (b) केिि 2, 4 और 5
अयिकार है। (c) केिि 1, 4 और 5 (d) उपरदक्त सभी
2. ऊर्ध्ान िर आरक्षण का उद्े श्य जायत पदानुक्रम से उत्पन्न उिर : 1 - (d), 2 - (c) , 3 - (d)
हदने िािी सामायजक यिषमता कद सोंबदयित करना है ।
3. श्रेयणिदों के भीतर िोंयचत समूहदों के यिए सकारात्मक
नीयतिाों प्रदान करने के यिए क्षैयतज आरक्षण सभी
ऊर्ध्ान िर समूहदों में कटौती करता है ।
नीचे यदए गए कूट का उपिदग करके सही उिर चुनें:
(a) केिि एक

131
अथि व्य िस्था कथन-II: भारत में बहुआिामी गरीबदों की सोंख्या में 9.89
प्रयतर्त अोंकदों की उल्लेखनीि यगरािट दजन की गई है , जद
1. 'जीएसटी पररषद' के सोंबोंि में यनम्नयिल्कखत कर्नदों पर यिचार 2015-16 में 24.85% से बढकर 2019-21 में 14.96% हद
करें : गई है ।
1. इसकी स्र्ापना राष्टरपयत द्वारा सोंयििान के अनुिेद उपरदक्त कर्नदों के सोंबोंि में यनम्नयिल्कखत में से कौन सा सही
279A(1) के अनुसार की गई र्ी। है ?
2. पररषद के सदस्यदों में केंि से केंिीि यिि मोंत्री (अध्यक्ष), (a) कर्न-I और कर्न-II ददनदों सही हैं और कर्न-II कर्न-
केंिीि राज्य मोंत्री (यिि) र्ायमि हैं । I के यिए सही स्पष्टीकरण है
3. प्रत्येक राज्य एक यिि िा करािान प्रभारी मोंत्री िा यकसी (b) कर्न-I और कर्न-II ददनदों सही हैं और कर्न-II,
अन्य मोंत्री कद सदस्य के रूप में नायमत कर सकता है । कर्न-I के यिए सही स्पष्टीकरण नहीों है
उपरदक्त कर्नदों में से कौन सा/से सही है /हैं ? (c) कर्न-I गित है िेयकन कर्न-II सही है
(a) केिि एक (b) केिि दद (d) कर्न-I सही है िेयकन कर्न-II गित है
(c) सभी तीन (d) उपरदक्त में से कदई नहीों उिर : 1 - (c) , 2 - (d), 3 - (c) , 4 - (b)

2. यनम्नयिल्कखत कर्नदों पर यिचार करें : पयाि ि रर्


1. चियनत िस्तु कीमतदों में अत्ययिक अल्कस्र्रता कद
1. 'एम्बरग्रीस' के सोंबोंि में यनम्नयिल्कखत कर्नदों पर यिचार करें :
अिर्दयषत करने के यिए 2014-15 में मूल्य ल्कस्र्रीकरण
कथन-I: एम्बरग्रीस सोंरयक्षत र्ुक्राणु व्हेि के पाचन तोंत्र से
कदष की स्र्ापना की गई र्ी।
उत्पन्न हदता है ।
2. 2018-19 के बजट में "ऑपरे र्न ग्रीन्स" की घदषणा की
कथन-II: िह एक दु िनभ पदार्न है यजसका उपिदग कस्तूरी
गई।
नदट् स के सार् इत्र बनाने के यिए यकिा जाता है ।
ऊपर यदए गए कर्नदों में से कौन सा/से सही नही ों है /हैं ?
उपरदक्त कर्नदों के सोंबोंि में यनम्नयिल्कखत में से कौन सा सही
(a) केिि 1 (b) केिि 2
है ?
(c) 1 और 2 ददनदों (d) न तद 1 और न ही 2
(a) कर्न-I और कर्न-II ददनदों सही हैं और कर्न-II कर्न-
I के यिए सही स्पष्टीकरण है
3. यनम्नयिल्कखत कर्नदों पर यिचार करें :
(b) कर्न-I और कर्न-II ददनदों सही हैं और कर्न-II,
कथन-I: इों टरनेर्नि इों स्टीट्यूट फॉर मैनेजमेंट डे ििपमेंट
कर्न-I के यिए सही स्पष्टीकरण नहीों है
(IMD) द्वारा जारी िैयश्वक प्रयतस्पिानत्मकता सूचकाों क 2023 में
(c) कर्न-I गित है िेयकन कर्न-II सही है
भारत 43िें स्र्ान पर है ।
(d) कर्न-I सही है िेयकन कर्न-II गित है
कथन-II: डे नमाकन, आिरिैंड और ल्कस्वट् जरिैंड ने यिश्व
प्रयतस्पिानत्मकता रैं यकोंग में र्ीषन तीन स्र्ान प्राप्त यकए।
2. यनम्नयिल्कखत जदयडिदों पर यिचार करें :
उपरदक्त कर्नदों के सोंबोंि में यनम्नयिल्कखत में से कौन सा सही
रािरीय उद्यान स्थान
है ?
1. कूनद राष्टरीि उद्यान - मध्य प्रदे र्
(a) कर्न-I और कर्न-II ददनदों सही हैं और कर्न-II कर्न-
2. बन्नेरघट्टा राष्टरीि उद्यान - तयमिनाडु
I के यिए सही स्पष्टीकरण है
3. ग्रेट यहमाििन नेर्नि पाकन - जम्मू और कश्मीर
(b) कर्न-I और कर्न-II ददनदों सही हैं और कर्न-II,
उपरदक्त यदए गए िुग्दों में से कौन सा/से सही सुमेयित नही ों
कर्न-I के यिए सही स्पष्टीकरण नहीों है
है /हैं ?
(c) कर्न-I गित है िेयकन कर्न-II सही है
(a) केिि 1 (b) केिि 2 और 3
(d) कर्न-I सही है िेयकन कर्न-II गित है
(c) केिि 2 (d) 1, 2 और 3

4. 'राष्टरीि बहुआिामी गरीबी सूचकाों क: समीक्षा 2023 की प्रगयत'


3. 'एों थ्रदपदसीन िुग' के सोंदभन में यनम्नयिल्कखत कर्नदों पर यिचार
के सोंबोंि में यनम्नयिल्कखत कर्नदों पर यिचार करें :
करें :
कथन-I: भारत में 2015-16 और 2019-21 के बीच 13.5
1. एों थ्रदपदसीन िुग कद एक र्ब्द के रूप में िषन 2000 में
करदड िदग बहुआिामी गरीबी से बाहर यनकिे , यजसमें उिर
पॉि क्रुटज़ेन और िूजीन स्टदएमर द्वारा गढा गिा र्ा।
प्रदे र्, यबहार, मध्य प्रदे र् में सबसे तेजी से कमी आई।

132
2. िह ितनमान भूिैज्ञायनक समि अोंतराि कद दर्ानता है , उपरदक्त कर्नदों में से कौन सा/से सही नही ों है /हैं ?
यजसमें मानि प्रभाि के कारण पृथ्वी के पाररल्कस्र्यतकी (a) केिि 1 (b) केिि 2
तोंत्र में आमूि-चूि पररितनन हुए हैं । (c) 1 और 2 ददनदों (d) न तद 1 और न ही 2
3. इस िुग से जुडी घटनाओों में ग्लदबि िायमिंग, समुि के 4. यनम्नयिल्कखत कर्नदों पर यिचार करें :
स्तर में िृल्कद्ध, समुि का अम्लीकरण, बडे पैमाने पर यमट्टी 1. अि साल्वाडदर दु यनिा का पहिा दे र् है यजसने बाजार
का क्षरण, घातक गमी की िहरदों का आगमन र्ायमि पूोंजीकरण के यहसाब से सबसे बडी यक्रप्टदकरें सी
है । यबटकॉइन कद अपनी कानूनी यनयिदा के रूप में
उपरदक्त कर्नदों में से कौन सा/से सही है /हैं ? अपनािा है ।
(a) केिि एक (b) केिि दद 2. यक्रप्टदकरें सी और केंिीि बैंक यडयजटि मुिाएों ददनदों
(c) तीनदों (d) उपरदक्त में से कदई नहीों ब्लॉकचेन-आिाररत यडयजटि मुिाएों हैं
उिर : 1 – (b), 2 – (b), 3 – (c) उपरदक्त कर्नदों में से कौन सा/से सही नही ों है /हैं ?
(a) केिि 1 (b) केिि 2
र्िज्ञान एिों प्रौद्य र्गकी (c) 1 और 2 ददनदों (d) न तद 1 और न ही 2

1. यनम्नयिल्कखत कर्नदों पर यिचार करें :


5. 'नॉन-फोंयजबि टदकन (एनएफटी)' के सोंदभन में, यनम्नयिल्कखत
1. चोंििान-3 भारत का तीसरा चोंि यमर्न है और सार् ही
कर्नदों पर यिचार करें :
चोंिमा की सतह पर रदबदयटक िैंडर की सॉफ्ट िैंयडों ग
1. कदई भी चीज़ यजसे यडयजटि रूप में बदिा जा सकता
कराने का दू सरा प्रिास है ।
है िह एनएफटी हद सकती है ।
2. चोंििान-2 यमर्न आों यर्क रूप से यिफि हद गिा
2. एनएफटी ब्लॉकचेन पर काम करता है क्दोंयक िह
क्दोंयक इसके िैंडर और रदिर चोंिमा पर सॉफ्ट-िैंयडों ग
उपिदगकतानओों कद यडयजटि सोंपयि का पूणन स्वायमत्व
नहीों कर सके।
दे ता है ।
उपरदक्त कर्नदों में से कौन सा/से सही नही ों है /हैं ?
उपरदक्त कर्नदों में से कौन सा/से सही नही ों है /हैं ?
(a) केिि 1 (b) केिि 2
(a) केिि 1 (b) केिि 2
(c) 1 और 2 ददनदों (d) न तद 1 और न ही 2
(c) 1 और 2 ददनदों (d) न तद 1 और न ही 2
उिर : 1 – (d), 2 – (c) , 3 – (a), 4 – (d), 5 – (d)
2. 'भारत 6जी अिािोंस (बी6जीए)' के सोंदभन में , यनम्नयिल्कखत
कर्नदों पर यिचार करें :
रक्षा और सु र क्षा
1. इसमें भारत में 6G तकनीक के यिकास पर ध्यान दे ने के
1. अक्सर समाचारदों में दे खा जाने िािा "सस्पेंर्न ऑफ
सार् सािनजयनक और यनजी कोंपयनिाों , अनुसोंिान सोंस्र्ान
ऑपरे र्ोंस (एसओओ) समझौता" यकससे सोंबोंयित है:
और मानक र्ायमि हैं।
(a) मयणपुर (b) जम्मू और कश्मीर
2. 6G तकनीक की उन्नयत का समर्नन करने के यिए,
(c) अरुणाचि प्रदे र् (d) यमजदरम
सरकार ने दू रसोंचार प्रौद्यदयगकी यिकास यनयि िदजना के
उिर: 1 - (a)
माध्यम से 240.51 करदड का अनुदान आिोंयटत यकिा
है ।
नीचे यदए गए कूट का उपिदग करके सही उिर चुनें: र्ारिीय समाि और सामार्िक न्याय
(a) केिि 1 (b) केिि 2 1. 'इिामृत्यु' के सोंदभन में यनम्नयिल्कखत कर्नदों पर यिचार करें :
(c) 1 और 2 ददनदों (d) न तद 1 और न ही 2 1. अरुणा रामचन्द्र र्ानबाग बनाम भारत सोंघ (2011)
मामिे में सुप्रीम कदटन ने माना र्ा यक असािारण
3. 'नेर्नि ररसचन फाउों डेर्न (एनआरएफ) यििेिक, 2023' के पररल्कस्र्यतिदों में यनल्किि इिामृत्यु की अनुमयत दी जा
सोंदभन में, यनम्नयिल्कखत कर्नदों पर यिचार करें : सकती है ।
1. इसमें यिज्ञान और इों जीयनिररों ग अनुसोंिान बदडन 2. ल्कस्वट् ज़रिैंड ने इिामृत्यु पर प्रयतबोंि िगा यदिा है
(एसईआरबी) स्र्ायपत करने का प्रस्ताि है । िेयकन डॉरर िा यचयकत्सक की उपल्कस्र्यत में सहािता
2. इसका उद्े श्य एक र्ीषन यनकाि के रूप में राष्टरीि से मरने की अनुमयत दे ता है।
अनुसोंिान फाउों डेर्न की स्र्ापना करना है , यजसके उपरदक्त कर्नदों में से कौन सा/से सही है /हैं ?
पदे न अध्यक्ष प्रिानमोंत्री हदोंगे। (a) केिि 1 (b) केिि 2

133
(c) 1 और 2 ददनदों (d) न तद 1 और न ही 2 (d) कर्न-I सही है िेयकन कर्न-II गित है
उिर: 1 - (c) उिर : 1 - (b), 2 - (c) . 3 - (c) , 4 - (d)

किा और सों स्कृ र्ि समािार ों में स्थान

1. यनम्नयिल्कखत जदयडिदों पर यिचार करें : 1. यनम्नयिल्कखत जदयडिदों पर यिचार करें :


किा रूप - राज् आईिैंड - िे श
1. नमदा किा - अरुणाचि प्रदे र् 1. कच्चार्ीिू - मािदीि
2. कािीघाट पेंयटों ग - पयिम बोंगाि 2. बेिे द्वीप - भारत
3. पट्टयचत्रा - आों ध्र प्रदे र् 3. कािीमोंतन - इों डदनेयर्िा
उपरदक्त िुग्दों में से कौन सा/से सही सुमेयित नही ों है /हैं ? उपरदक्त िुग्दों में से कौन सा/से सही सुमेयित नही ों है /हैं ?
(a) केिि 1 (b) केिि 1 और 3 (a) केिि 1 (b) केिि 1 और 3
(c) केिि 2 (d) उपरदक्त सभी (c) केिि 2 (d) 1, 2 और 3

2. 'मेसदयियर्क िुग' के सोंदभन में, यनम्नयिल्कखत कर्नदों पर यिचार 2. 'कास पठार' के सोंदभन में, यनम्नयिल्कखत कर्नदों पर यिचार करें :
करें : 1. कास पठार के तिछट के एक हायििा अध्यिन से
1. भारत में इसका प्रसार 9,000 ई.पू. से 4,000 ई.पू. तक िगभग 8664 साि पहिे प्रारों यभक-मध्य-हदिदसीन के
र्ा। दौरान भारतीि ग्रीष्मकािीन मानसून में एक महत्वपूणन
2. इसकी यिर्ेषता माइक्रदयिथ्स (छदटे ब्लेड िािे पत्थर के बदिाि का पता चिा है ।
उपकरण) की उपल्कस्र्यत है । 2. िह पयिमी घाट में ल्कस्र्त है ।
उपरदक्त कर्नदों में से कौन सा/से सही है /हैं ? उपरदक्त कर्नदों में से कौन सा/से सही है /हैं ?
(a) केिि 1 (b) केिि 2 (a) केिि 1 (b) केिि 2
(c) 1 और 2 ददनदों (d) न तद 1 और न ही 2 (c) 1 और 2 ददनदों (d) न तद 1 और न ही 2

3. 'सोंर्ाि यििदह िा हूि' के सोंदभन में यनम्नयिल्कखत कर्नदों पर 3. यनम्नयिल्कखत जदयडिदों पर यिचार करें :
यिचार करें : झीि - राज्
1. यििदह हरे -भरे दायमन-ए-कदह क्षेत्र में हुआ र्ा। 1. पैंगदोंग त्सद - िद्ाख
2. इसका नेतृत्व दद भाइिदों यसिू और कान्हू ने यकिा र्ा। 2. कदल्लेरु - तयमिनाडु
उपरदक्त कर्नदों में से कौन सा/से सही है /हैं ? 3. िेम्बनाड झीि - हररिाणा
(a) केिि 1 (b) केिि 2 उपरदक्त िुग्दों में से कौन सा सही सुमेयित है ?
(c) 1 और 2 ददनदों (d) न तद 1 और न ही 2 (a) केिि 1 (b) केिि 2 और 3
(c) केिि 3 (d) 1, 2 और 3
4. 'िैयश्वक र्ाोंयत सूचकाों क' के सोंबोंि में यनम्नयिल्कखत कर्नदों पर उिर : 1 - (a), 2 - (c) . 3 - (a)
यिचार करें :
कथन-I: िह इों स्टीट्यूट फॉर इकदनॉयमक्स एों ड पीस (IEP) ख़बर ों में य िनाएाँ
द्वारा प्रयतिषन जारी यकिा जाता है ।
कथन-II: भारत इसमें 125िें स्र्ान पर है । 1. यनम्नयिल्कखत कर्नदों पर यिचार करें :
उपरदक्त कर्नदों के सोंबोंि में यनम्नयिल्कखत में से कौन सा सही कथन-I: SWAMIH एक श्रेणी-II AIF (िैकल्कल्पक यनिेर्
है ? कदष) ऋण यनयि है जद SEBI (भारतीि प्रयतभूयत और यियनमि
(a) कर्न-I और कर्न-II ददनदों सही हैं और कर्न-II कर्न- बदडन ) के सार् पोंजीकृत है ।
I के यिए सही स्पष्टीकरण है कथन-II: िह फोंड यिि मोंत्रािि द्वारा प्रािदयजत है ।
(b) कर्न-I और कर्न-II ददनदों सही हैं और कर्न-II, उपरदक्त कर्नदों के सोंबोंि में यनम्नयिल्कखत में से कौन सा सही
कर्न-I के यिए सही स्पष्टीकरण नहीों है है ?
(c) कर्न-I गित है िेयकन कर्न-II सही है

134
(a) कर्न-I और कर्न-II ददनदों सही हैं और कर्न-II कर्न- (c) सभी तीन (d) उपरदक्त में से कदई नहीों
I के यिए सही स्पष्टीकरण है
(b) कर्न-I और कर्न-II ददनदों सही हैं और कर्न-II, 3. 'अटि ििद अभ्युदि िदजना' यकसके तत्वाििान में है :
कर्न-I के यिए सही स्पष्टीकरण नहीों है (a) सामायजक न्याि और अयिकाररता मोंत्रािि
(c) कर्न-I गित है िेयकन कर्न-II सही है (b) मयहिा एिों बाि यिकास मोंत्रािि
(d) कर्न-I सही है िेयकन कर्न-II गित है (c) सहकाररता मोंत्रािि
(d) स्वास्थ्य और पररिार कल्याण मोंत्रािि
2. यनम्नयिल्कखत में से कौन सा पीएम-प्रणाम िदजना का उद्े श्य है ?
1. सौर पोंपदों और यग्रड से जुडे सौर और अन्य निीकरणीि 4. 'समर्न िदजना' यकन क्षेत्रदों कद ियक्षत करती है ?
ऊजान यबजिी सोंिोंत्रदों (आरईपीपी) की स्र्ापना कद बढािा 1. कपडा 2. ऑटदमदबाइि
दे ना 3. खनन 4. िातुकमन
2. यमट्टी कद बचाना और उिनरकदों के यटकाऊ, सोंतुयित नीचे यदए गए कूट का उपिदग करके सही उिर चुनें:
उपिदग कद बढािा दे ना। (a) केिि 3 और 4 (b) केिि 2
3. पोंजाब, हररिाणा और उिर प्रदे र् में परािी जिाने की (c) 2, 3 और 4 (d) केिि 1
समस्या कद कम करना।
नीचे यदए गए कूट का उपिदग करके सही उिर चुनें: उिर : 1 - (b), 2 - (a), 3 - (a), 4 - (d)
(a) केिि एक (b) केिि दद



135
136

You might also like